FNP Practice Questions - COMBO

Pataasin ang iyong marka sa homework at exams ngayon gamit ang Quizwiz!

2. All of the following are infections that affect mostly the labia and vagina except: A) Bacterialvaginosis B) Candidiasis C) Trichomoniasis D) Chlamydia trachomatis

D) Chlamydia trachomatis Infections that commonly affect the labia and vagina include bacterial vaginosis, candidiasis, and trichomoniasis. Chlamydia trachomatis commonly affects the cervix, endometrial lining , fallopian tubes, and pelvic cavity.

All of the following drugs interfere with the metabolism of oral contraceptives except: A) Tetracycline B) Rifampin C) Phenytoin (Dilantin) D) Ciprofloxacin (Cipro)

D) Ciprofloxacin (Cipro) Cipro is the only medication listed that will not interfere with the metabolism and absorption of OCPs.

Which of the following drugs is recommended by the Centers for Disease Control and Prevention (CDC) as first-line treatment for treating infections by the organ- ism Bacillus anthracis (anthrax)? A) Clindamycin (Cleocin) B) Fluconazole (Diflucan) C) Penicillin G injection D) Ciprofloxacin (Cipro)

D) Ciprofloxacin (Cipro) The CDC's first-line treatment recommendation for anthrax is Cipro.

Henry, a 50-year-old male, complains of marked scalp tenderness accompanied by a bad headache on his left temple. He reports a sudden loss of vision in the left eye for the past several hours. The neurologic exam is normal except for the loss of vision in the left eye. Which of the following is most likely? A) Cluster headache B) Migraine headache with aura C) Migraine headache without aura D) Giant cell arteritis

D) Giant cell arteritis Loss of vision does not occur with headaches. Loss of vision, along with marked scalp tenderness with a bad headache on the left temple, are symptoms of giant cell arteritis. Giant cell arteritis is inflammation of the arteries in the head at the temple area, which causes these symptoms.

All of the following are classified as activities of daily living (ADLs) except: A) Ability to feed self (self-feeding) B) Ability to manage bladder and bowel elimination C) Personal hygiene and grooming D) Grocery shopping

D) Grocery shopping Grocery shopping, housework, and managing one's finances are considered instrumental ADLs (IADLs).

Sources of legal risk for the nurse practitioner would include all of the following except: A) Invasive procedures B) Electronic medical record entries C) Prescribing medication D) In-service training

D) In-service training Legal risks for the NP include invasive procedures, electronic medical record entries, and prescribing of medications. Inservice training does not directly affect the patient; therefore, legal risks are not a problem.

The following clinical signs are seen in Parkinson's disease except: A) Pill-rolling tremor B) Difficulty initiating involuntary movement C) Shuffling gait with cogwheel rigidity D) Increased facial movements due to tics

D) Increased facial movements due to tics Clinical signs of Parkinson's dis- ease include pill-rolling tremors, difficulty initiating involuntary movements, and shuffling gait with cogwheel rigidity. Facial movement decreases and is known as generalized rigidity with masked facies.

All of the following statements are correct regarding licensure for nurse practitioners except: A) It ensures a minimum level of professional competency B) It grants permission for an individual to practice in a profession C) It requires verification of educational training from an accredited graduate program D) It reviews information via a nongovernmental agency

D) It reviews information via a nongovernmental agency Licensure for nurse practitioners includes a minimum level of professional competency, grants per- mission for one to practice in the profession, and verifies completion of an accred- ited graduate program.

Which of the following physical exam findings is most specific for systemic lupus erythematosus (SLE)? A) Swollen and painful joint involvement B) Fatigue and myalgia C) Stiffness and swelling of multiple joints D) Malar rash

D) Malar rash A malar rash is the butterfly-shaped rash on the middle of the face that is caused by a type of photosensitivity reaction. It is associated with SLE. The other answer options are found with other diseases, such as rheumatoid arthritis, polymyalgia rheumatica, and so on.

Systemic lupus erythematosus (SLE) is more common among the following racial backgrounds except: A) AfricanAmericans B) Asians C) Hispanics D) Mediterranean

D) Mediterraneans Lupus is more common among women of AfricanAmerican, Asian, and Hispanic racial backgrounds compared with those with Mediterranean background (Italians, Greeks, etc.).

Ken has type 2 diabetes mellitus and a "sensitive stomach." Which medication is least likely to cause him gastrointestinal distress? A) Naproxen sodium (Anaprox) B) Aspirin (Bayer's aspirin) C) Erythromycin (E-mycin) D) Sucralfate (Carafate)

D) Sucralfate (Carafate) Naproxen sodium, aspirin, and erythromycin all have gastrointestinal side effects. Sucralfate is used to protect the stomach lining by building a protective layer over the stomach lining; it allows healing to occur

A 72-year-old female complains to you of a crusty and non-healing small ulcer on her upper lip that she has had for several years. Which of the following would you recommend? A) Triamcinolone acetonide (Kenalog) cream BID for 2 weeks B) Triple antibiotic ointment BID x 2 weeks C) Hydrocortisone 1% cream BID for 2 weeks D) The patient needs to be evaluated by a dermatologist

D) The patient needs to be evaluated by a dermatologist Non healing ulcers of the skin are a risk for skin cancer and should be evaluated by a dermatologist for treatment.

20-3. Which of the following, as defined by the American Nurses Association, is not a main population group with unique health concerns? A. Elderly patients B. Obese patients C. Transgender patients D. Female patients

D. Female patients

What structure of the eye is responsible for 20/20 vision? A. Rods B. Cones C. Optic disc D. Fovea and macula

D. Fovea and Macula

A COPD patient with a GOLD spirometric classification of 2, with a self-report of 3 exacerbations in the past year and CAT score of 12 would be characterized as which risk group? A. Group A B. Group B C. Group C D. Group D

D. Group D

16-25. Which clinical manifestation is characteristic of fire ant stings? A. Round, bluish colored papules B. Single, large bull's-eye rash C. Honey-colored crusting on an erythematous base D. Groups of red spots with blisters on top

D. Groups of red spots with blisters on top

11-21. Which of the following would not meet the criteria identifying low-risk headache? A. History of similar headache B. A 27-year-old patient with new onset headache C. A pericranial headache associated with neck muscle tenderness D. Headaches triggered by the Valsalva maneuver

D. Headaches triggered by the Valsalva maneuver

11-33. Common signs of Parkinson disease include all of the following except: A. Shuffling/short steps B. Bradykinesia C. Masked face D. Hyperphonia

D. Hyperphonia

16-8. A patient suffering from psoriasis is concerned about possible adverse effects from steroid overuse. The nurse practitioner knows adverse effects can include all of the following except: A. Striae B. Hypopigmentation C. Thinning skin D. Hyperpigmentation

D. Hyperpigmentation

13-6. When considering a diagnosis of Addison's disease, potential differential diagnoses include: A. Cushing syndrome B. Osteoporosis C. Depression D. Hyperthyroidism

D. Hyperthyroidism

On a 12-lead ECG, which leads normally exhibit upright QRS? A. I, II, AVR, AVF, V1, V2 B. AVR, AVL, AVF, V1, V2, V5 C. II, III, AVL, V1, V5, V6 D. I, II, III, AVF, V5, V6

D. I, II, III, AVF, V5, V6

12-8. In assessing a patient for PTSD, you should expect the following to be reported except: A. A feeling of detachment B. Hyperarousal C. Sleep issues D. Inability to recall the precipitating event

D. Inability to recall the precipitating event

11-14. Which of the following is most indicative of delirium (as opposed to dementia)? A. Memory impairment B. Sleep disturbance C. Permanence D. Inattention

D. Inattention

Which complaint would not be of concern in a patient newly started on norethindrone/ethinyl estradiol? A. Headache B. Abdominal pain C. Diplopia D. Irregular bleeding

D. Irregular bleeding

11-23. Which of the following tests may be positive in the patient with meningitis? A. Kernig and Barlow B. Galeazzi and Brudzinski C. Galeazzi and Trendelenburg D. Kernig and Brudzinski

D. Kernig and Brudzinski

The nurse practitioner is ordering intravenous fluids for a child with severe dehydration secondary to gastroenteritis who weighs 10 kilograms. Which of the following is most appropriate? A. 5% dextrose in water at 35 mL/hour B. 3% normal saline at 40 mL/hour C. Lactated Ringer solution at 42 mL/hour D. Lactated Ringer solution 200 mL bolus

D. Lactated Ringer solution 200 mL bolus

14-14. Sequelae associated with multiple myeloma include all of the following except: A. Osteolytic lesions B. Hypercalcemia C. Renal failure D. Liver failure

D. Liver failure

19-34. Reimbursement for services by private insurance companies is based on reimbursement by: A. Blue Cross; Blue Shield B. Medicaid C. Health Maintenance Organizations D. Medicare

D. Medicare

4-14. The following patients are being seen in the oncology clinic: Melissa F., who is receiving whole brain irradiation for metastatic breast cancer; Jason S., who is receiving interferon for Kaposi sarcoma; and Sandra M., who is receiving treatment with cyclophosphamide for ovarian cancer. The nurse practitioner knows that these three patients are at risk for which common side effect? A. Increased energy prior to treatment B. Insomnia C. Tachycardia D. Oral candidiasis

D. Oral candidiasis

13-20. Diagnosis of hypoglycemia is made by obtaining a blood glucose sample during the event. If a sample can not be taken at that time, which of the following diagnostic tests can be conducted to replicate the conditions in which hypoglycemia would be present? A. 12-hour fasting blood glucose B. Insulin-induced hypoglycemia test C CT scan D. Oral glucose tolerance test

D. Oral glucose tolerance test

18-11. When describing palliative care to an elderly patient, which statement best describes the treatment option? A. Palliative care is focused on care and comfort, not cure B. Palliative care focuses on symptom relief, but only at the end of life C. Palliative care is paid for by Medicare Part A Medicaid, and insurance D. Palliative care may include curative treatment

D. Palliative care may include curative treatment

16-3. A mother has brought in her 7-year-old son for examination. According to the mother, the boy has been feeling "under the weather" and has had a mild temperature, mild sore throat, and an on-and-off headache for a week. While her son states he feels better and his fever and sore throat are now gone, he still looks "feverish" to the mother, as the skin on his cheeks has turned bright red as if he were slapped. What is the likely cause of his condition? A. Child abuse B. Toxoplasma gondi C. Coxsackievirus D. Parvovirus B19

D. Parvovirus B19

You receive a callback from the mother of a 5-year-old patient seen in clinic yesterday by another provider. The mother is requesting antibiotics. She states her child is not getting better and that the other provider did nothing to help her child. Prior to getting on the phone with the mother, you look through the on the patient's chart for which piece of information? A. Patient's height and weight at most recent visit B. Mother's history of calling the clinic C. Patient's allergy list D. Patient's diagnosis and treatment plan from most recent visit

D. Patient's diagnosis and treatment plan from most recent visit

12-19. A patient who is new to your practice states she has been treated for bipolar disorder. She requests you continue her current medications, which are lamotrigine, dextroamphetamine, and sertraline. She has current prescription bottles in her name for these medications. Your plan includes a diagnostic assessment and: A. Prescribing all three medications, since these medications are evidenced-based practice for bipolar disorder B. Not prescribing any medications until you receive her records from previous healthcare providers C. Not prescribing any medications and referring her to a mental health provider D. Prescribing lamotrigine and sertraline only and referring her for ADHD testing

D. Prescribing lamotrigine and sertraline only and referring her for ADHD testing

11-40. A patient is newly diagnosed with myasthenia gravis. Which of the following includes symptoms usually associated with myasthenia gravis? A. Bowel and bladder dysfunction and spasticity B. Ascending paralysis with ultimate respiratory symptoms C. Cogwheel rigidity and loss of coordination D. Progressive weakness that is worse at the day's end

D. Progressive weakness that is worse at the day's end

A 53-year-old male presents to the clinic with a chief complaint of sudden onset shortness of breath and non-radiating chest pain that he describes as sharp and substernal. He rates it 6/10. His vital signs are: T 100.5 F, HR 119, RR 21, BP 152/90. Upon exam, he appears anxious and you note an accentuated second heart sound and rales. The FNP suspects which of the following diagnoses? A. Acute myocardial infarction B. Pneumothorax C. Pleurisy D. Pulmonary embolism

D. Pulmonary embolism

15-3. An otherwise healthy adult has a cough, congestion, and fever. The nurse practitioner should be aware that: A. A fever of 102.0°F (38.9°C) indicates need for antibiotics B. Antibiotics are required for respiratory infections C. Acute onset indicates bacterial infection D. Respiratory infections are often viral

D. Respiratory infections are often viral

Group A strep pharyngitis may result in which of the following? A. Rheumatic fever, Kawasaki syndrome, acute glomerulonephritis B. Scarlet fever, acute glomerulonephritis, hemolytic uremic syndrome C. Hemolytic uremic syndrome, rheumatic fever, scarlet fever D. Rheumatic fever, acute glomerulonephritis, scarlet fever

D. Rheumatic fever, acute glomerulonephritis, scarlet fever

An adult patient is being scheduled for an intravenous pyelogram (IVP). What is an important preprocedure assessment the nurse practitioner should make? A. Ability to urinate independently B. Mobility status C. Oral fluid intake D. Current medication list

D. Current medication list

In order to cover meals eaten within 30-60 minutes, which type of insulin should be prescribed by the nurse practitioner? A. Intermediate-acting insulin B. Long-acting insulin C. Rapid-acting insulin D. Short-acting insulin

D. Short-acting insulin

18-9. Which of the following risk factors increases an elderly patient's risk for developing a pressure ulcer? A. Use of a walker B. Sleep apnea C. Alcohol use D. Smoking

D. Smoking

13-16. Treatment options for Cushing syndrome can include: A. Corticosteroid replacement B. Parathyroid gland removal C. Propranolol D. Tapering off exogenous steroids

D. Tapering off exogenous steroids

11-36. Patient and family education on Parkinson disease may include making the patient and family aware that: A. Parkinson treatment will slow and eventually reverse the symptoms B. The altered gait and rigidity seen in Parkinson disease necessitates the use ofa wheelchair for all patients C. Antidepressant medications, specifically SSRIS, can be useful to treat both drooling and depression D. The patient is at risk for social isolation, as others may interpret the patient's slowed move- ments and stony expression for slow thinking and social disinterest

D. The patient is at risk for social isolation, as others may interpret the patient's slowed move- ments and stony expression for slow thinking and social disinterest

3-10. The term prevalence refers to: A. Health conditions required through statute, ordinance, or administrative rule to be reported to a public health agency when diagnosed in an individual B. A group of cases of a specific disease or illness clearly in excess of what one would normally expect in a particular geographic area C. Illness or lack of health caused by disease, disability, or injury D. The proportions of people in a population who have some attribute or condition at a given point in time, or during a specified time period

D. The proportions of people in a population who have some attribute or condition at a given point in time, or during a specified time period

11-16. Which documentation reflects a positive Romberg test? A. Unable to walk heel to toe in a straight line B. Unable to maintain balance with one foot outstretched C. Unable to maintain upper arms outstretched (i.e., one arm drops suddenly) with eyes closed D. Unable to maintain balance when standing with feet together with arms extended and eyes closed

D. Unable to maintain balance when standing with feet together with arms extended and eyes closed

The most common presenting symptom of pancreatic cancer is midepigastric pain that is characterized as being worse at all of the following times except: A. At night B. After eating C. When lying flat D. When sitting forward

D. When sitting forwardo

The nurse practitioner (NP) is evaluating a teenager who reports having taken 42 extra-strength acetaminophen (Tylenol) caplets. What will the NP order as an antidote? A. magnesium sulfate (Epsom salt) B. Calcium gluconate (Kalcinate) C. acetylsalicylic acid (Aspirin) D. acetylcysteine (Mucomyst)

D. acetylcysteine (Mucomyst)

The nurse practitioner has diagnosed a young adult with acute uncomplicated pyelonephritis. Which is the appropriate first-line treatment? A. amoxicillin-clavulanate (Amoxil) B. trimethoprim-sulfamethoxazole (Bactrim DS) C. nitrofurantoin (Macrobid) D. ciprofloxacin (Cipro)

D. ciprofloxacin (Cipro)

A child has recently been diagnosed with giardiasis. Which of the following medications will the nurse practitioner prescribe? A. ivermectin (Stromectal) B. mesalamine (Apriso) C. metoclopramide (Reglan) D. metronidazole (Flagyl)

D. metronidazole (Flagyl)

Outpatient treatment for diverticulitis includes which antibiotic regimen? A. ciprofloxacin (Cipro) and doxycycline (Vibramy- cin) for 10 days B. amoxicillin-clavulanate (Augmentin) and cipro- floxacin (Cipro) for 10 days C. trimethoprim-sulfamethoxazole (Bactrim) and levofloxacin (Levaquin) for 10 daysaa D. metronidazole (Flagyl) and ciprofloxacin (Cipro) for 10 days

D. metronidazole (Flagyl) and ciprofloxacin (Cipro) for 10 days

The nurse practitioner is caring for an 18-month-old who has had vomiting and diarrhea for one day. Which of the following antiemetics should the nurse practitioner avoid prescribing? A. chlorpromazine (Thorazine) B. metoclopramide (Reglan) C. ondansetron (Zofran) D. promethazine (Phenergan)

D. promethazine (Phenergan)

A 32-year-old G1PO presents to the office with complaints of daily heartburn. She is already taking calcium carbonate several times a day and is asking if there is anything else she can do. You write which of the following prescriptions as your first-line treatment? A. pantoprazole (Protonix) 40 mg PO daily B. doxylamine (Unisom) 10 mg qhs C. diphenhydramine (Benadryl) 25 mg PO TID D. ranitidine (Zantac) 75 mg PO qhs

D. ranitidine (Zantac) 75 mg PO qhs

11-35. The nurse practitioner's 40-year-old patient has a recent diagnosis of early-onset Parkinson disease. The patient's tremor is very mild. Which treatment is the most appropriate for this patient? A. Deep brain stimulation (DBS) B. levodopa (Larodopa) C. haloperidol (Haldol) D. ropinirole (Requip)

D. ropinirole (Requip)

16-20. When prescribing a shampoo for seborrheic derma- titis, the FNP should ensure the shampoo includes which of the following ingredients? A. griseofulvin B. Neosporin C. Perfume D. selenium sulfide

D. selenium sulfide

Which of the following sets of clinical manifestations is consistent with acute angle-closure glaucoma? A. Gradual increase in intraocular pressure resulting in eye pain B. sudden increase in intraocular pressure without eye pain C. Gradual increase in intraocular pressure without eye pain D. sudden increase in intraocular pressure with eye pain

D. sudden increase in intraocular pressure with eye pain

An NP is giving dietary counseling to an alcoholic male who has been recently diagnosed with folic acid deficiency anemia. Which of the following foods should the NP recommend? A. Tomatoes, oranges, bananas B. Cheese, yogurt, milk C. Lettuce, beef, dairy products. D. Spinach, liver, and whole wheat bread

D: Spinach liver and whole wheat

16-13. If malignant melanoma is suspected, which of the following diagnostic methods should be utilized? A. Shave biopsy B. Fine-needle biopsy C. Punch biopsy D. Excisional biopsy

D. Excisional biopsy

Treatment for mild preeclampsia includes all of the following except: A) Bed rest except for bathroom privileges B) Close monitoring of weight and blood pressure C) Close follow-up of urinary protein, serum creatinine, and platelet count D) A prescription of methyldopa (Aldomet) to control blood pressure

D) A prescription of methyldopa (Aldomet) to control blood pressure Recommended care for women diagnosed with preeclampsia includes bed rest with bath- room privileges, weight and BP monitoring, and closely following urine protein and serum protein, creatinine, and platelet counts. Oral medications are not used as first-line treatment.

The signs and symptoms of dementia may include all of the following except: A) Personality changes B) Difficulty in verbalizing C) Difficulty in recognizing familiar objects D) Abstract thinking ability is increased

D) Abstract thinking ability is increased Characteristics of dementia include irreversible symptoms with a gradual onset. Short-term memory loss is an early sign of dementia. As symptoms progress, the patient may become incoherent, unable to talk, walk, feed self, or perform self-care.

You would associate a positive iliopsoas muscle test result with: A) Left cerebral vascular accident B) Urinary tract infection C) Heel fractures D) Acute abdomen

D) Acute abdomen A positive iliopsoas muscle test is seen with an acute abdomen. This test is performed by asking the patient to actively flex the thigh at the hip. A "positive psoas sign" is noted when the patient exhibits pain in the right lower quadrant due to the inflamed tissue. The right iliopsoas muscle lies under the appendix, so pain may suggest appendicitis.

All of the following factors have been found to increase the risk of atrial fibrillation in predisposed individuals except: A) Hypertension B) Excessive alcohol intake in susceptible individuals C) Theophylline (Theodur) and pseudoephedrine (Sudafed) D) Acute esophagitis

D) Acute esophagitis Factors that influence the risk of atrial fibrillation include hypertension, excessive alcohol consumption, and medications such as Theodur and Sudafed.

Which of the following is most likely to cause delirium? A) Dehydration B) Multiple brain infarcts C) Malnutrition D) Acute infection

D) Acute infection Delirium is an acute decline in mental status and is temporary. Common causes are fever, shock, drugs, alcohol, severe dehydration, and acute infection.

You are performing a pelvic exam on a 25-year-old sexually active woman. You palpate a tender and warm cystic mass on the lower edge of the left labia majora, which is red. The most likely diagnosis is: A) Skene's gland cyst B) Cystocele C) Lymphogranuloma venereum D) Bartholin's gland abscess

D) Bartholin's gland abscess Bartholin's glands are located in the base of the labia minora at about the 4 o'clock and 8 o'clock positions. Their function is to provide moisture for the vestibule. They are small (about pea sized) unless they become clogged or infected. If glands become clogged or infected, an abscess may form and glands will enlarge and become painful.

Which of the following cranial nerves is evaluated when a wisp of cotton is lightly brushed against the corner of the eye? A) CN II B) CN III C) CN IV D) CN V

D) CN V Cranial nerves are assessed as follows: II (optic): distance vision, near vision . III, IV, VI (oculomotor, trochlear, abducens): EOMs, visual fields of gaze. V1 (trigeminal 3 branches: V19 ophthalmic), V2 (maxillary), V3 (mandibular): motor portion, clench jaws; sensory portion, corneal reflex/facial sensation

4-21. Which calcium channel blocker has little to no effect on conduction through the SA and AV nodes? A. procainamide (Pronestyl) B. verapamil (Calan) C. amlodipine (Norvasc) D. diltiazem (Cardizem)

C. amlodipine (Norvasc)

18-3. The cardiac changes associated with normal physiological aging can have effects across the body systems. Which result below is not normally expected? A. Poor skin healing B. Reduced cardiac ejection fraction C. Increased blood pressure D. Multiple-infarct dementia

D. Multiple-infarct dementia

13-36. Diagnostic testing for thyroid cancer can include: A. T3 and T4 B. Large needle biopsy C. Endoscopy D. Neck ultrasound

D. Neck ultrasound

13-7. A hallmark sign of gestational diabetes (GD) in the expectant mother is/are: A. Glycosuria B. Vision changes C. Polydipsia D. No signs

D. No signs

4-1. All of the following may effect a drug's absorption except: A. Size of the drug molecule B. Food in the gut C. Available surface area D. P450 enzyme system

D. P450 enzyme system

Constipation is considered chronic if it lasts longer than _____________ weeks. A. 8 В. 10 С. 12 D. 14

С. 12

The span of the normal adult liver is: A) 15 to 18 cm in the midclavicular line B) 12 to 16 cm in the right midclavicular line C) 2 to 6 cm in the midsternal line D) 4 to 8 cm in the midsternal line

A) 15 to 18 cm in the midclavicular line This range is considered the normal span for adults.

14-10. A definitive sign/symptom associated with Hodgkin lymphoma (HL) is: A. Cough B. Pruritus C. Night sweats D. Reed-Sternberg cells

D. Reed-Sternberg cells

The bacterium responsible for the highest mortality in pts with CAP is: A. Strep Pneumo B. Mycoplasma pneumo C. Moraxella catarrhalis D. Haemophilus influenzae

Streptococcus pneumoniae

The following abnormal lab result may be seen in patients with acute mononucleosis except: A. Lymphocytosis and/ or atypical lymphocytes B. Positive EBV titers for (Ig) M and Ig G C. Elevated liver function tests D. Elevated BUN and Ct

A Lymphocytosis and/ or atypical lymphocytes- Mono caused by virus not infection. More likely to see EBV titers, elevated LFTs, and BUN CT elevation

Rovsing's sign is associated with which of the following? A) An acute abdomen, such as during a ruptured appendix B) Knee instability C) Damage to the meniscus of the knee D) Acute cholelithiasis

A) An acute abdomen, such as during a ruptured appendix Rovsing's sign identifies an acute abdomen, such as acute appendicitis. Rovsing's maneuver is per- formed with the patient in the supine position, palpating deep into the left lower quadrant of the abdomen and having referred pain to the right lower quadrant.

When Molluscum contagiosum is found on the genital area of children, which of the following is the best explanation? A) It should raise the suspicion of child sexual abuse B) It is not considered a sexually transmitted disease C) It is caused by atypical bacteria D) It is caused by the poxvirus and will resolve on its own

A) It should raise the suspicion of child sexual abuse Molluscum contagiosum is spread by skin-to-skin contact. Lesions found in the genital area of young children should be evaluated for suspicion of child sexual abuse.

A 14-year-old female who is sexually active is brought in by her mother for an immunization update. According to the mother, her daughter had 1 dose of hepatitis B vaccine. Which of the following vaccines would you administer at this visit? A) Td and hepatitis B B) DTaP (diphtheria, tetanus, acellular pertussis) and hepatitis B C) Hepatitis B only D) MMR (measles, mumps, rubella) and Td

A) Td and hepatitis B The CDC recommends Td and hepatitis B for 14-year-old females. It is recommended for her to continue the hepatitis series, which includes a total of 3 injections.

Thiazide diuretics have been shown to have a beneficial effect on the bones. Hypertensive women with osteopenia or osteoporosis benefit from thiazide diuretics. What is the mechanism of action for its effect on the bones? A) Thiazide diuretics decrease calcium excretion by the kidneys and stimulate osteoclast production B) Thiazide diuretics increase both calcium and magnesium retention by the kidneys C) Thiazide diuretics increase bone mineral density (BMD) D) Thiazide diuretics influence electrolyte excretion by the kidneys

A) Thiazide diuretics decrease calcium excretion by the kidneys and stimulate osteoclast production This positive side effect of thiazides results in a decrease in calcium bone loss and an increase in the bone mineral density.

3-11. Which of the following tests carries a grade A recommendation for a 67-year-old male who has smoked a pack of cigarettes every day for the last 25 years? A. Abdominal ultrasound B. Chest CT C. PSA D. Pulmonary function test

A. Abdominal ultrasound

Regarding benzodiazepines, which of the following is true? A. They provide sedative effects B. They produce general anesthesia C. They have analgesic actions D. They require weeks for an effect

A. They provide sedative effects

Orchitis is caused by which of the following? A. Mumps virus B. Measles virus C. Chlamydia Trachomatis D. Chronic UTIs that are not correctly treated

A: Mumps: Orchitis (or inflammation of one or both of the testicles) occurs with the mumps

Which of the following laboratory tests is positive in a large number of patients with systemic lupus erythematosus? A) Antinuclear antibody (ANA) B) Rheumatoid factor C) Antiparietal antibody D) Immunoglobulin

Antinuclear antibody (ANA) Screening tests for systemic lupus erythematosus include antinuclear antibody. The rheumatoid factor test is performed to diagnose rheumatoid arthritis. Antiparietal antibody testing is done to evaluate for anti- bodies against the parietal cells. The parietal cells make a substance that the body needs to absorb Vitamin B12. Immunoglobulin testing is done to assess for the amount of antibodies in the blood for a specific disease.

A bulla is defined as: A) A solid nodule less than 1 cm in size B) A superficial vesicle filled with serous fluid greater than 1 cm in size C) A maculopapular lesion D) A shallow ulcer

B) A superficial vesicle filled with serous fluid greater than 1 cm in size A bulla is defined as a superficial vesicle filled with serous fluid greater than 1 cm in size.

Precocious puberty is defined as the onset of secondary sexual characteristics before the age of: A) Age 7 in girls and age 8 in boys B) Age 8 in girls and age 9 in boys C) Age 9 in girls and age 10 in boys D) Age 9 for both girls and boys

B) Age 8 in girls and age 9 in boys Precocious puberty is defined as onset of secondary sexual characteristics by the age of 8 years in girls and 9 years in boys.

13-18. About 80 % of Cushing sufferers have: A. Irregular menses B. Hypertension C. Hypotension D. Hypocortisolism

B. Hypertension

16-31. Which condition is associated with a herald patch? A. Tinea corporis B. Pityriasis rosea C. Carbuncles D. Impetigo

B. Pityriasis rosea

For which of the following would the nurse practitioner not recommend stretching exercises as a part of the initial treatment plan? A. Tendonitis B. Sprain C. Plantar fasciitis D. Bursitis

B. Sprain

A 12 y.o. male's peak inspiratory flow rate results show 60-80% of the predicted range. How would you classify his asthma? A. Mild intermittent B. Mild Persistent C. Moderate persistent D. Severe asthma

C moderate persistent- 60-80% peak flow tests is moderate persistent and anything less than 60% is severe

Which of the following patients is least likely to become an alcoholic? A) A patient whose father has a history of alcoholism B) A patient whose wife complains that he drinks too much C) A patient who drinks one cup of wine nightly with dinner D) A patient who feels he drinks all the time

C) A patient who drinks one cup of wine nightly with dinner Excessive use or exposure to alcohol puts that patient at risk for becoming an alcoholic. A patient who drinks one glass of wine at dinner has a lower risk of becoming an alcoholic than someone who has been exposed to alcohol while growing up, one who drinks all the time, or one whose family believes he is drinking excessively.

During a routine physical exam on an 82-year-old, the nurse practitioner palpates an irregular mass on the midabdomen that is not tender and is about 2 cm in size. Which of the following is the best initial imaging test to further evaluate the abdominal mass? A) CT scan of the abdomen B) KUB study C) Abdominal ultrasound D) MRI of the abdomen

C) Abdominal ultrasound The ultrasound or sonogram is used as an initial imaging test for abdominal tumors. A CT scan can be ordered at a later time, but it is not considered an initial imaging test in the primary care area.

Human papilloma virus infection in women has been associated with the development of: A) Ectopic pregnancy B) Infertility C) Cervical cancer D) Pelvic inflammatory disease

C) Cervical cancer HPV is a virus associated with cervical cancer. HPV vaccine is now available for girls and boys between the ages of 15-21 years of age to help prevent 4 strains of this virus that is linked to cervical cancer.

In most states, patients younger than age 18 years may consent to healthcare without parental or legal guardian consent except for which of the following? A) Contraception B) Pregnancy C) School physicals D) STD evaluation and treatment

C) School physicals Any student younger than 18 years of age must have paren- tal permission to have a school physical exam done

You are reviewing a Pap smear report on a 25-year-old female. Which of the following cells should be on a Pap smear to be classified as a satisfactory specimen? A) Clue cells and endometrial cells B) Vaginal cells and cervical cells C) Squamous epithelial cells and endocervical cells D) Leukocytes and RBCs

C) Squamous epithelial cells and endocervical cells Squamous epithelial cells and endocervical cells must be obtained when performing a Pap smear to be considered satisfactory to evaluate the cells from the endocervix.

11-29. Which is most accurate about the progression of multiple sclerosis? A. All MS patients experience occasional remissions B. Most MS patients completely lose the ability to walk C. About one-half of MS patients eventually experience cognitive deficits D. MS patients' life expectancy is the same as the general population

C. About one-half of MS patients eventually experience cognitive deficits

11-5. A 72-year-old patient presents with unilateral facial drooping, slurred speech, and vision changes. The first step in management is: A. Aspirin B. Echocardiogram C. Referral to the emergency department D. CBC, CMP, and troponin levels

C. Referral to the emergency department

Which of the following medications is least likely to be associated with Stevens-Johnson syndrome? A. acetaminophen (Tylenol) B. allopurinol (Zyloprim) C. morphine (Roxanol) D. naproxen (Anaprox)

C. morphine (Roxanol)

16-43. Which of the following laboratory findings would be consistent with the diagnosis of urticaria? A. Hyphae on KOH prep B. Neutropenia C. Decreased erythrocyte sedimentation rate D. Eosinophilia

D. Eosinophilia

A patient has a well at his home. Which of the following is the likely cause of his gastroenteritis? A. Shigella B. Salmonella C. Rotavirus D. Giardia

D. Giardia

13-15. Signs and symptoms of Cushing syndrome can include: A. Bronzed skin B. Weight loss C. Orthostatic hypotension D. Round face

D. Round face

15-9. The nurse practitioner is caring for a 4-year-old who attends day care and has two 1.5 cm furuncles on the lower extremity. In order to provide coverage for methicillin-resistant Staphylococcal aureus (MRSA), which of the following should the nurse practitioner prescribe? A. ciprofloxacin (Cipro) B. doxycycline (Vibramycin) C. minocycline (Minosin) D. trimethoprim-sulfamethoxazole (Bactrim DS)

D. trimethoprim-sulfamethoxazole (Bactrim DS)

A high school teacher complains of cough X6 weeks. Worse when lying down. He has episodes of heartburn, which he treats with OTC TUMS. He chews mints for his "bad breath" which is most likely causing his cough? A. Asthma B. Gastroesophageal Reflux C. Pneumonia D. Chronic Post nasal drip

GERD: Common cause of chronic cough, worse when lying down because acid splashes up into upper esophagus

A small abscess on a hair follicle on the eyelid is called: A. Hordeolum B. Aortic Stenosis C. Pinguecula D. Ptosis

Hordeolum- painful acute bacterial infection of a hair follicle on the eyelid

All of the following are factors important in determining the peak flow rate except: A. Weight B. Height C. Age D. Gender

Weight

A woman is being evaluated by the nurse practitioner for complaints of dyspareunia. A microscopy slide reveals a large number of atrophic squamous epithelial cells. The vaginal pH is 4.0. There are very few leukocytes and no RBCs are seen on the wet smear. Which of the following is most likely? A) Atrophic vaginitis B) Bacterial vaginosis C) Trichomoniasis D) This is a normal finding

A) Atrophic vaginitis Symptoms of atrophic vaginitis include painful intercourse, atrophic squamous epithelial cells, and a decrease in pH. Vaginal atrophy is caused by lack of or imbalance of estrogen. Normal pH of the vagina is 4.0-5.0 (acidic).

The posterior fontanel should be completely closed by: A) 3 months B) 4 months C) 5 months D) 6 months

A) 3 months The posterior fontanelle normally closes by 3 months of age. The anterior fontanelle closes between 12 and 18 months of age.

The most current recommendation from the Joint National Commission on the Evaluation and Treatment of High Blood Pressure in Adults (JNC 8) for the blood pressure goal in diabetics is: A) <140/90 B) <130/85 C) <130/80 D) <125/75

A) < 140/90 JNC 8 bp goals include < 140/90 in patients with diabetes.

Grey-Turner's sign is highly suggestive of which of the following conditions? A) Acute pancreatitis B) Acute appendicitis C) Acute diverticulitis D) Gastric cancer

A) Acute pancreatitis Grey-Turner's sign is the acute onset of bluish discoloration located on the flank area that is caused by bruising. It is usually associated with severe acute pancreatitis, but it can also be found in some cases of ruptured ectopic pregnancy.

Cullen's sign is most commonly associated with which of the following? A) Acute pancreatitis B) Myocardial infarction C) Acute pyelonephritis D) Preeclampsia

A) Acute pancreatitis Cullen's sign is commonly seen in acute pancreatitis. It is a yellowish-blue skin color change around the umbilicus. It is thought to occur due to the pancreatic enzymes that run along the ligament and subcutaneous tis- sues around the umbilicus.

A newly diagnosed middle-aged type 2 diabetic wants to start an exercise program. All of the following statements are true except: A) If the patient is unable to eat due to illness, anti-diabetic agents can be continued with frequent glucose monitoring B) Strenuous exercise is contraindicated for type 2 diabetics because of a higher risk of hypoglycemic episodes C) Exercise increases the body's ability to metabolize glucose D) Patients who exercise vigorously in the afternoon may have hypoglycemic episodes in the evening or at night if they do not eat

B) Strenuous exercise is contraindicated for type 2 diabetics because of a higher risk of hypoglycemic episodes Exercise is recommended because exercise helps to use the glucose stores and reduce blood sugar. Blood sugar should be monitored closely, especially if on insulin, when exercising to avoid hypoglycemia.

A college freshman who is on oral contraceptives calls the nurse practitioner's office asking for advice. She forgot to take her pills 2 days in a row during the second week of the pill cycle and wants to know what to do. What is the best advice? A) Start a new pack of pills and dispose of the old one B) Take 2 pills today and 2 pills the next day; use condoms for the rest of the cycle C) Stop taking the pills right away and start a new pill cycle in 2 weeks D) Take 1 pill now and 2 pills the next day and use condoms

B) Take 2 pills today and 2 pills the next day; use condoms for the rest of the cycle When forgetting to take the birth control pill on 2 consecutive days, it is recommended that she take 2 pills today and 2 pills tomorrow, then continue the rest of her pack. Stress the importance of the use of condoms for protection against pregnancy and STIs.

At what Tanner stage does puberty start? A) Tanner Stage I B) Tanner Stage II C) Tanner Stage III D) Tanner Stage IV

B) Tanner Stage II Puberty is defined as the period in life when secondary sexual characteristics begin to develop, identified as Tanner Stage II for boys and girls.

When an adolescent male's penis grows more in length than width, at which of the following Tanner stages is he classified? A) Tanner Stage II B) Tanner Stage III C) Tanner Stage IV D) Tanner Stage V

B) Tanner Stage III Tanner Stage III in males consists of penis lengthening and darker, coarse pubic hair, which begins to curl

Which of the following statements is false regarding jaundice in breastfed infants? A) Breastfed infants have a higher incidence of hyperbilirubinemia compared with formula-fed infants B) The mechanism of breast milk jaundice is still not known C) Phytotherapy is usually not indicated for these infants D) It usually starts in the first week of life

B) The mechanism of breast milk jaundice is still not known Jaundice occurs after the first week of life for breastfed infants. Breastfed infants usually have a higher incidence of jaundice, and the condition may last for 2-3 weeks. Bilirubin is broken down and excreted in the urine and stool. The mechanism of breast milk jaundice is still unknown.

All of the following are true statements regarding Munchausen syndrome except: A) It is considered a mental illness B) The patient has a medical illness that causes an anxiety reaction and denial C) The patient fakes an illness in order to gain attention from health care providers D) The patient has an inconsistent medical history along with a past history of frequent hospitalizations

B) The patient has a medical illness that causes an anxiety reaction and denial Munchausen syndrome is a psychiatric disorder in which the patient fakes a medical illness or disorder to gain attention from health care provid- ers. These patients commonly use the emergency department frequently to gain attention.

The bacille Calmette-Guerin (BCG) vaccine is used to immunize a person against which of the following? A) Enterobiasis B) Tuberculosis C) Anthrax D) Smallpox

B) Tuberculosis The BCG vaccine is given routinely in some countries where tuberculosis is endemic (or epidemic). One of the few exceptions for the BCG vaccine in the United States is for health care workers who see a high percentage of patients who are infected with M. tuberculosis strains resistant to both isoniazid and rifampin. BCG is considered a biohazardous material (U.S. Black Box Warning) and proper handling and disposal must be followed.

Which of the following is a good example of how the "utilitarian" principle is applied? A) Helping a patient decide the type of treatment that he/she wants B) Using limited societal financial resources on programs that will positively affect the largest number of people possible and have the lowest possible negative outcomes C) Minimize the bad outcome when choosing treatment choices for a patient D) Health caregivers should be more careful when using health care financial resources

B) Using limited societal financial resources on programs that will positively affect the largest number of people possible and have the lowest possible negative outcomes Generally, the utilitarian principle refers to societal programs that will affect or benefit the largest number of people in a positive manner. It is not used to refer to an individual or to one person.

Women who are pregnant during the winter months are recommended to have which of the following? A) Increased intake of Vitamin C and folate B) Vaccination against the influenza virus C) Increased caloric intake of fruits and vegetables D) Heavier winter clothes to avoid chilling the fetus

B) Vaccination against the influenza virus Influenza vaccine is recommended for all pregnant patients for prevention due to the decrease in immune status during pregnancy. The vaccine is safe to use during pregnancy.

What type of breath sounds are best heard over the base of the lungs? A) Fine breath sounds B) Vesicular breath sounds C) Bronchial sounds D) Tracheal breath sounds

B) Vesicular breath sounds Vesicular breath sounds are heard best over the base of the lungs. Vesicular sounds are soft and/or blowing, heard throughout inspiration, and fade away with expiration. Bronchial sounds are heard over the bronchi, the largest tubes in the anterior chest. Sounds are loud and high pitched. Tracheal breath sounds are heard over the trachea. These sound harsh and similar to air being blown through a pipe.

Which of the following is the confirmatory test for the HIV screening test? A) ELISA test for HIV B) Western blot C) HIV polymerase chain reaction test D) HIV antibody

B) Western blot A positive Western blot test confirms an HIV screening test.

A 35-year-old woman is complaining of gradual weight gain, lack of energy, and amenorrhea. The urine pregnancy test is negative. The CBC shows a hemoglobin of 13.5 g and MCV 84. The nurse practitioner suspects that the patient may have hypothyroidism. The TSH is 10 mU/L. Which of the following is the next step in the evaluation? A) Check the thyroid profile B) Check the total T3 and T4 levels C) Check for antithyroid peroxidase antibodies D) Recheck the TSH in 4 to 6 months

A) Check the thyroid profile The upper limit of the serum TSH level is about 5.0 mU/L (range of 0.5 to 5.0). With an elevated TSH of 10, it is important to rule out hypothyroidism. The next step in this patient's evaluation is to order a thyroid profile test. Serum assays measure bound and unbound (free) forms of thyroxine (T4) and triiodothyronine (T3). Classic findings of hypothyroidism are a low total T4, low T3-resin uptake (THBI), and low free T4 index.

Some pharmacologic agents may cause confusion in the elderly. Which of the following pharmacologic agents is most likely to cause confusion in this population? A) Cimetidine (Tagamet), digoxin (Lanoxin), diphenhydramine (Benadryl) B) Acetaminophen (Tylenol), aspirin (Bayer), indomethacin (Indocin) C) Sucralfate (Carafate), docusate sodium (Surfak), psyllium (Metamucil) D) Cephalexin (Keflex), amoxicillin (Amoxil), clarithromycin (Biaxin)

A) Cimetidine (Tagamet), digoxin (Lanoxin), diphenhydramine (Benadryl) Medications that commonly cause confusion in the elderly include cimetidine, lanoxin, and diphenhydramine.

A 35-year-old smoker is being evaluated for birth control choices. The patient has a history of PID along with an embolic episode after her last pregnancy. Which of the following methods of birth control would you recommend? A) Condoms and the vaginal sponge (Today Sponge) B) Estrogen patches C) Intrauterine device D) Depo-Provera (depot medroxyprogesterone)

A) Condoms and the vaginal sponge (Today Sponge) Contraindications for hormonal contraception include: Migraine headaches; cigarette smoking or obesity in women older than 35 years; history of thromboembolic disease; hypertension or vascular disease if over 35 years of age; systemic lupus erythematosus with vascular disease, nephritis, or antiphospholipid antibodies; breastfeeding (may use progestin-only pills); hypertriglyceridemia; CAD; CHF; and strokes.

Which chronic illness disproportionately affects the Hispanic population? A) Diabetes mellitus B) Hypertension C) Alcohol abuse D) Skin cancer

A) Diabetes mellitus Diabetes mellitusis 2-3 times higher in Mexican Americans versus non-Hispanic Americans.

Mr. Jones, who has been on pravastatin (Pravachol) 20 mg at bedtime for the past few months, complains of lately feeling extremely fatigued. The patient also noticed that his urine has been a darker color during the past 2 weeks. Which of the following is the best treatment plan to follow? A) Discontinue his pravastatin and order a liver function profile B) Continue the pravastatin but on half the dose C) Schedule him for a complete physical exam D) Schedule him for a liver function profile

A) Discontinue his pravastatin and order a liver function profile Side effects of statin drugs include myalgia, fatigue, and elevated liver enzymes. Routine labs should include liver enzymes to assess the effects on the liver.

A 30-year-old male patient refuses to take his afternoon dose of pills. The nurse tells him of the possible consequences of his action, but the patient still refuses to cooperate. Which of the following is the best course for the nurse to follow? A) Document in the patient's record his behaviors and the action taken by the nurse B) Reassure the patient that he will be fine after taking the medicine C) Document only the patient behavior D) Document only the nurse's action

A) Document in the patient's record his behaviors and the action taken by the nurse If a patient is non-compliant, documentation of the patient behavior and the actions taken by the nurse both must be documented.

Which of the following is recommended treatment for erythema migrans or early Lyme disease? A) Doxycycline (Vibramycin) 100 mg PO BID x 21 days B) Ciprofloxacin (Cipro) 250 mg PO BID x 14 days C) Erythromycin (E-mycin) 333 mg PO TID x 10 days D) Dicloxacillin 500 mg PO BID x 10 days

A) Doxycycline (Vibramycin) 100 mg PO BID x 21 days The CDC-recom- mended treatment for erythema migrans or Lyme disease is doxycycline 100 mg PO BID x 21 days.

A 25-year-old male with schizophrenia comes in for a routine annual physical. He is a heavy smoker and has a BMI of 28. The patient has been on olanzapine (Zyprexa) for 10 years. Regarding the patient's prescription, which of the following laboratory tests is recommended for monitoring for adverse effects of atypical antipsychotics? A) Fasting blood glucose, fasting lipid profile, and weight B) Urinalysis, serum creatinine, 24-hour urine for protein and creatinine clearance C) Liver function tests only D) CBC with differential, liver function tests, weight

A) Fasting blood glucose, fasting lipid profile, and weight Patients on atypical antipsychotics commonly gain weight and are at risk for obesity, hyperglycemia, and type 2 diabetes. Zyprexa will increase lipids (cholesterol, LDL, and triglycerides). Atypical antipsychotics also increase the risk of death among frail elders and older adults living in nursing homes

The majority of serum alpha fetoprotein is produced by the: A) Fetal liver B) Mother's liver C) Placenta D) Fetal neural tube

A) Fetal liver Serum alpha fetoprotein is produced by the fetal liver.

In the majority of children, the first permanent teeth start to erupt at the age of 6 years. Which of the following are the first permanent teeth to erupt in this time period? A) First molars B) Second molars C) Lower or upper incisors D) Canines

A) First molars The first molars are the first permanent teeth to develop; they appear at approximately 6 years of age

When initially treating adults for bronchitis, which would the NP be least likely to order? A. Expectorants B. Antibiotics C. Bronchodilators D. Antitussives

B. Antibiotics

The nurse practitioner is caring for a 28-year-old woman with myoclonic seizures that are well controlled on valproic acid. The woman states that she would like to become pregnant in the next year. What should the NP consider regarding the woman's anticonvulsant medication? A. Continue the present therapy B. Consider changing to lamotrigine C. Decrease the valproic acid dose D. Add a second anticonvulsant

B. Consider changing to lamotrigine

On physical exam, a 53-year-old Caucasian female patient presents with an anterior perianal fistula, mouth ulcers, weight loss, fever, and crampy abdominal pain. The nurse practitioner will do more testing, but she suspects the patient has: A. Ulcerative colitis B. Crohn's disease C. Diverticulitis D. Irritable bowel syndrome

B. Crohn's disease

What is the mechanism of action of sulfonylureas used for type 2 diabetes? A. Decrease hepatic production of glucose B. Enhance insulin secretion by binding to pancreatic beta cell receptor sites C. Mimic endogenous insulin by binding to cell wall receptors D. Stimulate insulin secretion in a short-acting manner

B. Enhance insulin secretion by binding to pancreatic beta cell receptor sites

What is the best procedure for evaluating corneal abrasion? A. Tonometry B. Fluorescein stain C. Visual Field test D. Funduscopy

B. Fluorescein stain

A patient reports to the nurse practitioner perianal discomfort and pain. The patient has a history of constipation. Which of the following traits has an increased risk for perianal fissures? A. Patients aged 45-65 B. History of trauma and childbirth C. Intake of low-fiber diet and adequate hydration D. Presence of both internal and external hemorrhoids

B. History of trauma and childbirth

What is a key diagnostic finding in open-angle glaucoma? A. Papilledema B. Increased intraocular pressure C. Eye pain D. Sluggish pupils

B. Increased intraocular pressure

11-48. The nurse practitioner knows that febrile seizures are most highly associated with which age group? A. Newborns B. Infants C. Young adults D. Older adults

B. Infants

All of the following statements reflect inadequate breast milk production except: A) Full-term infant is at birth weight by the second week of life B) Less than 6 wet diapers per day or less than 4 stools per day C) Infant is nursing fewer than 8 times per 24-hour period D) Weight loss of > 10% of birth weight

A) Full-term infant is at birth weight by the second week of life The full- term infant should be back to birth weight at 2 weeks of age. The infant should be nursing every 2-4 hours and should wet 6-10 diapers a day (24 h).

Females with polycystic ovarian syndrome are at higher risk for: A) Heart disease and breast cancer B) Uterine fibroids and ovarian cancer C) Premature menopause D) PID (pelvic inflammatory disease)

A) Heart disease and breast cancer Females with PCOS are at higher risk of developing heart disease, stroke, and breast cancer.

19-28. Which is not a provision of the Affordable Care Act? A. It monitors for fraud and healthcare waste B. It ties health insurance coverage to employment with certain employers C. It allows for a penalty to be levied against persons who choose to be uninsured D. It prevents healthcare companies from dropping patients with pre-existing conditions or raising their premiums

B. It ties health insurance coverage to employment with certain employers

Which diagnostic test is most appropriate for confirming the diagnosis of oral candidiasis? A. Bacterial culture and sensitivity B. KOH prep C. Positive pathergy test D. Tzanck smear

B. KOH prep

Which of the following is considered an abnormal result on a Weber test? A) Lateralization to one ear B) No lateralization in either ear C) Air conduction lasts longer than bone conduction D) Bone conduction lasts longer than air conduction

A) Lateralization to one ear The Weber test is performed by placing a tuning fork on top of the head. A normal Weber test is when no lateralization occurs. Lateralization to one ear occurs with hearing impairment.

The complications of untreated gout include: A) Loss of joint mobility and renal failure B) Loss of joint mobility and liver failure C) An increased risk of urinary tract infections D) Bladder cancer

A) Loss of joint mobility and renal failure Complications of untreated gout are loss of joint mobility and renal failure. High uric acid levels can also lead to kidney stones.

The most common cause of cancer deaths in males is: A) Lung cancer B) Prostate cancer C) Colon cancer D) Skin cance

A) Lung cancer Lung cancer is the most common cause of cancer deaths in men. Prostate cancer and colon cancer are the second and third causes of cancer death in men.

A 19-year-old male athlete complains of acute knee pain after a football game. The nurse practitioner elicits McMurray's sign, which is positive on the patient's injured knee. This is a test for: A) Meniscal injury B) Inflammation of the knee joint C) Osteophytes of the knee joint D) Tenosynovitis

A) Meniscal injury With an acute knee injury, the knee should be assessed using McMurray's sign. A positive McMurray's sign indicates a meniscal injury. Inflammation of the knee, osteophytes, and tenosynovitis would not elicit a positive McMurray's sign.

Epidemiologic studies show that Hashimoto's disease occurs most commonly in: A) Middle-aged to older women B) Smokers C) Obese individuals D) Older men

A) Middle-aged to older women Hashimoto's disease commonly occurs in middle-aged to older women.

Stella, a new mother, complains to you that she has been feeling irritable and jittery almost daily for the past few months. She complains of frequent palpitations and more frequent bowel movements along with weight loss. Her BP is 160/70, her pulse is 110, and she is afebrile. All of the following conditions should be considered in the differential diagnosis for this patient except: A) Mitral regurgitation B) Graves' disease C) Generalized anxiety disorder D) Illicit drug use

A) Mitral regurgitation Signs and symptoms of mitral regurgitation do not include frequent bowel movements with weight loss.

A nurse practitioner is taking part in a community outreach program for a local hospital. Most of her audience has a diagnosis of hypertension. They are all interested in learning more about a proper diet. When discussing potential sources of potassium and magnesium, which of the following is the best advice? A) Most fruits and vegetables B) Whole grains and sausages C) Processed corned beef and yogurt D) Mushrooms and sauerkraut

A) Most fruits and vegetables Fruits and vegetables are higher in potassium and magnesium.

Orchitis is caused by which of the following? A) Mumps virus B) Measles virus C) Chlamydia trachomatis D) Chronic urinary tract infections that are not treated adequately

A) Mumps virus Orchitis is caused by the mumps virus.

A test called the visual fields by confrontation is used to evaluate for: A) Peripheral vision B) Central distance vision C) Narrow-angle glaucoma D) Accommodation

A) Peripheral vision The visual fields of confrontation test is used to evaluate peripheral vision. The Snellen chart is used to measure central distance vision. A tonometer is used to assess for glaucoma. The ophthalmoscope is used to assess for cataracts.

Which of the following is the best course of treatment for this patient? A) Refer him to an orthopedic specialist B) Refer him to a chiropractor C) Advise him that the clicking noise will resolve within 2 to 4 weeks D) AdvisehimtouseanAcebandagewrapduringthefirst2weeksforkneesup- port and to see you again for reevaluation

A) Refer him to an orthopedic specialist Referral to orthopedics is advised for evaluation of the need for treatment and surgery. Delaying referral and treatment can be detrimental if the diagnosis is not correct.

Which of the following laboratory values may be found elevated alone on the liver function panel of patients who are alcohol abusers? A) Serum GGT (gamma glutamyl transaminase) B) Serum ALT (alanine aminotransferase) C) Serum bilirubin D) Blood urea nitrogen

A) Serum GGT (gamma glutamyl transaminase) The serum gamma glutamyl transaminase alone will be elevated with alcohol abusers.

All of the following are factors important in determining the peak expiratory flow volume except: A) Weight B) Height C) Age D) Gender

A) Weight Peak expiratory flow volume is determined by using height, gender, and age.

Erysipelas is an infection of the skin most commonly caused by which of the following class of organisms? A) Streptococci B) Staphylococi C) Gram-negative bacteria D) Fungi

A)Streptococci Erysipelas is a skin infection commonly caused by group A beta- hemolytic streptococci. This infection is usually more superficial than other bacterial infections of the skin, such as cellulitis.

12-32. Which of the following patients may be most at risk for a completed suicide? A. A 75-year-old recent widower B. A 19-year-old female college student C. A married mother with financial concerns D. A Roman Catholic 22-year-old male

A. A 75-year-old recent widower

A previously healthy 68-year-old patient presents with a 4-day history of productive cough, temperature of 101.5 F, BP 115/80, pulse 90, and respiratory rate of 25. Physical examination shows the patient to be alert and oriented, with diminished breath sounds in the RLL. BUN is 16 and WBC is elevated. On CXR, the RLL demonstrates infiltrates. Which of the following is true? A. According to CURB-65, this patient has a score of 1 and can be treated as an outpatient B. According to CURB-65, this patient has a score of 2 and can be treated as an outpatient C. According to CURB-65, this patient has a score of 4 and can be treated as an outpatient with a combination of antibiotics rather than a single-agent regimen D. According to CURB-65, this patient has a score of 4 and hospitalization is needed

A. According to CURB-65, this patient has a score of 1 and can be treated as an outpatient

Which of the following would the NP use to determine hearing loss? A. Audiogram B. Tympanogram C. Pneumatic otoscopy D. Computed tomography

A. Audiogram

A 13 y.o. pt has a throat culture that is positive for strep throat. She reports that her younger brother was recently diagnosed with strep and was treated. The pt has a severe allergy to penicillin and reports erythromycin makes her very nauseated. Which of the following ABX is the best choice? A. Azythromycin B Cephalexin (keflex) C. Cefuroxime axetil (Ceftin) D. Levofloxacin (Levaquin)

A. Azythromycin: Don't use Levaquin in kids, 10% cross reactivity between penicillin and Keflex, Erythromycin commonly has nausea as side effect but not allergy, azythromycin better tolerated.

A 68-year-old male with a history of HTN and CAD presents to the office with progressive SOB and chest tightness. The NP notices a heart rate of 102 and a blood pressure of 95/78. Which diagnostic test will help the NP differentiate between a diagnosis of pneumonia vs. CHF? A. BNP В. СВС С. СМР D. ABG

A. BNP

The diagnosis of preeclampsia can be made with which of the following findings? A. BP 150/92 and UPCR 0.4 B. BP 140/90 and +2 protein on dipstick C. BP 142/96 and +3 bilateral pitting edema D. BP 150/78 and trace urine protein on dipstick

A. BP 150/92 and UPCR 0.4

A patient who suffered a recent MI presents two weeks after undergoing revascularization via PTCA. He was prescribed metoprolol 25 mg PO BID but has developed dry mouth and erectile dysfunction. Which medication should be substituted due to betablocker intolerance? A. Calcium channel blocker B. ARB C. Anti-arrhythmic D. ACE inhibitor

A. Calcium channel blocker

On a routine visit, a 62-year-old female with type 1 DM presents to the office reporting chest pain unrelieved by rest, diaphoresis, and tachycardia. The EKG reveals changes in leads II, III, and aVF. What should the nurse practitioner do next? A. Call 911 immediately B. Repeat the ECG once the heart rate comes dowm C. Order an exercise stress test D. Order an anticoagulant

A. Call 911 immediately

Appropriate next steps for a patient newly diagnosed with sinus bradycardia who is stable and without symptoms would include: A. Cardiology referral B. Preparing for immediate transport C. Giving atropine IV D. Tilt table test

A. Cardiology referral

Which of the following is likely to be reported by a patient experiencing chest pain related to pericarditis? A. Chest pain relieved by sitting forward B. Chest pain relieved by lying down C. Chest pain relieved by deep inspiration D. Chest pain relieved by rest

A. Chest pain relieved by sitting forward

Which of the following is most consistent with a patient who may be experiencing an MI? A. Chest pressure unrelieved by 3 doses of sublingual nitroglycerin B. Burning chest pain relieved by position change C. Chest tightness relieved by rest D. Aching chest pain relieved by ibuprofen

A. Chest pressure unrelieved by 3 doses of sublin- gual nitroglycerin

21-1. Sensitivity is the ability of a test to: A. Detect true positives B. Detect true negatives C. Rule in disease D. Differentiate between two conditions

A. Detect true positives

3-4. Improvement in sleep health is an objective of Healthy People 2020 for what reason? A. Inadequate sleep has implications for driving safety, work productivity, and quality of life B. 75% of adult population in the U.S. reports some sort of sleep disturbance C. Sleep apnea can be successfully treated with zolpidem D. Sleep apnea significantly increases the risk of diabetes and glaucoma

A. Inadequate sleep has implications for driving safety, work productivity, and quality of life

A patient that is newly diagnosed with hypertension returns for their first follow-up after being on Lisinopril 10 mg daily for 1 month. If blood pressure is 152/94, what would be the next step? A. Increase the dosage B. Change to a different class C. Stop the medication and focus on lifestyle changes D. Workup for secondary hypertension

A. Increase the dosage

Which of the following clinical symptoms is most commonly seen with the initial presentation of colorectal cancer? A. Iron deficiency anemia B. Nausea C. Flatulence D. Indigestion

A. Iron deficiency anemia

3-1. All of the following are true of Healthy People 2020 except: A. It is funded by the pharmaceutical industry B. It is a national agenda that communicates a vision for improving health and achieving health equity of the U.S. population C. It provides a set of more than 1,200 specific, measurable objectives with targets to be achieved over the decade D. Initiatives are organized by general health and wellness topics, age, and special needs populations such as maternal health or vulnerable population groups

A. It is funded by the pharmaceutical industry

Bone loss from irritable bowel disease generally results from: A. Malabsorption from the inflamed GI tract B. Increased osteoblast activity related to insulin resistance C. Chronic antibiotic use D. NSAID use

A. Malabsorption from the inflamed GI tract

A patient presents with acute onset of dyspnea, pleuritic chest pain, and hypoxia. No clear cause can be determined by clinical examination. General chemistries are normal and a chest x-ray is negative. The FNP suspects a pulmonary embolism and will order which of the following diagnostic tests to confirm the diagnosis? A. Meltidetector-row computed tomography angiography B. Coagulation studies C. V/Q scan D. D-dimer

A. Meltidetector-row computed tomography angiography

A comorbidity frequently associated with irritable bowel syndrome is: A. Migraine headaches B. Diabetes mellitus C. Obstructive sleep apnea D. Cirrhosis

A. Migraine headaches

A patient with a history of Stage-C HF presents to the office for a routine follow-up. The patient is currently being managed on lisinopril (Zestril), digitalis (Lanoxin), and furosemide (Lasix). What should be included as part of the plan? A. Monitor potassium B. Discontinue digitalis (Lanoxin) C. Assess liver function D. ECG monthly

A. Monitor potassium

12-6. Which of the following is commonly reported by individuals with an anxiety disorder? A. Muscle tension B. Hypersomnolence C. Skin rashes such as hives D. Constipation

A. Muscle tension

13-32. Which of the following is the most sensitive indicator of overall thyroid function? A. TSH B. Free T3 C. Thyroxine D. Free T4

A. TSH

4-9. Edith L. is a 36-year-old pregnant woman who presents at the clinic with symptoms of fever, chills, and body aches. Symptoms started last night, and she tests positive for influenza A. Which of the following would you prescribe? A. oseltamivir (Tamiflu) B. zanamivir (Relenza) C. peramivir (Rapivab) D. amantadine (Symmetrel)

A. oseltamivir (Tamiflu)

Which of the following is considered an objective finding in patients who have a case of suppurative otitis media? A) Erythema of the tympanic membrane B) Decreased mobility of the tympanic membrane as measured by tympanogram C) Displacement of the light reflex D) Bulging of the tympanic membrane

B) Decreased mobility of the tympanic membrane as measured by tympanogram Fluid behind the ear drum will decrease the mobility of the TM when measured by a tympanogram. When perforation occurs, discharge will flow through the pars tensa portion of the ear drum.

Which of the following is indicated for initial treatment of an uncomplicated case of Helicobacter pylori negative peptic ulcer disease? A) Omeprazole (Prilosec) B) Misoprostol (Cytotec) C) Ranitidine (Zantac) D) Pepto-Bismol tablets

B) Misoprostol (Cytotec) Misoprostol is recommended for short-term, uncomplicated PUD; it acts by decreasing gastric acid production and enhancing mucosal resistance to injury.

Hegar's sign is considered a: A) Positive sign of pregnancy B) Probable sign of pregnancy C) Presumptive sign of pregnancy D) Problem in pregnancy

B) Probable sign of pregnancy Hegar's sign is softening of the lower portion of the uterus and is considered a probable sign of pregnancy.

The mother of a 16-year-old boy is concerned that her son is not developing normally. On physical exam, the patient is noted to have small testes with no pubic or facial hair. What is the most appropriate statement to the mother? A) Her son is developing normally B) Her son's physical development is delayed and should be evaluated by a pediatric endocrinologist C) Her son should be rechecked in 3 months; if he still does not have secondary sexual characteristics, a thorough hormonal workup should be initiated D) Her son's physiological development is slower than normal but is within the lower limit of normal for his age group

B) Her son's physical development is delayed and should be evaluated by a pediatric endocrinologist A 16-year-old male without secondary sexual characteristics should be referred to an endocrinologist. If there is no testicular development by 14 years of age, referral to an endocrinologist is recommended.

The mother of a 12-month-old infant reports to the nurse practitioner that her child had a high fever for several days, which spontaneously resolved. After the fever resolved, the child developed a maculopapular rash. Which of the following is the most likely diagnosis? A) Fifth disease (erythema infectiosum) B) Roseola infantum (exanthema subitum) C) Varicella D) Infantile maculopapular rashes

B) Roseola infantum (exanthema subitum) Roseola infantum is a common viral rash that is caused by the human herpes virus; the most common ages of onset are between 6 months and 2 years. The rashes are maculopapular (small round pinkcolored) rashes that first appear on the trunk and then spread to the extremities.

A 25-year-old female presents with an onset of severe right-sided pelvic pain for the past 48 hours. She reports small amounts of vaginal bleeding. The pain is aggravated by jumping or any movement that jars her pelvis. The best initial intervention is which of the following? A) Follicle-stimulating hormone (FSH) B) Serum quantitative pregnancy test C) Pelvic ultrasound D) CBC with white cell differentials

B) Serum quantitative pregnancy test Severe right-sided pain with vaginal bleeding in a 25-year-old patient requires an evaluation for pregnancy, by performing a serum quantitative pregnancy test. Ectopic pregnancy and/or miscarriage should be ruled out.

The cover/uncover test is a screening tool for: A) Color blindness B) Strabismus C) Visual acuity D) Cataracts

B) Strabismus The cover/uncover test screens for strabismus. Color blindness is evaluated by using the Ishihara tool. Visual acuity is evaluated using the Snellen chart. Cataracts are screened by using the ophthalmoscope and shining a light into the eyes at an angle, approximately 15 inches away. If the red reflex is not elicited, then cataracts are present

19-16. Which of the following is not an example of an ethical dilemma? A. Nursing staff is restricted from using a cell phone in patient areas, but the nurse practitioner wants a cell phone handy to return calls to patients B. The nurse practitioner has prescribed an asthma inhaler for her patent, and the patient has decided to purchase the generic version instead of the brand name because it is cheaper C. The nurse practitioner wants to provide patient- centered care but works in a facility where general safety takes precedence over patient- centered care D. The nurse practitioner has noticed that colleagues are taking photos to post on social media without patient permission

B. The nurse practitioner has prescribed an asthma inhaler for her patent, and the patient has decided to purchase the generic version instead of the brand name because it is cheaper

11-50. Mr. R. is a 63-year-old patient with a history of hypertension, hyperlipidemia, and lung cancer. He has had a new onset of fatigue, nausea/vomiting, and memory loss for the last two weeks. Which statement is true regarding the patient's symptoms? A. The symptoms would be expected based on the patient's age B. The symptoms could be related to metastases of lung cancer to the brain C. The symptoms could be related to a transient ischemia attack (TIA) D. The symptoms could be related to worsening of hypertension

B. The symptoms could be related to metastases of lung cancer to the brain

14-21. Which of the following is not a sign or symptom of thrombocytopenia? A. Jaundice B. Tinnitus C. Hematuria D. Excessive menstrual flow

B. Tinnitus

The nurse practitioner suspects mitral valve prolapse in a 62-year-old female patient complaining of shortness of breath and dizziness. Which test would the nurse practitioner order initially to confirm this diagnosis? A. Cardiac stress test B. Transthoracic echocardiogram C. Blood culture D. Cardiac catheterization

B. Transthoracic echocardiogram

16-21. Which of the following is NOT recommended for the treatment of atopic dermatitis? A. Repeated moisturizing with lotion to face and cream to body B. Twice daily antihistamine cream to affected areas C. Fragrance-free detergent, fabric softener, and dryer sheets D. Unscented, sensitive-skin Dove soap or Cetaphil cleanser

B. Twice daily antihistamine cream to affected areas

18-4. The nurse practitioner knows that the musculoskeletal changes associated with normal physiological aging do not lead to: A. Altered posture/gait B. Vestibular dysfunction C. Loss of stature D. Increased risk for fall

B. Vestibular dysfunction

A patient has had repeat attacks of gout over a one-to two-year period. Which medication should the nurse practitioner choose for long-term treatment for this patient? A. ibuprofen (Motrin) B. allopurinol (Zyloprim) C. hydrocortisone (Cortef) D. colchicine (Colcrys)

B. allopurinol (Zyloprim)

The nurse practitioner is prescribing medication for a patient with gastroesophageal reflux disease (GERD). Which of the following medication classes would be an inappropriate choice? A. Antacids B. Antispasmodics C. H2 blockers D. Proton pump inhibitors

B. antispasmodics

Which of the following medications may cause hypohidrosis? A. lactulose (Cephulac) B. atropine/diphenoxylate (Lomotil) C. arlex (Sorbitol) D. cimetidine (Tagamet)

B. atropine/diphenoxylate (Lomotil)

Type 2 auto-immune hepatitis (AIH) usually strikes __________. A. the elderly B. children and young womenm C. young adults, typically male D. middle-aged adults

B. children and young womenm

All of the following pharmacologic treatments have been approved for long-term use in the setting of constipation except: A. lubiprostone B. docusate C. polyethylene glycol D. psyllium

B. docusate

A 68-year-old female has been diagnosed with a pulmonary embolism. The FNP places the patient on anticoagulant therapy using which of the following preferred medications? A. alteplase (Activase) B. fondaparinux (Arixtra) C. streptokinase (Streptase) D. heparin sodium (Heparin)

B. fondaparinux (Arixtra)

The nurse practitioner is seeing a patient for acute exacerbation of asthma. The patient reports increasing shortness of breath and wheezing the last two days. He ran out of his albuterol inhaler and has been using his grandmother's inhaler (formoterol) with no improvement. What is the best rationale for the ineffectiveness of the formoterol? A. His grandmother nay have used all of the medicine in her inhaler B. formoterol is a long-acting beta agonist and is not effective for acute symptoms C. formoterol is an inhaled corticosteroid and is not effective for acute symptoms D. He is not using the inhaler correctly

B. formoterol is a long-acting beta agonist and is not effective for acute symptoms

16-4. A few months after entering a nursing home, the 75-year-old patient has developed a painful vesicular rash in a dermatomal distribution on her right chest and an additional forehead rash. The nurse practitioner suspects __________and will seek a referral to a __________. A. herpes simplex; dermatopathologist B. herpes zoster; ophthalmologist C. shingles; infectious disease specialist D. crusted scabies; wound specialist

B. herpes zoster; ophthalmologist

13-14. Medication management options for Cushing syndrome may include: A. rifampin (Rifadin) B. ketoconazole (Nizoral) C. prednisone (Sterapred) D. hydroxychloroquine (Plaquenil)

B. ketoconazole (Nizoral)

Kayla complains of pain on her tongue and a white coating to her tongue. She is diagnosed with oral candidiasis. The nurse practitioner knows this diagnosis is a complication of which medication? A. metoprolol tartrate (Lopressor) B. mometasone (Asmanex) C. lamotrigine (Lamictal) D. colchicine (Colcrys)

B. mometasone (Asmanex)

Which medication should be discontinued if eye pain or irritation occurs? A. cyclosporin B. ocular lubricants C. brimonidine D. gentamicin

B. ocular lubricants

All of the following are appropriate treatments in a patient diagnosed with an URI except: A. oxymetazoline 0.05% nasal (Afrin) B. theophylline 300 mg (Theo-Dur) C. loratadine 10 mg (Claritin) D. dextromethorphan 10 mg (Robitussin)

B. theophylline 300 mg (Theo-Dur)

Constipation is defined as hard, lumpy stool that occurs fewer than __________ times per week. A. two B. three C. four D.five

B. three

4-17. While performing patient education about drug-food interactions, the nurse practitioner cautions a patient to avoid grapefruit juice because he takes which of the following medications? A. lisinopril (Zestril) B. verapamil (Calan) C. warfarin (Coumadin) D. timolol (Timoptic)

B. verapamil (Calan)

Which of the following viral infections is associated with occasional abnormal forms of lymphocytes during an acute infection? A. Cytomegalovirus (CMV) B. Epstein-Barr virus (EBV) C. HPV D. Coxsackie Virus

B: Epstein Barr Virus (EBV)

An 8 y.o. boy is seen by the NP and the mother reports he has had a fever X2 days, not eating well due to painful sores inside pt's mouth. Temp is 101F, Pulse is 88 BPM, and respirations are 14. Np notices several small blisters and shallow ulcers on the pt's pharynx and oral mucosa. He also has small red rashes on both palms and soles. Which of the following is the most likely? A. Herpes simplex infection B. Hand Foot and mouth disease C. Varicella infection D. Secondary syphilis infection

B: Hand foot and mouth disease: spread by fecal/ oral route and direct contact with toys etc., caused by coxsacksie virus, treatment is supportive.

All of the following are associated with emphysema except: A) A barrel-shaped chest B) Pursed-lip breathing C) A chest radiograph result with infiltrates and flattening of the costovertebral angle D) Dyspnea when at rest

C) A chest radiograph result with infiltrates and flattening of the costovertebral angle Emphysema is characterized by having a barrel-shaped chest, pursed- lip breathing, and dyspnea when at rest. Infiltrates on an x-ray indicate bacterial infection, such as pneumonia

The following children are considered at higher risk for tuberculosis (TB) except: A) A child who has recently been diagnosed with leukemia B) An infant whose family is homeless C) A child who was born in Japan D) A Hispanic child with asthma who is using a steroid inhaler

C) A child who was born in Japan Risk factors for tuberculosis include compromised immune system, high-risk populations (minorities, foreign-born people, prisoners, nursing home residents, indigents, migrant workers and health care providers). Steroid therapy and blood disorders also increase the risk of TB.

Pulsus paradoxus is best described as: A) An increase in systolic blood pressure on inspiration B) A decrease in diastolic blood pressure on exhalation C) A decrease in systolic blood pressure on inspiration D) An increase in diastolic blood pressure on expiration

C) A decrease in systolic blood pressure on inspiration Pulsus paradoxus: With inspiration, systolic pressure drops due to the increased pressure (positive pres- sure). Some pulmonary risks of having increased pressure include asthma and emphysema

The following conditions are absolute contraindications for the use of oral contraceptives except: A) Hepatomas B) History of emboli that resolved with heparin therapy 15 years ago C) A family history of migraines with aura D) A history of gallbladder disease during pregnancy

C) A family history of migraines with aura All of the items are contraindications, but a family history does not substantiate a need to avoid oral contraceptives.

What is the pedigree symbol for a diseased male? A) An empty square B) An empty circle C) A filled-in square D) A filled-in circle

C) A filled-in square A filled-in square is a diseased or affected male and a filled-in circle is a diseased or affected female. An empty square is a healthy male and an empty circle is a healthy female.

Which of the following should you expect to find on a wet-mount slide of a patient diagnosed with bacterial vaginosis? A) Tzanck cells B) A large amount of leukocytes and epithelial cells C) A large amount of bacteria coating the squamous cells and very few leukocytes D) Epithelial cells and a small amount of blood 108

C) A large amount of bacteria coating the squamous cells and very few leukocytes Results of the wet mount for bacterial vaginosis include bacteria coating the squamous cells (clue cells) and very few leukocytes or RBCs

The following findings are considered benign lesions of the skin except: A) Lentigo B) Seborrheic keratosis C) Actinic keratosis D) Rosacea

C) Actinic keratosis Actinic keratosis is a small, raised skin lesion on skin that has been in the sun for a long period of time. Actinic keratoses are usually benign, but can develop into skin cancer; therefore, further evaluation is needed to deter- mine if removal is required.

Which of the following antihypertensive medications should the nurse practitioner avoid when treating patients with emphysema? A) Calcium channel blockers B) Angiotensin-converting enzyme (ACE) inhibitors C) Beta-blockers D) Diuretics

C) Beta-blockers Beta-blockers should be avoided in patients with a history of emphysema. Studies have shown evidence of a reduction in forced expiratory volume in 1 second (FEV1), increased airway hyperresponsiveness, and inhibition of bronchodilator response to beta agonists in patients receiving non-selective beta- blockers and high doses of cardioselective beta-blockers.

A 55-year-old nurse brings her mother, who is 82 years of age, to the emergency department of a local hospital. She reports she found her mother on the floor when she checked on her that morning. Her mother was awake and oriented, but needed help getting up. Her mother states that she thinks she passed out. She is being evaluated by a physician who orders an EKG and x-rays of both hips. Regarding laboratory testing, which of the following is important to perform initially? A) Urinalysis B) Serum electrolytes C) Blood glucose D) Hemoglobin and hematocrit

C) Blood glucose Checking the blood glucose is indicated for patients with syncopal and near syncopal episodes. The NP has performed a thorough history of the incident (diabetic, rapid onset or slow, position, provocation), medical history, health history, and the medications.

Which cranial nerve (CN) is being evaluated when patients are instructed to shrug their shoulders? A) CN IX B) CN X C) CN XI D) CN XII

C) CN XI Cranial nerves IX, X, XI, and XII are: glossopharyngeal, vagal, spinal accessory, and hypoglossal, respectively.

19-22. An FNP has decided to work in dermatology and has been made aware that suturing wounds may be part of the job description. A colleague advises that suturing is outside the scope of practice for an FNP because it is "associated with surgery and thus involves acute care." Which is the best response? A. Suturing is always outside the FNP scope of practice because most FNP programs do not teach this skill B. Many NP schools teach suturing, and NPs can also take continuing medical education for further training on suturing. It is within the FNP scope of practice as long as the FNP is trained in and competent with this skill C. Only physician's assistants and MDs are allowed to suture D. Only MDs are allowed to suture

B. Many NP schools teach suturing, and NPs can also take continuing medical education for further training on suturing. It is within the FNP scope of practice as long as the FNP is trained in and competent with this skill

14-12. Non-Hodgkin of more than 60 cancers. lymphoma (NHL) represents a group Which of the following is not contained within this group? A. Mycosis fungoides B. May-Hegglin anomaly C. Sézary syndrome D. Primary central nervous system lymphoma

B. May-Hegglin anomaly

Which test is used to rule out a meniscal tear? A. Lachman B. McMurray C. Moving patellar apprehension D. Valgus stress

B. McMurray

Which nerve's compression is responsible for the pain and tingling associated with carpal tunnel syndrome? A. Axillary nerve B. Median nerve C. Radial nerve D. Ulnar nerve

B. Median nerve

15-10. A 34-year-old male patient has been diagnosed with HIV. The patient asks why more than one antiret- roviral drug is needed. Of the following statements, which is the best reason for using more than one drug class? A. More drugs mean more cure B. More drugs inhibit viral replication C. More drugs prevent drug interactions D. More drugs decrease CD4+ T cells

B. More drugs inhibit viral replication

14-17. Hypercalcemia associated with multiple myeloma can cause symptoms such as: A. Irritability B. Nausea/vomiting C. Diarrhea D. Decreased thirst

B. Nausea/vomiting

Which of the following is not a preventive measure for epistaxis? A. Using a water-based lubricant in the nares B. Not taking hot showers C. Avoiding forceful nose blowing D. Using a humidifier

B. Not taking hot showers

14-24. A patient has recently been diagnosed with leukemia. Which of the following symptoms would the nurse practitioner expect to see given this diagnosis? A. Nausea, headache, and confusion B. Petechiae, fatigue, and fever C. Abdominal pain, irritability, and psychosis D. Hypotension, bradycardia, and weight loss

B. Petechiae, fatigue, and fever

Persistent cough is the most common presenting symptom is adults in ambulatory care. It is important for the nurse practitioner to distinguish the difference between acute bronchitis and other common causes of cough. All of the following are differentials for acute bronchitis except: A. Upper respiratory infection B. Pharyngitis C. Pneumonia D. Allergic rhinitis

B. Pharyngitis

21-4. The elements of clinical inquiry known as PICO stand for: A. Problem, interaction, cause, outcome B. Population, intervention, comparison, outcome C. Patient, implementation, control, outliers D. Prognostic factor, intervention, cause, outcome

B. Population, intervention, comparison, outcome

16-41. A patient presents with patches on both elbows that are not improving, despite use of a topical steroid cream. They are nonpruritic, pink, sharply demarcated papular plaques with a silvery scale. What is the nurse practitioner's diagnosis of this condition? A. Eczema B. Psoriasis C. Rosacea D. Xerosis

B. Psoriasis

Which test is frequently used to diagnose cholecystitis? A. Abdominal x-ray B. RUQ ultrasound C. Urinalysis D. MRI of abdomen

B. RUQ ultrasound

11-24. When considering the possibility of bacterial menin- gitis, which is the most likely presentation? A. Intermittent fever, neck pain B. Rapid onset of fever, headache, and nuchal rigidity C. Severe headache with low-grade fever D. Gradual onset of fever, headache, and nuchal rigidity

B. Rapid onset of fever, headache, and nuchal rigidity

The "Five Rights" of medication administration can help the nurse practitioner remember how to safely write a prescription. It is prudent to remember which additional two "rights" to ensure best prescribing practices? A. Pharmacy and location B. Rationale and documentation C. Nurse practitioner and patient's addresses D. Response and follow-up

B. Rationale and documentation

16-23. The nurse practitioner in the clinic is treating a patient with painful, blistering burns on the left third, fourth, and fifth fingers, extending slightly onto the dorsa. What is the appropriate plan? A. Apply silver sulfadiazine cream (Silvadene) to the blisters B. Refer for burn specialty care C. Open the blisters and apply antibiotic cream D. Use a nonadherent dressing to wrap loosely

B. Refer for burn specialty care

A 28-year-old male presents to the clinic with a 15-day history of cough that started out dry, but has progressed to productive, with occasional orange-yellow sputum. He denies fever, chills, dyspnea, and nasal congestion, but complains of a burning sensation in his chest when he coughs. The physical exam is unremarkable. As part of his treatment, the nurse practitioner prescribes: A. A macrolide antibiotic, such as azithromycin, for 10 days B. Rest, increased intake of fluids, and use of a humidifier C. An antitussive, rest, and use of an air purifier D. An antihistamine, an antitussive, and increased intake of fluids

B. Rest, increased intake of fluids, and use of a humidifier

A 55-year-old female patient with a history of mitral stenosis presents to your clinic for follow-up. The nurse practitioner may expect to see which finding in the patient's history? A. Rheumatoid arthritis B. Rheumatic fever C. Alopecia areata D. Graves disease

B. Rheumatic fever

A 15 m.o. child who is eating and behaving normally is found to have a high fever. A few days after, the fever resolves and the infant breaks into a maculopapular rash. This is a description of which of the following? A. Erythema infectiosum B. Roseola Infantum C, Fifth disease D. Scarlet fever

B. Roseola Infantum (Also known as fifth disease). Slapped cheek appearance. The happy rash, kids don't seem sick

The mother of a 12 m.o baby reports to the NP that her baby had a high fever for several days that resolved spontaneously. A few days later she broke out into a maculopapular rash. Which of the following is the most likely DX? A. Fifths disease (erythema infectiosum) B. Roseola infantum (exanthema subitum) C. Varicella D. Infantile maculopapular rashes

B. Roseola Infantum (exanthum subitum) is a common viral rash that is caused by the human herpes virus. The most common ages of onset is 6months -2 yrs old. The rashes are maculopapular (round and pink) and appear first on the trunk and then spread to the extremities

11-19. A headache induced by uncontrolled hypertension would be described as a: A. Primary headache B. Secondary headache C. Tertiary headache D. Chronic headache

B. Secondary headache

An 18-year-old female presents to the office for insertion of the Skyla LNG IUD. As you prepare for the procedure, which information in her chart would cause you to reschedule her appointment for a later date? A. She was recently diagnosed with a right-sided ovarian cyst B. She is currently being treated for pelvic inflammatory disease C. She has poorly controlled type 1 diabetes mellitus D. She takes amphetamine/dextroamphetamine for attention deficit disorder

B. She is currently being treated for pelvic inflammatory disease

A 74-year-old female complains of low back pain. She reports a history of osteopenia, steroid use for her chronic rheumatoid arthritis, and a fall yesterday down the back two steps. What is the likely cause of her low back pain? A. Kyphosis B. Spinal fracture C. Poor posture D. Cauda equina syndrome

B. Spinal fracture

16-11. A 58-year-old African American male presents to the clinic with complaints of a non-tender sore on his upper lip that appeared approximately two months ago. PMH is non-contributory, though he has a 30 pack-per-year smoking history. Upon examination, the FNP notes a slightly elevated, erythematous plaque with well-defined borders and no telangiectasias. The suspected diagnosis is: A. Basal cell carcinoma B. Squamous cell carcinoma C. Actinic keratosis D. Psoriasis

B. Squamous cell carcinoma

12-15. When assessing a patient with a diagnosis of anorexia nervosa, it is very important at every encounter to evaluate for: A. Associated eating disorders B. Suicide ideation, behaviors, and risk factors for self-harm C. Family history of eating disorders D. Substance use disorder

B. Suicide ideation, behaviors, and risk factors for self-harm

4-2. The primary means of medication excretion is through: A. The liver B. The kidneys C. Exhaled air D. Sweat from pores

B. The kidneys

A research participant tells the nurse practitioner that he wants to withdraw from the study. Regarding this case, the nurse practitioner is aware that all research study consent forms should contain which of the following information? A) The patient's demographic information B) The possible risks of the study C) Information that a research subject can voluntarily withdraw from the study at any time without any penalties or adverse consequences D) The benefits from the study

C) Information that a research subject can voluntarily withdraw from the study at any time without any penalties or adverse consequences Information that should be included in the written consent forms is that the research subject can voluntarily withdraw from the study at any time without any penalties or adverse consequences. In addition, the human subject should be verbally informed of this right.

Which of the following anatomic areas are involved with sensorineural hearing loss? A) Outer ear B) Middle ear C) Inner ear D) Cranial nerve VIIII

C) Inner ear Sensorineural hearing loss (i.e., presbycusis) involves damage to both the hair cells in the cochlea (sensory portion) and cranial nerve VIII (neural).

The red blood cells in pernicious anemia will show: A) Microcytic and hypochromic cells B) Microcytic and normochromic cells C) Macrocytic and normochromic cells D) Macrocytic and hypochromic cells

C) Macrocytic and normochromic cells The blood cells in pernicious anemia will display macrocytic, normochromic cells. RBCs in iron-deficiency anemia are microcytic and hypochromic.

You note a high-pitched and blowing pansystolic murmur in a 70-year-old male. It is Grade II/VI and is best heard at the apical area. Which of the following is most likely? A) Ventricular septal defect B) Tricuspid regurgitation C) Mitral regurgitation D) Mitral stenosis

C) Mitral regurgitation Mitral regurgitation is best heard at the apical area, and manifests as a high-pitched, blowing pansystolic murmur.

Jane, a middle-aged nurse, complains of localized pain on the sole of her left foot that is located between the third and fourth toes. The pain is aggravated by weight bearing and feels like "a pebble in my shoe." During the physical examination, the nurse practitioner palpates a tender nodule in the metatarsal interspace on the left foot. No redness or swelling is noted. Which of the following conditions is being described? A) Plantar wart B) Foreign body C) Morton's neuroma D) Metatarsalgia

C) Morton's neuroma Morton's neuroma is characterized by experiencing pain, burning, or numbness with weight bearing between two toes. Some experience a sense or feeling like they are walking on a pebble between two adjacent toes.

All of the following are true statements about diverticula except: A) Diverticula are located in the colon B) A low-fiber diet is associated with the condition C) Most diverticula in the colon are infected with gram-negative bacteria D) Supplementingwithfibersuchaspsyllium(Metamucil)isrecommended

C) Most diverticula in the colon are infected with gram-negative bacte- ria Diverticuli in the colon can be infected with both gram-negative and gram- positive bacteria.

The following statements about benign prostatic hypertrophy are correct except: A) It is seen in up to 50% of males older than 50 B) Dribbling and nocturia are common patient complaints C) Saw palmetto is always effective in reducing symptoms D) The PSA value is usually slightly elevated

C) Saw palmetto is always effective in reducing symptoms Research does not support the theory that saw palmetto reduces the symptoms of benign prostatic hyperplasia in all cases.

A 48-year-old woman is told by a physician that she is starting menopause. All of the following are possible findings except: A) Hot flashes B) Irregular menstrual periods C) Severe vaginal atrophic changes D) Cyclic mood swings

C) Severe vaginal atrophic changes As women reach menopause, changes that may occur include hot flashes, irregular menstrual periods, and cyclic mood swings. Vaginal changes, such as dryness and thinning, may also begin to occur

All of the following infections are reportable diseases except: A) Lyme disease B) Gonorrhea C) Nongonococcal urethritis D) Syphilis

C) Nongonococcal urethritis The public health department requires all agencies to report Lyme disease, gonorrhea, and syphilis. Statistics of these reportable diseases are kept in each state.

All of the following services are covered under Medicare Part A except: A) Inpatienthospitalizations B) Medicines administered to a patient while hospitalized C) Nursing home care D) Surgeons

C) Nursing home care Medicare A coverage includes inpatient hospitalization and skilled care given in a certified skilled nursing facility. Most nursing home care is custodial care (help with bathing, dressing, using a bathroom, and eating). This care is not covered by Medicare A.

All of the following patients have an increased risk of developing adverse effects from Metformin (Glucophage) except: A) Patients with renal disease B) Patients with hypoxia C) Obese patients D) Patients who are alcoholics

C) Obese patients Metformin is used as a hyperglycemic agent for NIDDM. However, increased mortality has been associated with drug-induced lactic aci- dosis. Contraindications for the use of metformin include patients with significant renal and hepatic disease, alcoholism, and conditions associated with hypoxia (cardiac/pulmonary problems), sepsis, dehydration, high doses, and advanced age. Metformin is used in obese patients and education regarding the lactic acidosis risk should be discussed with the patient and family.

A neighbor's 14-year-old son, who is active in basketball, complains of pain and swelling on both knees. On physical exam, there is tenderness over the tibial tuberosity of both knees. Which of the following is most likely? A) Chondromalacia patella B) Left knee sprain C) Osgood-Schlatter disease D) Tear of the medial ligament

C) Osgood-Schlatter disease Osgood-Schlatter disease is characterized by bilateral pain over the tibial tuberosity upon palpation, along with knee pain and edema with exercise.

A 15-year-old basketball player who is 6 ft tall is seen for complaints of painful lumps on his knees. Upon inspection, the nurse practitioner notes a bonelike growth on the upper tibia midline below the kneecap on both knees. The patient has full range of motion with no joint tenderness, redness, or swelling. Which of the following conditions is best described? A) Osteosarcoma of the tibia B) Juvenile rheumatoid arthritis C) Osgood- Schlatter disease D) Paget's disease of the bone

C) Osgood-Schlatter disease Osgood-Schlatter disease is characterized by having pain over the tibial tuberosity with palpation of a bony mass over the anterior tubercle of one or both knees. Exercise worsens the pain

Kyphosis is a late sign of: A) Old age B) Osteopenia C) Osteoporosis D) Osteoarthritis

C) Osteoporosis Kyphosis is a curvature of the spine that causes a rounding of the back, which leads to a slouching posture. Severe thinning of the bones (osteoporosis) contributes to this curvature in the spine. Symptoms that may occur with severe cases of kyphosis include difficulty breathing, fatigue, and back pain.

During the eye exam of a 50-year-old hypertensive patient who is complaining of an onset of a severe headache, you find that the borders of the disc margins on both eyes are blurred. What is the name of this clinical finding? A) Normal optic disc B) Optic neuropathy C) Papilledema D) Hypertensive retinopathy

C) Papilledema Papilledema is defined as having optic disc swelling that is caused by increased intracranial pressure. It is commonly seen in both eyes and can develop over a time period of hours or weeks. Headache is a common complaint.

Acute bronchitis is best characterized by: A) Fever and wheezing B) Purulent sputum and fever C) Paroxysms of coughing that is dry or productive of mucoid sputum D) A gradual onset and fatigue

C) Paroxysms of coughing that is dry or productive of mucoid sputum Acute bronchitis is characterized by spastic cough that is dry or productive with mucus.

A 25-year-old woman's last menstrual period was 6 weeks ago. She is complaining of nausea with vomiting in the morning and fatigue. Her breasts feel bloated. The nurse practitioner suspects that she is pregnant. Her symptoms would be considered: A) Positive signs of pregnancy B) Probable signs of pregnancy C) Presumptive signs of pregnancy D) Possible signs of pregnancy

C) Presumptive signs of pregnancy Presumptive signs of pregnancy are symptoms experienced by the woman, such as amenorrhea, breast tenderness, nausea/ vomiting, fatigue, and increased urinary frequency.

Which of the following is responsible for the symptoms of dysmenorrhea? A) Estrogen B) Human chorionic gonadotropin C) Prostaglandins D) Progesterone

C) Prostaglandins Prostaglandins are hormones the body produces prior to menses; they eventually cause the uterus to contract to shed the endometrial lining. Contractions cause pain. The greater the amount of prostaglandins that are released, the more pain one will experience. Contractions of the uterus cause vasoconstriction of blood supply to the uterus, which in turn will cause pain.

Which of the following pathogenic bacteria are commonly found in the lungs of older children and adults with cystic fibrosis? A) Streptoccocus pneumoniae B) Chlamydia pneumoniae C) Pseudomonas aeruginosa D) Staphylococcus aureus

C) Pseudomonas aeruginosa Pseudomonas aeruginosa is a common chronic lung infection seen in older children and adults with cystic fibrosis. This infection is difficult to get rid of and sometimes is the cause of death.

What is the least common pathogen found in community-acquired atypical pneumonia? A) Moxarella catarrhalis B) Streptococcus pneumoniae C) Pseudomonas aeruginosa D) Mycoplasma pneumonia

C) Pseudomonas aeruginosa Pseudomonas aeruginosa is an uncommon cause of CAP but is a particularly difficult bacterial infection to treat.

A 62 y.o. male with COPD complains to his NP that his ipratropium bromide (Atrovent) is not working. He reports he still feels SOB after using it 4X per day for 3 months. Which of the following actions is the next step for the NP? A. Increase pt's dose to 3 inhalations QID B. Continue the atrovent and start the pt on O2 via NC C. Continue the atrovent and add albuterol 2 inhalations QID D. Start the pt on O2 at night time abd PRN during the day

C: COntinue atrovent and add albuterol. The plan of care is always start with an anticholinergic (Atrovent) and then add short acting beta agonist

A cauliflower like growth with fowl smelling discharge is seen in an otoscopic exam of the left ear of an 8 y.o. boy with a hx of chronic otitis media. No tympanic membrane, ossicles, are visible and the pt seems to have difficulty hearing out of the affected ear. Which condition is the best described? A. Chronic perforation of TM with secondary bacterial infetcion B. Chronic Mastoiditis C. Cholesteatoma D. Cancer of the middle ear

C: Cholesteatoma

Lead poisoning can cause which type of anemia? A. A mild macrocytic anemia B. Normocytic anemia C. Microcytic anemia D. A mild hemolytic anemia

C: Microcytic anemia: Lead poisoning can cause microcytic anemia. Causes anemia by mimicking healthful minerals such as Calcium, iron and zinc. Is absorbed by the bones where it disrupts the formation of RBCs.

Acute Bronchitis is best characterized by: A. Fever and wheezing B. Purulent Sputum and fever C. Paroxysms of coughing that is dry or with productive of mucoid sputum D. A gradual onset and fatigue

C: Paroxysms of coughing: Bronchitis is characterized by intermittent cough that is either dry or productive with mucous

12 y.o. complaining of 2 week history of facial pressure, that worsens when she bends over. She c/o tooth pain in her upper molars on the right side of her face. On physical exam, her lung and heart sounds are normal. Which of the following is the most likely DX? A. An acute dental abscess B. Chronic sinusitis C. Acute sinusitis D. Severe allergic rhinitis

C; Acute sinusitis: s/s include facial pressure, headache, pain that worsens when bending over, eye/ear pain, aching in upper jaw/ teeth, reduced smell/ taste and cough.

All of the following do not require parental consent to be obtained by the nurse practitioner except: A) A 17-year-old who wants to be treated for a sexually transmitted infection B) A 12-year-old who wants a serum pregnancy test C) A 15-year-old who wants birth control pills D) A 14-year-old who wants to be treated for dysmenorrhea

D) A 14-year-old who wants to be treated for dysmenorrhea Treatment for teenagers may be done without parental consent for sexually transmitted infections, pregnancy testing, contraception counseling and treatment. Parental consent is required for any type of physical exam for other problems

Pulsus paradoxus is more likely to be associated with: A) Sarcoidosis B) Acute bronchitis C) Status asthmaticus D) Bacterial pneumonia

C) Status asthmaticus Pulsus paradoxus is most likely to be seen with status asthmaticus. With inspiration, systolic pressure drops due to the increased pres- sure (positive pressure). Some pulmonary risks of having increased pressure include asthma and emphysema. Cardiac causes for pulsus paradoxus include tamponade, pericarditis, and cardiac effusion.

Which of the following T-scores is indicative of osteoporosis? A) T-score of 0 to -1.0 B) T-score of -1.0 to -2.0 C) T-score of -2.5 or less D) The diagnosis of osteoporosis is based on an x-ray series of the spine

C) T-score of -2.5 or less Osteoporosis is defined as having a T-score of -2.5 or less. Risk factors should also be considered when interpreting T-scores for diagnosis.

During a sports physical of a 14-year-old female, you note her breast development. The areola and the breast tissue are all in one mound. In which Tanner stage is this patient? A) Tanner Stage I B) Tanner Stage II C) Tanner Stage III D) Tanner Stage IV

C) Tanner Stage III Tanner stages for breast development:I , prepuberty; II, breast bud; III, breast and areola one mound; IV, breast and areola secondary mound; V, adult pattern.

Swim therapy for a 13-year-old with cerebral palsy is an example of: A) Primary prevention B) Secondary prevention C) Tertiary prevention D) Health prevention

C) Tertiary prevention Tertiary prevention is any type of rehabilitation for a particular condition. Examples include physical rehab (swimming), cardiac rehab, and/or speech therapy.

Spermatogenesis occurs at the: A) Vas deferens B) Seminal vesicles C) Testes D) Epididymis

C) Testes Spermatogenesis occurs in the testes, taking approximately 3 months to develop.

Which of the following methods is used to diagnose gonorrheal pharyngitis or proctitis? A) Serum chlamydia titer B) Gen-Probe C) Thayer-Martin culture D) Culture and sensitivity of the purulent discharge

C) Thayer-Martin culture Thayer-Martin culture is recommended for screening to diagnose gonorrheal pharyngitis or proctitis.

All of the following are correct statements regarding oral contraceptives except: A) The actual failure rate of oral contraceptives is 3% B) Desogestrel belongs to the progesterone family of drugs C) The newer low-dose birth control pills do not require back-up during the first 2 weeks of use D) Oral contraceptives are contraindicated for women 35 years of age or older who smoke

C) The newer low-dose birth control pills do not require backup during the first 2 weeks of use Low-dose OCPs require a second backup method of contraception during the first 2 weeks of use.

A sexually active woman is complaining of amenorrhea and vaginal spotting. On exam, her left adnexa is tender and cervical motion tenderness is positive. Which test should the nurse practitioner initially order? A) Flat plate of the abdomen B) Complete blood count (CBC) with white cell differentials C) Urine pregnancy test D) Pelvic ultrasound

C) Urine pregnancy test A urine pregnancy test should be performed to rule out miscarriage and/or tubal pregnancy.

In small children with acquired immunodeficiency syndrome (AIDS), which of the following vaccines is contraindicated? A) Diphtheria and tetanus (Td) B) Hepatitis B and mumps C) Varicella D) Td and oral polio

C) Varicella The data regarding efficacy of the varicella vaccine are insufficient; therefore, varicella vaccine is contraindicated in HIV-infected individuals.

Physiologic anemia of pregnancy is due to: A) An increase in the cardiac output at the end of the second trimester B) A physiologic decrease in the production of RBCs in pregnant women C) An increase of up to 50% of the plasma volume in pregnant women D) An increase in the need for dietary iron in pregnancy

C)An increase of up to 50% of the plasma volume in pregnant women Physiologic anemia of pregnancy is caused by the increased volume of plasma during pregnancy when compared to the production of RBCs.

A college student has recently been informed that he has an HPV (human papilloma virus) infection on the shaft of his penis. Which of the following may reveal subclinical lesions on the penile skin? A) Perform a KOH (potassium hydroxide) exam B) Scrape off some of the affected skin and send it for a culture and sensitivity C) Apply acetic acid to the penile shaft and look for acetowhite changes D) Order a serum herpes virus titer

C)Apply acetic acid to the penile shaft and look for acetowhite changes Lesions of HPV infection will turn white with application of acetic acid.

For the adult patient, what is the maximum amount of acetaminophen that can be taken per day? A. 1 gram B. 2 grams C. 4 grams D. 5 grams

C. 4 grams

16-38. Which of the following individuals is most likely to experience rosacea? A. 5-year-old Caucasian male B. 14-year-old African American male C. 48-year-old Caucasian female D. 69-year-old African American female

C. 48-year-old Caucasian female

16-40. Which of the following statements regarding rosacea is incorrect? A. The cause is unknown B. Rosacea may cause dry, red eyes C. A biopsy is necessary for diagnosis D. A cure does not exist

C. A biopsy is necessary for diagnosis

The nurse practitioner knows the most likely patients to succumb to chronic viral hepatitis of the following are: A. Any patients with hepatitis A B. Adults with hepatitis B C. Adults with hepatitis C D. Any patients with hepatitis D

C. Adults with hepatitis C

Treatment for hemorrhoids includes: A. Adequate hydration and low fiber to bulk and soften the stool B. Cool sitz baths to reduce itchiness and irritation C. Analgesic creams for pain management D. Oral nitroglycerin to promote relaxation of the anal sphincter

C. Analgesic creams for pain management

What should the FNP prescribe for acute dacryocystitis? A. Antiviral medication B. Dacryocystorhinostomy C. Antimicrobial medication D. Cold compresses

C. Antimicrobial medication

11-7. An 82-year-old patient presents with a history of TIA and confusion. Which test is the most sensitive for stroke? A. Ataxic gait B. Troponin, elevated C. Arm weakness/pronator drift D. Facial drooping, bilateral

C. Arm weakness/pronator drift

20-7. The nurse practitioner's role related to the social determinants of health is best described by which of the following actions? A. Focus on individual behavior to impact health and wellness B. Encourage people to take responsibility for their own health behaviors C. Assess and intervene for the root causes of disease and health disparities D. Lecture people about how to change wrong health behaviors to right behaviors

C. Assess and intervene for the root causes of dis- ease and health disparities

11-27. Multiple sclerosis can be described as a(n): A. Autoimmune disorder typically featuring upper and lower motor neuron dysfunction, hyperre- flexia, atrophy, and fasciculations B. Demyelinating autoimmune disorder with damage typically seen in the white matter of the brain and spinal cord; seen predominately in children/teens C. Autoimmune disorder typically featuring inflammation of the spinal cord, an inflamed optic nerve, and ataxia D. Autoimmune disorder typically featuring ascending paraplegia and potentially fatal respi- ratory muscle weakness

C. Autoimmune disorder typically featuring inflammation of the spinal cord, an inflamed optic nerve, and ataxia

11-10. A medication like donepezil can be useful to treat mild and moderate dementia because it: A. Blocks cholinesterase, resulting in a reverse of cognitive symptoms B. Stimulates the release of cholinesterase, stopping the progression of cognitive decline C. Blocks cholinesterase, resulting in stabilization of cognitive decline D. Stimulates the release of cholinesterase, improv- ing cognitive function

C. Blocks cholinesterase, resulting in stabilization of cognitive decline

GERD food triggers include: A. Wintergreen B. Cheddar cheese C. Chocolate D. Saltine crackers

C. Chocolate

19-32. Which of the following is true regarding diagnosis coding? A. As long as CPT codes are documented, diagnosis codes do not need to be included B. Coding should follow ICD-9 guidelines C. Coding should be accurate and thorough and should explain the patient encounter D. Coding for a patient visit is the responsibility of the billing department

C. Coding should be accurate and thorough and should explain the patient encounter

Which of the following symptoms is not common to GERD? A. Regurgitation B. Heartburn C. Diverticulitis D. Dysphagia

C. Diverticulitis

4-15. Which of the following is a chemotherapeutic (anti-cancer) dosing principle? A. Drugs should be administered at low doses but with increased frequency B. Drugs are more beneficial if major toxicities are nonoverlapping C. Drugs should be administered infrequently to minimize side effects D. Drugs are rarely effective in combination

C. Drugs should be administered infrequently to minimize side effects

12-5. The diagnostic criteria for generalized anxiety disorder (GAD) include all of the following except: A. Irritability B. Apprehension C. Early morning awakening D. Difficulty concentrating

C. Early morning awakening

Which is not included in the ACR criteria to diagnose SLE? A. Proteinuria B. Leukopenia C. Elevated potassium D. Malar rash

C. Elevated potassium

12-2. Donna, a 20-year-old single mother, lives in a rundown apartment in a dangerous area of her city. After her 12-hour shift as a cashier, she comes home irritable. She admits she yells, berates, and belittles her 7-year-old son. Lately, she has taken to locking him in the basement when she is too tired. Together, these behaviors may potentially be considered: A. Neglect B. Physical abuse C. Emotional abuse D. Not abusive, as she does not hit her child

C. Emotional abuse

14-9. The gold standard of diagnosis for lymphoma is: A. Fine-needle aspiration B. Core biopsy C. Excisional biopsy of an intact node D. Chest x-ray of lymph node sites

C. Excisional biopsy of an intact node

When treating infectious mononucleosis, the NP should recommend which of the following? A. Antibiotics, analgesics, fluids, rest B. Antivirals, analgesics, fluids, rest C. Fever and pain reduction, fluids, rest D. Allow sports participation, fluids, analgesics

C. Fever and pain reduction, fluids, rest

Which of the following signs is highly suspicious for radial head subluxation? A. Numbness and tingling in the affected hand B. Noticeable deformity of the affected arm C. Fixed, straight position of the affected arm by the side D. Significant edema of the affected arm

C. Fixed, straight position of the affected arm by the side

When educating a patient with Meniere disease about prevention of episodes, the NP teaches the patient to restrict all of the following except: A. Sodium B. Nicotine C. Fluid intake D. Caffeine

C. Fluid intake

19-15. The nurse practitioner is often called upon to advo- cate for vulnerable patients. The nurse practitioner can expect to fulfill this role: A. Only when the ombudsman is not available B. Only in cases where patients who do not have family members available to weigh in on their care C. For all patients D. Only in situations where the patient is confused or unable to make decisions

C. For all patients

An undesirable late-onset complication of GERD is: A. Testicular torsion B. Gastroparesis C. Gastric structure D. Diverticulum formation

C. Gastric structure

14-23. Which of the following is a risk factor for the development of acute lymphoblastic leukemia? A. Agent orange exposure B. Benzene exposure C. Genetic syndrome D. Cystic fibrosis

C. Genetic syndrome

Which next step should be included in the instructions following the administration of carbamide peroxide? A. Keep the eyes closed for 2-3 minutes B. Wait 15 minutes prior to inserting contact lenses C. Gently irrigate the ear canal with warm water D. Make sure vision is clear before driving

C. Gently irrigate the ear canal with warm water

4-5. The P450 enzyme system affects the way medications are metabolized. Which of the following is an example of the P450 system in action? A. The ease that a medication is bound by plasma binding proteins B. A bigger molecule having a harder time crossing membranes C. Grapefruit juice interacting with a medication's metabolism D. The need to lower the dose of a medication because of poor kidney function

C. Grapefruit juice interacting with a medication's metabolism

18-15. An example of an advance care directive is the designation of which of the following? A. Funeral director B. Primary care provider C. Healthcare advocate D. Guardian of trust

C. Healthcare advocate

16-46. A patient presents with a painful, pimple-like rash in her axilla. She states she has had this once or twice before. What is the most likely diagnosis? A. Acne vulgaris B. Folliculitis C. Hidradenitis suppurativa D. Urticaria

C. Hidradenitis suppurativa

All of the following are risk factors for developing pancreatic cancer except: A. Obesity B. Family history C. History of Addison disease D. History of diabetes mellitus

C. History of Addison disease

Which of the following objective findings does not result from embolic events related to infective endocarditis? A. Janeway lesions B. Osler nodes C. Impetigo D. Roth spots

C. Impetigo

11-52. The purpose of debulking of a brain tumor is to: A. Remove all of the tumor B. Disrupt cancer cells' ability to multiply C. Improve quality and length of life D. Prevent further growth of the tumor

C. Improve quality and length of life

16-44. A patient has psoriasis on both arms. What is the most appropriate treatment recommendation? A. Tumor necrosis modulators B. Psoralen with ultraviolet A light therapy C. Medium-potency topical corticosteroids D. Anthralin or calcipotriene

C. Medium-potency topical corticosteroids

Which of the following is not a danger sign for neck or back pain? A. Patient with neck pain who was involved in an MVA two days ago B. Patient with low back pain who is currently tak- ing chemotherapy for stage III colon cancer C. Patient with neck pain and a history of smoking two packs per day for 30 years D. Patient with neck pain that is worsening and radiates to the left jaw

C. Patient with neck pain and a history of smoking two packs per day for 30 years

a 49-year-old male presents with a pulmonary embolism. The NP would expect to find which clinical symptoms upon assessment of this patient? A. Lower extremity edema, stridor, hypotension B. Crackles, S4 gallop, bradycardia C. Rales, accentuated second heart sound, wheezing D. Hemoptysis, hypotension, diminished heart sounds

C. Rales, accentuated second heart sound, wheezing

What do Koplik spots occur with? A. Leukoplakia B. Rubella C. Rubeola D. Behcet syndrome

C. Rubeola

3-12. The ability of a test to correctly identify whose with the disease (true positive rate) is known as: A. False positivity B. Incidence rate C. Sensitivity D. Specificity

C. Sensitivity

13-5. Symptoms of adrenal crisis can include A. Excitability В. Нурertension C. Severe leg, back, or abdominal pain D. Craving of sweets

C. Severe leg, back, or abdominal pain

19-27. The Affordable Care Act (ACA) introduced major healthcare reforms, but did not provide for: A. Coverage to patients with pre-existing conditions B. Capping of patients out-of-pocket expenditures C. Single-payer healthcare D. Coverage of young people (dependents still on their parents' healthcare plan) through age 26

C. Single-payer healthcare

A 72-year-old male has just been diagnosed with lung cancer. Which of the following recommendations should be made by the nurse practitioner as part of his immediate treatment plan? A. Morphine IVP for pain control B. Chest x-ray every three months C. Smoking cessation D. Serial CT scans every 3 months to monitor the size of the tumor

C. Smoking cessation

A.J. is a 22-year-old college student who complains of feeling his heart race. He states it is intermittent and has happened 3-4 times weekly for the last month. Each episode lasts 5-10 minutes. His vital signs are HR 92 and regular, BP 124/82, RR 16, SpO2 99%, pain 0/10. Pertinent history would include all of the following except: A. Caffeine usage B. Prescription medications C. Sodium intake D. Illicit drug use

C. Sodium intake

An elderly pt with productive cough + fever diagnosed with pneumonia. All of the following organisms are capable of causing CAP except: A. Haemophilus influenzae B. Mycoplasma pneumoniae C. Treponema pallidum D. Strep Pneumo

C. Treponema pallidum is a spirochete that causes syphilis

How is herpes stomatitis differentiated on physical examination? A. Large white patches or plaques on tongue B. round ulcers on a gray base in oral cavity C. Ulcerations on an erythematous base D. Small white spots on buccal mucosa

C. Ulcerations on an erythematous base

17-5. Which rash is characteristic of varicella? A. Polymorphous rash B. "Slapped cheek" and lacy rash C. Vesicular lesions in various stages D. Koplik spots

C. Vesicular lesions in various stages

The FNP is caring for a patient who describes recent stress in her life and presents with a recurrent mouth lesion. The lesion came up again 2 days ago, is on the interior lip border, is ulcerative and painful, and has an erythematous base. What is the medication treatment of choice for this patient? A. Colchicine (Colcrys) B. amoxicillin (Amoxil) C. acyclovir (Zovirax) D. mycostatin (Nystatin)

C. acyclovir (Zovirax)

Which medication is considered first-line treatment for acute bacterial rhinosinusitis? A. Doxycycline (Monodox) B. levofloxacin (Levaquin) C. amoxicillin-clavulanate (Augmentin) D. moxifloxacin (Avalox)

C. amoxicillin-clavulanate (Augmentin)

Which medication would be a first-line antihypertensive for a new hypertension diagnosis in a patient who is African American with a family history of diabetes? A. metoprolol tartrate (Toprol) B. clonidine transdermal patch (Catapres) C. chlorthalidone (Thalitone) D. benazepril (Lotensin)

C. chlorthalidone (Thalitone)

11-8. George, age 65, has had several transient ischemic attacks. After a complete physiological exam and workup, the nurse practitioner would prescribe which medication? A. atenolol (Tenormin) B. warfarin (Coumadin) C. clopidogrel (Plavix) D. nitroglycerine (Nitrostat)

C. clopidogrel (Plavix)

A patient has been diagnosed with irritable bowel syndrome (IBS) by the nurse practitioner. Which medication is the nurse practitioner likely to prescribe? A. cimetidine (Tagamet) B. dexamethasone (Decadron) C. dicyclomine (Bentyl) D. infliximab (Remicade)

C. dicyclomine (Bentyl)

A 53-year-old female presents to the office with complaints of vaginal discomfort and burning on urination. Her last period was two years ago. Your exam findings are as follows: pregnancy test is negative, external vagina with normal appearance, internal vagina is pale light pink, cervix with mild stenosis, scant clear vaginal discharge, bimanual exam without tenderness, vaginal pH is 5, whiff test is negative, wet mount shows no buds/clue cells/trichomonads, urine DIP is negative. Which of the following prescriptions do you recommend for this patient? A. hydrocortisone 1% topical cream (Cortef) B. metronidazole gel 0.75% (MetroGel) C. estradiol 0.01% cream (Estace) D. clotrimazole 1 % cream (Cleocin)

C. estradiol 0.01% cream (Estace)

A patient with LLQ abdominal pain, fever, nausea, vomiting, and diarrhea, presents to the primary care clinic. Your initial workup includes a urinalysis and CBC with differential. The patient wants symptomatic relief while awaiting a diagnosis. Which medication would be contraindicated? A. acetaminophen (Tylenol) B. ibuprofen (Advil) C. loperamide (Imodium A-D) D. promethazine (Phenergan)

C. loperamide (Imodium A-D)

Of the following treatments, which is not included in the plan for strep pharyngitis? A. throat lozenges B. ibuprofen (Motrin) C. prednisone (Deltazone) D. penicillin V (PenVK)

C. prednisone (Deltazone)

11-32. Which of the following treatments is matched correctly with the targeted symptom? A. Neurontin for fatigue B. baclofen for nerve pain C. propranolol for tremors D. dalfampridine for nystagmus

C. propranolol for tremors

15-4. Which of the following white blood cell elevations as noted on the differential is associated with severe bacterial infections? A. Eosinophils B. Lymphocytes C.Monocytes D. Neutrophils

C.Monocytes

Which medication would cause concern prior to prescribing ipratropium bromide (Atrovent)? A. albuterol (Proair) B. fluoxetine (Prozac) C. montelukast (Singulair) D. Tamsulosin (Flomax)

D. Tamsulosin (Flomax)

16-39. The nurse practitioner is prescribing monotherapy for a patient with mild acne. Which product will be prescribed? A. benzoyl peroxide (Benzagel) B. isotretinoin (Accutane) C. Topical retinoid (Retin A) D. Regular facial moisturizer

A. benzoyl peroxide (Benzagel)

For which of the following medications is the patient required to keep the eyes closed for 2-3 minutes without blinking or squinting? A. cyclosporin (Restasis) B. brimonidine (Alphagan) C. pilocarpine (Salagen) D. carbachol (Miostat)

A. cyclosporin (Restasis)

18-1. The nurse practitioner knows that __________ is not a normal physiological change that occurs in the elderly. A. dementia B. loss of muscle mass C. reduced cardiac function D. loss of visual acuity

A. dementia

15-7. Which of the following is not a likely diagnostic test for vector-borne illnesses? A. Enzyme-linked immunosorbent assay B. Nucleic acid test C. Polymerase chain reaction D. Tensilon test

D. Tensilon test

Which of the following is the most common cause of cancer deaths for women in the United States? A) Breast cancer B) Lung cancer C) Colon cancer D) Heart disease

B) Lung cancer Lung cancer is the most common cause of cancer deaths in women as well as men.

Cluster headaches are most often seen in: A) Adolescent females B) Middle-aged men C) Elderly men D) Postmenopausal women

B) Middle-aged men Cluster headaches are most commonly seen in middle- aged men.

You note that your 11-year-old female patient is at Tanner Stage II. You would advise her mother that menarche will probably start in: A) 1 to 2 years B) 2.5 to 3 years C) 4 years D) It is dependent on the girl's genetic makeup

A) 1 to 2 years Menarche normally begins approximately 1-2 years following Tanner Stage II. Average age is 11.8 years old.

One of the developmental milestones for this age group is the ability to draw a stick figure "person" with 2 to 3 separate body parts. What is the age group that this finding is associated with? A) 3 year old B) 4 year old C) 5 year old D) 6 year old

A) 3 year old The ability to draw a person with 2 to 3 separate body parts is one of the developmental milestones to assess in children who are 3 years of age. Others are climbs up stairs with alternating feet without handrails; rides a tricycle; knows name, age, and gender; and can speak in 3-word sentences that can be understood by most strangers.

According to the guidelines outlined in JNC 8 in those younger than 60 (without comorbitidites), the normal blood pressure should be: A) <140/90 B) <130/85 C) <120/80 D) <110/75

A) <140/90 The JNC VIII Guidelines recommended that bp be less than 140/90 in those less than 60. Those older than 60, BP should be less than 150/90.

All of the following clinical findings are considered benign oral findings except: A) A patch of leukoplakia B) Fordyce spots C) Torus palatinus D) Fishtail uvula

A) A patch of leukoplakia An early sign of HIV is a hairy patch of leukoplakia on the tongue.

Which of the following would be classified as a second-degree burn? A) A severe sunburn with blistering B) Burns that involve the subcutaneous layer of skin C) A reddened finger after touching a hot iron D) Burns that involve escha

A) A severe sunburn with blistering Burns: First degree: erythema without blistering, painful; second degree: red skin with superficial blisters, painful; third degree: entire skin layering, involving the subcutaneous tissue and may include soft tissue facia, painless.

An 80-year-old male with hypertension and hyperlipidemia presents with complaints of the quick onset of severe low back pain that is accompanied by abdominal pain which is gradually worsening. The patient appears pale and complains that he does not feel well. During the abdominal exam, the nurse practitioner detects a soft pulsatile mass just above the umbilicus as she palpates this area with her hand. Which of the following conditions is most likely? A) Abdominal aortic aneurysm B) Cauda equina syndrome C) Acute diverticulitis D) Adenocarcinoma of the colon

A) Abdominal aortic aneurysm Elderly males who are ex-smokers are at higher risk for abdominal aortic aneurysm. It is usually asymptomatic and is discovered incidentally during a routine chest x-ray or abdominal ultrasound. Although small aneurysms are usually not detectable during abdominal exams, the larger aneurysms may be palpable during an abdominal exam, but abdominal obesity will obscure the findings. The symptoms in this case point toward a rapidly dissecting aneurysm. The best action in this case is to call 911 stat.

A positive obturator sign might signify which of the following conditions? A) Acute appendicitis B) Acute pancreatitis C) Acute cholecystitis D) Acute hepatitis

A) Acute appendicitis A positive obturator sign may indicate acute appendicitis. The test is performed by having the patient lie down supine, and rotate the hip, using full range of motion. A positive sign is when pain is experienced with movement or flexion of the hip.

An urgent care nurse practitioner is assessing a 45-year-old White woman with a BMI of 32 for complaints of intermittent right upper quadrant abdominal pain over the past few weeks that is precipitated by eating fried foods and peanut butter. On exam, the patient's heart and lungs are normal. There is no pain over the costovertebral angle. During abdominal exam, the bowel sounds are present in all quadrants. While palpating deeply on the right upper quadrant during deep inspiration, the patient complains of severe sharp pain to the nurse practitioner. Which of the following is best described? A) Acute cholecystitis B) Acute appendicitis C) Acute gastroenteritis D) Acute diverticulitis

A) Acute cholecystitis Acute cholecystitis can be diagnosed when deep palpation of the right upper quadrant causes the patient to complain of pain.

All of the following are possible causes for secondary hypertension except: A) Acute pyelonephritis B) Pheochromocytoma C) Renovascular stenosis D) Coarctation of the aorta

A) Acute pyelonephritis Pheochromocytoma is an adrenal gland tumor in which there is increased production of the hormones adrenaline and noradrena- line, which may cause long-term elevation in blood pressure. Renovascular steno- sis is a narrowing of one or both arteries leading to the kidneys. It can cause severe hypertension and irreversible kidney damage. Coarctation of the aorta is a con- genital heart defect of the aorta; it is a narrowing of the aorta that causes the heart to work harder to get blood to flow through the narrow aortic passageway to other organs, which in turn causes an increase in blood pressure.

A 28-year-old male nurse of Hispanic descent reports a history of a cold that resolved 2 weeks ago except for a dry cough and pain over his right cheek that worsens when he bends down. The patient denies fever. He tells the employee health nurse practitioner that he is very allergic to both cephalexin (Keflex) and erythromycin. The vital signs show that the patient's temperature is 99.2 degrees Fahrenheit, a pulse of 72 beats per minute, and a respiratory rate of 12 breaths per minute. Which of the following conditions is most likely? A) Acute sinusitis B) Acute bronchitis C) Fever secondary to the previous viral URI (upper respiratory infection) D) Munchausen's syndrome

A) Acute sinusitis Acute sinusitis symptoms include cough, facial pain, and low-grade fever.

A 50-year-old woman of Irish descent presents with history of lethargy, feeling weak, nausea, anorexia with diarrhea and abdominal pain. The woman's skin appears tanned, but she denies prolonged sun exposure. During physical exam, the skin appears tan with hyperpigmentation of the nipple area, the gums, and the lips. The electrolyte panel reveals hyperkalemia and hyponatremia. She reports craving salty foods. Which of the following is most likely? A) Addison's disease B) Cushing's disease C) Metabolic syndrome D) Cutaneous drug reaction

A) Addison's disease Addison's disease is also known as primary adrenal insufficiency. The most common cause of damage to the adrenal cortex (the outer layer of the gland) is autoimmune destruction. The adrenal cortex produces glucocorticoids (cortisol) and mineralocorticoids (aldosterone). Aldosterone regulates sodium retention and potassium excretion through the kidneys (affects blood pressure). Electrolyte abnormalities are high potassium and low sodium. In primary disease (Addison's), serum cortisol is low, ACTH is high, and serum aldosterone is low. If the patient is not treated, severe stress (illness, accident) may cause an adrenal crisis ("Addisonian" crisis), which can be fatal.

Clara is a 20-year-old college student who reports to the student health clinic with a laceration in her left hand. She tells the nurse practitioner that she cut her hand while working in her garden. Her last Td booster was 5.5 years ago. Which of the following is correct? A) Administer a booster dose of the Td vaccine B) Administer the Td vaccine and the Td immunoglobulin (HyperTet) C) Administer Td immunoglobulin (HyperTet) only D) She does not need any Td immunoglobulin (Hypertet) or a Td booster

A) Administer a booster dose of the Td vaccine Td booster vaccine is recommended for recent injuries if it has been more than 5 years since the last Td.

Atrophic macular degeneration of the aged (AMD) is the leading cause of blindness in the elderly in the United States Which of the following statements is correct? A) It is a slow or sudden painless loss of central vision B) It is a slow or sudden painless loss of peripheral vision C) It is an occlusion of the central retinal vein causing degeneration of the macular area D) It is commonly caused by diabetic retinopathy

A) It is a slow or sudden painless loss of central vision Macular degeneration causes the loss of central vision.

A multigravida who is at 28 weeks of gestation has a fundal height of 29 cm. Which of the following is the best recommendation for this patient? A) Advise the mother that her pregnancy is progressing well B) Order an ultrasound of the uterus C) Refer her to an obstetrician for an amniocentesis D) Recommend bed rest with bathroom privileges

A) Advise the mother that her pregnancy is progressing well From 20-35 weeks' gestation, fundal height should equal weeks gestation. If greater than 2 cm within the gestational week of pregnancy, further testing should be performed to evaluate fetal growth

A nurse practitioner is doing a funduscopic exam on a 35-year-old female during a routine physical exam. He notices that she has sharp disc margins and a yellowish- orange color in the macular area. The ratio of veins to arteries is 3:2. What is the next most appropriate action? A) Advise the patient that she had a normal exam B) Advise the patient that she had an abnormal exam C) Refer the patient to the emergency room D) Refer the patient to an ophthalmologist

A) Advise the patient that she had a normal exam A normal funduscopic exam would include sharp disc margins, with the macula appearing yellow-orange in color. The ratio comparing the artery to vein width is 2:3.

All of the following agents are used to control the inflammatory changes seen in the lungs of asthmatics except: A) Albuterol inhaler (Proventil) B) Triamcinolone (Azmacort) C) Montelukast (Singulair) D) Cromolyn sodium inhaler (Intal)

A) Albuterol inhaler (Proventil) Proventil inhaler is a short-acting bronchodilator and is used for immediate relief of shortness of breath. It acts quickly to dilate the tubes in the lungs. Medications that have steroids and leukotrienes help the inflamed channels to remain open and clear but take longer to get into the system to work.

The Jarisch-Herxheimer reaction is best described as: A) An immune-mediated reaction precipitated by the destruction of a large number of spirochetes due to an antibiotic injection B) Severe chills and elevated blood pressure C) Caused by infection with either Chlamydia trachomatis or gonorrheal infection of the liver capsule D) Associated with certain viral illnesses

A) An immune-mediated reaction precipitated by the destruction of a large number of spirochetes due to an antibiotic injection The Jarisch-Herxheimer reaction is defined as an immune-mediated reaction precipitated by the destruction of a large number of spirochetes due to the antibiotic injection.

When an infant is found to have tufts of fine dark hair on the sacrum, which of the following tests is recommended? A) An ultrasound of the sacrum B) A plain radiograph of the lumbo sacral spine C) No imaging test is necessary D) Genetic testing

A) An ultrasound of the sacrum An infant with tufts of fine dark hair on the sacrum should be evaluated for occult spina bifida. The first imaging test to order is an ultrasound of the lower spine.

You note bony nodules located at the proximal interphalangeal joints on both the hands of your 65-year-old female patient. Which of the following is most likely? A) Bouchard's node B) Heberden's node C) Osteoarthritic nodules D) Tophi deposits

A) Bouchard's node Bony nodules at the proximal interphalangeal joints of the hands are called Bouchard's nodes. Heberden's nodes are nodules on the distal interphalangeal joints. Tophi deposits are seen with gout, where high levels of uric acid in the blood occur and cause nodules in the joint that can eventually destroy the bone. Osteoarthritic nodules develop in the joints of the hands.

A 55-year-old male patient describes to you an episode of chest tightness in his substernal area that radiated to his back while he was jogging. It was relieved immediately when he stopped. Which of the following conditions does this best describe? A) Angina pectoris B) Acute myocardial infarction C) Gastroesophageal reflux disease D) Acute costochondritis

A) Angina pectoris Symptoms of angina pectoris include a pressing, squeezing, or crushing pain, usually in the chest or back, arms, neck, or ear lobes. Weakness, fatigue, and shortness of breath and pain radiating in the arms, shoulders, jaw, neck, and/or back may also occur. Angina pectoris is most commonly brought on by physical exertion, but can also be triggered by stress and other stressors to the body.

Which of the following is recommended by JNC 7 as first-line treatment for hypertension in patients with microalbuminuria? A) Angiotensin-converting enzyme (ACE) inhibitors B) Diuretics C) Calcium channel blockers D) Beta-blockers

A) Angiotensin-converting enzyme (ACE) inhibitors First line of treatment for hypertension in patients with microalbuminuria is ACE inhibitors. ACE inhibitors are renal protective.

A medium-pitched harsh mid-systolic murmur is best heard at the right second ICS of the chest. It radiates into the neck. Which of the following is the correct diagnosis? A) Aortic stenosis B) Pulmonic stenosis C) Aortic regurgitation D) Mitral stenosis

A) Aortic stenosis The murmur associated with aortic stenosis can be auscul- tated as harsh and high pitched in the right second intercostal space; it typically radiates to the carotid arteries and apex.

You can determine a pulse deficit by counting the: A) Apical and radial pulses at the same time, then finding the difference between the two B) Apical pulse first, then the radial pulse, and subtracting to find the difference between the two C) Apical pulse and the femoral pulse at the same time and finding the difference between the two D) Radial pulse first, then counting the femoral pulse, and subtracting to find the difference between the two

A) Apical and radial pulses at the same time, then finding the difference between the two The pulse deficit is determined by counting the apical and radial pulses at the same time, then subtracting to find the difference between the two.

Patients who are being screened for TB and are immunocompromised should be evaluated for anergy. Which of the following is the best description of anergy testing? A) Apply Candida or mumps antigen to the right forearm and the PPD on the left forearm and read results in 48 to 72 hours B) Apply Candida or mumps antigen and PPD on left forearm only and check for a reaction in 24 hours C) Mix the Candida or mumps antigen with the PPD and apply it to both forearms D) Apply the Candida or mumps antigen 24 hours before the PPD on the left forearm

A) Apply Candida or mumps antigen to the right forearm and the PPD on the left forearm and read results in 48 to 72 hours Anergy is when the immune system is unable to perform a healthy, normal immune response when the body is challenged with a particular antigen.

The nurse practitioner notices a gray ring on the edge of both irises of an 80-year- old female. The patient denies visual changes or pain. She reports that she has had the "ring" for many years. Which of the following is the most likely diagnosis? A) Arcus senilis B) Pinguecula C) Peripheral cataracts D) Macular degeneration

A) Arcus senilis Arcus senilis is frequent in older adults. Arcus senilis does not affect vision.

While doing a cardiac exam on a 45-year-old male, you note an irregular rhythm with a pulse rate of 110 beats per minute. The patient is alert and is not in distress. What is the most likely diagnosis? A) Atrial fibrillation B) Ventricular fibrillation C) Cardiac arrhythmia D) First-degree right bundle branch block

A) Atrial fibrillation Atrial fibrillation is chaotic electrical activity of the heart, caused by several ectopic foci in the atria without any signs of distress.

A 13-year-old patient has a throat culture that is positive for strep throat. She reports that her younger brother was recently diagnosed with strep throat and treated. The patient has a severe allergy to penicillin and reports that erythromycin makes her very nauseated. Which of the following antibiotics is the best choice? A) Azithromycin (Zithromax) B) Cephalexin (Keflex) C) Cefuroxime axetil (Ceftin) D) Levofloxacin (Levaquin)

A) Azithromycin (Zithromax) If the patient has a severe penicillin allergy, there is a 10% chance of cross-reactivity to cephalosporins (especially first generation). Because the patient is a child, the levofloxacin is contraindicated. Nausea is a com- mon adverse reaction to erythromycin (it is not an allergic reaction). The best option is to use azithromycin because of its minimal GI adverse effects. Azithromycin has fewer drug interactions compared with other macrolides.

A chest radiograph shows an area of consolidation on the lower lobe. Which of the following conditions is most likely? A) Bacterial pneumonia B) Acute bronchitis C) COPD D) Atypical pneumonia

A) Bacterial pneumonia Consolidation is visible on x-ray when bacterial pneumonia is present. Consolidation is not present in the lungs with bronchitis, COPD, or atypical pneumonia.

What is the most common type of skin cancer? A) Basal cell skin cancer B) Squamous cell skin cancer C) Melanoma D) Actinic keratose

A) Basal cell skin cancer Skin cancer is the most common type of cancer overall, but the most common type of skin cancer is basal cell carcinoma of the skin. Regarding mortality, the skin cancer that causes the most deaths (from skin cancer) is melanomas (65% of skin cancer deaths).

Terazosin (Hytrin), an alpha blocker, is used to treat which of the following conditions? A) Benign prostatic hypertrophy and hypertension B) Chronic prostatitis and atrial fibrillation C) Urinary tract infections and arrhythmias D) Benign prostatic hypertrophy and chronic prostatitis

A) Benign prostatic hypertrophy and hypertension Hytrin is used to treat benign prostatic hypertrophy and hypertension.

Laws governing nurse practitioner authority are determined by the: A) Board of Nursing B) State Legislature C) American Nursing Association D) State Medical Association

A) Board of Nursing Each state has laws establishing a Board of Nursing that regulates the role of the nurse practitioner.

The S1 heart sound is caused by: A) Closure of the atrioventricular valves B) Closure of the semilunar valves C) Opening of the atrioventricular valves D) Opening of the semilunar valves

A) Closure of the atrioventricular valves Closure of the AV valves causes the S1 sound heard when auscultating heart sounds.

A 14-year-old teen is worried that she has not started to menstruate like most of her friends. During the gynecological examination, the nurse practitioner tells the mother, who is in the room with the patient, that her daughter is starting Tanner Stage II. What are the physical exam findings during this stage? A) Breast buds and some straight pubic hair B) Fully developed breasts and curly pubic hair C) Breast tissue with the areola on a separate mound with curly pubic hair D) No breast tissue and no pubic hair

A) Breast buds and some straight pubic hair Tanner Stage II in females includes breast buds and few straight, fine pubic hairs.

The nurse practitioner is educating a new patient with Raynaud's phenomenon about lifestyle changes that have been found helpful in decreasing exacerbations of the disorder. Which of the lifestyle recommendations is false? A) Caffeine-containing drinks and foods such as chocolate have been found helpful with decreasing the incidence of exacerbations B) Wear gloves or mittens during cold weather and be careful when handling frozen foods C) Stop smoking and exercise at least 3 times a week D) Decrease emotional stress and lifestyle stressors

A) Caffeine-containing drinks and foods such as chocolate have been found helpful with decreasing the incidence of exacerbations Lifestyle changes associated with decreasing exacerbations are wearing gloves or mittens during cold weather, taking care with handling frozen foods (wear gloves) and avoiding vasoconstricting agents (caffeine, smoking, cocaine, amphetamines) and emotional stress. Exercise and reducing lifestyle stress are recommended. Usually involves the fingers and/or toes due to severe arteriolar vasospasm causing ischemia. During an exacerbation, the fingers change color into white, blue, and red (think of the American flag as a reminder). Raynaud's is classified either as primary (Raynaud's disease) or secondary (Raynaud's phenomenon). Individuals with secondary Raynaud's have a higher risk of autoimmune disorders such as scleroderma, Sjogren syndrome, and systemic lupus erythematosus. Affects mostly young women (ages 15 to 30 years).

Which of the following antihypertensive medications has beneficial effects for an elderly White female with osteoporosis? A) Calcium channel blocker B) Angiotensin-converting enzyme (ACE) inhibitor C) Beta-blocker D) Diuretic

A) Calcium channel blocker Calcium channel blockers act by blocking the cal- cium channels in the heart muscle and the blood vessels, thereby keeping more calcium in the bones.

The red reflex is elicited by shining a light in the eyes of the infant at an angle with the light about 15 in. away. The nurse practitioner is screening for? A) Cataracts B) Strabismus C) Blindness D) The blinking response

A) Cataracts Screening for cataracts is performed by shining a light into the eyes at an angle, approximately 15 inches away. If the red reflex is not elicited, then cataracts are present.

The cones in the retina of the eye are responsible for: A) Central vision B) Peripheral vision C) Night vision D) Double vision

A) Central vision The cones of the eyes are responsible for central vision.

An adult patient was recently discharged from the hospital with a prescription of clindamycin. The patient reports that he took his last dose yesterday. He presents in the primary care clinic with complaints of the recent onset of watery diarrhea from 10 to 15 times a day with abdominal cramping. He denies fever and chills. Which of the following conditions is most likely in this patient? A) Clostridium difficile-associated diarrhea (CDAD) B) Giardiasis C) Pseudomembranous colitis D) Irritable bowel syndrome

A) Clostridium difficile-associated diarrhea (CDAD) An important risk factor for Clostridium difficile-associated diarrhea (CDAD) and C. difficile colitis is antibiotic therapy and hospitalization. Almost any antibiotic can cause the condition, but the most common are clindamycin, cephalosporins, and fluoroquinolones. Diarrhea can occur during therapy as well as after therapy (5 to 10 days; up to 10 weeks). Pseudomembranous colitis is a complication of C. difficle colitis.

All of the following findings are associated with the secondary stage of an infection by the organism Treponema pallidum except: A) Condyloma acuminata B) Maculopapular rash of the palms and soles C) Lymphadenopathy D) Condyloma lata

A) Condyloma acuminata Findings consistent with the diagnosis of syphilis, caused by the Treponema pallidum organism, include maculopapular rash of the palms and soles, lymphadenopathy, and condyloma lata. Condyloma acuminata (genital warts) are caused by the human papilloma virus (HPV) and spread to oth- ers by skin-to-skin contact.

A possible complication from Bell's palsy is: A) Corneal ulceration B) Acute glaucoma C) Inability to swallow D) Loss of sensation in the affected side

A) Corneal ulceration Due to the paralysis caused by the seventh cranial nerve damage, the eyelid on the affected side may not close voluntarily. This leads to dryness, which can in turn result in ulceration.

All of the following measures have been found to help lower the risk of osteoporosis except: A) Drinking organic juice B) Eating low-fat dairy foods C) Performing weight-bearing exercises D) Vitamin D supplementation

A) Drinking organic juice Vitamin D levels must be sufficient for the body to absorb calcium. Eating foods high in vitamin D and calcium along with calcium and vitamin D supplements are advised to protect the bones and prevent bone loss. Performing weight-bearing exercises daily also increases bone strength.

An elderly Hmong, who is from Thailand, is seen by the nurse practitioner for a follow-up visit. He is accompanied by his eldest daughter. The patient presented 6 weeks ago with complaints of the recent onset of morning headaches. The patient was diagnosed with Stage 2 hypertension and prescribed hydrochlorothiazide, 125 mg tablet once a day. On this visit, Mr. Nguyen tells the nurse practitioner that the new medicine cured the headache and he stopped taking the medicine. What is the best plan to follow during this visit? A) Educate the patient about hypertension, how the medicine works on his body, and the importance of taking his pill daily B) Reassure the patient that he can resume his prescription medicine again the next morning C) Tell the patient that you will lower the dose of the hydrochlorothiazide to 12.5 mg daily D) Speak to the patient in a loud voice and confront him about his behavior

A) Educate the patient about hypertension, how the medicine works on his body, and the importance of taking his pill daily When Hmong (Thailand, Burma, Vietnamese) see a medical doctor for a symptom, they expect to be treated and "cured" of their illness. When the symptoms disappear, many will stop taking the medicine. When medication is to be taken on a long-term basis, it is important to educate the patient (and the patient's family) about the disease (i.e., hypertension), how the medicine works on the body, and the reason why he has to take his medicine daily. Hmong and Vietnamese are very polite and consider speaking in a loud voice, staring, or confrontation to be rude behavior.

The mother of an 8-year-old boy reports the presence of a round red rash on the child's left lower leg. It appeared 1 week after the child returned from visiting his grandparents, who live in Massachusetts. During the skin exam, the maculopapular rash is noted to have areas of central clearing making it resemble a round target. Which of the following is best described? A) Erythema migrans B) Rocky Mountain spotted fever C) Meningococcemia D) Larva migrans

A) Erythema migrans Erythema migrans is an annular lesion that enlarges and with time has central clearing. This is commonly caused by a bite from an infected tick and can cause Lyme disease if not treated appropriately. Lyme disease is most commonly seen in the northeastern United States.

Which of the following would you recommend on an annual basis for an elderly type 2 diabetic? A) Eye exam with an ophthalmologist B) Follow-up visit with a urologist C) Periodic visits to an optometrist D) Colonoscopy

A) Eye exam with an ophthalmologist Elderly patients with type 2 diabetes should have a dilated eye exam done annually by an ophthalmologist. It is also recommended to see a podiatrist once or twice a year. Preventive care also includes receiving a flu shot annually, receiving a pneumovax vaccine if over 60 years of age, and taking a baby aspirin of 81 mg each day.

All of the following statements are correct regarding the Td vaccine except: A) Fever occurs in up to 80% of the patients B) A possible side effect is induration on the injection site C) The Td is given every 10 years D) The DPT and DT should not be given beyond the seventh birthday

A) Fever occurs in up to 80% of the patients Side effects of the Td vaccine include induration at the injection site. Td is given in adults every 10 years. The DPT and DT should not be given beyond 7 years of age. Fever may occur, but studies do not support 80% of patients having fever.

Which of the following conditions is associated with three stages of rashes? A) Fifth disease B) Erythema infectiosum C) Varicella D) Rocky Mountain spotted fever

A) Fifth disease Fifth disease has three stages: Prodromal stage: Begins with symptoms of an upper respiratory infection, such as low-grade fever, headache, chills, and malaise. Second stage: A red rash appears on the cheeks, known as the "slapped cheek" rash. Usually resolves in 2-3 days. Third stage: The rash moves to the arms and legs and becomes a "lacy"-appearing rash that is flat/purple in appearance. May last for a few weeks.

A patient has recently been diagnosed with migraine headache with aura. She wants advice on what she should eliminate from her diet to reduce her risk of headaches. The best response is to advise this patient to avoid: A) Foods with aspartame (Equal) because they can trigger migraines B) Foods with monosodium glutamate because they promote diuresis C) Foods with garlic because they promote hypertension D) Foods with nitrates because they do not induce flushing

A) Foods with aspartame (Equal) because they can trigger migraines Dietary triggers for migraine headaches include foods that contain aspartame, tyramine, nitrites, MSG, and/or red wine. These foods are thought to cause a change in the blood vessels and increased blood flow to the brain.

An adolescent male presents to the nurse practitioner for a wellness exam. The patient's face is long and narrow with a prominent forehead and chin, and he has large ears. The mother reports that her son has intellectual disabilities and autistic disorder. Which of the following conditions is being described? A) Fragile X disorder B) Marfan's syndrome C) Turner's syndrome D) Down syndrome

A) Fragile X disorder Fragile X syndrome is the most common form of inherited intellectual disability. The disorder is associated with a higher incidence of autism (especially boys). Males are affected more severely than females. The facial features can vary. The classic facie is a long, narrow face with a prominent forehead and chin and large ears. Other features include hyperlaxity of the joints, flat feet, high arched palate, and others. Definitive diagnosis is by genetic testing.

Erythromycin 200 mg with sulfisoxazole 600 mg suspension (Pediazole) is contraindicated in which of the following conditions? A) G6PD deficiency anemia B) Lead poisoning C) Beta thalassemia minor D) B12 deficiency anemia

A) G6PD deficiency anemia Glucose-6-phosphate dehydrogenase (G6PD) defi- ciency is a hereditary condition that occurs when the red blood cells break down, causing hemolysis, due to a missing or lack of a sufficient enzyme that is needed to help the red blood cells work efficiently. Certain foods and medications may trigger this reaction and hemolysis will occur. Some of these medications include antimalarial drugs, aspirin, nitrofurantoin, NSAIDs, quinidine, quinine, and sulfa medications.

Erythromycin 200 mg with sulfisoxazole 600 mg suspension (Pediazole) is contraindicated in which of the following conditions? A) G6PD deficiency anemia B) Lead poisoning C) Beta thalassemia minor D) B12 deficiency anemia

A) G6PD deficiency anemia Glucose-6-phosphate dehydrogenase (G6PD) deficiency is a hereditary condition that occurs when the red blood cells break down, causing hemolysis, due to absence or lack of sufficient G6PD, an enzyme that is needed to help the red blood cells work efficiently. Certain foods and medications may trigger this reaction. Some of the medications include antimalarial drugs, aspirin, nitrofurantoin, nonsteroidal anti-inflammatory drugs (NSAIDs), quinidine, quinine, and sulfa medications.

A 40-year-old female is positive for anti-hepatitis C virus (HCV). Which test is appropriate for follow-up? A) HCV RNA B) HCV antibodies C) HCV core antigen D) Hepatitis C surface antigen

A) HCV RNA HCV RNA is performed following a positive anti-hepatitis C virus test.

Which of the following tests would you recommend to patients to confirm the diagnosis of beta thalassemia or sickle cell anemia? A) Hemoglobin electrophoresis B) Bone morrow biopsy C) Peripheral smear D) Reticulocyte count

A) Hemoglobin electrophoresis Patients with the diagnosis of beta thalassemia and/or sickle cell anemia would be screened using hemoglobin electrophoresis to identify the blood disorder.

A middle-aged Black male complains of a history of outbreaks of painful large nodules and pustules on both his axillae that resolve after treatment with antibiotics. On physical examination, the nurse practitioner notices large red nodules that are tender to palpation. In addition, several pustules are present along with multiple scars on the skin. The nurse practitioner advises the patient that the condition is caused by a bacterial infection of the sweat glands in the axillae. Which of the following conditions is being described? A) Hidradenitis suppurativa B) Severe nodular acne C) Granuloma inguinale D) Cat scratch fever

A) Hidradenitis suppurativa Hidradenitis suppurativa is a chronic skin condition, commonly found in the apocrine glands in the axilla and/or groin, that causes painful nodules under the skin. These abscesses tend to open and drain fluid and pus. Significant scarring of the skin may result from these outbreaks.

A small abscess on a hair follicle on the eyelid is called: A) Hordeolum B) Pterygium C) Pinguecula D) Ptosis

A) Hordeolum Hordeolum is a painful, acute bacterial infection of the hair follicle on the eyelid. Pinguecula is a thickening of the bulbar conjunctiva, located on the inner and outer margins of the cornea. Pterygium is a thickening of the conjunctiva located on the nasal or temporal cornea. Pinguecula and pterygium are both caused by the UV light of long-term sun exposure. Sunglasses with UV protection are recommended to prevent damage to the conjunctiva. Ptosis is the drooping of the upper eyelid.

A positive Chvostek's sign is associated with: A) Hypocalcemia B) Hypernatremia C) Hypokalemia D) Hyperkalemia

A) Hypocalcemia Chvostek's sign is contraction of the facial muscles when the facial nerve is tapped briskly in front of the ear (anterior to the auditory canal). Low calcium levels cause tetany and neuromuscular disturbances. Acute hypocalcemia with symptoms (tetany, weakness, arrhythmias) should be referred to the emergency department. Conditions such as acute or chronic renal failure, Vitamin D or magnesium deficiency, or acute pancreatitis increase the risk of hypocalcemia

What is the name of the immune process that is responsible for anaphylactic reactions? A) IgE-mediated reaction B) Serious allergic reaction C) Antibody reaction D) Atopic reaction

A) IgE-mediated reaction Anaphylaxis is an immunoglobulin E (IgE)-mediated reaction (also known as IgE-mediated type 2). IgE immediate reactions such as anaphylaxis trigger mast cell degranulation and release of potent mediators such as histamines, leukotrienes, and prostaglandins, which immediately induce con- striction of smooth muscle, swelling, vasodilation, and other pathologic changes in the body that may be fatal.

All of the following are considered selective serotonin reuptake inhibitors (SSRIs) except: A) Imipramine (Elavil) B) Fluoxetine (Prozac) C) Sertraline (Zoloft) D) Paroxetine (Paxil CR)

A) Imipramine (Elavil) Prozac, Zoloft, and Paxil CR are selective serotonin reuptake inhibitors. Elavil is a tricyclic antidepressant.

A 44-year-old female who is undergoing treatment for infertility complains of not having a menstrual period for a few months. The night before, she started spotting and is now having cramp-type pains in her pelvic area. Her blood pressure (BP) is 160/80, the pulse rate is 110, and she is afebrile. Her labs reveal a mild anemia with mild leukocytosis. On pelvic exam, the uterine fundus is found above the symphy- sis pubis. The cervical os is dilated at 3 cm. Which of the following is most likely? A) Inevitable abortion B) Threatened abortion C) Incomplete abortion D) Acute pelvic inflammatory disease

A) Inevitable abortion Inevitable abortion is defined as vaginal bleeding with pain and cervical dilation and/or effacement. Threatened abortion is defined as vaginal bleeding with absent or minimal pain and a closed, long, and thick cervix. Incomplete abortion is moderate to diffuse vaginal bleeding, with the passage of tissue and painful uterine cramping or contractions. Acute pelvic inflammatory disease is a sudden onset of inflammation and pain that affects the pelvic area, cervix, uterus, and ovaries, which is caused by infection.

The Phalen test is used to evaluate: A) Inflammation of the median nerve B) Rheumatoid arthritis C) Degenerative joint changes D) Chronic tenosynovitis

A) Inflammation of the median nerve The Phalen test is used to identify inflammation of the median nerve seen with carpal tunnel syndrome. Phalen's test is performed by examining the wrist with full flexion of the wrist for 60 seconds. Positive findings are reproduction of the symptoms such as numbness and tin- gling of the thumb, index finger, and middle finger areas.

All of the following vaccines are contraindicated in pregnant women except: A) Influenza B) Mumps C) Varicella D) Rubella

A) Influenza Influenza vaccine is recommended for all pregnant patients for prevention, due to the decrease in immune status during pregnancy. The vaccine is safe to use during pregnancy. The other vaccines are live viruses and are contraindicated during pregnancy.

Joe, a 20-year-old Asian male, reports pain in his right knee after twisting it playing soccer. The injured knee locks up when he attempts to straighten his leg. Which of the following is most likely? A) Injury to the meniscus of the right knee B) Injury to the patella of the right knee C) Injury to the ligaments of the right knee D) Rupture of the quadriceps tendon

A) Injury to the meniscus of the right knee Pain in the knee after twisting it and "locking up" while attempting to straighten the leg is most consistent with an injury to the meniscus.

What is the significance of a positive Lachman sign? A) Instability of the affected knee caused by damage (i.e.,rupture) to the anterior cruciate ligament (ACL) of the knee B) Posterior cruciate ligament laxity which may cause locking of the affected knee C) Achilles tendon rupture D) Patellar tendon rupture

A) Instability of the affected knee caused by damage (i.e., rupture) to the anterior cruciate ligament (ACL) of the knee A positive Lachman sign is highly suggestive of damage to the anterior cruciate ligament of the knee. The anterior drawer sign may also be positive. There is laxity of the abnormal knee joint (compared with the normal knee). The Lachman test or maneuver is considered more sensitive for ACL damage compared with the anterior drawer test.

Which of the following foods would you advise a new mother to introduce to her 6-month-old infant first? A) Iron-fortified rice cereal B) Plain rice cereal C) Iron-fortified pureed chicken meat D) Plain pureed carrots

A) Iron-fortified rice cereal At 6 months of age, it is recommended that the infant be fed iron-fortified rice cereal. Introducing only one new food at a time is recommended; in case an allergy does develop, you will be able to identify which food caused the allergy.

What is the name of the physiologically active compound that is derived from soybeans? A) Isoflavones B) Estrogen C) Progesterone D) Resveratrol

A) Isoflavones Soy isoflavones mimic the action of estrogen in the body. It is derived from soybeans and soybean products (soy milk, tofu). Resveratrol is found in red grapes and red wine. It is thought to help prolong life and is considered one of the reasons that the Mediterranean diet increases longevity

You are reviewing the bilirubin level on a 3-day-old full-term neonate. You note that it is 10 mg/dL. The infant has a slight yellow color to his skin, mucous membranes, and sclera. The infant is feeding well, is not irritable, and has 8 to 10 wet diapers per day. Which of the following is a true statement? A) Keep on monitoring the infant's bilirubin level until it returns back to normal in about 1 week B) Recommend that the infant be treated with phototherapy 10 minutes a day until the bilirubin level is back down to a normal range C) Refer the infant to a neonatologist as soon as possible D) Refer the infant to the neonatal intensive care unit

A) Keep on monitoring the infant's bilirubin level until it returns back to normal in about 1 week Bilirubin is excreted through the urine and feces. Increased fluids and wetting 8-10 diapers a day is sufficient fluid intake/excretion to help bring down the bilirubin level. Levels should continue to be monitored and should improve in approximately 1 week.

A patient who recently returned from a vacation in Latin America complains of a severe headache and stiff neck that are accompanied by a high fever for the past 12 hours. While examining the patient, the nurse practitioner flexes both the patient's hips and legs and then tells the patient to straighten them against resistance. The name of this test is: A) Kernig's maneuver B) Brudzinski's maneuver C) Murphy's sign D) Homan's sign

A) Kernig's maneuver Kernig's maneuver is performed by having the patient flex both hips and legs and then straighten the legs against resistance, testing for meningitis. Flexion of hip/knees is a positive sign for meningitis. Brudzinski's maneuver is performed by placing the patient's hands behind his head, and gently tucking chin to chest. Murphy's sign is elicited by having the patient inspire with the tips of the examiner's fingers placed on the right upper quadrant, at the liver border, under the ribs. Pain on inspiration is suggestive of cholecystitis. Homan's sign is flexion of the foot, causing pain in the posterior calf area, suggestive of a DVT.

The Lachman maneuver is used to detect which of the following? A) Knee instability B) Nerve damage of the knee due to past knee injuries C) The integrity of the patellar tendon D) Tears on the meniscus of the knee

A) Knee instability The Lachman maneuver is a test performed to assess for knee instability. Knee instability indicates a tear of the anterior cruciate ligament.

A red, raised serpiginous-shaped rash is noted by the nurse practitioner on the right foot of a 4-year-old child brought in for a preschool physical by the mother. The child complains of severe itch and keeps scratching the lesion. The mother reports that the child frequently played in the yard without shoes or sandals on. Which of the following is most likely? A) Larva migrans B) Erythema migrans C) Tinea pedis D) Insect bites

A) Larva migrans Larva migrans affects the intestines of dogs/cats and is caused by the eggs of parasites (worms) from the animals' intestine. Children are at high risk of developing this if they come in contact with dirt that is contaminated with dog/cat feces. Eating foods that are grown in contaminated soil and/or raw liver are other means of transfer of the infection. After the eggs hatch, the parasite can migrate to other organs of the body if left untreated

Human papilloma virus (HPV) infection of the larynx has been associated with: A) Laryngeal neoplasia B) Esophageal stricture C) Cervical cancer D) Metaplasia of esophageal squamous cells

A) Laryngeal neoplasia HPV infection of the larynx has been associated with laryngeal neoplasia; HPV infection of the cervix is associated with cervical cancer.

The following abnormal lab results may be seen in patients with acute mononucleosis except: A) Lymphocytosis and/or atypical lymphocytes B) Positive EBV titers for immunoglobulin (Ig) M and Ig G C) Elevated liver function tests D) Elevated creatinine and BUN

A) Lymphocytosis and/or atypical lymphocytes Common symptoms of mononucleosis include positive EBV titers, elevated liver enzymes, and elevated creatinine and BUN.

Which of the following is considered a spiritual illness by Latinos/Hispanics that can cause symptoms such as loss of appetite, crying, diarrhea, and weakness or death among infants and small children? A) Mal ojo or mal de ojo B) Chronic nightmares C) Trabajo D) Malo

A) Mal ojo or mal de ojo Mal ojo or mal de ojo is a spiritual illness that can cause symptoms such as loss of appetite, crying, diarrhea, colic, fear, weakness, or death. A curandero or curandera is usually consulted and does spiritual cleansing of the patient. It may take several cleansings (limpia) to cure the patient. Trabajo means "work" and malo means "bad"; these are being used as distractors.

Which of the following maneuvers is contraindicated in patients with acute prostatitis? A) Massaging the infected prostate B) Serial urine samples C) Rectal exams D) Palpation of the epididymis

A) Massaging the infected prostate Massaging the infected prostate is contraindicated for acute prostatitis because the massage can spread the bacteria from the prostate into the blood system and lead to sepsis.

Patient confidentiality is breached when: A) Medical information is given to a spouse B) Records are subpoenaed C) Reports are sent to the Public Health Department D) Records are released to insurance companies

A) Medical information is given to a spouse Patient confidentiality is breached when medical information is given to a spouse or any other individual without consent of the patient.

A 25-year-old woman complains of dysuria, severe vaginal pruritis, and a malodorous vaginal discharge. Pelvic examination reveals a strawberry-colored cervix and frothy yellow discharge. Microscopic exam of the discharge reveals mobile organisms that have flagella. The correct pharmacologic therapy for the condition is: A) Metronidazole (Flagyl) B) Ceftriaxone sodium (Rocephin) C) Doxycycline hyclate (Vibramycin) D) Clotrimazole (Gyne-Lotrimin)

A) Metronidazole (Flagyl) Trichomoniasis symptoms include dysuria, severe vaginal pruritis, and malodorous vaginal discharge. Wet prep will show trichomonads that are pear-shaped and have several flagella (whiplike tails) at one end. CDC recommendation for treatment is metronidazole.

Medicare Part A will pay for all of the following services except: A) Minor surgery in a walk-in surgical center B) Plastic surgery to repair facial damage from a burn C) Kidney transplantation D) Medical supplies and drugs that are used while the patient is in the hospital

A) Minor surgery in a walk-in surgical center Medicare Part A will pay for medically necessary inpatient care and supplies. Therefore, any type of surgery that is done in outpatient settings such as a walk-in surgical center will not be reimbursed. If plastic surgery is medically necessary (i.e., plastic surgery to repair facial damage from a burn), then it will be reimbursed. Costs for organ transplantation such as kidney transplants are reimbursed.

12-23. Which of the following is the most appropriate action to take for a 21-year-old male presenting with the symptom of "hearing voices"? A. Complete a mental status exam (MSE) B. Order labs for substance use C. Complete a physical examination D. Call the crisis team

A. Complete a mental status exam (MSE)

Precocious puberty is defined as: A) Onset of puberty before age 8 in girls and age 9 in boys B) Onset of puberty before age 9 in girls and age 10 in boys C) Onset of puberty before age 10 in girls and age 11 in boys D) Onset of puberty at an earlier age than the child's parent(s)

A) Onset of puberty before age 8 in girls and age 9 in boys The definition of precocious puberty is the onset of puberty before age 8 in girls and age 9 in boys.

Jenny, a 21-year-old, complains to you of a 1-week episode of dysuria, frequency, and a strong odor to her urine. This is her second episode of the year. What is the most appropriate follow-up for this patient? A) Order a urinalysis and urine for culture and sensitivity (C&S) and treat the patient with antibiotics B) Order a urine C&S and hold treatment until you get the results from the lab C) Treat the patient with a 7-day course of antibiotics and order a urine for culture and sensitivity (urine C&S) now and after she completes her antibiotics D) Treat the patient with a stronger drug such as ofloxacin (Floxin) for 10 days

A) Order a urinalysis and urine for culture and sensitivity (C&S) and treat the patient with antibiotics The best treatment for this patient is to order the urinalysis and urine C&S to identify the organism causing the infection. Treatment may begin while waiting for the culture results. However, occasionally treatment may have to be changed after the culture/sensitivity results return, due to resistance to the antibiotic used.

Stella works in the housekeeping department of a hospital. She presents to the employee health clinic with a complaint of a needlestick to her left thumb. The needle was in one of the garbage bags from the emergency room. The patient had a little bleeding that stopped spontaneously. Which of the following is the next step? A) Order an enzyme-linked immunosorbent assay (ELISA) test as soon as possible B) Recommend a tetanus booster in 1 week C) Offer the patient hepatitis B immunoglobulin D) Order a chest x-ray

A) Order an enzyme-linked immunosorbent assay (ELISA) test as soon as possible Employee health clinic protocols for needlesticks recommend ordering an ELISA test as soon as possible to establish baseline blood work for the employee.

A second triple screen on a 35-year-old primigravida reveals abnormally low lev- els of the alpha fetoprotein and estriol and high levels of human chorionic gonad- otropin. Which of the following interventions is the best choice for this patient? A) Order an ultrasound B) Order a computed tomography (CT) scan of the abdomen C) Order a 24-hour urine for protein clearance D) Assess for a history of illicit drug or alcohol use

A) Order an ultrasound Abnormally low levels of alpha fetoprotein and estriol and high levels of human chorionic gonadotropin are abnormal during pregnancy. An ultrasound should be ordered to further evaluate the fetus for characteristics of Down syndrome and/or fetal demise.

A charitable foundation plans to build a community youth center in a large urban area with a history of gang violence. What type of health prevention activity is being done in this area? A) Primary prevention B) Secondary prevention C) Tertiary prevention D) Health prevention

A) Primary prevention A community youth center with good staffing can be an effective method of drawing the children out of the streets into a safer environment. It can reduce the risk of children becoming victims of gang violence. In addition, staff members can serve as role models or mentors for the older children.

All of the following are included in the criteria used to diagnose patients with AIDS except: A) Profound fatigue B) Thrush C) Kaposi's sarcoma D) Hairy leukoplakia of the tongu

A) Profound fatigue Criteria used to diagnose AIDS include hairy leukoplakia of the tongue, Kaposi's sarcoma, and thrush.

A 60-year-old female truck driver presents to the outpatient urgent care clinic of a hospital complaining of the worsening of her low back pain the past few days. She describes the pain as "sharp and burning" and points to the left buttock. She reports that the pain started on the mid-buttock of the left leg and recently started to go down the lateral aspect of the leg toward the top of the foot. During the physical exam, the ankle jerk and the knee jerk reflex is 1+ on the affected leg and 2+ on the other leg. The pedal, posterior tibialis, and popliteal pulses are the same on both legs. Which of the following tests should the nurse practitioner consider for this patient? A) Order both a plain radiograph and computed tomography (CT) scan of the spine as soon as possible B) Write a prescription for ibuprofen 800 mg PO QID with a muscle relaxant and advise the patient to follow up with her primary care provider within 3 days C) Refer the patient to an orthopedic surgeon D) Ordering an imaging study of the spine is premature because the majority of low back pain cases resolve within 10 to 12 weeks

A) Order both a plain radiograph and computed tomography (CT) scan of the spine as soon as possible The patient has a severe case of sciatica that is worsening quickly (may progress to cauda equina). In addition, older age and an abnormal neurologic exam are red flags for potentially serious underlying pathology (osteoporosis fracture, cancer, infection, spondylolisthesis). Because the patient's symptoms are worsening, a CT scan is the best choice. A plain radiograph can only detect bony pathology, but a CT scan can detect nerve root compression, herniated disc, cancer, and spinal stenosis (narrowing of the spinal canal). In addition, the patient needs to follow up with a neurologist as soon as possible.

The atypical antipsychotic drugs have many adverse effects. Which of the following side effects is most likely to be seen with this drug class? A) Orthostatic hypotension and sedation B) Malignant hypertension and headache C) Skin hyperpigmentation and alopecia D) Severe anxiety and increased appetite

A) Orthostatic hypotension and sedation Orthostatic hypotension and sedation are common side effects of atypical antipsychotics such as olanzapine (Zyprexa), quetiapine (Seroquel), and risperidone (Risperdal). It is also a common side effect of the older antipsychotics like haloperidol (Haldol). Antipsychotics do not cause severe anxiety and hyperphagia (increased appetite). They lower anxiety and cause sedation, sleepiness, anorexia, and hypotension and increase the risk of sud- den death in frail elders

The positive signs of pregnancy include: A) Palpation of the fetus and auscultation of the fetal heart tones by the nurse practitioner B) Palpation of the fetus and a positive quantitative serum pregnancy test C) Fetal heart tones and a positive quantitative serum pregnancy test D) Fetal heart tones and feeling of movement of the baby by the mother

A) Palpation of the fetus and auscultation of the fetal heart tones by the nurse practitioner Presumptive signs of pregnancy are symptoms experienced by the woman, such as amenorrhea, breast tenderness, nausea/vomiting, fatigue, and increased urinary frequency. Probable signs of pregnancy are signs detected by the examiner, such as an enlarged uterus. Positive signs of pregnancy are direct evidence of pregnancy such as audible fetal heart tones or cardiac activity on ultrasound.

Which of the following medications is indicated for the treatment ofobsessive-compulsive disorder? A) Paroxetine (Paxil CR) B) Haldoperidol (Haldol) C) Lorazepam (Xanax) D) Imipramine (Elavil

A) Paroxetine (Paxil CR) Antidepressants are the most common medications used for OCD. Those antidepressants that are approved for OCD by the Food and Drug Administration (FDA) include clomipramine (Anafranil), fluvox- amine (Luvox), fluoxetine (Prozac), paroxetine (Paxil, Pexeva), and sertraline (Zoloft).

A 30-year-old chef complains of pruritic hives over her chest and arms but denies difficulty swallowing or breathing. She reports a family history of allergic rhinitis and asthma. Which of the following interventions is most appropriate? A) Perform a complete and thorough history B) Prescribe an oral antihistamine such as diphenhydramine 25 mg PO QID C) Give an injection of epinephrine 1:1000 intramuscularly stat D) Call 911

A) Perform a complete and thorough history Prior to prescribing medications, a complete and thorough history must be performed to determine possible causes of hives. The patient denied difficulty with swallowing and breathing, so there was no medical emergency to require calling 911.

Which of the following is most likely to be found in patients with a long-standing case of iron deficiency anemia? A) Pica B) Fatigue C) Pallor D) Irritability

A) Pica

Human chorionic gonadotropin (hCG) is produced by the: A) Placenta B) Hypothalamus C) Anterior pituitary D) Ovaries

A) Placenta HCG is produced by the placenta.

A 10-year-old male who was recently accepted into his school's soccer team has a history of exercise-induced asthma. The child wants to know when he should take his albuterol inhaler. The nurse practitioner would advise the patient: A) Premedicate himself 20 minutes prior to starting exercise B) Wait until he starts to exercise before using the inhaler C) Premedicate 60 minutes before starting exercise D) Wait until he finishes his exercise before using his inhaler

A) Premedicate himself 20 minutes prior to starting exercise Exercise-induced asthma is best controlled by using the Proventil inhaler (bronchodilator) approximately 20 minutes prior to exercise, to prevent vasospasm of the bronchioles and shortness of breath with exercise. These bronchodilators usually last approximately 4 hours. They also work quickly to open up the bronchioles if an acute attack/shortness of breath occurs

3-6. Daily exercise and staying current with routine-recommended immunizations are examples of what level of prevention? A. Primary prevention B. Secondary prevention C. Tertiary prevention D. Primordial prevention

A. Primary prevention

Which of the following is recommended as first-line treatment for essential tremor? A) Propranolol (Inderal) B) Phenytoin (Dilantin) C) Amitriptyline (Elavil) D) Fluoxetine (Prozac)

A) Propranolol (Inderal) Propranolol (Inderal) is approved for "treatment" of essential tremor. It helps control the symptoms. Essential tremors are permanent and cannot be cured. Before prescribing, order an EKG. Do not use beta- blockers if a patient has second- or third-degree heart block or chronic lung disease.

A 75-year-old patient has a history of benign prostatic hypertrophy (BPH). During the physical exam, which of the following clinical findings during the prostatic exam is correct? A) Prostate feels firm and uniformly enlarged B) Prostate feels boggy and enlarged C) Prostate feels harder than normal D) Presence of tender nodules

A) Prostate feels firm and uniformly enlarged The prostate should feel firm and will be uniformly enlarged. A boggy and warm prostate is present with acute prostatitis.

Which bacterium is the most common pathogen seen in otitis externa infections? A) Pseudomonas aeroginosa B) Streptococcus pyogenes C) Haemophilus influenza D) Moraxella catarrhalis

A) Pseudomonas aeroginosa The most common bacterium is Pseudomonas. The second most common bacterium is Staphylococcus aureas. Polymyxin and neomycin combination ear drops (Cortisporin) are the first-line treatment for otitis externa. Other ear drops that are also effective are the quinolone ear drops (ofloxacin, ciprofloxacin topical drops).

A 55-year-old woman who has type 2 diabetes is concerned about her kidneys. She has a history of 3 urinary tract infections within the past 8 months. She denies dysuria and frequency at this visit. Which of the following is the best initial course to follow? A) Recheck the patient's urine and order a urine for culture and sensitivity B) Order an IVP (intravenous pyelogram) C) Advise the patient to follow up with a urologist D) Evaluate the patient for a possible kidney infection

A) Recheck the patient's urine and order a urine for culture and sensitivity A urinary tract infection is defined as the presence of 100,000 organisms per mL of urine in asymptomatic patients or greater than 100 organisms per mL or urine with pyuria (more than 7 WBCs/mL) in a symptomatic patient

Lifestyle modifications are an important aspect in the treatment of hypertension. Which of the following statements is incorrect? A) Reduce intake of sodium, potassium, and calcium B) Reduce intake of sodium and saturated fats C) Exercise at least three to four times per week D) Maintain an adequate intake of potassium, magnesium, and calcium

A) Reduce intake of sodium, potassium, and calcium Lifestyle modifications for hypertension include exercise 3-4 times a week, diet modifications of reduced intake of sodium and saturated fats, and adequate dietary intake of potassium, magnesium, and calcium

A 28-year-old male nurse tells the employee health nurse practitioner that he was treated for a UTI twice the previous year. The patient denies fever, flank pain, or urethral discharge during the visit. Which of the following is the best follow-up for this patient? A) Refer the patient to a urologist B) Prescribe the patient ofloxacin (Floxin) for 2 weeks instead of 1 week C) Advise the patient that he needs to void every 2 hours when awake D) Refer the patient to the local ED, because he has a very high risk of sepsis

A) Refer the patient to a urologist It is recommended that frequent UTIs in males be referred to a urologist for evaluation and treatment.

Which of the following clinical findings can mimic a case of testicular torsion but is not considered an emergent condition? A) The "blue dot" sign B) One swollen testicle with yellow-colored penile discharge C) An acute onset of dysuria and frequency D) Avaricocele

A) The "blue dot" sign The "blue dot" sign is located underneath the skin of the testicle and appears as a round, blue to purple mass. It is not an emergent condi- tion. Also known as the torsion of testicular appendage.

An 87-year-old male reports to the nurse practitioner that his grandson locks him in the bedroom when the grandson goes out of the house and sometimes with- holds food from him if he does not give the grandson spending money. The patient appears frail, with poor grooming, and has a strong odor of urine on his clothing. Which of the following is the best action for the nurse practitioner to take? A) Report the patient's grandson for elder abuse to protective health services of the state B) Call the grandson from the waiting room and educate him about the importance of proper grooming for his grandfather C) Advise the grandson that if the patient reports the same complaints the next time he is seen by the nurse practitioner, he will be reported for elder abuse to the state authorities D) Advise the patient that he should call his son as soon as possible

A) Report the patient's grandson for elder abuse to protective health services of the state Speaking with the grandson and warning him about elder abuse and reporting may result in harm to the patient and/or refusal to return to the clinic in the future for follow-up of the patient. Option D is a vague answer (call his son as soon as possible for what?).

All of the following foods are best avoided by individuals with celiac sprue except: A) Rice cereal B) Blueberry muffins C) Organic wheat bread D) Rye bread

A) Rice cereal Patients can eat any food except those that contain the protein gluten. Foods containing wheat, barley, and rye should be avoided.

The nurse practitioner examines a 4-week-old boy whose mother reports that he has cried for at least 3 hours a day at the same time of day since birth. What is the main goal in the clinical evaluation of this infant? A) Rule out any physiologic cause for the crying spells B) Make sure that the infant is well clothed C) Evaluate the environment D) Order laboratory and diagnostic testing

A) Rule out any physiologic cause for the crying spells For infants that cry for several hours during the day, ruling out a physiological problem that may be causing the distress is recommended.

The bell of the stethoscope is best used for auscultation of which of the following? A) S3 and S4 and low-pitched tones B) S3 and S4 only C) S1 and S2 and high-pitched tones D) S1 and S2 only

A) S3 and S4 and low-pitched tones The bell of the stethoscope is used to auscultate the S3, S4, and low-pitcehd tones. Remember the word "below" (Bell-Low).

Which of the following drugs is most likely to cause sexual dysfunction in males? A) SSRIs B) ACE inhibitors C) Amphetamines D) Atypical antidepressants

A) SSRIs A common side effect of SSRIs (e.g., Prozac, Paxil, Zoloft) is sexual dysfunction in males. For depressed males, atypical antidepressants such as bupropion (Wellbutrin) cause less sexual dysfunction.

A 40-year-old nurse complains of a new onset of back pain secondary to her job on the medical-surgical floor of a hospital. She reports lifting some obese patients while working the previous night shift. She reports to the worker's compensation clinic where she was referred. She describes the pain as starting in her right but- tocks area and radiating down the back of her thigh. It becomes worse when she sits down for long periods. You would suspect: A) Sciatica B) Acute muscle spasm C) Cauda equina syndrome D) Acute muscle strain

A) Sciatica Sciatica is defined as pain that begins in the buttock area and radiates down one leg. Other symptoms include weakness and tingling sensation. Acute muscle spasm and strain do not cause tingling down the leg. Cauda equina syndrome is an emergent issue, in which there is neurological involvement and patient complaints of weakness and loss of bladder and bowel control.

On auscultation of the chest, a split S2 is best heard at: A) Second intercostal space, right sternal border B) Second intercostal space, left sternal border C) Fifth intercostal space, midclavicular line D) Fourth intercostal space, left sternal border

A) Second intercostal space, right sternal border A split S2 is best heard at the second ICS, right of the sternal border.

An elderly man of Mediterranean descent has a routine CBC done for an annual physical. The following are his lab test results: hemoglobin is 12.0 g/dL, hematocrit is 39%, and MCV is72 fL. His PSA result is 3.2 ng/mL. UA shows no leukocytes and few epithelial cells. Which of the following laboratory tests are indicated for this patient? A) Serum iron, serum ferritin, total iron-binding capacity (TIBC), and the red cell distribution width (RDW) B) Serum vitamin B12 and folate level with a peripheral smear C) CBC with white cell differential and UA D) Urine culture and sensitivity with microscopic exam of the urine

A) Serum iron, serum ferritin, total iron-binding capacity (TIBC), and the red cell distribution width (RDW)

If left untreated, Zollinger-Ellison syndrome can cause which of the following? A) Severe ulceration of the stomach or duodenum B) Toxic megacolon C) Chronic diarrhea D) Malabsorption of fat-soluble vitamins

A) Severe ulceration of the stomach or duodenum Untreated Zollinger-Ellison syndrome can lead to severe ulceration of the stomach or duodenum. Zollinger- Ellison syndrome occurs when tumors (gastrinomas) in the intestine, pancreas, or lymph nodes near the pancreas produce excessive amounts of gastrin, which in turn will increase the amount of acid produced by the stomach. High amounts of acid in the stomach will produce ulceration of the stomach or duodenum.

Which of the following drugs can increase the risk of bleeding in patients who are on anticoagulation therapy with warfarin sodium (Coumadin)? A) Trimethoprim/sulfamethoxazole (Bactrim DS) B) Carafate (Sucralfate) C) Losartan (Cozaar) D) Furosemide (Lasix)

A) Trimethoprim/sulfamethoxazole (Bactrim DS) Coumadin interacts with Bactrim and will increase the risk of bleeding; therefore, it is contraindicated.

Your newly diagnosed diabetic patient reports to you that she had severe hives and swollen lips when she took Bactrim for a bladder infection 2 months ago. Which of the following statements is correct? A) She cannot take any pills in the sulfonylurea class B) She can take some of the pills in the sulfonylurea class C) She can take any of the pills in the sulfonylurea class D) None of the above

A) She cannot take any pills in the sulfonylurea class If an allergic reaction occurs with Bactrim, the patient must not take any medications in the sulfonylurea class.

A 15-year-old White male is brought in by his father for a physical exam. He is concerned that his son is "too short" for his age. The father reports that when he was the same age, he was much taller. His son wants to try out for the football team, but the father is concerned because his son might be "too short" to join. Which of the follow- ing physical exam findings is worrisome? A) Small smooth testicles with no pubic or facial hair B) Smooth testicles with rugated scrotum that is a darker color than his normal skin color C) Smooth testicles with coarse and curly pubic hair D) Straight pubic and axillary hair with a long thin penis

A) Small smooth testicles with no pubic or facial hair Small, smooth testicles with no pubic or facial hair (Tanner Stage I) is a worrisome finding at the age of 14 years because it signifies that the boy is not in the pubertal stage yet. The average age of onset of puberty among boys is 12 years (range 10 to 14 years). The maximum growth spurt in boys occurs about 2 years after the onset of puberty. Boys start about 1 year later than girls and continue to grow until their early 20s (college).

Carol, a 30-year-old type 2 diabetic, is on regular insulin and lente insulin in the morning and in the evening. She denies changes in her diet or any illness, but recently started attending aerobic classes in the afternoon. Because of her work- outs, her blood sugars have dipped below 50 mg/ dL very early in the morning. Her fasting blood sugar before breakfast is now elevated and higher than normal. Which of the following is best described? A) Somogyi phenomenon B) Dawn phenomenon C) Raynaud's phenomenon D) Insulin resistance

A) Somogi phenomenon The Somogi phenomenon is when nocturnal hypoglycemia (2 a.m to 3 a.m.) stimulates the liver to produce glucagon to raise the blood sugar. The fasting blood glucose levels will be elevated from this glucagon production.

The most common type of skin malignancy is: A) Squamous cell cancer B) Basal skin cancer C) Melanoma D) Dysplastic nevi

A) Squamous cell cancer Squamous cell cancer is the most frequent type of skin cancer. It can occur in normal skin or inflamed skin. Common areas involved are those which are exposed to the sun or UV light. The earliest form of squamous cell cancer is known as Bowen's disease, in which the cancer has not involved the tissue nearby.

The bacterium responsible for the highest mortality in patients with community- acquired pneumonia is: A) Streptococcus pneumoniae B) Mycoplasma pneumoniae C) Moraxella catarrhalis D) Haemophilus influenzae

A) Streptococcus pneumoniae Steptococcus pneumoniae is known for being responsible for the highest mortality in patients with community-acquired pneumonia.

Which of the following drug classes is recommended for the treatment of post- herpetic neuralgia? A) TCAs (tricyclic antidepressants) B) SSRIs (selective serotonin reuptake inhibitors) C) Atypical antidepressants D) Benzodiazepines

A) TCAs (tricyclic antidepressants) Tricyclic antidepressants and anticonvulsants are recommended for post-herpetic neuralgia. These medications may help with the neuropathic pain.

A 30-year-old male with a history of gout is walking to the examination room and the nurse practitioner notices that he is limping. When the patient sits down, the nurse practitioner notes that the metatarsophalangeal joint of the great toe is very swollen and is bright red. The patient reports that he was attending a party the night before and drank. The patient is requesting a prescription to treat his painful toe. The nurse practitioner prescribes the patient indomethacin (Indocin) 50 mg TID PRN and colchicine. Regarding colchicine, which of the following instructions is correct? A) Take 1 pill every hour to every 2 hours until relief is obtained or adverse gastrointestinal effects occur, such as abdominal pain, nausea, or diarrhea B) Take 1 pill every hour until relief is obtained up to 24 hours C) Take 1 pill every 4 to 6 hours until the pain is relieved D) Take 2 to 3 pills QID until relief is obtained or adverse GI effects occur, such as abdominal pain, nausea, or diarrhea

A) Take 1 pill every hour to every 2 hours until relief is obtained or adverse gastrointestinal effects occur, such as abdominal pain, nausea, or diarrhea Colchicine acts as an anti-inflammatory and helps to suppress gouty attacks. It is usually taken as 1 tablet (0.6 mg) every 1 to 2 hours until relief is obtained (or adverse gastrointestinal effects occur, such as abdominal pain, nausea, or diarrhea). Prescribe the patient only 10 tablets at a time (do not refill) during a flare-up. The maximum dose is 6 mg/day. Many patients will develop GI adverse effects even before the pain is relieved. Colchicine can also be taken daily in small amounts for prophylaxis.

The Pap smear result for a 20-year-old sexually active college student who used condoms inconsistently reveals a large number of white blood cells and blood along with inflammatory changes. During the speculum exam, the nurse practitioner who examined the patient noticed that the patient's cervix bled very easily (friable) and a small amount of purulent discharge was present on the cervical surface. No cervical motion tenderness was noted during the bimanual vaginal exam. What is the next step in the management of this patient? A) The NP needs to do cervical cultures to verify gonorrhea B) Prescribe metronidazole vaginal cream for the patient over the phone C) Call the patient and tell her she needs a repeat Pap smear in 6 months D) Advise her to use a Betadine douche at bedtime x 3 days

A) The NP needs to do cervical cultures to verify gonorrhea Cultures should be taken at the time of the Pap smear, as the patient may not return for later diagnostic testing.

The Pap smear result on a 20-year-old sexually active student who uses condoms inconsistently shows a large amount of inflammation. Which of the following is the best follow-up action? A) The NP needs to do cervical cultures to verify the presence of gonorrhea B) Prescribe metronidazole vaginal cream for the patient over the phone C) Call the patient and tell her she needs a repeat Pap smear in 6 months D) Advise the patient to use a Betadine douche at bedtime x 3 days

A) The NP needs to do cervical cultures to verify the presence of gonor- rhea Cultures should be taken at the time of the Pap smear, as the patient may not return for later diagnostic testing.

Nurse practitioners and clinical nurse specialists derive their legal right to practice from: A) The Nurse Practice Act of the state where they practice B) The laws of the state where they practice C) The Medicare statute D) The Board of Nursing in the state where they practice

A) The Nurse Practice Act of the state where they practice The Nurse Practice Act provides nurses with the legal right to practice nursing. The State Board of Nursing is the agency that enforces the Nurse Practice Act. The Medicare statute provides the funds for paying for health services at the age of 65 years and older.

When evaluating the blood pressure on both the arms and legs of an infant who has a diagnosis of coarctation of the aorta, which of the following is the correct finding? A) The blood pressure is higher in the arms than in the legs B) Only the diastolic blood pressure is higher in the legs than in the arms C) The blood pressure is higher in the legs than the arms D) The blood pressure is lower in both arms than in the legs

A) The blood pressure is higher in the arms than in the legs In coarctation of the aorta, blood pressure is higher in the arms than in the legs due to the narrowing in the aorta. Blood pressure must rise to get adequate blood flow to the lower extremities; therefore, the blood pressure above the coarctation rises to compensate for this.

RhoGAM's mechanism of action is: A) The destruction of Rh-positive fetal RBCs that are present in the mother's circulatory system B) The destruction of maternal antibodies against Rh-positive fetal RBCs C) The stimulation of maternal antibodies so that there is a decreased risk of hemolysis D) The destruction of maternal antibodies against fetal RBCs

A) The destruction of Rh-positive fetal RBCs that are present in the mother's cirulatory system RhoGAM is given to mothers with Rh-negative blood when the fetus has Rh-positive blood. RhoGAM protects the mother from developing antibodies by destroying the Rh-positive fetal RBCs in the mother's blood system.

A 30-year-old female complains of having no period for the last 12 weeks. She is sexually active and has been using condoms inconsistently. The patient has a his- tory of irregular menstrual cycles and severe dysmenorrhea. The urine pregnancy test result is positive. Which of the following is a true statement regarding this pregnancy? A) The fundus of the uterus should be at the level of the symphysis pubis B) The cervix should be dilated about 0.5 in. at this time of gestation C) "Quickening" starts during this period D) Hegar's sign is present during this period of pregnancy

A) The fundus of the uterus should be at the level of the symphysis pubis At 12 weeks gestation, the fundus of the uterus should be located approximately at the symphysis pubis.

A patient who is on warfarin sodium (Coumadin) is advised to avoid eating large amounts of leafy green vegetables because: A) The high Vitamin K levels will increase bleeding time B) They have too much ascorbic acid, which can interact with the medicine C) The high fiber content will decrease the absorption of the Coumadin D) The vitamins in the vegetables will bind and inactivate Coumadin

A) The high Vitamin K levels will increase bleeding time Foods containing Vitamin K may enhance the anticoagulation effect of Coumadin, thus increasing bleeding time.

A positive Coombs test on an Rh-negative pregnant woman means: A) The mother has autoantibodies against Rh-positive red blood cells (RBCs) B) The fetus has autoantibodies against maternal RBCs C) The mother does not have Rh factor against fetal RBCs D) The fetus does not have RBC autoantibodies

A) The mother has autoantibodies against Rh-positive red blood cells (RBCs) The mother's autoantibodies can attack the fetus's Rh-positive red blood cells and cause destruction of these cells, which can cause severe anemia and com- plications in the fetus. Today this is preventable with the administration of anti- RhD immunoglobulin (Rho(D) immune globulin) to an Rh-negative mother at 28 weeks gestation and after birth if the newborn is Rh positive.

During a sports physical, you note that the vision of an 18-year-old male athlete is 20/30 in both eyes. Which of the following statements is true? A) The patient can see at 20 ft what a person with normal vision can see at 30 ft B) The patient can see at 30 ft what a person with normal vision can see at 20 ft C) The patient cannot engage in contact sports D) The patient needs to be referred to an ophthalmologist

A) The patient can see at 20 ft what a person with normal vision can see at 30 ft When vision results are 20/30 in both eyes, this means that the patient can see at 20 ft what a person with normal vision can see at 30 ft.

The best form of aerobic exercise for a patient with severe rheumatoid arthritis is: A) Yoga B) Swimming C) Riding a bicycle D) Passive range of motion

A) Yoga The American College of Rheumatology states that exercise is beneficial for everyone, including those with RA, and currently recommends 150 minutes of moderate-intensity aerobic activity each week. Safe forms of aerobic exercise, such as walking, aerobic dance, and aquatic exercise, help arthritis patients to control weight, and improve sleep, mood, and overall health.

16-32. Which of the following is the best description of paronychia? A. Swollen, red, painful B. Swollen, blue, painful C. Painful redness without swelling D. Swollen, red, painless

A. Swollen, red, painful

During the physical exam of a 60-year-old adult, the nurse practitioner performs an abdominal exam. The nurse practitioner is checking the left upper quadrant of the abdomen. During percussion, an area of dullness is noted beneath the lower left ribcage. Which of the following is a true statement regarding the spleen? A) The spleen is not palpable in the majority of healthy adults B) The spleen is 8 cm to 10 cm in the left midaxillary line at its longest axis C) The spleen is 2 cm to 6 cm between the ninth to eleventh ribs on the left midaxillary line D) The splenic size varies depending on the patient's gender

A) The spleen is not palpable in the majority of healthy adults The spleen is located on the left upper quadrant of the abdomen under the diaphragm and is protected by the lower ribcage. In the majority of adults, it is not palpable. The spleen's longest axis is from 11 cm to 20 cm. Any spleen larger than 20 cm is enlarged. The best test for evaluating splenic (or hepatic) size is the abdominal ultrasound. Disorders that can cause splenomegaly include mononucleosis, sickle cell disease, congestive heart failure, bone marrow cancers (myeloma, leukemia), and several other diseases.

Research nomenclature assigns which of the following symbols to indicate a sub- population or a subgroup within the total population? A) The symbol "n = " B) The symbol "N = " C) The symbol "p = " D) The symbol "P = "

A) The symbol "n = " The correct symbol to indicate a subpopulation of a sample is the small letter "n." For example, if a research study has a total population of 100 (N = 100), but it is divided into 2 equal groups, then each group has 50 subjects (n = 50 for each group)

Which of the following is the correct statement regarding the size of the arterioles and veins on the fundi of the eye? A) The veins are larger than the arterioles B) The arterioles are larger than the veins C) The arterioles are half the size of the veins D) The veins and the arterioles are equal in size

A) The veins are larger than the arterioles On funduscopic exam of the eye you will see that the veins are larger in size than the arterioles.

All of the following statements are true regarding domestic abuse except: A) There is no delay in seeking medical treatment B) The pattern of injuries is inconsistent with the history reported C) Injuries are usually in the "central" area of the body instead of the extremities D) Pregnant women have a higher risk of domestic abuse

A) There is no delay in seeking medical treatment When assessing for domes- tic abuse, the most common body area that is abused is the "swim-suit" area, which is usually covered by clothing. Suspect abusive relationships when the history is inconsistent with the injury. Most victims do not seek medical attention until after several episodes of violence. Studies have shown that the incidence of battery escalates during pregnancy.

Café-au-lait spots look like tan-to-light brown stains that have irregular borders. They can be located anywhere on the body. Which of the following is a correct statement? A) They are associated with neurofibromatosis or von Recklinghausen's disease B) They may be considered as precancerous after a biopsy C) They are more common in darker-skinned children D) They are associated with Wilson's disease

A) They are associated with neurofibromatosis or von Recklinghausen's disease Café-au-lait spots are caused by an increase in melanin content, often with the presence of giant melanosomes. They have irregular borders and vary in color from light to dark brown. Neurofibromatosis causes tumors to grow in the nervous system, and these tumors commonly cause skin changes that are seen as café-au-lait spots.

Pete J., a 20-year-old White male, is being seen for a physical exam by the nurse practitioner. He complains of pruritic macerated areas in his groin for the past 2 weeks. Which of the following is the most likely? A) Tinea cruris B) Tinea corporis C) Tinea capitis D) Tinea pedis

A) Tinea cruris Tinea cruris is a fungal infection found in the groin area. Tinea corporis involves the body, tinea capitis involves the head, and tinea pedis involves the feet.

Fetal TORCH infections can cause microcephaly, mental retardation, hepatospleno- megaly, and intrauterine growth retardation. The acronym TORCH stands for: A) Toxoplasma gondii, other infections, rubella, cytomegalovirus, and herpes B) Toxic shock syndrome, ocular infections, rubella, cytomegalovirus, and herpes zoster C) Tetanus, ophthalmic infections, roseola, cancer, and head abnormalities D) Toxins, other infections, roseola, candidiasis, and head abnormalities

A) Toxoplasmagondii, other infections, rubella, cytomegalovirus, and herpes The acronym TORCH stands for Toxoplasma gondii, other infections, rubella, cytomegalovirus, and herpes. Although several of the conditions listed in the other answer options can also cause fetal problems, they are not included in the TORCH acronym.

A 56-year-old man complains of episodes of lancinating pain that shoots up to his right cheek when he eats or drinks. He has stopped drinking cold drinks because of the pain. Which of the following is most likely? A) Trigeminal neuralgia B) Cluster headache C) Acute sinusitis D) Sinus headache

A) Trigeminal neuralgia Pain shooting up the right cheek with food or drink is seen with trigeminal neuralgia.

14-19. Which of the following is not a possible cause of thrombocytopenia? A. Cholecystitis B. Sepsis C. Measles, mumps, and rubella vaccine (MMR) D. Mononucleosis

A. Cholecystitis

Which of the following is an accurate description of eliciting for Murphy's sign? A) Upon deep inspiration by the patient, palpate firmly in the right upper quad- rant of the abdomen below the costovertebral angle B) Bend the patient's hips and knees at 90 degrees, then passively rotate hip exter- nally, then internally C) Ask the patient to squat, then place the stethoscope on the apical area D) Press into the abdomen deeply, then release it suddenly

A) Upon deep inspiration by the patient, palpate firmly in the right upper quad- rant of the abdomen below the costovertebral angle Murphy's sign is positive if pain occurs when performing RUQ palpation while the patient takes a deep inspiration, causing the patient to abruptly stop the deep breath due to pain. As the liver is palpated, it will descend and push up against the inflamed gallbladder and cause sharp pain (cholecystitis).

Jason, an 8-year-old with type 1 diabetes, is being seen for a 3-day history of fre- quency and nocturia. He denies flank pain and is afebrile. The urinalysis result is negative for blood and nitrites but is positive for a large amount of leukocytes and ketones. He has a trace amount of protein. Which of the following is the best test to order initially? A) Urine for culture and sensitivity B) 24-hour urine for protein and creatinine clearance C) 24-hour urine for microalbumin D) An intravenous pyelogram

A) Urine for culture and sensitivity An 8-year-old male patient with the diagnosis of diabetes has a high risk of urinary tract infections. A large amount of leukocytes in the urinalysis is abnormal and he has been having symptoms of frequency and nocturia for the past 3 days. The urine culture would be ordered because he has a high risk of infection. The urine culture and sensitivity (C&S) is the best evaluation for diagnosing a urinary tract infection.

A 30-year-old primigravida is diagnosed with a possible threatened abortion. The result of the urine pregnancy test is positive. Which of the following statements is true regarding a threatened abortion? A) Vaginal bleeding and cramping are present, but the cervix remains closed B) Vaginal bleeding and cramping are present along with a dilated cervix C) The fetus and placenta are all expelled D) The products of conception and the placenta remain inside the uterus along with a dilated cervix

A) Vaginal bleeding and cramping are present, but the cervix remains closed Threatened abortion is defined as vaginal bleeding and cramping with- out the presence of cervical dilation.

A new mother is planning on breastfeeding her newborn infant for at least 6 months. She wants to know whether she should give the infant vitamins. Which of the following vitamin supplements is recommended by the American Academy of Pediatrics (APA) latest guideline during the first few days of life? A) Vitamin D drops B) Multi vitamin drops C) It is not necessary to give breastfed infants vitamin supplements because breast milk contains enough vitamins and minerals that are necessary for the infant's growth and development D) Folic acid drops

A) Vitamin D drops According to the APA, all infants should be given Vitamin D supplementation within the first few days of life. Mothers who plan to breastfeed their infants should be taught how to use Vitamin D drops. Infant formula is supplemented with Vitamin D (and many other vitamins, minerals, and omega-3 oil), so there is no need to give it separately.

Which of the following is the best method for diagnosing candidiasis in the primary care setting? A) Wet smear B) Tzanck smear C) KOH (potassium hydroxide) smear D) Clinical findings only

A) Wet smear The wet prep smear is best used to diagnose candidiasis in the primary care setting. To perform the wet prep, the vaginal smear is placed on a slide, then a drop of saline solution is applied. Organisms viewed under the microscope can include WBCs, clue cells, bacteria, yeast cells, and trichomonads.

Which of the following groups has been recommended to be screened for thyroid disease? A) Women 50 years or older B) Adolescent females C) Elderly males D) School-age children

A) Women 50 years or older Screening for thyroid disease is recommended for women 50 years of age and older.

Diagnosis of acute bronchitis is made by: A. Clinical presentation B. Chest X-ray C. Sputum culture D. White blood cell count (WBC)

A. Clinical presentation

A menopausal woman with osteopenia is attending a dietary education class. Which of the following foods are recommended? A) Yogurt and sardines B) Spinach and red meat C) Cheese and red meat D) Low-fat cheese and whole grain

A) Yogurt and sardines Postmenopausal women are advised to increase their dietary intake of calcium and vitamin D to help protect their bones from osteope- nia/osteoporosis. Foods that are high in calcium and vitamin D include yogurt and sardines.

11-39. A patient diagnosed with myasthenia gravis is discussing her frustration with the muscle weakness associated with the disease, especially in relation to ADLs and exercising. Which statement is the best response by the nurse practitioner? A. "It is best to complete the tasks first thing in the morning." B. "It is best to complete the tasks mid-afternoon, after you have eaten lunch." C. "It is best to complete the tasks in the evening, during the commercials of your favorite shows." D. "It is best to complete the tasks at bedtime to relax you before sleeping."

A. "It is best to complete the tasks first thing in the morning."

A 45-year-old patient presents with no known risk factors for hyperlipidemia (HL), LDL 98 mg/dL HDL 42 mg/dL, and triglycerides 720 mg/dL. The nurse practitioner should consider making which of the following statements? A. "The addition of a fibric acid such as fenofibrate is recommended at this time to lower your triglyceride levels." B. "We should recheck your levels in 4-6 weeks as Lesson triglyceride levels do fluctuate. C. "Your triglyceride levels are high, but we gener- ally do not initiate drug therapy until levels are greater than 1000 mg/dL." D. "Your triglycerides levels are very high, but you are not at risk for acute pancreatitis at this time.

A. "The addition of a fibric acid such as fenofibrate is recommended at this time to lower your triglyceride levels."

A 5-month-old infant is diagnosed with influenza A. Her mother asks for information about keeping her fever down. Based on your knowledge of OTC antipyretics, which of the following statements by the nurse practitioner is best? A. "You may use acetaminophen for her fever." B. "You may use aspirin for her fever." C. "You may use ibuprofen for her fever." D. "You may use a combination of acetaminophen and ibuprofen for her fever."

A. "You may use acetaminophen for her fever."

Which of the following patients would be at greatest risk for infective endocarditis? A. 35-year-old patient who is a chronic injection drug user В. 49-year-old patient undergoing chemo treat- ments for stage II breast cancer C. 55-year-old patient who has coronary artery disease D. 64-year-old patient with chronic migraines

A. 35-year-old patient who is a chronic injection drug user

12-9. Which of the following patients meets the criteria for major depressive disorder? A. A 32-year-old female with symptoms of emptiness, lack of pleasure, weight loss, insomnia, inappropriate guilt, and lack of focus nearly every day for the past month B. A 32-year-old female with symptoms of a depressed mood for the past two months after losing her job C. A 20-year-old male with inability to initiate or maintain sleep and lack of motivation for the past two weeks D. A 20-year-old male with symptoms of fatigue, being tired, insomnia, and lack of focus for the past month

A. A 32-year-old female with symptoms of emptiness, lack of pleasure, weight loss, insomnia, inappropriate guilt, and lack of focus nearly every day for the past month

13-1. The most common cause of Addison's disease is: A. A malfunctioning immune system that attacks the adrenal glands B. Metastatic cancer that attacks the kidneys and adrenal glands C. Steroid use that leads to the suppression of the hypothalamic-pituitary-adrenal axis D. Medication use

A. A malfunctioning immune system that attacks the adrenal glands

19-2. What is a covered entity under HIPAA? A. A medical office that files electronic insurance claims B. A public school system that requires immunizations for entry into school C. A gym that has clients fill out a health questionnaire prior to joining D. A nursing school that provides free blood pres- sure screenings to the public

A. A medical office that files electronic insurance claims

Which of the following is true about Schedule II controlled substances? A. A prescription may only be called into the pharmacy in the event of an emergency B. They are highly addictive, and currently do not have any medical use C. Refills may be written, faxed, or called in to the pharmacy D. The patient will need a new prescription after five refills or six months

A. A prescription may only be called into the pharmacy in the event of an emergency

12-3. Which of the following may require mandatory reporting from the nurse practitioner? A. A withdrawn 5-year-old patient with multiple fractures of varying stages of healing B. A 70-year-old patient who states her 40-year-old daughter has been stealing her Social Security check C.A 19-year-old still living withhis parents who has not completed high school is disruptive, and, according to his parents, is "acting out" D. An anxious 29-year-old housewife who states that her husband is "always angry" at her and appears to have caked-on make-up concealing a black eye

A. A withdrawn 5-year-old patient with multiple fractures of varying stages of healing

During a routine follow-up appointment, an echocardiogram of a patient with a history of hypertension reveals left ventricular dysfunction and an ejection fraction of 45 %. The patient denies weight gain, difficulty breathing, or shortness of breath and is currently taking atenolol to manage HTN. Based on this finding, which medication is likely to be initiated? A. ACE inhibitor B. ARB C. Calcium channel blocker D. Digitalis

A. ACE inhibitor

a 28 y.o. male nurse of Hispanic descent reports hx of a cold that resolved two weeks ago except for a dry cough, and pain in his cheek that worsens when he bends down. Pt denies fever, tells NP he is very allergic to keflex and erythromycin. Vitals: Temp 99.2F, HR: 72, and RR: 12 breaths/min. Which of the following is most likely? A. Acute sinusitis B. Acute bronchitis C. Fever 2ndary to recent URI D. Munchausen's syndrome

A. Acute Sinusitis: acute sinusitis sx include facial pain, cough, and low grade fever

Maiya is a 42-year-old female complaining of intermittent epigastric pain that she rates as 7/10. The pain started 2 days ago after eating a cheeseburger at a fast food restaurant. She has felt feverish but hasn't checked her temperature. She also complains of nausea and decreased appetite and denies diarrhea, change in bowels, dysuria, or vaginal complaints. Based on this information, you should begin a workup for which diagnosis? A. Acute cholecystitis B. Acute pancreatitis C. Acute diverticulitis D.Acute appendicitis

A. Acute cholecystitis

Kallie is a 35-year-old female who complains of widespread musculoskeletal pain and fatigue for the last year. After ruling out other possible diagnosis, she is diagnosed with fibromyalgia. You are educating her on her diagnosis. Which information is correct teaching for fibromyalgia? A. Adequate sleep and exercise are often first-line treatment B. Opioid medications are first-line treatment for pain control C. She should avoid sulfa antibiotics going forward to prevent exacerbations D. She will need to have her ANA level checked every three months to predict exacerbations

A. Adequate sleep and exercise are often first-line treatment

11-28. A 40-year-old Caucasian male with hypertension who presents with fatigue, tingling, numbness in the left lower extremity, and changes in vision asks the nurse practitioner if he has multiple sclerosis. Which of the following additional risk factors below are also symptoms that are suggestive of the disorder? A. Age, ethnicity B. Ethnicity, gender C. Gender, medical condition D. Medical condition, age

A. Age, ethnicity

14-6. You examine a 75-year-old female who presents with history of DM and rheumatoid arthritis. Her DM has been fairly well controlled with insulin. No significant abnormal findings are found on exam. Lab tests reveal: -Hgb 10.1 g/dL (normal (12-16 g/dL) -Hct 31.3% (normal 35-46%) -RBC 3.65 million mm3 (normal 4.7-6.1 million mm3) -MCV 81 fL (normal 81-96 fL) -TIBC 195 mcg/dL (250-450 mcg/dL) -Iron 80 mcg/dL (normal 25-170 mcg/dL) -Ferritin 25 ng/dL (normal 12-156 ng/dL) These values are most consistent with: A. Anemia of chronic disease B. Iron deficiency anemia C. Thalassemia D. Pernicious anemia

A. Anemia of chronic disease

16-36. Which medication classification is commonly used to treat rosacea? A. Antibiotics B. Antifungals C. Non-steroidal anti-inflammatory drugs D. Corticosteroids

A. Antibiotics

A 30-year-old man presents to the office for an annual visit. His best friend was recently diagnosed with testicular cancer and is asking whether he should get himself tested. He states he has completed a thorough testicular self exam and has not felt any masses, but he is still worried. Based on the current cancer screening guidelines, you give him what advice? A. Asymptomatic men such as himself are low-risk and should not undergo testing B. Even though he is low-risk, it doesn't hurt to send him for a baseline testicular ultrasound just in case C. As long as he did not have cryptorchidism as a child, he will not get testicular cancer D. The average age of diagnosis is 50-69, which makes his friend an exception, so he shouldn't worry about it

A. Asymptomatic men such as himself are low-risk and should not undergo testing

11-6. When considering the diagnosis of acute stroke, which of the following are likely to be a part of the presentation? A. Ataxia, unilateral facial numbness, headache B. Slurred speech, coordinated gait, orientation x 3 C. Severe headache, confusion, bilateral facial weakness D. Myopia, altered level of consciousness, severe headache

A. Ataxia, unilateral facial numbness, headache

A patient suspected of having heart failure is sent for further diagnostic examination, including a blood test to assess BNP levels. What is the significance of the BNP level? A. BNP is secreted by the ventricles of the heart in response to excessive stretching of cardiomyocytes B. BNP elevations are only accurate in diagnosing diastolic dysfunction C. BNP is a good marker for differentiating between renal and pulmonary causes of dyspnea D. In decompensated heart failure, BNP levels decrease

A. BNP is secreted by the ventricles of the heart in response to excessive stretching of cardiomyocytes

Which statement is true regarding the use of backup contraception following the initial start of combined oral contraceptives (COC)? A. Backup contraception should be used for the first seven days of the menstrual cycle after a Sunday start B. Backup contraception is not needed with the quick start method C. Backup contraception is needed for the first full menstrual cycle following the first day start method D. Backup contraception should be used for the first 14 days of the menstrual cycle with the Sunday start method

A. Backup contraception should be used for the first seven days of the menstrual cycle after a Sunday start

16-14. Mohs surgery, a technique in which a lesion is excised and examined under a microscope to deter- mine if the edges are free from cancer, is an appro- priate treatment option for which of the following lesions? A. Basal cell carcinoma B. Malignant melanoma C. Actinic keratosis D. Seborrheic keratosis

A. Basal cell carcinoma

16-42. Which treatment recommendation would the nurse practitioner make for the patient diagnosed with alopecia areata? A. Begin daily meditation, yoga, or tai chi practice B. Take vitamin D; 2000 international units daily C. Stop wearing restrictive headbands and hats D. Shampoo with an organic, dye-free product

A. Begin daily meditation, yoga, or tai chi practice

An 8-year-old has been diagnosed with acute post- streptococcal glomerulonephritis. Which finding does the nurse practitioner anticipate? A. Blood pressure 128/88 B. Blood pressure 105/73 C. Heart rate 80 D. Urine output 2 mL/kg/hr

A. Blood pressure 128/88

19-9. Which of the following criteria must be present for medical malpractice to occur? A. Breach of standard of care, injury, proximal cause B. Negligence, duty of care, proximal cause C. Incomplete documentation, poor communication, injury D. Duty of care, lack of evidence for practice, injury

A. Breach of standard of care, injury, proximal cause

NCCN guidelines recommend which of the following to diagnose pancreatic cancer? A. CT scan and EUS B. MRI and EUS C. CT scan and MRCP D. Ultrasound and ERCP

A. CT scan and EUS

An 18-year-old patient complaining of a two-day history of fever, running nose, nasal congestion, and a sore throat is suspected of having a URI. All of the following will be included in the appropriate diag- nostic workup except: A. Chest x-ray B. Rapid strep test C. Rapid influenza test D. Throat culture

A. Chest x-ray

Which laboratory tests are appropriate for evaluating a patient with suspected mononucleosis? A. Complete blood count (CBC), heterophile antibody test B. Rapid antigen detection test, complete blood count (CBC) C. Only a heterophile antibody test is necessary D. Lymphocyte count, rapid plasma regain test

A. Complete blood count (CBC), heterophile antibody test

3-16. Which type of vaccine involves manipulation of the bacterial cell wall to make the immune response more effective in infancy? A. Conjugate B. Live C. Polysaccharide D. Toxoid

A. Conjugate

Which is the safest contraceptive method for a woman with active breast cancer? A. Copper IUD (Paragard) B. Depo-Provera C. Combined oral contraceptives D. Mirena IUD

A. Copper IUD (Paragard)

Fluorescein stain and a Wood's lamp are useful in diagnosing which disorders? A. Corneal abrasions and herpes keratitis B. Glaucoma and allergic conjunctivitis C. Herpes keratitis and bacterial conjunctivitis D. Corneal abrasions and glaucoma

A. Corneal abrasions and herpes keratitis

21-8. The nurse practitioner's current research features observational study of an entire population at a specific point in time. This is also known as a: A. Cross-sectional study B. Longitudinal study C. Placebo effect D. Randomized controlled trial

A. Cross-sectional study

Therapeutic lifestyle changes are recommended for a patient who recently experienced an MI. What should be included as part of the plan? A. Decreasing sodium in the diet B. Cutting down to smoking half a pack of ciga rettes a day C. Walking for 15 minutes once a week D. Lowering the amount of fiber consumed

A. Decreasing sodium in the diet

19-7. Which of the following is not a way to prevent a malpractice claim? A. Delegate interpretation of lab results and patient callback to nursing staff B. Document that a patient was called back about test results and told to repeat testing in three months C. Manage hypertension based on the latest evidence-based recommendations D. Understand and follow the state's Nurse Practice Act

A. Delegate interpretation of lab results and patient callback to nursing staff

16-12. A 28-year-old female presents to the clinic for an annual skin examination. The nurse practitioner notes a mole on the patient's left leg that is dark brown, circular, slightly raised, and 8 mm in diameter. The nurse practitioner is most concerned about which characteristic? A. Diameter B. Color C. Shape D. Slight elevation

A. Diameter

Which of the following is recommended for erythema migrans treatment in early stages of lyme disease? A. Doxycyline (Vibramycin) 100mg PO BID X21 days B. Ciprofloxacin (Cipro) 250 mg PO BID X14 days C. Erythromycin (E-mycin) 333mg PO TID X10 days D. Dixoxacillin 500mg PO BID X 10 days

A. Doxycyline 100mg PO BID X21 days- Recommended by the CDC as gold standard treatment

11-2. In prescribing prednisone for the patient with Bell palsy, the nurse practitioner considers that: A. Early implementation of treatment is correlated with better recovery of nerve function B. Treatment for Bell palsy is controversial, and little data exists that suggests the drug will be effective C. A low dose steroid over four weeks is recommended D. Medication will strengthen the nerve to help eliminate facial and eye drooping

A. Early implementation of treatment is correlated with better recovery of nerve function

4-12. Which is not a mechanism of action of chemotherapy? A. Ensuring the RNA inside a cancer cell is unchanged B. Cell apoptosis due DNA damage C. Destruction of checkpoint proteins D. Intervention at some point in the cell life cycle of DNA replication

A. Ensuring the RNA inside a cancer cell is unchanged

19-24. States that authorize full practice authority for an FNP are indicating that the FNP can: A. Evaluate and diagnose patients, order and interpret diagnostic tests, and initiate and manage treatment B. Evaluate and diagnose patients and order and interpret diagnostic tests, but may need oversight to initiate and manage treatment C. Evaluate and diagnose patients, but may need oversight to order and interpret diagnostic tests and to initiate and manage treatment D. None of the above: there is no true "full practice authority" for FNPS as they must work under the supervision of an MD

A. Evaluate and diagnose patients, order and interpret diagnostic tests, and initiate and manage treatment

12-13. Anorexia nervosa must be considered in children and adolescents who: A. Fail to maintain the normal developmental trajectory B. Lose weight without physiological cause C. Think they are overweight despite maintaining a normal BMI D. Participate in multiple sports to maintain weight

A. Fail to maintain the normal developmental trajectory

Which of the following is not included in Wells criteria? A. Family history of DVT B. Swelling of the entire leg C. Recent cast of the leg D. Active cancer diagnosis

A. Family history of DVT

14-11. Which of the following is not a risk factor/condition associated with lymphoma? A. Family history of rheumatoid arthritis B. Personal history of ulcerative colitis C. Exposure to ionizing radiation D. HIV

A. Family history of rheumatoid arthritis

12-21. The risk of developing schizophrenia is related to an interaction between genetics and environmental factors such as: A. Family history, exposure to viruses, and psychosocial factors B. Family history, other mental health diagnoses, and home location C. Family history, marital status, and nutritional status D. Family history, other medical diagnoses, and education level

A. Family history, exposure to viruses, and psychosocial factors

Which of the following is more characteristic of irritable bowel syndrome than inflammatory bowel disease? A. Fecal urgency B. Bloody stools C. Weight loss D. Tenesmus

A. Fecal urgency

The nurse practitioner has just splinted a patient's wrist. What should the NP assess next? A. Finger sensation and color B. Ability to lift the arm C. Pain relief D. Sensation of the elbow

A. Finger sensation and color

All of the following induce a bronchodilation response except: A. Fluticasone HFA B. Albuterol HFA C. Tiotropium HFA D. Theophylline

A. Fluticasone HFA

14-13. Most bone cancer originates: A. From elsewhere in the body (metastasizing from other cancer sites such as the breast) В. In the bone itself C. As a sequela from bone trauma D. From monoclonal gammopathy of undeter mined significance (MGUS)

A. From elsewhere in the body (metastasizing from other cancer sites such as the breast)

19-20. Which definition is correctly matched with the AANP classification defining the NP scope of practice? A. Full practice: the nurse practitioner can independently evaluate/diagnose patients and order/ interpret diagnostic tests B. Reduced practice: the NP is not able to provide patient care without supervision, delegation, or team management by an outside health discipline C. Restricted practice: the NP needs a collaborative agreement "with an outside health discipline" to be able to provide patient care D. Temporary practice: the NP can independently evaluate/diagnose patients and order/interpret diagnostic tests between the time of graduation and the time of passing the certification exam

A. Full practice: the nurse practitioner can inde- pendently evaluate/diagnose patients and order/ interpret diagnostic tests

Select the appropriate GTPAL score for the woman with the following obstetrical history: vaginal births at 39w3d and 40w0d, cesarean delivery for twins at 35w4d, miscarriage at 11 weeks, termination at 6 weeks, all children born are still alive A. G5T2P1A2L4 B. G6T2P1A2L4 C. G5P4 D. G6P2

A. G5T2P1A2L4

19-5. Which action would be a violation of HIPAA rules? A. Giving lab results to a patient's girlfriend because she said the patient told her to call and get them B. Discussing the treatment plan with another provider who a patient has seen in consultation C. Reporting child abuse to the Department of Child Services without the parent's consent D. Calling a patient by name in the waiting room

A. Giving lab results to a patient's girlfriend because she said the patient told her to call and get them

A 22-year-old male calls the answering service at 2 AM with a report that he was awakened from sleep with severe, debilitating testicular pain that lasted 10 minutes and then spontaneously resolved. He denies pain at this time but is afraid it will happen again. Which of the following is the nurse practitioner's best response? A. Go to the emergency room now for further evaluation B. Take acetaminophen 650 mg PO x 1 in case the pain comes back, and make an appointment to be seen as soon as the office opens C. Explain that this can be a normal symptom that is more common with younger men and it is nothing to worry about D. Prescribe azithromycin (Zithromax) 1 gram PO x 1, as this is a symptom of chlamydia

A. Go to the emergency room now for further evaluation

13-26. All of the following are obesity types considered harmful due to a large production of cytokines and likeliness of resulting in metabolic syndrome except: A. Gynecoid B. Visceral C. Intra-abdominal D. Central

A. Gynecoid

13-12. Which might prompt a diagnosis of diabetes mellitus (DM)? A. Hemoglobin AIC levels of >6.5 % on two separate tests B. An oral glucose tolerance test of 130 mg/dL C. A fasting plasma glucose test of 120 mg/dL D. A one-time random glucose test result of 150 mg/dL

A. Hemoglobin AIC levels of >6.5 % on two sepa- rate tests

Which of the following tests would you recommend to patients to confirm DX of beta thalassemia or sickle cell anemia? A.Hemoglobin Electrophoresis B. Bone Marrow Biopsy C. Peripheral Smear D. Reticulocyte count

A. Hemoglobin Electrophoresis

16-4. Upon examination of a 2-month-old, the nurse practitioner notes a positive Barlow sign and a positive Ortolani sign. Which diagnostic test is appropriate? A. Hip ultrasound B. Renal-bladder ultrasound C. Hip CT scan D. Abdominal CT scan

A. Hip ultrasound

13-17. The nurse practitioner would not expect the Cushing patient to exhibit: A. Hyperkalemia B. Striae C. Hyperglycemia D. Hirsutism

A. Hyperkalemia

14-22. A routine CBC/differential done on a 28-week pregnant patient reveals a platelet count of 97,000/microL. The plan of care for this patient should include: A. Immediate referral to the patient's obstetrician, as the low platelet count could indicate HELLP syndrome B. Immediate referral to the patient's obstetrician, as the low platelet count could indicate preterm labor C. Advising the patient to notify the obstetrician if she develops bleeding gums, nosebleeds, blood in urine, or blood in stool D. Discontinuing prenatal vitamins, as the vitamins can cause low platelets

A. Immediate referral to the patient's obstetrician, as the low platelet count could indicate HELLP syndrome

Which type of insulin should be prescribed specifically to cover insulin needs for about half the day or overnight? A. Intermediate-acting insulin B. Long-acting insulin C. Rapid-acting insulin D. Short-acting insulin

A. Intermediate-acting insulin

19-11. Recent laws have allowed for Medicaid expansion, permitting more Americans to be eligible for health services. What is true regarding Medicaid expansion? A. It allowed more people to qualify under income alone (at generally 138 % of the federal poverty level) B. It created Medicare Part E, which expanded eligibility to those 60 or older, even without disabilities C. It reduced drug copays to no more than $10 per prescription D. It extended coverage to all permanent U.S. residents, even those who were permanent residents for fewer than five years

A. It allowed more people to qualify under income alone (at generally 138 % of the federal poverty level)

Which of the following is characteristic of irritable bowel syndrome? A. It is typically a diagnosis of exclusion B. It is induced by autoimmune activity C. Diagnosis relies heavily on data from labs and imaging D. Diagnosis is confirmed by colonoscopy and biopsy

A. It is typically a diagnosis of exclusion

When Molluscum Contagiosum is found in the genital area of children, which of the following is the best explanation? A. It should raise suspicion of sexual abuse B. It is not considered an STI C. It is caused by Atypical bacteria D. It is caused by the poxvirus and will resolve on it's own

A. It should raise concern for sexual abuse: Molluscum Contagiosum is spread by skin to skin contact, if found in the genital area it should raise concern for sexual abuse

Metastasis of pancreatic cancer is common to the regional lymph nodes and the: A. Liver B. Brain C. Bones D. Kidneys

A. Liver

14-5. Jack is diagnosed with microcytic, hypochromic ane- mia (low MCV, low MCHC). Iron and iron binding capacity and ferritin labs are ordered. What finding suggests iron deficiency anemia? A. Low iron, high TIBC, low ferritin B. Low iron, low TIBC, normal ferritin C. High iron, low TIBC, high ferritin D. Normal iron, normal TIBC, high ferritin

A. Low iron, high TIBC, low ferritin

__________ is a medication that is not strongly associated with drug-induced hepatitis/hepatotoxicity. A. Metformin (Glucophage) B. Tetracycline (Sumycin) C. Morphine (Roxanol) D. Griseofulvin (Gris-PEG)

A. Metformin (Glucophage)

Tara is a 35-year-old patient newly diagnosed with asthma. PMH includes seasonal allergies. Social history includes previous 1 pack-per-day smoker for 10 years. She has a history of coughing and wheezing during the spring and fall, but she attributes this to smoking. She stopped smoking 2 years ago, and the coughing and wheezing have not resolved. She admits she sleeps poorly 3-4 nights out of the week because of coughing. She is unable to walk for exercise as much as she would like because she gets short of breath. She also has coughing and wheezing every morning when she wakes up. You are waiting on her spirometry results. How would you classify her asthma based on the information you have been given in her history? A. Moderate B. Intermittent C. Severe D. Mild

A. Moderate

13-21. The symptoms of hypoglycemia can mimic all of the following serious clinical conditions except: A. Myocardial infarction B. Transient ischemic attack C. Cerebrovascular accident D. Cardiogenic shock

A. Myocardial infarction

In the patient diagnosed with allergic rhinitis, which medication would be used as rescue therapy? A. Oral antihistamines B. Corticosteroid nasal spray C. Mast cell stabilizers D. Leukotriene modifiers

A. Oral antihistamines

17-8. Which complication may occur with mumps? A. Orchitis B. Osteomyelitis C. Myocarditis D. Aortic aneurysm

A. Orchitis

The NP is performing a cardiovascular assessment on a 45-year-old female patient who complains of chest pain that worsens with deep breathing. While locating the PMI, the NP identifies pain with palpation of the 5th intercostal space. What should the NP do next? A. Order an NSAID to treat musculoskeletal pain B. Send the patient to the ER immediately C. Order an ECG D. Refer the patient to a cardiologist

A. Order an NSAID to treat musculoskeletal pain

A 70-year-old man presents to the office with a three-year history of bilateral knee pain, exacerbated by walking for long periods of time. He denies any new injuries, but he did play football in high school. He has no joint swelling, but reports some stiffness in the mornings for approximately 30 minutes and every time he rises from a sitting position. Based on the history, what diagnosis is suspected and what testing would be most beneficial? A. Osteoarthritis; x-rays of the knees B. Osteoarthritis; x-rays of the knees, CBC, ESR, RF, anti-CCP C. Rheumatoid arthritis; x-rays of the knees, CBC, ESR, RF, anti-CCP D. Gouty arthritis; aspiration of the knee joints

A. Osteoarthritis; x-rays of the knees

Duodenal ulcers often present with: A. Pain 1-3 hours after a meal that can typically be relieved by eating again B. Pain 1-3 hours after a meal that typically cannot be relieved by eating again C. Pain immediately after a meal that can be relieved by eating again ldns D. Pain immediately after a meal that cannot be relieved by eating again

A. Pain 1-3 hours after a meal that can typically be relieved by eating again

Which of the following is not a common manifestation of infectious mononucleosis? A. Paroxysmal coughing B. Fatigue C. Sore throat D. Lymphadenopathy

A. Paroxysmal coughing

An 85-year-old patient with a history of hypertension and hyperlipidemia presents to the office with his wife for a routine annual physical. She is concerned about her husband's risk for developing an MI and asks about warning signs. What do the patient and his spouse need to know? A. Patients over the age of 80 may present with atypical symptoms such as confusion and disorientation B. Extreme fatigue and epigastric discomfort are usually the initial symptoms of MI in men C. Assure the patient and his wife that he will not have an MI if he continues his daily aspirin D. Cough and gum pain are the classic symptoms of MI in older men

A. Patients over the age of 80 may present with atypical symptoms such as confusion and disorientation

The patient states "I am really fearful of breast cancer. I have so much cancer in my family." How should the nurse practitioner respond? A. Perform a three-generation family pedigree B. Refer the patient to a breast surgeon C. Prescribe tamoxifen (Nolvadex) for risk reduction D. Include breast MRI as part of the patient's annual imaging surveillance

A. Perform a three-generation family pedigree

The NP is caring for a 6-year-old who had one of his front (permanent) teeth kicked out. The child is crying hysterically. In addition to referring the child to a pediatric dentist immediately, what should be done with the avulsed tooth? A. Place it in a container of milk B. Put the tooth in ice or water C. Allow the child to hold the tooth in his mouth D. Scrub the tooth vigorously to disinfect it

A. Place it in a container of milk

A nurse practitioner is obtaining a medical history from a patient. Which preexisting condition may lead the nurse practitioner to suspect that a patient has colorectal cancer? A. Polyps B. Duodenal ulcers C. Hemorrhoids D. Weight gain

A. Polyps

11-20. A 27-year-old female presents to the clinic with a chief complaint of unilateral, frontal headaches. She reports the headaches occur 3-4 times per month and have not responded well to OTC ibuprofen therapy. Which of the following is a critical data element of her history of present illness? A. Pregnancy status B. Diet changes C. Sensitivity to high-pitched sound D. Immunization status

A. Pregnancy status

A female healthcare worker presents to employee health for her annual tuberculin skin test (TST). Her previous results have been negative. When she returns 48 hours later, the results show 10mm of induration surrounded by erythema. She is asymptomatic and her chest x-ray is normal. The most appropriate management is to: A. Prescribe isoniazid for a duration of 9 months to prevent conversion to active disease B. Repeat the Mantoux tuberculin skin test in 3 months C. Monitor for signs of active disease and perform a repeat chest x-ray in one year D. Prescribe isoniazid, rifampin, and pyrazinamide for a duration of 9 months

A. Prescribe isoniazid for a duration of 9 months to prevent conversion to active disease

A 60-year-old female presents to the office for follow-up after being seen two weeks ago for a painless labial lesion and associated swollen, though non-painful, inguinal lymph nodes. She had a VDRL drawn, which resulted positive, so reflex testing was performed. Today you will tell her that her FTA-ABS was also positive and give her which of the following diagnoses? A. Primary syphilis B. Hepatitis B virus C. Human immunodeficiency virus D. Herpes zoster virus

A. Primary syphilis

What bacteria most commonly causes otitis externa? A. Pseudomonas aeroginosa B. Strep pyogens C. Haemophilis influenza D. Maraxella catarrhalis

A. Pseudomonas aeroginosa is the most common. Staphylococcus is the second most common seen in otitis externa. Treat with neomycin ear drops of Ofloxacin ear drops

An 80-year-old male comes into the office one week after discharge from the hospital after undergoing an ORIF for a hip fracture. He presents with tachypnea, anxiety, and sudden sharp, stabbing pain worsening with deep breaths. Which disorder is suspected? A. Pulmonary embolism B. Aortic stenosis C. Pneumonia D. Angina

A. Pulmonary embolism

All of the following are included in the treatment plan for early stage colorectal cancer except: A. Radiation and palliative care B. Surgery C. Smoking cessation D. Colostomy

A. Radiation and palliative care

Patients eventually diagnosed with pancreatic cancer may present with all of the following symptoms except: A. Rectal bleeding B. Dark-colored urine ) C. Jaundice D. Stomach bloating

A. Rectal bleeding

17-12. A 30-month-old child has a positive Hirschberg reflex. What is the best action taken by the nurse practitioner? A. Refer to the pediatric ophthalmologist B. Recheck the reflex again in 6 months C. Refer to a local optometrist D. Document as normal in the record

A. Refer to the pediatric ophthalmologist

11-53. Following recent eradication of a brain tumor, the nurse practitioner should counsel the patient to: A. Report new hearing changes B. Follow up yearly C. Follow a low-protein diet D. Drive with extra caution

A. Report new hearing changes

A 52-year-old woman presents to the clinic with a three-month history of pain in the MCP joints of both hands with synovitis. She has associated fatigue and morning stiffness lasting for one hour. Which tests would be most beneficial to assist in diagnosis? A. Rheumatoid factor, anti-CCP antibodies, x-rays of the hands B. Rheumatoid factor, x-rays of the hands C. CBC, rheumatoid factor D. CBC, ESR, CRP

A. Rheumatoid factor, anti-CCP antibodies, x-rays of the hands

The NP is performing a physical exam on a 58-year-old male with a long-standing history of HTN and COPD. The NP detects JVD with HOB 60 degrees, an S3 gallop, and 2+ pitting pedal edema. What is the most likely diagnosis? A. Right-sided heart failure B. Biventricular failure C. Left-sided heart failure D. Diastolic failure

A. Right-sided heart failure

All of the following are non-atherosclerotic etiologies of MI except: A. Sarcoidosis B. Cocaine use C. Chest trauma D. Aortic stenosis

A. Sarcoidosis

16-17. Where does seborrheic dermatitis most frequently occur? A. Scalp B. Dorsa of feet C. Antecubital spaces D. Lower legs

A. Scalp

B.W. is a 34-year-male who complains that his left knee has been painful for three days and is getting progressively worse. He describes the pain as throb- bing and rates the pain as a 7/10. ROS is positive for fever. He denies fatigue, weight loss, and rash. PMH is unremarkable. He denies past gastrointestinal or genitourinary infection. Surgical history includes previous arthroscopic surgery to left knee two years ago. Social history includes smoking 2 packs per day. On exam, the left knee is erythematous, warm, and edematous. Which diagnosis is most probable? A. Septic arthritis B. Fibromyalgia C. Reactive arthritis D. SLE

A. Septic arthritis

According to GOLD, pharmacologic treatment regimen should be guided by: A. Severity of symptoms B. Abnormal findings on chest radiograph C. Impaired FEV1 measurement on spirometry D. Immunization status

A. Severity of symptoms

12-11. A 38-year-old male has been taking an SSRI for the past four months for depression. He is experiencing new symptoms of sexual dysfunction and is having difficulty reaching orgasm. You advise him that: A. Sexual dysfunction is a common side efffect to an SSRI and may not resolve without changing the classification of his medication B. Changing to another SSRI will would most likely be beneficial C. This is a common side effect to an SSRI, but is only transient and the symptoms will resolve D. This is an uncommon side effect to SSRIS, and he will be need to be referred to a urologist

A. Sexual dysfunction is a common side efffect to an SSRI and may not resolve without changing the classification of his medication

4-19. Daria is a 27-year-old female who is being started on propranolol for migraine prophylaxis. In addition, she is taking metformin and glipizide for diabetes. It is important to include what information when teaching her about this new medication? A. Signs and symptoms of low blood sugar may not be as apparent B. It is okay to stop the propranolol if her migraines don't improve C. If a rash occurs, it is a minor side effect and should go away D. She should expect her total cholesterol and LDL levels to improve

A. Signs and symptoms of low blood sugar may not be as apparent

What type of diet should the patient with gastroenteritis eat? A. Slow return to a regular diet B. Clear liquids until all diarrhea resolves C. Glucose-containing sports drinks D. Nothing by mouth until diarrhea resolves

A. Slow return to a regular diet

Which of the following is correct regarding diagnosing asthma in a 2-year-old? A. Spirometry is unreliable in patients under the age of 5 B. Patients under 4 years old cannot be diagnosed with asthma C. Eczema is not seen concomitantly with asthma D. If on exam the patient is not wheezing, then asthma is unlikely

A. Spirometry is unreliable in patients under the age of 5

Patients with HTN should be educated regarding: A. Stopping smoking B. Decreasing alcohol intake to 3 drinks/day C. Exercising 60 mins/day, 6 days/week D. Avoiding over-the-counter guaifenesin

A. Stopping smoking

16-45. Trephination is used to treat which of the following disorders? A. Subungual hematoma B. Hidradenitis suppurativa C. Psoriasis D. Urticaria

A. Subungual hematoma

30 y.o. c/o pruritic hives over chest/ arms but denies SOB, difficulty swallowing. Reports family hx allergic rhinitis, & asthma. Which of the following interventions is most appropriate? A. Perform thorough History B. Prescribe PO antihistamine such as benadryl 25mg PO QD C. IM Epinephrine 1:1000 STAT D. Call 911

A. Take a thorough history prior to prescribing medications: find cause of rash. No airway difficulty/ swallowing so no need for 911

20-2. A 65-year-old female of Korean descent presents to the clinic with symptoms of a urinary tract infection. The woman is a recent immigrant from Korea and is accompanied by her nephew who is in the room to translate for her. The nurse practitioner provides culturally competent care for this patient in all of the following ways except: A. Telling the nephew to leave the room unless a HIPAA form has been signed B. Providing written information to the patient about her health concerns and treatment in Korean C. Using a translation app to allow the patient to talk into the app and translate her health concerns into English and vice versa D. Offering the patient a toll-free phone number to Korean translators

A. Telling the nephew to leave the room unless a HIPAA form has been signed

3-8. A patient who has experienced a stroke with hemiparesis and is now participating in occupational therapy is practicing what type of prevention? A. Tertiary prevention B. Secondary prevention C. Primary prevention D. Disease mitigation

A. Tertiary prevention

18-13. Studies indicate that only 1 in 23 cases of elder abuse are reported. What is one possible reason for this finding? A. The elder may worry about incriminating a family member B. The mandatory reporting requirement for adults only begins at age 75 C. Many elders have dementia and may not realize they are being abused D. Cases of children's abuse are more important

A. The elder may worry about incriminating a family member

11-41. A family member of a patient with Guillain-Barré syndrome asks about the paralysis associated with the disorder. Which statement by the NP best describes the paralysis? A. The paralysis is temporary; the disease is self- limiting" B. "The paralysis is progressive and permanent; we can talk about what that looks like for you." C. "The disease does not cause paralysis, just a tin- gling sensation in the lower extremities." D. "The paralysis is like chronic fatigue; it gets worse throughout the day"

A. The paralysis is temporary; the disease is self- limiting"

4-4. Chirality refers to: A. The shape of the medication molecule B. The shape of a drug receptor C. A drug that blocks a matching receptor D. A drug that shows affinity for certain receptors

A. The shape of the medication molecule

A 45-year-old female complains of neck pain after lifting boxes while moving into a new apartment. Sharp pain radiates down the left arm, 8/10, aggravated by flexion and rotation. She has a history of a cervical fusion at C5-C6 three years ago. On exam, how would you anticipate C4-C7 radiculopathy to present? A. Tingling that starts at the neck and radiates down the shoulder, to the back of the left arm, to the tips of fingers 1-3 B. Left biceps reflex 2+ C. Positive Patrick sign D. Decreased sensation to left ear

A. Tingling that starts at the neck and radiates down the shoulder, to the back of the left arm, to the tips of fingers 1-3

The NP is performing a physical exam on a patient who reports sharp chest pain after being involved in a car accident. Which finding indicates a life- threatening condition? A. Tracheal deviation B. Shoulder pain C. Chest bruising D. Headache

A. Tracheal deviation

Which of the following lab tests will be the most useful to determine if a patient is having an MI? A. Troponin B. Myoglobin C. CK-MB D. ESR

A. Troponin

18-6. Medication nonadherence in the elderly is often caused by all of the following except: A. Undesirable side effects B. Diminished hearing C. High cost of medication D. Decline in cognitive function

A. Undesirable side effects

12-26. A screening tool that aids in the evaluation and diagnosis process of ADHD in children is the: A. Vanderbilt Scale B. SCARED Scale C. Y-BOCS D. YMRS

A. Vanderbilt Scale

The NP sees a patient with a history of HF for a one-month follow-up. Digoxin was previously added to his regimen of enalapril and furosemide. AV heart block is present on the patient's ECG. Which of the following manifestations of digoxin toxicity is the patient most likely to report? A. Visual disturbances B. Constipation C. Polyphagia D. Hyperactivity

A. Visual disturbances

Esme is a 42-year-old hair dresser who complains of bilateral aching and swelling in her legs. The swelling and pain are worse at the end of the day. She is a nonsmoker. BMI is 32, HR 75, BP 126/84, RR 18, and SpO2 98 %. She walks 45 minutes twice a week for exercise without difficulty. She has tried to change her shoes and add insoles to her shoes with no improvement. Proper treatment for this patient would include: A. Wearing graded compression socks of 20-30 mmHg for 3 months and then return for follow-up B. Starting an exercise program that includes walking to the point of pain, resting until pain resolves, and start walking once pain has resolved C. Referring to vascular surgery D. Taking an aspirin 325 mg daily

A. Wearing graded compression socks of 20-30 mmHg for 3 months and then return for follow-up

According to Rome III criteria, a patient diagnosed with irritable bowel syndrome must incur all of the following except: A. Weight changes of greater than 6 kg in the past 3 months B. A change in bowel habits for at least 6 months C. Recurrent abdominal pain or discomfort D. Symptoms on at least 3 days per month for 3 months

A. Weight changes of greater than 6 kg in the past 3 months

Presenting symptoms of lung cancer may include all of the following except: A. Wheezing B. Dyspnea C. Chest discomfort D. Anorexia

A. Wheezing

Irritable bowel syndrome is more prevalent in: A. Women B. Men C. Elderly D. Young children

A. Women

16-24. Which of the following is not considered a predisposing factor for the development of lower extremity ulcers? A. Young age B. Diabetes mellitus C. Venous insufficiency D. Obesity

A. Young age

16-22. Which of the following is the preferred choice when prescribing an antibiotic for an adult patient with a human or animal bite? A. amoxicillin (Augmentin) clavulanate B. cephalexin (Keflex) C. doxycycline (Vibramycin) D. trimethoprim-sulfamethoxazole (Bactrim DS)

A. amoxicillin (Augmentin) clavulanate

Drug therapy for irritable bowel syndrome is dependent on whether the disorder manifests with diarrhea, constipation, or both. For IBS-D, which drug therapy would be most effective? A. eluxadoline (Viberzi) B. linaclotide (Linzess) C. lubiprostone (Amitiza) D. amoxicillin/clavulanate (Augmentin)

A. eluxadoline (Viberzi)

4-16. Which of the following is an antineoplastic drug that is also used to treat psoriasis and rheumatoid arthritis? A. methotrexate (Trexall) B. mercaptopurine (Purixan) C. allopurinol (Zyloprim) D. procarbazine (Matulane)

A. methotrexate (Trexall)

A 61-year-old patient is being discharged from hospital after a recent MI and new onset DM. According to most recent guidelines, which treatment is an example of a high-intensity statin drug to initiate upon discharge? A. rosuvastatin 20 mg (Crestor) B. pravastatin 40 mg (Pravachol) C. simvastatin 10 mg (Zocor) D. lovastatin 20 mg (Mevacor)

A. rosuvastatin 20 mg (Crestor)

16-5. The patient has greasy yellow flakes on her scalp and behind her ears. The patient says she washes her hair weekly. The nurse practitioner recommends that the patient change her hair care regimen. She believes the patient is suffering from and may benefit from the daily use of. A. seborrheic dermatitis; ketoconazole shampoo (Nizoral) B. scalp psoriasis; ingenol mebutate (Picato) C. head lice; permethrin cream (Elimite) D. alopecia; clobetasol propionate spray (Clobex)

A. seborrheic dermatitis; ketoconazole shampoo (Nizoral)

16-9. A patient does not want to continue ciclopirox nail lacquer 8 % (Penlac) for onychomycosis and wants to know what treatment might be most effective. Which of the following would the nurse practitioner likely prescribe? A. terbinafine (Lamisil) 250 mg by mouth once daily for six weeks B. Vicks VapoRub applied nightly to each affected nail C. ciclopirox (Loprox) cream applied nightly D. doxycycline (Vibramycin) 40 mg orally daily for 6 weeks

A. terbinafine (Lamisil) 250 mg by mouth once daily for six weeks

Which medication has the following mechanism of action: decreases inflammation and mucus production and causes bronchodilation by selectively binding leukotriene receptors? A. zafirlukast (Accolate) B. omalizumab (Xolair) C. ciclesonide (Alvesco) D. cromolyn sodium (Intal)

A. zafirlukast (Accolate)

Which of the following is associated with having 3 stages of rashes? A. Fifth disease B. Erythema infectiosum C. Varicella D. Rocky mountain spotted fever

A: Fifth disease: (Prodromal phase) begins with sx of URI such as fever, malaise, headache, and chills. Second stage: Rash begins on the cheeks known as the "slapped cheek" usually resolves in 2-3 days. Third stage: rash moves to arms and legs and appears "lacey" that is flat and purple in appearance. May last 3-4 weeks

10 y.o. male who was recently accepted to his school's soccer team has a hx of exercise induced astnma and he wants to know when he should take his albuterol inhaler. The NP should advise him: A. Premedicate himself 20 min before starting exercise B. Wait until he starts to exercise to use the inhaler Premedicate 60 min prior to starting exercise D. Wait until he finishes his exercise to use inhaler

A: Premedicate 20 min prior to starting exercise

Which of the following is the correct statement regarding the size of the arterioles and veins on the fundi of the eye? A. Veins are larger than arterioles B. The arterioles are larger than veins C. The arterioles are half the size of the veins D. The veins and the arterioles are equal in size

A: The veins are larger than the arterioles- this should be apparent on a fundoscopic exam

A chest radiograph shows an area of consolidation on the lower lobe. Which of the following conditions is the most likely? A. Bacterial pneumonia B. Acute bronchitis C. COPD D. Atypical pneumonia

A; Bacterial pneumonia: Consolidation is not seen on CXR in the other three options

The mother of an 8 y.o. boy reports presence of round red rash on the child's left lower leg. It appeared 1 week after the child returned from his grandparents house in Massachusets. During the skin exam, the NP notes a maculopapular rash with areas of clearing making it resemble a large target. Which of the following is best described? A. Erythema Migrans B. Rockey Mountain spotted fever C. Meningococcemia D. Larva Migrans

A; Erythema Migrans: "Bulls eye rash" seen in Lyme disease. Needs immediate treatmemt with ABX, seen in NE region of USA

What cholesterol level is classified as borderline? A) 180 to 199 mg/dL B) 200 to 239 mg/dL C) > 240 mg/dL D) > 300 mg/dL

B) 200 to 239 mg/dL Total cholesterol levels should be < 200. The levels 200-239 would be the best answer.

Which of the following individuals is more likely to be affected by alpha thalassemia anemia? A) 53-year-old Greek patient B) 25-year-old Chinese patient C) 62-year-old Russian patient D) 38-year-old African American patient

B) 25-year-old Chinese patient Alpha thalassemia minor/trait or disease is more prevalent among Asians such as Chinese and Filipinos. Beta thalassemia minor/ trait or disease is more common in the countries in the Mediterranean area, such as Greece and Italy.

A 22-year-old male presents to the urgent care clinic with burns caused by a hot oil spill while frying food. He denies facial involvement, dyspnea, or weakness. During the physical exam, the nurse practitioner notices bright red skin with numerous bullae on the right arm and hand and bright red skin on the right thigh and the right lower leg. On a pain scale of 1-10, he reports the pain as 8. Based on the Rule of Nines, what is the TBSA of this patient's burns? A) 36% B) 27% C) 18% D) 9%

B) 27% This patient has a TBSA of 27% and should be referred to the emergency department as soon as possible. Check the "ABCs" and monitor the patient for shock. Do not puncture bullae.

Which of the following individuals is most likely to be at higher risk for osteoporosis? A) 70-year-old female of African ancestry who walks daily for exercise B) 42-year-old obese woman from Cuba who has been taking prednisone 10 mg daily for the last 12 years to control her severe asthma C) 55-year-old Caucasian female who is an aerobics instructor D) 4-year-old Asian female who has been on high-dose steroids for 1 week

B) 42-year-old obese woman from Cuba who has been taking prednisone 10 mg daily for the last 12 years to control her severe asthma Risk factors for osteoporosis include postmenopause, early menopause, use of chronic steroids, smoking, excessive use of alcohol, sedentary lifestyle, insufficient intake of calcium and Vitamin D in the diet, and being an Asian or Caucasian female.

A nursing home resident reports to his physician that his previous roommate was recently started on tuberculosis treatment. A Mantoux test and chest x-ray are ordered for the patient. What is the minimum size of induration considered positive for this patient? A) 3 mm B) 5 mm C) 10 mm D) 15 mm

B) 5 mm To perform the Mantoux test, a dose of 5 tuberculin units in 0.1 mL solution is injected intradermally into the forearm and read 48 to 72 hours later. Results are measured by the size of the induration, not ery- thema. Standard results of 10 mm or less are considered negative for a low- risk population. For a high-risk population, such as a recent exposure, 5 mm of induration or greater is a positive result.

All of the following children are within the parameters of normal growth and development for their age group except: A) A 2-month-old who coos and smiles B) A 14-month-old who understands complex commands C) A 20-month-old who can walk without support D) A 3-year-old who can speak in three- to four-word sentences

B) A 14-month-old who understands complex commands A 14-month-old child should developmentally be able to say "mama" and "dada," know his own name, and know at least 2-4 words. A 2-year-old is able to understand simple commands.

Native Americans view illness or disease as being caused by which of the following? A) Poor blood circulation in the body B) A punishment by the "spirits" for wrongful actions against others or for failure to follow spiritual rules C) An imbalance of the flow of energy in the body D) An imbalance of the hot and cold energy forces in the body

B) A punishment by the "spirits" for wrongful actions against others or for failure to follow spiritual rules Native Americans believe that illness is punishment by the spirits for wrong actions or from failure to follow spiritual rules. Shamans "cure" the illness by performing rituals and using herbal medicines. Medicine pouches that are tied to the patient by a string are believed to help cure the illness. Do not remove such pouches without the patient's (or parent's) permission.

When assessing a patient suspected of having vertigo, which description provided by the patient is most consistent with the diagnosis? A) A sensation of imbalance while walking B) A sensation of spinning or rotating C) A sensation of "passing out" D) A sensation of lightheadedness when changing positions from reclining to standing

B) A sensation of spinning or rotating Vertigo is defined as having a sensation of spinning or rotating

A positive psoas and obturator sign is highly suggestive of which of the following conditions? A) Ectopic pregnancy B) Acute appendicitis C) Peritonitis D) Abdominal aortic aneurysm

B) Acute appendicitis Both the psoas and obturator signs are associated with acute appendicitis. When the appendix becomes inflamed or ruptured, the blood and pus irritate the psoas and/or obturator muscles, which are both located in the retroperitoneal area. Both muscles are hip flexors and assist with hip movement.

Extreme tenderness and involuntary guarding at McBurney's point is a significant finding for possible: A) Acute cholecystitis B) Acute appendicitis C) Acute gastroenteritis D) Acute diverticulitis

B) Acute appendicitis Symptoms of an acute abdomen, such as appendicitis, include extreme tenderness and involuntary guarding at McBurney's point.

The nurse practitioner would test the obturator and iliopsoas muscle to evaluate for: A) Cholecystitis B) Acute appendicitis C) Inguinal hernia D) Gastriculcer

B) Acute appendicitis. Signs and symptoms of an acute abdomen include invol- untary guarding, rebound tenderness, boardlike abdomen, and a positive obtu- rator and psoas sign. A positive obturator sign occurs when pain is elicted by internal rotation of the right hip from 90 degrees hip/knee flexion. The psoas sign is positive when pain occurs with passive extension of the thigh while the patient is lying on his/her side with knees extended, or when pain occurs with active flexion of his/her thigh at the hip.

A homeless 47-year-old male with a history of injection drug use (IDU) and alcohol abuse presents to the Public Health Clinic with a recent history of fever, night sweats, fatigue, and weakness. The patient has recently noticed some thin red streaks on his nailbed and red bumps on some of his fingers that hurt. During the cardiac exam, the nurse practitioner hears a Grade 3/6 murmur over the mitral area.The subcutaneous red-purple nodules are tender to palpation.The thin red lines on the nailbeds resemble subungual splinter hemorrhages. Which of the following conditions is most likely? A) Pericarditis B) Acute bacterial endocarditis C) Rheumatic fever D) Viral cardiomyopathy

B) Acute bacterial endocarditis Bacterial endocarditis is also known as infective endocarditis (IC). It is a serious bacterial infection of the heart valves and the endocardial surface. The bacteria most commonly involved are Staphylococcus and Streptococcus species. Subcutaneous red painful nodules on the finger pads are called Osler's nodes. Subungual splinter hemorrhages on the nailbeds are caused by microemboli. Janeway's lesions are caused by bleeding under the skin (usually located on the palms and the soles) and are painless red papules and macules. Other findings are conjunctival hemorrhages, petechaie, cardiac fric- tion rubs, arrhythmias, murmurs, and others. Three blood cultures obtained at separate sites 1 hour apart are used to identify the causative organism. Some of the risk factors are damaged prosthetic valves, history of rheumatic fever and injection drug use

Which of the following diagnoses is most likely in this patient? A) Rhabdomyolysis B) Acute drug-induced hepatitis C) Acute mononucleosis D) A major depressive episode

B) Acute drug-induced hepatitis Liver enzymes, such as ALT and AST, can be elevated with the use of medications, such as statin medications. High liver enzymes can cause an acute drug-induced hepatitis.

A concerned new mother reports to you that her son, who is 3 years of age, is not toilet trained yet. Which of the following is an appropriate reply? A) Recommend a referral to a pediatric urologist B) Advise the mother that her child is developing normally C) Recommend a bed-wetting alarm D) Recommend a voiding cystogram

B) Advise the mother that her child is developing normally Toilet training begins at approximately 2 years of age and may take 1-2 years to complete. Males who are not toilet trained by 3 years of age may still be developing normally.

The mother of a 4-week-old infant is concerned that her infant's eyes are crossed for a few seconds occasionally. The nurse practitioner would: A) Recommend referral to a pediatric ophthalmologist B) Advise the mother that this is a normal finding in infants up to 2 months of age C) Recommend that multivitamin supplements be given to the infant daily D) Educate her on how to patch the infant's eye every 4 hours

B) Advise the mother that this is a normal finding in infants up to 2 months of age Infants' eyes commonly cross over at times, and this is a normal finding up to 2 months of age.

A mother brings in her 6-year-old daughter to see the nurse practitioner (NP). She complains that the school nurse found a few nits on her daughter's hair. The mother states that the school has a "no nits" policy regarding head lice and the child cannot go back to school until all the nits have been removed. The child was treated with permethrin shampoo (Nix) twice about 3 months ago. During the physical exam, the NP sees a few nits that are about 2 inches away from the scalp. The child denies itchiness on her scalp. Which of the following is the best action for the NP to follow? A) Prescribe lindane (Kwell) for the child because she may have head lice that are resistant to permethrin B) Advise the mother to use a nit comb after spraying the child's hair with white distilled vinegar, waiting for 15 minutes, and then rinsing the child's hair C) Advise the mother to re-treat the girl with permethrin cream instead of shampoo D) Reassure the mother that the nits will probably drop off after a few weeks

B) Advise the mother to use a nit comb after spraying the child's hair with white distilled vinegar, waiting for 15 minutes, and then rinsing the child's hair According to the CDC, nits that are more than 1⁄4 inch from the child's scalp are usually not viable. The child also does not have an itchy scalp. One method of removal is to soak the child's head with distilled vinegar (and then rinse after), which will break down the protein of the nit casings, making it easy to comb them out of the hair.

Metronidazole (Flagyl) produces the disulfiram (Antabuse) effect when combined with alcoholic drinks or medicine. You would educate the patient to avoid: A) Alcoholic drinks during the time she takes the medicine B) Alcoholic drinks 1 day before, during therapy, and a few days after therapy C) Alcoholic drinks after she takes the medicine D) There is no need to avoid any food or drink.

B) Alcoholic drinks 1 day before, during, therapy and a few days after therapy Alcohol is contraindicated during the use and up until 3 days following the administration of Flagyl. Severe side effects may occur, such as tachycardia, flushing, tingling sensations, nausea, and vomiting.

Which of the following effects is seen in every woman using Depo-Provera (medroxyprogesterone injection) for more than 5 years? A) Melasma B) Amenorrhea C) Weight loss D) Headaches

B) Amenorrhea One common side effect seen in women who have been taking Depo-Provera for more than 5 years is amenorrhea. It is a progesterone hormone that causes cessation of periods.

A mother of a 7-year-old boy tells the family nurse practitioner that his teacher has complained to her of her son's frequent episodes of daydreaming. While the child is at home, the mother reports that sometimes her son seems not to hear her. The child "blanks out" for a short period of time. Which of the following is most likely? A) A partial seizure B) An absence seizure (petit mal seizure) C) A grand mal seizure D) A jacksonian seizure

B) An absence seizure (petit mal seizure) Petit mal seizure is a seizure that usually lasts less than 15 seconds, and the child appears to not be listening. The child appears to be "blacked out" or daydreaming when this occurs.

The "gold standard" for the diagnosis of active Helicobacter pylori infection of the stomach or duodenum is: A) A Helicobacter pylori titer B) An endoscopy with tissue biopsy C) An upper GI series D) A urea breath test

B) An endoscopy with tissue biopsy The "gold standard" for diagnosing Helicobacter pylori infection is to perform tissue biopsy via endoscopy.

All of the following factors are not associated with an increased risk of osteopenia in teenage girls except: A) Drinking one glass of low-fat milk daily B) Anorexia nervosa C) Participation in sports D) A normal BMI (basal metabolic index)

B) Anorexia nervosa Anorexia nervosa increases the risk for osteopenia in teenage girls due to the poor intake of foods that are high in calcium and Vitamin D.

When initially treating an adult for acute bronchitis, which of the following should the nurse practitioner be least likely to order? A) Expectorants B) Antibiotics C) Bronchodilators D) Antitussives

B) Antibiotics Acute bronchititis is normally a viral infection, so expectorants, bronchodilators, and antitussives would be prescribed for the cough. Antibiotics are not effective against viral infections.

You notice a medium-pitched harsh systolic murmur during an episodic exam. It is best heard at the right upper border of the sternum. What is most likely? A) Mitral stenosis B) Aortic stenosis C) Pulmonic stenosis D) Tricuspid regurgitation

B) Aortic stenosis Aortic stenosis is best heard at the right upper border of the sternum, radiating to the neck with a medium-pitched systolic murmur. Mitral stenosis is heard at the apex of the heart and sounds like a low-pitched diastolic rumbling murmur. It is suggested that this murmur is heard best with the bell side of the stethoscope.

A 45-year-old man fell asleep while smoking in his bedroom and started a fire. According to the patient, he refused to go to the emergency department because he had only minor burns. About 12 hours later, he presents to a walk-in urgent care center complaining of a new cough that is productive of saliva with clear mucus with small carbonaceous black particles. His brows appear singed. Which of the following is the priority when evaluating this patient? A) Perform a medical history, including prescription, OTC, and herbal medicines B) Assess the patient for respiratory distress C) Evaluate the patient for asthma and atopy D) Use the Rule of Nines to evaluate the total body surface area (TBSA)

B) Assess the patient for respiratory distress Assess the patient for respiratory distress ASAP. Follow the "ABCs" and assess the patient for any life-threatening symptoms. Smoke inhalation lung injury is the main cause of death in thermal burn victims.

A new patient is being interviewed by the nurse practitioner. The patient reports that she had a gastrectomy procedure 5 years ago to treat severe obesity. Currently, her BMI is 25 and the patient denies complications from the procedure. The nurse practitioner is aware that the patient is at higher risk for which of the following disorders? A) Folate deficiency anemia B) B12-deficiency anemia C) Iron-deficiency anemia D) Normocytic anemia

B) B12-deficiency anemia Intrinsic factor is made by the parietal cells, which are located on the fundus of the stomach. Intrinsic factor is needed to effectively absorb Vitamin B12 (dairy, meat). Because the gastric fundus is damaged in these patients, they are at higher risk of B12-deficiency anemia (MCV greater than 100).

Which of the following would you recommend to this 55-year-old patient? A) Start an exercise program by starting with walking instead of jogging B) Consult with a cardiologist for further evaluation C) Consult with a gastroenterologist to rule out acute cholecystitis D) Take ibuprofen (Advil) 600 mg for pain every 4 to 6 hours PRN

B) Consult with a cardiologist for further evaluation Physical assessment and examination by a cardiologist are advised to rule out cardiac problems and deter- mine the safety of physical exercise.

A 30-year-old female who is sexually active complains of a large amount of milk- like vaginal discharge for several weeks. A microscopy slide reveals a large amount of squamous epithelial cells that have blurred margins. Very few white blood cells are seen. The vaginal pH is at 6.0. What is most likely? A) Trichomonas infection B) Bacterial vaginosis C) Candidal infection D) A normal finding

B) Bacterial vaginosis Bacterial vaginosis is a bacterial infection of the vagina. Signs and symptoms include copious off-white to gray discharge with foul odor without vaginal erythema or irritation. Wet prep will show positive for clue cells. When performing a wet prep, the "whiff test" will be positive for a strong "fishy" odor when vaginal discharge is mixed with one drop of KOH.

The first-line treatment consideration for managing acute alcohol withdrawal delirium includes: A) Intubation B) Benzodiazepines C) Avoidance of physical restraints to decrease agitation D) Antipsychotics

B) Benzodiazepines Acute alcohol withdrawal delirium is managed with benzodiazepines.

Which of the following classes of antihypertensive drugs should a patient be weaned off slowly to avoid the risk of severe rebound hypertension? A) Diuretics B) Beta-blockers C) ACE inhibitors D) Calcium channel blockers

B) Beta-blockers Reverse rebound hypertension can occur if beta-blockers are abruptly stopped. Recommendations include weaning off the beta-blockers when changing medications.

Chadwick's sign is characterized by: A) Softening of the cervix B) Blue coloration of the cervix and vagina C) Softening of the uterine isthmus D) Nausea and vomiting during the first trimester of pregnancy

B) Blue coloration of the cervix and vagina Chadwick's sign is defined as a bluish discoloration of the cervix and vagina. These changes are caused by the increased vascularity and congestion in the pelvic area during pregnancy.

An infant who does not have a history of reactive airway disease and allergy has both inspiratory and expiratory wheezing accompanied by fever and profuse clear nasal discharge. Which of the following is most likely? A) Tracheobronchitis B) Bronchiolitis C) Croup D) A small foreign body that is lodged on the left main bronchus

B) Bronchiolitis Symptoms of tracheobronchitis include prominent dry, non- productive cough; later, coughing up phlegm is common. Bronchiolitis is a viral infection caused by RSV that is commonly seen during the winter/spring months in infants and young children. Typical signs/symptoms include fever, inspiratory/expiratory wheezing, with clear drainage. Croup is a viral infection with the classic "barking" cough; the patient may have a runny nose, but typically no fever. When a child swallows a foreign object, choking, wheezing, and shortness of breath may occur, but no fever or clear drainage is present.

A patient who is complaining of a new onset of severe headache is being examined. The patient is instructed to lie down on the examining table while the nurse practitioner flexes his head and neck forward to his chest. The patient reacts by quickly flexing his hip and knee. What is the name of this positive finding? A) Kernig's sign B) Brudzinski's sign C) Rovsing's sign D) Drawer's sign

B) Brudzinski's sign Brudzinski's sign is positive for meningitis when a patient spontaneously flexes his/her hips and knees after lying down and having his/her head and neck flexed forward to the chest.

Which cranial nerve innervates the extraocular muscles of the eyes? A) CN II, CN III, and CN VI B) CN III, CN IV, and CN VI C) CN IV, CN V, and CN VII D) CN V, CN VI, and CN VIII

B) CN III, CN IV, and CN VI The extraocular muscles of the eyes are innervated by cranial nerves III, IV, and VI.

Which cranial nerve is being evaluated when Rinne testing is done? A) CN VII B) CN VIII C) CN IX and X D) CN XI

B) CN VIII The acoustic nerve, cranial nerve VIII, is being evaluated when the Rinne test is performed.

A 67-year-old female with a 50 pack/year history of smoking presents for a routine annual physical examination. She complains of being easily short of breath and is frequently fatigued. Physical examination reveals diminished breath sounds, hyperresonance, and hypertrophied respiratory accessory muscles. Her CBC results reveal that her hematocrit level is slightly elevated. Her pulmonary function test (PFT) results show increased total lung capacity. What is the most likely diagnosis for this patient? A) Bronchogenic carcinoma B) COPD (chronic obstructive pulmonary disease) C) Chronic bronchitis D) Congestive heart failure

B) COPD (chronic obstructive pulmonary disease) Common complaints and signs/symptoms of COPD include shortness of breath, fatigue, chronic cough, wheezing, difficulty performing tasks and speaking, pursed-lip breathing, digital clubbing, diminished breath sounds, use of accessory muscles, and polycythemia due to chronic hypoxia. Pulmonary function tests/ spirometry include forced expira- tory volume, forced vital capacity, and FEV1/FVC ratios.

All of the following describe normal behavior for a 3-year-old child except: A) Speaks in 3- to 4-word sentences that are understood by most strangers B) Can draw a cross C) Can draw a circle D) Can ride a tricycle

B) Can draw a cross Developmental stages in children include the following: 1 year: walk; 2 years: walks up steps with the same foot; 3 years: pedals a tricycle and copies a circle; 4 years: rides a bicycle and copies a cross and draws a person with 2 parts

124. Which of the following viral infections is associated with occasional abnormal forms of lymphocytes during an acute infection? A) Cytomegalovirus (CMV) B) Epstein-Barr virus (EBV) C) Human papilloma virus (HPV) D) Coxsackie virus

B) Epstein-Barr virus (EBV) A test for an acute infection of the Epstein-Barr virus will show abnormal forms of lymphocytes.

The mother of a 13-year-old male with Down syndrome is in the family nurse practitioner's office and wants a sports physical done for her son. She reports that he wants to join the football team in his school. You would tell the mother that her son: A) Can play a regular football game as long as he wears maximum protective football gear B) Cannot play some contact sports because of an increased risk of cervical spine injury C) Can play certain contact sports after he has been checked for cervical instability D) None of the above

B) Cannot play some contact sports because of an increased risk of cervical spine injury Children with the diagnosis of Down syndrome are at higher risk for atlantoaxial instability, congenital heart defects, and early onset of Alzheimer's disease. Therefore, due to the risk of cervical spine injury, some contact sports are not advised

Fitz-Hugh-Curtis syndrome is associated with which following infection? A) Syphilis B) Chlamydia trachomatis C) Herpes genitalis D) Lymphogranuloma venereum

B) Chlamydia trachomatis Fitz-Hugh-Curtis is a complication of having pelvic inflammatory disease that was caused by a vaginal infection, such as gonorrhea or Chlamydia trachomatis. This causes inflammation and infection in the pelvic cavity. Left untreated, this infection can cause adhesions that stretch from the peritoneum to the liver.

411. Which of the following is the most common cause of nongonococcal urethritis? A) Escherichia coli B) Chlamydia trachomatis C) Neisseria gonorrhoeae D) Mycoplasma

B) Chlamydia trachomatis The most common cause of nongonococcal urethritis is Chlamydia trachomatis.

Patients who are diagnosed with gonorrhea should also be treated for which of the following infections? A) Chancroid B) Chlamydia trachomatis C) Herpes genitalis D) PID (pelvic inflammatory disease)

B) Chlamydia trachomatis When diagnosed with gonorrhea, the patient should also be treated for Chlamydia trachomatis.

What is the most common cause of left ventricular hypertrophy in the United States? A) Chronic atrial fibrillation B) Chronic hypertension C) Mitral valve prolapse D) Pulmonary hypertension

B) Chronic hypertension The most common cause of LVH is chronic hypertension.

The heart sound S2 is caused by: A) Closure of the atrioventricular valves B) Closure of the semilunar valves C) Opening of the atrioventricular valves D) Opening of the semilunar valves

B) Closure of the semilunar valves The heart sound S2 is caused by closure of the semilunar valves.

A 40-year-old male complains to the nurse practitioner of severe stabbing pains behind his left eye for the past 2 days. It is accompanied by some nasal congestion and rhinorrhea, which is clear in color. The patient denies pharyngitis and fever. Which of the following conditions is most likely? A) Migraine headache with aura B) Cluster headache C) Tic douloureux D) Cranial neuralgia

B) Cluster headache Signs/symptoms of cluster headaches include severe stabbing pain behind the eyes, with nasal congestion and rhinorrhea. Migraine head- aches with aura include visual changes, such as blind spots or flashing lights that appear before the onset of the headache. Trigeminal neuralgia (tic douloureux) is a unilateral headache from compression or inflammation of the trigeminal nerve (cranial nerve 5).

A physician is referring one of his patients to a nurse practitioner. What type of relationship will exist between the physician and nurse practitioner? A) Consultative B) Collaborative C) Professional D) Advocate

B) Collaborative A collaborative relationship exists when a health caregiver refers a patient to others (physicians, specialists, physical therapy, etc.) to help with patient treatment and management. Consult reports and progress reports are sent to the primary caregiver to report the patient's progress. Consultative relationships are informal, such as talking to a colleague about a patient treatment

A nurse practitioner is writing a referral for a middle-aged diabetic who has an A1c of 8.5% despite being on 3 antidiabetic medications. She is referring the patient to an endocrinologist. What type of relationship exists between the nurse practitioner and the endocrinologist? A) Consultative relationship B) Collaborative relationship C) Referral relationship D) Formal relationship

B) Collaborative relationship A collaborative relationship is a formal process of sharing responsibility for treating a patient together (sharing progress reports); for example, referring a patient to a specialist or for rehabilitation.

A 68-year-old woman with hypertension and diabetes is seen by the nurse practitioner for a dry cough that worsens at night when she lies in bed. She has shortness of breath, which worsens when she exerts herself. The patient's pulse rate is 90/min and regular. The patient has gained 6 lbs over the past 2 months. She is on a nitro- glycerine patch and furosemide daily. The best explanation for her symptoms is: A) Kidney failure B) Congestive heart failure C) ACE inhibitor-induced coughing D) Thyroid disease

B) Congestive heart failure Signs and symptoms of congestive heart failure (CHF) include dyspnea on exertion, edema, fatigue, hemoptysis, cough, orthopnea, hypertension, and nocturnal dyspnea.

An nurse practitioner, who is a recent graduate, is asking an experienced nurse practitioner's opinion about managing a patient who has multiple health problems. What type of relationship exists between the nurse practitioners? A) Collaborative relationship B) Consultative relationship C) Referral relationship D) Formal relationship

B) Consultative relationship A consultative relationship is an informal process between two or more providers who exchange information about a patient occasionally.

Which of the following laboratory tests is used in primary care to evaluate renal function? A) Electrolyte panel B) Creatinine C) Alkaline phosphatase D) Blood urea nitrogen (BUN) to creatinine ratio

B) Creatinine Serum creatinine is measured to evaluate renal function. Creatinine is the end product of creatine metabolism. Creatinine clearance is not affected by fluid or dietary intake of meat.

Heberden's nodes are commonly found in which of the following diseases? A) Rheumatoid arthritis B) Degenerative joint disease C) Psoriatic arthritis D) Septic arthritis

B) Degenerative joint disease Heberden's nodes are bony nodules on the distal interphalangeal joints, commonly seen in degenerative joint disease.

There is a higher risk of balanitis in which of the following conditions? A) Renal insufficiency B) Diabetes mellitus C) Graves' disease D) Asthma

B) Diabetes mellitus Balanitis is an yeast infection of the glans of the penis. Men who are not circumcised and who have diabetes mellitus are at higher risk for developing balanitis.

A 56-year-old mechanic is brought to your office complaining of heavy pressure in the substernal area of his chest that is radiating to his jaw. The pain began while he was lifting up a tire. He now appears pale and is diaphoretic. His blood pressure is 100/60 mm Hg, and his pulse rate 50. What is the most appropriate action? A) Perform a 12-lead EKG B) Dial 9-1-1 C) Administer a morphine injection for pain D) Observe the patient in the office

B) Dial 9-1-1 Heavy chest pressure in the substernal area, radiating to the jaw, diaphoresis, low BP, and bradycardia are signs of cardiac distress and 9-1-1 should be called immediately for transfer to the emergency department.

A woman who is in the third trimester of pregnancy presents to the nurse practitioner for a physical exam. During the physical exam, the nurse practitioner finds all of the following cardiac changes associated with pregnancy except: A) Systolic ejection murmur B) Diastolic murmur C) Displaced apical impulse D) Louder S1 and S2

B) Diastolic murmur Diastolic murmurs are more likely to be pathologic. The heart is displaced in a more transverse position that is lateral to the midclavicular line. The systolic ejection murmur is due to increased stroke volume caused by increased cardiac output and higher basal heart rate.

When starting an elderly patient on a new prescription of levothyroxine (Synthroid), the nurse practitioner should keep in mind that the rationale for starting an elderly patient on a lower dose is which of the following? A) Due to its central nervous system effects B) Due to its cardiac effects C) Due to its renal effects D) Due to its hepatic effects

B) Due to its cardiac effects Levothyroxine (Synthroid) should be started on the lowest dose in elderly patients due to the severe side effects that can occur. Side effects include palpitations, tachycardia, anxiety, irritability, elevated BP, flushing, and insomnia.

Which of the following findings is most likely in young primigravidas with pregnancy-induced hypertension? A) Abdominal cramping and constipation B) Edema of the face and the upper extremities C) Shortness of breath D) Dysuria and frequency

B) Edema of the face and the upper extremities Common signs and symptoms of pregnancy-induced hypertension include edema of the face and the upper extremities, weight gain, blurred vision, elevated BP, proteinuria, and headaches.

A 53-year-old crossing guard complains of twisting his right knee while working that morning. The knee is swollen and tender to palpation. The nurse practitioner diagnoses a Grade II sprain. The initial treatment plan includes which of the following? A) Application of cold packs the first 24 hours followed by applications of low heat at bedtime B) Elevation of the affected limb and intermittent applications of cold packs for the next 48 hours C) Rechecking the knee in 24 hours and isometric exercises D) The application of an Ace bandage to the affected kne

B) Elevation of the affected limb and intermittent applications of cold packs for the next 48 hours Elevation of the injured knee above the heart will reduce the amout of swelling that can occur. Use of ice packs immediately after the injury is most effective and will reduce swelling in the tissue. Ice the affected area for 15 minutes at a time intermittently to prevent frostbite and further damage to tissue. Allowing 30-45 minutes between icings of the limb is recommended.

A 21-year-old female who is complaining of random palpitations is diagnosed with mitral valve prolapse (MVP). Her echocardiogram (EKG) reveals redundant and thickened leaflets. You note a Grade III/VI systolic murmur with an ejection click during physical examination. You would recommend: A) Endocarditis prophylaxis for most dental and urologic procedures B) Endocarditis prophylaxis is not necessary C) Lifetime anticoagulation therapy with warfarin sodium D) Endocarditis prophylaxis for dental procedures only

B) Endocarditis prophylaxis is not necessary Prophylaxis treatment for endocarditis is no longer recommended for MVP.

Duvall and Miller used developmental theory to describe the family. Which of the following statements is true? A) Each family is developmentally unique in comparison to other families B) Families demonstrate common forms of membership across developmental stages C) Families complete each developmental task separately D) Families change over time because of the influence of environmental factors

B) Families demonstrate common forms of membership across developmental stages Families progress through similar developmental stages and experience similar changes through this process.

A newborn infant who is small for gestational age is noted to have shortened palpebral fissures and microcephaly with a small jaw. This infant is most likely to be diagnosed with: A) Down syndrome B) Fetal alcohol syndrome C) Growth retardation D) Hydrocephalus

B) Fetal alcohol syndrome Classic symptoms of fetal alcohol syndrome include small palpebral fissures and microcephaly with a small jaw.

What is the best procedure for evaluating a corneal abrasion? A) Tonometry B) Fluorescein stain C) Visual field test D) Funduscopy

B) Fluorescein stain Fluorescein stain is an eye stain used to detect abrasions or foreign objects in the cornea of the eye. Orange dye (fluorescein) is used to stain the eye and a blue light is used to detect/visualize any foreign bodies or abrasions in the eye. Visual field test assesses vision. Tonometry measures the pressure inside the eye. The funduscopic exam is performed with an ophthalmoscope to visualize the inside of the eye.

You would advise a patient who is on a MAOI (monoamine oxidase inhibitor) prescription to avoid taking one of the following drugs because of increased potential for a serious reaction. Which of the following is this drug? A) Alprazolam (Xanax) B) Fluoxetine (Prozac) C) Erythromycin (E-mycin) D) Amoxicillin (Amoxil)

B) Fluoxetine (Prozac) Patients taking MAOIs should not also take SSRIs (Prozac), due to the possibility of developing serotonin syndrome. This causes excessive serotonin release in the brain, which will cause symptoms of severe anx- iety, restlessness, confusion, and muscle twitching. If not stopped, the patient may develop muscle contractions, renal failure, respiratory failure, coma, and even death.

A high school teacher complains of a dry cough for the past 6 weeks. It worsens when he is supine. He has episodes of heartburn, which he self-treats with an over- the-counter (OTC) antacid. He chews mints for his "bad breath." Which of the fol- lowing is a possible cause for this patient's cough? A) Asthma B) Gastroesophageal reflux C) Pneumonia D) Chronic postnasal drip

B) Gastroesophageal reflux GERD is a condition in which food comes up from the stomach/esophagus due to a weak sphincter. It usually worsens with lying down and can cause a cough and esophageal irritation if not treated.

A newborn's mother is discovered to be HBsAg (hepatitis B surface antigen) positive. Which of the following would you recommend for this infant? A) Give the baby hepatitis B immunoglobulin B) Give the baby both hepatitis B vaccine and hepatitis B immunoglobulin C) Give the baby hepatitis B vaccine only D) Send the baby home because he is not infected

B) Give the baby both hepatitis B vaccine and hepatitis B immunoglobulin For a mother who tests positive for HBsAg, the newborn infant should be given hepatitis B vaccine and hepatitis B immunoglobulin for protection.

Podagra is associated with which of the following? A) Rheumatoid arthritis B) Gout C) Osteoarthritis D) Septic arthritis

B) Gout Podagra is pain in the joint of the great toe due to the accumulation of uric acid and salts in the joint.

An 8-year-old is seen as a walk-in appointment by the nurse practitioner. The mother reports that her child has been febrile for 2 days and is not eating well due to painful sores inside the child's mouth. The child's temperature is 101 degrees Fahrenheit, the pulse is 88 beats/minute, and the respirations are at 14 breaths per minute. During the physical examination, the nurse practitioner notices several small blisters and shallow ulcers on the child's pharynx and the oral mucosa. The child has small round red rashes on both palms and soles. Which of the following conditions is most likely? A) Herpes simplex infection B) Hand, foot, and mouth disease (HFMD) C) Varicella infection D) Secondary syphilis infection

B) Hand, foot, and mouth disease (HFMD) Hand, foot, and mouth disease (HFMD) is caused by the coxsackie virus A16. The virus is found in the saliva, sputum, nasal mucus, feces, and blister fluid. It is transmitted through direct contact of the secretion or in fomites (e.g., preschool toys). Treatment is symptomatic.

During a well child visit, a new father wants to know if he can give fresh whole milk to his 6-month-old son. The nurse practitioner would recommend that: A) He can start giving whole milk but not skim milk by 6 months of age B) He should not give whole milk to his son until the boy is at least 12 months of age C) He can give whole milk to his son at anytime D) He should not give whole milk to his son without diluting it with water

B) He should not give whole milk to his son until the boy is at least 12 months of age The American Academy of Pediatrics does not recommend offering whole milk to children younger than 12 months of age.

A 63-year-old patient with a 10-year history of poorly controlled hypertension presents with a cluster of physical exam findings. Which of the following indicate target organ damage commonly seen in hypertensive patients? A) Pedal edema, hepatomegaly, and enlarged kidneys B) Hepatomegaly, AV nicking, bibasilar crackles C) Renal infection, S3, neuromuscular abnormalities D) Glaucoma, jugular vein atrophy, heart failure

B) Hepatomegaly, AV nicking, bibasilar crackles With long-term, uncontrolled hypertension, organ damage may occur. Organs commonly affected include: brain (stroke), eyes (retinopathy, AV nicking, bleeding, blindness), heart (heart dis- ease, left ventricular hypertrophy, MI, and/or CHF), and kidneys (renal failure, proteinuria).

A 35-year-old primigravida who is at 28 weeks of gestation is expecting twins. What would you would expect her alpha fetoprotein (AFP) values to be? A) Normal B) Higher than normal C) Lower than normal D) None of the above

B) Higher than normal Alpha fetoprotein is produced in the fetal and maternal liver. Higher levels of alpha fetoprotein are commonly seen in multiple gestations due to the growing fetuses and enlargement of the livers.

Mrs.Brands complains of dizziness when she moves her head. You suspect benign paroxysmal positional vertigo. The diagnosis is supported by the presence of: A) Tinnitus B) Horizontal nystagmus with rapid head movement C) New onset of hearing loss D) Duration of greater than 2 years

B) Horizontal nystagmus with rapid head movement Symptoms of benign paroxysmal positional vertigo include horizontal nystagmus with rapid head movement. By performing the Dix-Hallpike maneuver and/or the roll test, the symptoms of vertigo will appear.

You have diagnosed Tom J., a 30-year-old male, with contact dermatitis on the left side of the face secondary to poison ivy. You would recommend: A) Washing with antibacterial soap BID to reduce risk of secondary bacterial infection until it is healed B) Hydrocortisone cream 1% BID until it is healed C) Clotrimazole (Lotrimin) cream BID for 2 weeks D) Halcinonide (Halog) 1% ointment BID for 2 weeks

B) Hydrocortisone cream 1% BID until it is healed Treatment for contact dermatitis includes using a steroid cream BID until healed.

Which of the following conditions is the most common cause of sudden death among athletes? A) Brain aneurysm B) Hypertrophic cardiomyopathy C) Left ventricular hypertrophy D) Aortic stenosis

B) Hypertrophic cardiomyopathy Hypertrophic cardiomyopathy is the most common cause of sudden death in athletes.

You are examining a patient who has just been diagnosed with Bell's palsy. Bell's palsy is characterized by all of the following except: A) Drooling B) Inability to swallow C) Inability to close the eye on the affected side D) Drooping of the corner of the mouth on the affected side

B) Inability to swallow Bell's palsy affects cranial nerve VII, which affects the ability to smile, close the eye on the affected side; drooping and drooling on the affected side can also occur. Swallowing is not affected by the facial nerve.

About one-third of children in the United States are considered obese. Which of the following methods are appropriate interventions for obese school-aged children? A) Severe restriction of dietary carbohydrates B) Increase physical activity and outdoor play C) Prescribe appetite suppressants D) Over-the-counter herbal weight-loss pills

B) Increase physical activity and outdoor play Lifestyle changes such as increasing physical activity and spending more time outdoors are appropriate recommendations for children. Consider referral to a registered dietician for dietary recommendations and counseling. Participation in an exercise program designed for children (if available) is also appropriate. Severe caloric and/or carbohydrate restriction is not recommended for this age group.

A positive straight-leg raising test is indicative of which of the following? A) Myasthenia gravis B) Inflammation of the sciatic nerve/herniated disc C) Multiple sclerosis D) Parkinson's disease

B) Inflammation of the sciatic nerve/herniated disc To perform the straight- leg test, have the patient lie supine on an exam table. Lift the patient's leg toward the head while the knee is straight. If the patient experiences sciatic pain when the straight leg is at an angle of between 30 and 70 degrees, then the test is positive and a herniated disc is likely to be the cause of the pain. The straight-leg test should be done on the pain-free side first to find out which range of movement is normal and to enable the patient to distinguish between "normal" stretching of muscles and a different sort of pain.

Mr. R. J. is a 40-year-old asthmatic male with hypertension. For the past 6 months, he has been following a low-fat, low-sodium diet and walking 3 times a week. His BP readings from the past 2 visits were 160/95 and 170/ 100. On this visit, it is 160/90. What is the most appropriate action for the nurse practitioner to follow at this visit? A) Continue the lifestyle modifications and recheck his blood pressure again in 4 weeks B) Initiate a prescription of hydrochlorothiazide 12.5 mg PO daily C) Initiate a prescription of atenolol (Tenormin) 25 mg PO daily D) Refer the patient to a cardiologist for a stress EKG

B) Initiate a prescription of hydrochlorathiazide 12.5 mg PO daily The patient would be started on HCTZ 12.5 mg PO daily for his high BP. He has been trying lifestyle modifications for 6 months with diet changes and exercise.

Mrs. Nottam, who has a BMI of 29, has a 20-year history of primary hypertension. She has been on hydrochlorothiazide 25 mg PO daily with excellent results. On this visit, she is complaining of feeling thirsty all the time even though she drinks more than 10 glasses of water per day. She reports to the nurse practitioner that she has been having this problem for about 6 months. Upon reading the chart, the nurse practitioner notes that the last two fasting blood glucose levels have been 140 mg/dL and 168 mg/dL. A random blood glucose is at 210 mg/dL. Which of the following is the best treatment plan to follow at this visit? A) Order another random blood sugar test in 2 weeks B) Initiate a prescription of metformin (Glucophage) 500 mg PO BID C) Order a 3-hour glucose tolerance test D) Order a HgbA1c leve

B) Initiate a prescription of metformin (Glucophage) 500 mg PO BID Initiating metformin is recommended because she has met the criteria for diagnosis of diabetes. Criteria for diagnosing type 2 diabetes: fasting blood glucose 126 mg/dL or higher on 2 separate occasions, 2-hour blood glucose 200 mg/dL or higher during OCT with 75 g glucose load, symptoms of diabetes (polyuria, polydipsia, polyphagia) plus random blood glucose greater than 200 mg/dL.

What does a positive posterior drawer sign in a 10-year-old soccer player signify? A) An abnormal knee B) Instability of the knee C) A large amount of swelling on the knee D) An injury of the meniscus

B) Instability of the knee The drawer sign is performed on the knee or ankle to assess for knee instability. The affected knee will have more laxity when compared to the unaffected knee.

A 75-year-old woman presents complaining of a soft lump on her abdomen that is located on the periumbilical area. She tells the nurse practitioner that she does not know how long she has had the lump or whether it has changed in size or shape. She denies abdominal pain, problems with defecation, loss of appetite, weight loss, or trauma. When performing an abdominal exam, what is the best method to differentiate between an abdominal wall mass and an intra-abdominal mass? A) Palpate the abdominal wall while the patient is relaxed B) Instruct the patient to lift her head off the table while tensing her abdominal muscles to visualize any masses and then palpate the abdominal wall C) Instruct the patient to lie still for few seconds while you palpate the abdominal wall D) Palpate the abdomen deeply, then release the palpating hand quickly

B) Instruct the patient to lift her head off the table while tensing her abdominal muscles to visualize any masses and then palpate the abdominal wall An abdominal wall mass will become more prominent when the abdominal wall muscles are tense. If it is an intra-abdominal mass, it will be pressed down by the muscles and will become less obvious or disappear. Some of the most common abdominal wall masses are hernias (epigastric, umbilical, incisional). This patient had a periumbilical hernia (soft lump on her abdomen that is located on the periumbilical area that is painless).

Which of the following is contraindicated in the care of pregnant women with placenta previa? A) Echocardiogram B) Intravaginal ultrasound C) Abdominal ultrasound D) Pelvic ultrasound

B) Intravaginal ultrasound No type of vaginal exam should be performed in patients diagnosed with placenta previa. Intravaginal ultrasound and pelvic exams are contraindicated

Which of the following would be appropriate initial management of a second- degree burn? A) Irrigate with hydrogen peroxide and apply Silvadene cream BID B) Irrigate with normal saline and apply Silvadene cream BID C) Irrigate with tap water and apply Neosporin ointment BID D) Unroof all intact blisters and apply antibiotic ointment BID

B) Irrigate with normal saline and apply Silvadene cream BID Burns should be cleansed with saline solution and Silvadene cream applied BID to the site. Hydrogen peroxide is no longer recommended. Intact blisters should not be unroofed.

Which of the following is used to screen for color blindness in a 7-year-old boy? A) Snellen chart B) Ishihara chart C) Cover/uncover test D) Red reflex

B) Ishihara chart The Ishihara chart is used for screening for color blindness.

All of the following are false statements about atopic dermatitis except: A) Contact with cold objects may exacerbate the condition B) It does not have a linear distribution C) It is associated with bullae D) The lesions have vesicles that are full of serous exudate

B) It does not have a linear distribution Atopic dermatitis is a skin condition in which the lesions occur in a linear fashion. They may have many different stages, including erythematous papules and vesicles, with weeping, drainage, and/or crusting. Lesions are commonly found on the scalp, face, forearms, wrists, elbows, and backs of the knees. Commonly pruritic. Also known as eczema.

A 72-year-old woman has been on hydrochlorothiazide 12.5 mg for many years to control her Stage II hypertension. Her blood pressure (BP) at this visit is 168/96. She is currently complaining of pain on her right hip and on both knees. She has increased her dose of ibuprofen (Motrin) from 400 mg 3 times day (TID) to 800 mg TID. She is still in pain and would like something stronger. Which of the following statements is the best explanation of the effects of ibuprofen (Motrin) on her disease? A) It increases the chances of adverse effects to her health B) It inhibits the effect of renal prostaglandins and blunts the effectiveness of the diuretic C) It prolongs the therapeutic effects of hydrochlorothiazide and other diuretics D) None of the statements are true

B) It inhibits the effect of renal prostaglandins and blunts the effectiveness of the diuretic. NSAIDs and ASA inhibit the vasodilatory effects of prostaglandins, which predisposes the kidney to ischemia. NSAIDs and diuretics can cause acute prerenal failure by decreasing renal blood flow.

An 80-year-old woman complains about her "thin" and dry skin. Which of the following is the best explanation for her complaint? A) Genetic predisposition B) Loss of subcutaneous fat and lower collagen content C) Loss of sebaceous glands D) Damage from severe sun exposure

B) Loss of subcutaneous fat and lower collagen content Thinning of the skin in older females is due to the loss of subcutaneous fat and lower collagen content

Your patient of 10 years, Mrs. Leman, is concerned about her most recent diagnosis. She was told by her dermatologist that she has an advanced case of actinic keratosis. Which of the following is the best explanation for this patient? A) It is a benign condition B) It is a precancerous lesion and should be followed up with her dermatologist C) Apply hydrocortisone cream 1% BID for 2 weeks and most of it will go away D) It is important for her to follow up with an oncologist

B) It is a precancerous lesion and should be followed up with her dermatologist Actinic keratosis is a small, raised skin lesion from having been in the sun for a long period of time. Some actinic keratoses may develop into skin cancer; therefore, further evaluation is needed to determine if removal is required.

All of the following statements about phototherapy are correct except: A) Light from the blue to white spectrum is used B) It is not always necessary to use a shield for the infant's eyes C) Unconjugated bilirubin in the skin is converted to a water-soluble nontoxic substance that is excreted in the bile D) The infant's eyes should be shielded

B) It is not always necessary to use a shield for the infant's eyes When using phototherapy, the eyes should always be protected by using a shield or goggles to prevent damage to the eyes.

What is the median? A) It is the number that occurs the most frequently B) It is the middle number in a group of numbers C) It is the average number in a group of numbers D) It a measure of central tendency

B) It is the middle number in a group of numbers The median is the middle score of a group of numbers. For example, for this group of numbers (2, 3, 6, 7, 7, 8, 10), the number "7" is the median value.

The Somogyi effect is characterized by which of the following? A) It is a complication of high levels of growth hormone B) It is the physiologic spike of serum blood glucose in the early morning C) It is characterized by high fasting blood glucose in the morning D) It is a rare phenomenon that only occurs in type 1 diabetic patient

B) It is the physiologic spike of serum blood glucose in the early morning The Somogyi phenomenon is when nocturnal hypoglycemia (2-3 a.m.) stimulates the liver to produce glucagon to raise the blood sugar. The fasting blood glucose levels will be elevated from this glucagon production.

A pelvic exam on a woman who is 12 weeks pregnant would reveal that her uterus is located at which of the following areas? A) Between the umbilicus and the suprapubic bone B) Just rising above the suprapubic bone C) Between the suprapubic bone and the xiphoid process D) Between the umbilicus and the xiphoid proces

B) Just rising above the suprapubic bone At 12 weeks gestation, the uterus measures approximately the size of a grapefruit, which would be felt just above the suprapubic bone on bimanual exam.

What is the primary carbohydrate found in breast milk and commercial infant formulas? A) Fructose B) Lactose C) Glucose D) Sucrose

B) Lactose Lactose is the primary carbohydrate found in breast milk and formula

The sentinel nodes (Virchow's nodes) are found at the: A) Right axillary area B) Left supraclavicular area C) Posterior cervical chain D) Submandibular chain

B) Left supraclavicular area The sentinel nodes are found at the supraclavicular area of the chest. They are the first lymph nodes that a cancer lesion will drain into. Therefore, when cancer is diagnosed, these nodes are biopsied to see if the cancer has spread into the lymph system.

A 67-year-old retired clerk presents with complaints of shortness of breath and weight gain over a 2-week period. A nonproductive cough accompanies her symptoms. The lung exam is positive for fine crackles in the lower lobes with no wheezing. The exam is positive for egophony. Which of the following conditions is most likely? A) Acute exacerbation of asthma B) Left-heart failure C) Right-heart failure D) Chronic obstructive pulmonary disease

B) Left-heart failure Signs and symptoms of left-heart failure include tachypnea, labored breathing, and rales or crackles in the lower bases of the lungs, which can develop into pulmonary edema.

A kindergarten teacher is diagnosed with acute pharyngitis. On exam, the throat is a bright red color with no tonsillar exudate, and clear mucus is seen on the lower nasal turbinates. The result of her urinalysis shows a large amount of white blood cells and is positive for nitrites. The patient has a sulfa allergy and thinks she is also allergic to penicillins. Which of the following is the best treatment choice? A) Amoxicillin/clavulanic acid (Augmentin) 500 mg PO BID B) Levoquinolone (Levaquin) 250 mg PO daily C) Trimethoprim sulfamethoxazole (Bactrim DS) 1 tablet PO BID D) Clarithromycin (Biaxin) 500 mg PO BID

B) Levoquinolone (Levaquin) Pharyngitis and urinary tract infection are both covered by using Levaquin, which is a quinolone. Augmentin and Bactrim could not be used due to her allergies.

While performing a Pap smear on a postmenopausal patient, several areas of flat white skin lesions that are irregularly shaped are found on the patient's labia. The patient reports that the lesions are extremely itchy and have been present for several years without much change. Which condition is best described? A) Chronic scabies infection B) Lichen sclerosus C) Chronic candidal vaginitis D) A physiologic variant found in some older women

B) Lichen sclerosus Lichen sclerosus is a disease of the skin, in which white spots appear on the skin and change over time. It is most commonly seen in the genital and rectal areas, but can appear in other areas. The spots are usually shiny and smooth and can eventually spread into patches. The skin appears thin and crinkled. Then the skin tears easily, and bright red or purple bruises are common. Sometimes, the skin becomes scarred. If the disease is a mild case, there may be no symptoms

The gold-standard test for visualizing a torn meniscus or joint abnormalities is the: A) Computed tomography (CT) scan B) Magnetic resonance imaging (MRI) scan C) X-ray with special views of the affected knee D) Lachman's maneuver

B) Magnetic resonance imaging (MRI) scan MRI provides good visualization of soft tissues of the body (most cancers, brain, cartilage, muscles, inflammation, etc.). It is best used in tissues with high water content. Patients with metal implants such as cochlear implants and cardiac pacemakers should be carefully screened. The MRI does not use radiation, but uses strong magnetic and radio waves to visualize body structures.

A major risk factor for a Down syndrome infant is: A) Maternal age younger than 16 years B) Maternal age older than 35 years C) A positive family history of Down syndrome D) A positive family history of genetic disease

B) Maternal age older than 35 years Maternal age greater than 35 years during pregnancy is a risk factor for infant Down syndrome.

Koplik's spots are associated with: A) Poxvirus infections B) Measles C) Kawasaki's disease D) Reye's syndrome

B) Measles Signs and symptoms of the measles include fever over 101°F, coryza, cough, conjunctivitis, rash, and Koplik's spots on buccal mucosa.

An elderly female patient who is a retired nurse has recently been discharged from the hospital. A few days later, she started having random and recurrent episodes of dizziness, but denies passing out. The patient describes it as the sensation of the room spinning or moving, which is worsened by sudden head movement. During the episodes, she gets very nauseated and sometimes vomits. The patient reports that she was given IV antibiotics and one of them was tobramycin. Which of the following medications is helpful in treating her symptom of dizziness? A) Scopolamine patch (Transderm Scop) B) Meclizine (Antivert) C) Dimenhydrinate (Dramamine) D) Duloxetine (Cymbalta)

B) Meclizine (Antivert) The case is describing vertigo. Meclizine (Antivert) 12.5 mg to 50 mg TID to QID is used to treat vertigo. Do not forget to also treat nausea, which can be severe. Anti-nausea medicine like dimenhydrinate (Dramamine) or prochlorperazine (Compazine) are effective. Advise the patient that these drugs can cause drowsiness.

A 22-year-old male is brought to an urgent care center by his anxious mother. She reports that her son returned from a camping trip 2 days ago with a high fever and bad headache. Apparently, the patient had complained to her of a painful and stiff neck along with nausea shortly after he arrived. According to the mother, her son started breaking out in a rash the day before, some of which is turning a dark red to purple color. During the physical exam, the nurse practitioner evaluates the patient for Kernig's sign, which is positive. Which of the following conditions is most likely? A) Stevens-Johnson syndrome B) Meningococcemia C) Rocky Mountain spotted fever D) Erythema multiforme

B) Meningococcemia Kernig's maneuver is performed by flexing both hips and legs and having the patient straighten the legs against resistance, testing for men- ingitis. A positive test indicates meningitis. Other characteristics of meningitis include high fever, headache, stiff neck, and nausea/vomiting.

A 55-year-old woman who is on a prescription of clindamycin for a dental infection presents to the nurse practitioner with complaints of watery diarrhea for the past 4 days. She complains of abdominal cramping and bloating with diarrheal stools up to 10 times a day. She denies seeing blood or pus in her stool. There is no history of recent travel. The patient has been taking over-the-counter medicine with no relief. The nurse practitioner suspects that the patient has a mild case of Clostridium difficile colitis. Which of the following antibiotics is indicated for this infection? A) Ciprofloxacin (Cipro) 400 mg PO BID x 7 days B) Metronidazole (Flagyl) 500 mg PO TID x 10 days C) Levofloxacin (Levaquin) 750 mg PO daily x 7 days D) Trimethoprim-sulfamethoxazole (Bactrim DS) 1 tablet PO BID x 10 days

B) Metronidazole (Flagyl) 500 mg PO TID x 10 days First-line treatment for a mild case of Clostridium difficile colitis is metronidazole (Flagyl) 500 mg PO TID x 10 days. Discontinuation of the offending antibiotic (if possible) or switching to another antibiotic class is recommended. The role of probiotic supplementation is controversial. Complications are pseudomembranous colitis, toxic megacolon, and fulminant colitis.

A 35-year-old male presents complaining of the acute onset of episodes of dizziness with nausea that started a few days after he got over a cold. The patient describes it as the sensation of the room moving or of the room spinning. It is worsened by sudden head movement. During the episodes, he gets very nauseated. He also has tinnitus with hearing loss in his right ear. The patient is a type 2 diabetic and is on a prescription of metformin 500 mg PO BID and an ACE inhibitor. The blood glucose during the visit is 80 mg/dL. Which of the following conditions is most likely? A) Vasovagal presyncopal episode B) Ménière's disease C) Atypical migraine D) Hypoglycemia

B) Ménière's disease The classic triad of symptoms of Ménière's disease are vertigo, tinnitus, and hearing loss. The condition can resolve spontaneously or may be chronic. BPV has similar symptoms except that it does not cause hearing loss. Vertigo is due to the dysfunction of the labyrinthine system. Differential diagnoses are many (Ménière's, acute labyrinthitis, acoustic neuroma, etc.). Vasovagal syncope does not cause hearing loss or tinnitus, nor is it episodic.

Jean, a 68-year-old female, is suspected of having Alzheimer's disease. Which of the following is the best initial method for assessing the condition? A) Computed tomography scan of the brain B) Mini Mental Status Exam C) Obtain the history from the patient, friends, and family members D) EEG (electroencephalography)

B) Mini Mental Status Exam Begin with administering the MMSE for a baseline assessment, followed by taking a history from the patient and others.

During a routine physical exam of a 90-year-old woman, a low-pitched diastolic murmur Grade II/VI is auscultated. It is located on the fifth ICS on the left side of the midclavicular line. Which of the following is the correct diagnosis? A) Aortic regurgitation B) Mitral stenosis C) Mitral regurgitation D) Tricuspid regurgitation

B) Mitral stenosis Mitral stenosis is best heard at the fifth ICS to the left of the midclavicular line. It is ausculated as a low-pitched diastolic murmur, grade II/ VI. Aortic regurgitation is a high-pitched diastolic murmur, heard at the second ICS to the right of the sternum. Mitral regurgitation is a pansystolic murmur that radiates to the axilla, loud and high pitched when ausculated.

A 28-year-old student is seen in the school health clinic with complaints of a hacking cough that is productive of small amounts of sputum and a runny nose. He does not take any medications, denies any allergies, and has no significant medical history. Physical examination reveals a low-grade temperature of 99.9 degrees Fahrenheit, respirations of 16/min, a pulse of 90 beats per minute, and diffuse fine crackles in the base of the lungs. A chest radiograph (x-ray) shows diffuse infiltrates on the lower lobe of the right lung. The total white blood cell count is 10,500/uL. What is the most likely diagnosis? A) Streptococcal pneumonia B) Mycoplasma pneumonia C) Acute bronchitis D) Legionnaires disease

B) Mycoplasma pneumonia Mycoplasma pneumonia is the organism most com- monly seen in children and young adults. It is easily spread from droplets, from sneezing and coughing, in close proximity. Diagnosis is based on symptoms and x-ray results of infiltrates in lower lobes.

A lab technician has a 10.5-mm area of redness and induration in his left forearm after getting a Mantoux test 72 hours ago. The last test, which was done 12 months ago, was negative. He denies cough, night sweats, and weight loss. What is the next best intervention? A) Obtain a sputum culture and a chest x-ray B) Obtain a chest x-ray C) Obtain a sputum for C&S and an acid fast stain D) Obtain a CBC and chest x-ray

B) Obtain a chest x-ray Mantoux results are negative if results show less than 10 mm induration on the forearm for low-risk clients. If results are greater or equal to 10 mm, then a chest x-ray should be ordered for further screening.

A 68-year-old woman has recently been diagnosed with polymyalgia rheumatica. The nurse practitioner is discussing the treatment options with the patient. Which of the following medications is the first-line treatment for this condition? A) Etanercept (Enbrel) B) Oral prednisone C) Indomethacin D) Methotrexate

B) Oral prednisone Patients with PMR are treated with oral steroids (glucocorticoids). One of the hallmarks of the disorder is the dramatic improvement of symptoms after starting treatment with oral prednisone. Usually, the symptoms can be controlled with long-term (2-3 years) low-dose oral prednisone, which can be tapered when symptoms are under control. For most patients, PMR is a self-limiting illness (from a few months to 3 years).

A 65-year-old Hispanic woman has a history of type 2 diabetes. A routine urinalysis reveals a few epithelial cells and is negative for leukocytes, nitrites, and protein. Which of the following would you recommend next? A) Order a urine for C&S B) Order a 24-hour urine for microalbumin C) Because it is negative, no further tests are necessary D) Recommend a screening IVP (intravenous pyelogram)

B) Order a 24-hour urine for microalbumin Epithelial cells are cells that are present on the inside lining of the organs. Few epithelial cells can be normal; however, with her history of type 2 diabetes, a 24-hour urine for microalbumin should be ordered to assess kidney function.

A patient is positive for anti-HCV (hepatitis C virus antibody). What is the next step to further evaluate this patient? A) Refer the patient to a gastroenterologist B) Order a hepatitis C PCR (polymerase chain reaction) test C) Order a hepatitis B comprehensive panel D) The patient is immune to hepatitis C and no further testing is indicated

B) Order a hepatitis C PCR (polymerase chain reaction) test When a patient tests positive for anti-HCV (hepatitis C virus antibody), labs ordered should include hepatitis C PCR to further evaluate the patient.

You are following up a 65-year-old male who has been on a new prescription of fluvastatin (Lescol) for 6 weeks. During a follow-up visit, he reports feeling extremely fatigued and having dark-colored urine. He denies any generalized muscle soreness. Which of the following is the most appropriate treatment plan? A) Order a CBC with differential B) Order a liver function profile C) Recommend an increase in fluid intake and rest D) Order a urine for C&S test

B) Order a liver function profile Statin medications, such as Lescol, can affect liver function and increase liver enzymes, as well as cause the patient to feel weak, fatigued, and have muscle aches. Therefore, checking the liver function profile is recommended.

You are checking a 75-year-old woman's breast during an annual gynecologi- cal exam. The left nipple and areola are scaly and reddened. The patient denies pain and pruritis. She has noticed this scaliness on her left nipple for the past 8 months. Her dermatologist gave her a potent topical steroid, which she used twice a day for 1 month. The patient never went back for the follow-up. She still has the rash and wants an evaluation. Which of the following is the best intervention for this patient? A) Prescribe another potent topical steroid and tell the patient to use it twice a day for 4 weeks B) Order a mammogram and refer the patient to a breast surgeon C) Advise her to stop using soap on both breasts when she bathes to avoid drying up the skin on her areolae and nipples D) Order a sonogram and fine-needle biopsy of the breast

B) Order a mammogram and refer the patient to a breast surgeon A scaly, reddened rash on the breast that does not resolve after a few weeks of medical treatment may indicate breast cancer. She should have a mammogram performed and see a breast surgeon for evaluation and treatment.

You note the following result on a routine urinalysis of a 37-year-old primigravida who is at 30 weeks of gestation. Leukocyte = trace, nitrite = negative, protein = 2 +, blood = negative. Her weight has increased by 5 lbs during the past week. Which of the following is most likely? A) HELLP syndrome B) Pregnancy-induced hypertension (preeclampsia) C) Eclampsia of pregnancy D) Primary hypertension

B) Pregnancy-induced hypertension (preeclampsia) The classic triad of symptoms of preeclampsia includes hypertension, edema (weight gain), and proteinuria.

An elderly woman has been on digoxin (Lanoxin) for 10 years. Her electrocardio- graph (EKG) is showing a new onset of atrial fibrillation. Her pulse is 64/min. She denies syncope and dizziness. Which of the following interventions is most appropriate? A) Order an electrolyte panel and a digoxin level B) Order a serum thyroid-stimulating hormone (TSH), digoxin level, and an electrolyte panel C) Order a serum digoxin level and decrease her digoxin dose by half while waiting for results D) Discontinue the digoxin and order another 12-lead EKG

B) Order a serum thyroid-stimulating hormone (TSH), digoxin level, and an electrolyte panel Obtaining baseline blood work to evaluate for causes of new- onset atrial fibrillation is recommended prior to decreasing or stopping medications. Thyroid disease is a common cause of new-onset atrial fibrillation.

Glucosamine sulfate is a natural supplement that is used for which of the following conditions? A) Rheumatoid arthritis B) Osteoarthritis C) Osteoporosis D) Metabolic syndrome

B) Osteoarthritis Glucosamine sulfate has been found to have a beneficial effect on cartilage growth and repair. It may also have an anti-inflammatory effect. Many patients with OA who take it claim that it helps to relieve joint pain. It can take from 1 to 3 months of taking the medicine to feel its effects. Glucosamine is a compound made up of glucose and an amino acid.

What is the most common cause of infertility among women in the United States? A) Scarring of the fallopian tubes due to a history of pelvic inflammatory disease (PID) B) Ovulation disorders C) Age older than 35 years D) Endometriosis

B) Ovulation disorders Ovulation disorders are the top cause of female infertility (25%). There is no ovulation (anovulation) or infrequent ovulation that results in oligoamenorrhea (i.e., PCOS). The second cause is endometriosis (15%). About 10% of women in the United States (ages 15 to 44 years) have difficulty getting pregnant (CDC, 2009). PCOS is one of the most common causes of female infertility. Infertility in males is often caused by a varicocele (heats up the testes) and abnormal and/or low sperm count.

A 40-year-old cashier complains of periods of dizziness and palpitations that have a sudden onset. The EKG shows P waves before each QRS complex and a heart rate of 170 beats/minute. A carotid massage decreases the heart rate to 80 beats/min. These best describe: A) Ventricular tachycardia B) Paroxysmal atrial tachycardia C) Atrial fibrillation D) Ventricular fibrillation

B) Paroxysmal atrial tachycardia Signs/symptoms of paroxysmal atrial tachycardia include rapid, regular heart rate that begins and ends very quickly. The atria are beating at a very fast rate, but it is not life-threatening. Ventricular tachycardia is usually associated with heart disease, occurs when the ventricles are beating rapidly and inefficiently, and can lead to death if not treated. Atrial fibrillation is an irregular heartbeat that can be life threatening if not treated. Ventricular fibrillation occurs when the heartbeat is rapid and chaotic, and death will occur if the condition is not treated.

Which of the following is not a characteristic of delirium? A) Sudden onset B) Patient is coherent C) Worse in the evenings D) It has a brief duration

B) Patient is coherent Characteristics of delirium include an acute and dramatic onset of symptoms that is temporary, and usually will worsen in the evening. May last hours to days. Patient is incoherent and disoriented. Usually brought on by fever, shock, drugs, alcohol, or dehydration.

Hypovolemic shock would most likely occur with fractures of the: A) Spine B) Pelvis C) Femur D) Humerus

B) Pelvis Hypovolemic shock can occur from a fractured pelvis secondary to internal bleeding from a fractured bone fragment that lacerates an artery or vein. The pelvis can accommodate a large amount of fluid.

Which of the following conditions is a possible complication of severe eclampsia? A) Placenta previa B) Placenta abruptio C) Erythroblastosis fetalis D) Uterine rupture

B) Placenta abruptio Abruptio placenta is a possible complication of severe eclampsia. With elevated blood pressure, the placenta can pull away from the uterine lining and cause painful, bright red bleeding.

A 38-year-old multigravida who is at 32 weeks of gestation calls the family nurse practitioner complaining of bright red vaginal bleeding. There is no watery dis- charge. She complains that her uterus feels hard and is very painful. Which of the following conditions is most likely? A) Placenta previa B) Placenta abruptio C) A molar pregnancy D) An ectopic pregnancy

B) Placenta abruptio Abruptio placenta symptoms are bright red vaginal bleeding, board-like uterus on palpation, and pain. However, there can be concealed hemorrhage and the patient may not have vaginal bleeding. Placenta previa is painless bleeding. Ectopic and molar pregnancy would not progress to 32 weeks gestation

A 28-year-old multipara who is at 32 weeks of gestation presents to your office complaining of a sudden onset of small amounts of bright red vaginal bleeding. She has had several episodes and appears anxious. On exam, her uterus is soft to palpation. Which of the following is most likely? A) Placenta abruptio B) Placenta previa C) Acute cervicitis D) Molar pregnancy (hydatidiform mole)

B) Placenta previa Placenta previa occurs when abnormal implantation of the placenta occurs. A common symptom of placenta previa is painless, bright red bleeding.

Medicare Part D will reimburse for which of the following services? A) Preventive health care such as routine Pap smears and physical exams B) Prescription drugs C) Alcohol misuse/abuse counseling D) Over-the-counter drugs and vitamins

B) Prescription drugs Medicare Part D is a voluntary program that charges a premium. Like all Medicare services, patients need to enroll during the "open enrollment" periods during the year (there is a penalty for late enrollment). There is a drug formulary and not all drugs are available or reimbursed. Use of generic drugs is preferred, as there is a spending limit. Medicare Part B will reimburse for alcohol misuse/abuse treatment.

Folic acid supplementation is recommended for women who are planning pregnancy in order to: A) Prevent renal agenesis B) Prevent anencephaly C) Prevent kidney defects D) Prevent heart defects

B) Prevent anencephaly Folic acid supplementation during pregnancy has been shown to decrease the risk of neural tube defects of the fetus

Jane, a young primigravida, reports to you that she is starting to feel the baby's movements in her uterus. This is considered to be which of the following? A) Presumptive sign B) Probable sign C) Positive sign D) Possible sign

B) Probable sign Probable signs of pregnancy are symptoms of pregnancy that are felt by the woman.

The most important job of an institutional review board (IRB) is: A) Protecting the interests of the hospital or the research institution B) Protecting the rights of the human subjects who participate in research done at the institution C) Protecting the researcher and research team from lawsuits D) Evaluating research protocols and methodology for appropriateness and safety

B) Protecting the rights of the human subjects who participate in research done at the institution Every research institution has an institutional review board (IRB) whose job is to review all the research that is conducted in that institution. The IRB's most important role is to protect the rights of the human subjects who participate in research done at the institution of which the IRB is a part (e.g., research hospitals, universities).

You note, during a physical exam on a 6-year-old child, some pitting on the finger nails. This finding is correlated with: A) Iron-deficiency anemia B) Psoriasis C) Onychomycosis D) Vitamin C deficiency

B) Psoriasis Fingernail pitting is correlated with psoriasis. Psoriasis can cause pitting on all finger and toenails, along with thickening and irregular shape of the nail.

During a routine physical exam of an elderly woman, a triangular thickening of the bulbar conjunctiva on the temporal side is noted to be encroaching on the cornea. She denies any eye pain or visual changes. Which of the following is most likely? A) Corneal arcus B) Pterygium C) Pinguecula D) Chalazion

B) Pterygium Pterygium is a triangular growth on the white part of the eye that also extends onto the cornea. Corneal arcus is a ring around the edge of the cornea. Chalazion is a stye in the eye that may cause pain and swelling. Pinguecula is a benign growth on the conjuctiva caused by the degeneration of its collagen fibers. Thick, yellow fibers may be seen.

What type of follow-up should this patient receive? A) Refer him within 48 hours to a urologist B) Refer him to the emergency department as soon as possible C) Prescribe ibuprofen (Advil) 600 mg QID for pain D) Order a testicular ultrasound for further evaluation

B) Refer him to the emergency department as soon as possible Immediate referral to the emergency department is required. Success of treatment is usually 100% if treated within the first 6 hours and 0% if treated after 24 hours.

Which of the following is a true statement regarding pes planus in an infant? A) Pes planus should be evaluated by a pediatric orthopedist if it does not spontaneously correct itself by the age of 12 months B) The fat pads on an infant's feet can mimic pes planus C) It is always corrected by wearing special orthotic shoes D) It is also called talipes equinovarus

B) The fat pads on an infant's feet can mimic pes planus The fat pads on an infant's feet can resemble pes planus or flat feet.

Which type of exercise would you recommend to a 65-year-old arthritic patient who complains of a new onset of a painful, swollen left knee caused by gardening for 2 days? A) Quadriceps-strengthening exercises of the left knee followed by the application of cold packs for 20 minutes 4 times a day B) Rest the joint and apply cold packs intermittently for the next 48 hours C) Passive range of motion and cold packs D) A cool tub bath with warm packs on the knee to avoid stiffening of the joint

B) Rest the joint and apply cold packs intermittently for the next 48 hours New onset of a painful, swollen left knee caused by unusual activity should be treated with rest and alternating ice packs to rule out other causes of pain first

Mrs. Green, age 45, is complaining of generalized morning stiffness, especially in both her wrist and hands. It is much worse in the morning and lasts for a few hours. She also complains of fatigue and generalized body aches that have been present for the past few months. Which of the following is most likely? A) Osteoporosis B) Rheumatoid arthritis C) Osteoarthritis D) Gout

B) Rheumatoid arthritis Signs and symptoms of rheumatoid arthritis include symmetrical joint stiffness, swelling, and pain in the hands, wrists, elbows, ankles, and shoulders. Other common complaints include fatigue, malaise, fever, and generalized body aches.

A 15-month-old child who is eating and behaving normally is found to have a high fever. After a few days, the fever resolves and the infant breaks out in a maculopapular rash. This is a description of which of the following conditions? A) Erythema infectiosum B) Roseola infantum C) Fifth disease D) Scarlet fever

B) Roseola infantum The signs/symptoms of roseola infantum include high fever for a few days with a maculopapular rash occuring after the fever breaks. Fifth disease is a lacy-appearing rash that usually begins on the face and moves to the extremities and trunk. It is known by the "slapped cheek " appearance at onset. Erythema infectiosum is the same as Fifth disease. Scarlet fever is a red rash that feels like sandpaper that usually begins on the neck and trunk and spreads to the extremities. Patients will have other symptoms such as fever, sore throat, and beefy red tongue. This is caused by the Group A Streptococcus bacterium.

A 40-year-old White female with a BMI (body mass index) of 32 complains of col- icky pain in the right upper quadrant of her abdomen that gets worse if she eats fried food. During the physical exam, the nurse practitioner presses deeply on the left lower quadrant of the abdomen. After releasing her hand, the patient complains of pain on the right side of the lower abdomen. What is the name of this finding? A) Rebound tenderness B) Rovsing's sign C) Murphy's sign D) Psoas test

B) Rovsing's sign Rovsing's sign is referred pain to the opposite side of the abdomen after release of palpation.

A toddler with congenital heart disease is seen for a 1-week history of facial and lower-extremity edema accompanied by shortness of breath. The child's mother reports that the child's appetite has been poor. The chest x-ray reveals that the child has congestive heart failure (CHF). Which of the following heart sounds are found in these patients? A) S1 and S2 B) S1, S2, and S3 C) S1, S2, and S4 D) Still's murmur and S4

B) S1, S2, and S3 Congestive heart failure is the inability of the heart to pump a sufficient amount of blood to the organs to meet the body's requirements. It is common to hear S1, S2, and S3 heart sounds on exam. Common signs and symptoms include fatigue, shortness of breath with activity, and edema of lower extremities.

The Tdap vaccine should be encouraged at which time? A. Anytime, as long as it is during the pregnancy B. 28-34 weeks C. Immediately postpartum D. During the first trimester

B. 28-34 weeks

The cough associated with acute bronchitis typically lasts: A. 2 weeks B. 3 weeks C. 4 weeks D. 6 weeks

B. 3 weeks

A 16-year-old complains of a severe sore throat for 3 days along with a generalized rash and fever. The skin has the texture of fine sandpaper. This best describes: A) Kawasaki's disease B) Scarlet fever C) German measles D) Rubeola

B) Scarlet fever Scarlet fever is a bacterial infection caused by the group A Streptococcus infection. The patient will exhibit sore throat, high fever, and a fine sandpaper rash, which is characteristic of scarlet fever.

A 15-year-old female who attends a public school is referred to the nurse practitioner by one of her teachers. The teen has been missing school and is falling behind in her schoolwork. After closing the exam room door, the nurse practitioner starts to interview the teen, asking about her moods, her appetite, her sleep, whether she any plan of hurting herself or others, and other questions. What is the level of health prevention that the nurse practitioner is performing? A) Primary prevention B) Secondary prevention C) Tertiary prevention D) Dropout prevention program

B) Secondary prevention The nurse practitioner is evaluating the teenager for major depression. Because the teenager already has the disease (depression), it is a screening "test." All screening tests/labs (mammography, Pap smears, etc.) are secondary prevention activities.

A hypertensive middle-aged man who is Native American has recently been diagnosed with mild renal insufficiency. He has been on lisinopril (Accupril) for many years. Which of the following laboratory values should be carefully monitored? A) Hemoglobin, hematocrit, and the MCV (mean corpuscular volume) B) Serum creatinine and potassium levels C) Aspartate aminotransferase and alanine aminotransferase D) Serum sodium, potassium, and magnesium

B) Serum creatinine and potassium levels Native Americans have a much higher rate of kidney disease and renal failure when compared to other races. Native Americans have a 1 in 3 incidence of hypertension. Hypertension is the second leading cause of kidney failure. BUN, creatinine, estimated GFR, and urinalysis are performed to assess the function of the kidneys.

You would advise an 18-year-old female student who has been given a booster dose of MMR at the college health clinic that: A) She might have a low-grade fever during the first 24 to 48 hours B) She should not get pregnant within the next 4 weeks C) Her arm will be very sore on the injection site for 24 to 48 hours D) Her arm will have some induration on the injection site in 24to48hours

B) She should not get pregnant within the next 4 weeks MMR should not be administered to women known to be pregnant. In addition, women should be counseled to avoid becoming pregnant for 28 days following vaccination.

The nurse practitioner is evaluating a middle-aged female who is complaining of gradual weight gain, lack of energy, dry hair, and an irregular period over an 8-month period. The routine annual laboratory testing showed a TSH result of 10 mU/L. The nurse practitioner decides to order a thyroid profile. The TSH is 8.50 mU/L and the serum free T4 is decreased. During the physical exam, the patient's BMI is 28. The heart and lung exams are both normal. Which of the fol- lowing is best treatment plan? A) Advise the patient that because the TSH level has decreased, she does not have a thyroid problem anymore B) Start the patient on levothyroxine (Synthroid) 0.25 mcg PO daily C) Start the patient on Armour thyroid D) Refer the patient to an endocrinologist

B) Start the patient on levothyroxine (Synthroid) 0.25mcg PO daily The patient is symptomatic (weight gain, lack of energy, and irregular periods) with low free T4. Even though the TSH went down slightly, the free T4 remains low. An elevated TSH and low free T4 are indicative of hypothyroidism. The next step is to start the patient on levothyroxine (Synthroid) 0.25 mcg daily and recheck the TSH in 6 weeks. The goal is to normalize the TSH (between 1.0 and 3.5) and to ameliorate the patient's symptoms (increased energy, feels better, etc.). Armour thyroid (desic- cated thyroid) is a natural supplement composed of dried (desiccated) pork thyroid glands. It is used in alternative medicine as an alternative to synthetic levothyrox- ine/T4 (Synthroid).

A post menopausal woman's dual-energy x-ray absorption (DXA or DEXA) result shows osteopenia. Which of the following t-scores is indicative of osteopenia? A) T-score of -1.0 or higher B) T-score between -1. 0 to -2.5 C) T-score of less than -2.5 D) T-score of -2.50 to -3.50 D) Normal results

B) T-score between -1. 0 and -2.5 Osteopenia is defined as a t-score between -1. 0 and -2.5. Osteoporosis is defined as a t-score of less than -2.5.

A sexually active 16-year-old female is brought in by her mother for a physical exam. During the exam, the nurse practitioner notices some bruises on both breasts. All of the following are important areas to evaluate in this patient during this visit except: A) Depression B) Tanner stage C) Sexual history D) STDs

B) Tanner stage The stem of the question is asking for the important areas to evaluate in this patient "during this visit." This is a priority-type question. The priorities to evaluate in this patient are depression, STD testing, and sexual history. The Tanner staging does not have to be done "during this visit."

A 13-year-old adolescent female is brought in by her mother for a sports physical. The mother reports that the teen's last vaccines were given at the age of 6 years. Which of the following vaccines is recommended by the CDC for this patient? A) Td and HPV vaccines B) Tdap, MCV4, and the HPV vaccines C) DTap and the flu vaccine D) DT and MCV4 vaccines

B) Tdap, MCV4, and the HPV vaccines Vaccine questions usually are not this complicated, but there are several lessons that can be learned with this question. The 2012 CDC recommendations for the ages 13 to 18 years are the Tdap catch-up (if did not receive at age 11-12 years), HPV or Gardasil catch-up (if did not receive at age 11-12 years), and the MCV4 or meningococcal conjugate vaccine (Menactra). Only one dose of Tdap is recommended (lifetime). Thereafter, the Td form of the vaccine is indicated every 10 years.

Laura, who is 18 years old, is being followed up for acne by the nurse practitioner. The facial exam reveals that papules and pustules are present mostly on the forehead and the chin areas. The patient has been using prescription topicals and OTC medicated soap daily for 6 months without much improvement. Which of the following would you recommend next? A) Isotretinoin (Accutane) B) Tetracycline (Sumycin) C) Clindamycin topical solution (Cleocin T) D) Minoxidil (Rogaine)

B) Tetracycline (Sumycin) First-line treatment for acne vulgaris includes OTC medicated soap and water with topical antibiotic gels. The next step of treatment would be the initiation of oral tetracycline.

All of the following are true statements about the human papilloma virus vaccine (Gardasil) except: A) The Centers for Disease Control and Prevention (CDC) recommends the first dose at age 11 to 12 years B) The CDC does not recommend the HPV vaccine for males C) The vaccine is not complete until a total of 3 doses have been administered D) The minimum age the vaccine can be given is 9 years

B) The CDC does not recommend the HPV vaccine for males The HPV vaccine is now recommended for both males and females. It can be given until the age of 26 years, especially if the individual is at high risk. Do not use the vaccine if age is less than 9 years.

A 14-year-old female with amenorrhea is tested for pregnancy and has a positive result. The patient tells the NP that she is seriously considering terminating the pregnancy. She tells the NP that she wants to be referred to a Planned Parenthood clinic. The NP's personal beliefs and religious beliefs are pro-life. Which of the fol- lowing is the best action for the NP? A) The nurse practitioner should tell the patient about her personal beliefs and advise her against getting an abortion B) The NP advises the patient that an NP peer who is working with the NP can help answer her questions more thoroughly C) The NP should excuse herself from the case D) The nurse practitioner should refer the patient to an obstetrician

B) The NP advises the patient that an NP peer who is working with the NP can help answer her questions more thoroughly In general, discussing personal beliefs is considered unprofessional behavior. Respecting the patient's right to choose is an example of supporting patient autonomy.

The mother of a 4-month-old calls your office and reports that the infant has a fever of 101.4 degrees Fahrenheit. The infant received her immunizations yesterday. Which of the following is correct? A) The fever is most likely due to the combination of the MMR and polio vaccines B) The fever is most likely due to the pertussis component of the DTP vaccine C) The infant is probably starting a viral upper respiratory infection D) The infant had an allergic reaction to one of the vaccines given and should be brought to the emergency room

B) The fever is most likely due to the pertussis component of the DPT vaccine The pertussis component of the vaccine is most likely the cause of the fever.

The Patient Protection and Affordable Care Act is President Obama's plan for national insurance coverage. All of the following are true statements about this law except: A) Preexisting health conditions will be covered immediately B) The health plan will start to be effective in the year 2013 C) Young adults up to the age of 26 years who live with their parents will be covered under their parents' health insurance plan D) The health insurance plan mandates will fine employers who choose not to participate in the national health insurance plan

B) The health plan will start to be effective in the year 2013 President Obama's national health insurance plan is known the "Patient Protection and Affordable Care Act." It covers preexisting health conditions immediately and prohibits an insurance company from rejecting people with preexisting health conditions. There is a penalty for employers (and individuals) who choose not to participate in the national health plan.

A 70-year-old woman complains of left lower quadrant abdominal pain and fever for 2 days. Her blood pressure of 130/80, pulse 90/minute, respirations 14/minute and a temperature of 100.5°F. During the abdominal exam, the left lower quadrant of the abdomen is tender to palpation. The NP does not palpate a mass, guarding, or rigidity. Rovsing's sign is negative. Bowel sounds are present in all quadrants. The nurse practitioner is familiar with the patient, who is alert and is asking appropriate questions about her condition. The nurse practitioner suspects that the patient has acute diverticulitis. Which of the following treatment plans is appropriate for this patient? A) The patient should be referred to the physician as soon as possible B) The patient has a mild case of acute diverticulitis and can be treated with antibiotics in the outpatient setting with close follow-up C) This patient has a moderate to severe case of acute diverticulitis and needs to be admitted to the hospital for IV antibiotics D) Refer the patient to the emergency department as soon as possible

B) The patient has a mild case of acute diverticulitis and can be treated with antibiotics in the outpatient setting with close follow-up The patient has a mild case of acute diverticulitis and can be treated as an outpatient with antibiotics and a clear fluid diet. If outpatient treatment is selected, close follow-up (within 24 to 48 hours) is very important. Instruct patients to go to the hospital if symptoms get worse, if fever increases, if unable to tolerate PO treatment, and if pain worsens. Order CBC (leukocytosis, neutrophils, and possible shift to the left), chemistry profile, and urinalysis (to rule out renal causes)

What is the clinical significance of the above finding on a 35-year-old? A) The patient has a higher risk of blindness B) The patient should be evaluated for hyperlipidemia C) The patient should be evaluated by an ophthalmologist D) The patient should be evaluated for acute glaucoma

B) The patient should be evaluated for hyperlipidemia Arcus senilis is caused by lipid deposits deep in the edge of the cornea.

A split S2 is best heard at which of the following areas? A) The aortic area B) The pulmonic area C) The tricuspid area D) The mitral area

B) The pulmonic area The split S2 is best heard at the pulmonic area of the heart.

The major difference between the quasi-experimental design is which of the following? A) The quasi-experimental design is a type of observational study B) The quasi-experimental design uses convenience sampling instead of random sampling to recruit subjects C) The quasi-experimental design is also known as a survey D) The quasi-experimental design does not have an intervention group

B) The quasi-experimental design uses convenience sampling instead of random sampling to recruit subjects The quasi-experimental design uses an intervention (it is not an observational study or a survey). It has many similarities to an experimental study except that the human subjects are recruited by convenience and not at random (as in an experimental study).

The research term/symbol that is used to indicate the "total population" in a research study is: A) The symbol called "n = " B) The symbol called "N =" C) The symbol called "p =" D) The symbol called "P ="

B) The symbol called "N = " The correct symbol to indicate total number of subjects in a study (total population) is "N." For example, a research study has a total number of subjects or total population of 100 (N = 100). The small letter n is used to indicate a subpopulation. For example, a study uses a total population of N = 100 that is divided into 2 groups of 50 subjects (n = 50).

17-10. Which of the following newborns warrants intervention with phototherapy? A. 12 hours old, total bilirubin 1.5 mg/dL B. 36 hours old, total bilirubin 16 mg/dL C. 60 hours old, total bilirubin 17 mg/dL D. 72 hours old, total bilirubin 12 mg/dL

B. 36 hours old, total bilirubin 16 mg/dL

The following statements are all true regarding herpes zoster except: A) It is due to reactivation of latent varicella virus B) The typical lesions are bullae C) It is usually more severe in immunocompromised individuals D) Infection of the trigeminal nerve ophthalmic branch can cause corneal blindness

B) The typical lesions are bullae Herpes zoster occurs secondary to reactivation of the varicella virus. This infection can be more severe in the immunocompromised patient due to the inability to fight infection. This rash appears as a vesicular, painful rash, commonly in clusters, following one dermatome on one side of the body. When the trigeminal nerve is involved, there is an increased risk of corneal blindness.

During a sports physical exam, a1 6-year-old patient is noted to have a few beats of horizontal nystagmus on extreme lateral gaze that disappear when the eyes move back toward midline. Which statement best describes this clinical finding? A) It is caused by occult bleeding of the retinal artery B) This is a normal finding C) It is a sign of a possible brain mass D) This is a borderline result and requires further evaluation

B) This is a normal finding Horizontal nystagmus is a normal variation on physical exam

While reviewing some lab reports, the nurse practitioner notes that one of her teenage male patient's lab results is abnormal. The liver function tests are all normal except for a slight elevation in the alkaline phosphatase level. The patient is a member of a soccer team and denies any recent injury. What is the next step in this patient's evaluation? A) Order a liver ultrasound to rule out fatty liver B) This is a normal finding for his age group C) The patient needs to be evaluated further for pancreatic disease D) Refer the patient to a pediatric rheumatologist

B) This is a normal finding for his age group Elevated levels of alkaline phosphatase are commonly seen in growing children. Growth of bone tissue elevates the alkaline phosphatase levels.

Which of the following is not a relative contraindication for oral contraceptive pills? A) Active hepatitis A infection B) Thrombosis related to an IV needle C) Undiagnosed vaginal bleeding D) Migraine headache without focal aura

B) Thrombosis related to an IV needle Thrombosis related to either a known trauma or IV needle does not represent a contraindication for use of oral contraceptives.

The best screening test for detecting and monitoring both hyperthyroidism and hypothyroidism is: A) The total T3-4 B) Thyroid-stimulating hormone (TSH) C) Thyroid profile D) Palpation of the thyroid gland

B) Thyroid-stimulating hormone (TSH) The best "screening" test of hypothyroidism and hyperthyroidism is the TSH. If this result is abnormal, then further diagnostic tests should be performed.

Rocky Mountain spotted fever is caused by the bite of the: A) Mosquito B) Tick C) Insect D) Flea

B) Tick Rocky Mountain spotted fever is caused by a tick bite from a tick that was infected with the parasite Rickettsia rickettsii. The mortality rate is 1%-7% if left untreated.

The best test for diagnosing congenital glaucoma is which of the following? A) Fluorescein staining B) Tonometry C) Snellen vision exam D) The refractive index

B) Tonometry A tonometer is used to measure the intraocular pressure (IOP) of the eye to screen for glaucoma. Normal range IOP is 10-22 mm Hg.

Which of the following is useful in the primary care area when evaluating a patient for possible acute sinusitis or hydrocele? A) Checking for the cremasteric reflex B) Transillumination C) Ultrasound D) CT scan

B) Transillumination Transilliumination is a technique used with a light source, such as the otoscope, to visualize fluid below the skin surface, which will appear as a "glow." Ultrasound and CT scan are not readily available in the primary care setting; the patient must be sent to radiology for these tests to be performed

The nurse practitioner calls a patient to discuss the results of routine laboratory tests, which are all normal. She calls the patient twice and each time, the answering machine is on. Which of the following is the most appropriate action for the nurse practitioner? A) Because they are all normal, leave the laboratory results on the answering machine B) When the nurse practitioner is unable to speak with the patient directly, she should leave a message with her name and telephone number and instruct the patient to call back C) Some clinics have a policy of not calling patients back if their laboratory results are normal D) It is up to the physician to determine whether a laboratory result can be discussed with a patient on the telephone

B) When the nurse practitioner is unable to speak with the patient directly, she should leave a message with her name and telephone number and instruct the patient to call back It is against HIPAA regulations to leave laboratory results on an answering machine even if they are normal, routine laboratory tests. HIPAA is also known as the "Privacy Rule."

What does a KOH (potassium hydroxide) prep help the nurse practitioner diagnose? A) Herpes zoster infections B) Yeast infections C) Herpes simplex infections D) Viral infections

B) Yeast infections The KOH prep test is performed by placing a sample of dis- charge on a glass slide, with one drop of potassium hydroxide and a coverslip on top to evaluate for yeast infections. Budding spores and pseudohyphae will be seen with Candida, with a pH of 3.5-4.5.

Which of the following pharmacologic agents is the best choice for an elderly patient with insomnia? A) Diazepam (Valium) B) Zolpidem (Ambien) C) Temazepam (Restoril) D) Diphenhydramine (Benadryl)

B) Zolpidem (Ambien) Ambien has a quick onset of action (15 minutes) and a short half-life of 2 hours. Avoid diphenhydramine in the elderly, as there is a higher incidence of adverse effects (confusion, prolonged sedation) in this population. Avoid long-acting benzodiazepines such as diazepam (half-life 12 hours) and temazepam (half-life of 10 hours). Hypnotics can be used PRN up to 2 weeks; otherwise, benzodiazepines can cause addiction and withdrawal symptoms.

The ELISA and Western Blot tests are both used to test for HIV. Which of the following statements is correct? A) It is a test to detect viral RNA B) A positive ELISA screening does not mean the person has HIV infection C) It is a test to detect viruses D) It is a diagnostic test for the AIDS virus

B)A positive ELISA screening does not mean the person has HIV infection The ELISA test is always followed by a Western blot test to confirm diagnosis.

All of the following drugs can interact with theophylline (Theo-24) except: A) Erythromycin B) Montelukast (Singulair) C) Phenytoin sodium (Dilantin) D) Cimetidine (Tagamet)

B)Montelukast(Singulair) Medications that are contraindicated with theophylline include erythromycin, phenytoin sodium, and cimetidine.

During a breast exam of a 30-year-old nulliparous female, the nurse practitioner palpates several rubbery mobile areas of breast tissue. They are slightly tender to palpation. Both breasts have symmetrical findings. There are no skin changes or any nipple discharge. The patient is expecting her menstrual period in 5 days. Which of the following would you recommend? A) Referral to a gynecologist for further evaluation B) Tell her to return 1 week after her period so her breasts can be rechecked C) Advise the patient to return in 6 months to have her breasts rechecked D) Schedule the patient for a mammogram

B)Tell her to return 1 week after her periods so her breasts can be rechecked Prior to having menses, women will experience breast changes of tenderness with palpation, with symmetrical soft, mobile areas of breast tissue that are benign. Her symptoms should improve after her menses and repeating the exam in 1 week would be recommended.

Paige is a 35-year-old female who calls the primary care clinic about a sneeze, runny nose, and cough. She wants to know if she should come into the clinic. (The most important question to ask the patient is: A. "Do you smoke?" B. "How long have you had the symptoms?" C. "Have you traveled out of the country recently?" D. "Are you running a fever?"

B. "How long have you had the symptoms?"

11-12. The NP is providing care for a patient diagnosed with Alzheimer's dementia. The patient's son states that his mom often wanders. The NP discusses with the son ways to ensure that his mom remains safe. Which response requires immediate follow-up by the NP? A. "I heard that using motion detectors on doors within the home can be helpful" B. "I usually leave the house keys and car keys on a rack by the door because I often misplace them." C. "I keep emergency phone numbers posted on the refrigerator. D. "I was able to get a home health aide to stay with my mom during the day when I am not home.

B. "I usually leave the house keys and car keys on a rack by the door because I often misplace them."

13-31. Which of the following is the normal laboratory range for thyroid-stimulating hormone? A. 0.7-1.9 ng/dL B. 0.4-4.0 mIU/L C. 4.6-12 ug/dL D. 80-180 ng/dL

B. 0.4-4.0 mIU/L

13-9. Peak ages for developing type 1 diabetes include: A. 1-2 years old B. 10-14 years old C. 16-18 years old D. Early adulthood

B. 10-14 years old

13-8. Prediabetic patients can be identified by a fasting glucose blood test of: A. <100 mg/dL B. 100-125 mg/dL C. 126-150 mg/dL D. 150-200 mg/dL

B. 100-125 mg/dL

13-23. How many grams of fast-acting glucose should a patient ingest to treat asymptomatic and symptomatic hypoglycemia? A. 10-15 g B. 15-20 g C. 20-25 g D. 25-30 g

B. 15-20 g

13-24. To meet diagnostic criteria for metabolic syndrome, a patient's triglyceride level should be greater than or equal to: A. 100 mg/dL B. 150 mg/dL C. 200 mg/dL D. 250 mg/dL

B. 150 mg/dL

16-33. The FNP is caring for several individuals with skin infections. Which individual is most likely to have impetigo? A. 17-year-old team athlete B. 27-month-old in day care C. 45-year-old landscaper D. 78-year-old in long-term care

B. 27-month-old in day care

19-31. The Affordable Care Act provision restricting health insurers from denying coverage or increasing insurance premiums to persons with pre-existing conditions applies to all of the following situations except: A. A 25-year-old woman who is 32 weeks pregnant and applying for insurance during open enrollment B. A 30-year-old man who is self-employed with a private insurance plan purchased through a local insurance carrier, now newly diagnosed with diabetes C. A 25-year-old man applying for coverage under his parent's insurance plan who does not live with his parents D. A 43-year-old woman who tested positive for BRCA 1 mutation last year and is now applying for insurance through a new employer

B. A 30-year-old man who is self-employed with a private insurance plan purchased through a local insurance carrier, now newly diagnosed with diabetes

3-14. According to the USPSTF, which population has the highest risk of contracting HIV and should be screened? A. A 25-year-old patient with a negative HIV test last year B. A 35-year-old patient recently diagnosed with hepatitis B contracted from IV drug use C. A 42-year-old patient with a 10-year history of smoking marijuana D. A 72-year-old patient in a monogamous sexual relationship

B. A 35-year-old patient recently diagnosed with hepatitis B contracted from IV drug use

4-3. Which of the following describes and agonist? A. A chemical that has a harder time crossing the blood-brain barrier B. A chemical that binds to a receptor and stimulates cellular activity C. A chemical that binds to a receptor and blocks or inhibits cellular activity D. A chemical that has a longer half-life than a competing chemical

B. A chemical that binds to a receptor and stimulates cellular activity

18-12. What statement about advance care directives (ACDS) is incorrect? A. An advance care directive is related to end-of-life care B. A living will is more legally binding than a POLST C. An advance care directive describes what medical interventions one does and does not want D. An advance care directive defines the individual who is chosen to make healthcare decisions

B. A living will is more legally binding than a POLST

14-25. Splenomegaly may be an objective finding with all of the following leukemias except: A. Acute lymphocytic leukemia (ALL) B. Acute myelogenous leukemia (AML) C. Chronic lymphoblastic leukemia (CLL) D. Chronic myelogenous leukemia (CML)

B. Acute myelogenous leukemia (AML)

What should the nurse practitioner recommend to the parents of a 3-year-old with gastroenteritis? A. Provide sips of flat soda pop B. Allow sips of oral rehydration solution C. Nothing by mouth for 12 hours after a vomiting episode D. Any fluid she will drink is permissible

B. Allow sips of oral rehydration solution

Which of the following is true considering the use of antidiarrheal medication for the treatment of gastroenteritis? A. The drugs are curative for gastroenteritis B. Antidiarrheal medications are not recom- mended for use in children C. These medications help with hydration status D. Only bacterial causes require these drugs

B. Antidiarrheal medications are not recom- mended for use in children

Of the following measures, which is recommended for preventing otitis media? A. Allowing supine bottle feeding B. Avoiding secondhand smoke C. Placing in a day care setting D. Allowing pacifier use

B. Avoiding secondhand smoke

A new patient is being interviewed by an NP, and states she had a gastrectomy procedure 5 years ago. The NP knows that this patient is at risk for which of the following? A. Folate deficiency anemia B. b12 deficiency anemia C. Iron deficiency anemia D. Normocytic anemia

B. B12 deficiency anemia: intrinsic factor is made by parietal cells in fundus of the stomach. IF is needed to sufficiently absorb B12. Gastrectomy pts have damaged fundus usually so they are at risk.

15-1. Which of the following white blood cell elevations as noted on the differential is associated with parasitic infections? A. Banded neutrophils B. Basophils C. Lymphocytes D. Neutrophils

B. Basophils

19-12. Medicaid pays for nearly one-half of: A. Long-term care in the U.S. B. Births in the U.S. C. Hospice care D. Respiratory care

B. Births in the U.S.

16-15. A 31-year-old female presents to the clinic with concerns about a mole on her back that seemed to "appear overnight." Upon further investigation, you learn that her father has a history of squamous cell carcinoma, she had several blistering sunburns as a teenager, and she wears sunblock with SPF 20 most days. Upon physical exam, you note that she is fair- skinned, blonde, and blue-eyed. Which of the fol- lowing risk factors is most concerning for malignant melanoma? A. Family history of squamous cell carcinoma B. Blistering sunburns as a teenager C. Use of 20 SPF sunblock D. Fair-skinned, light eyes/hair

B. Blistering sunburns as a teenager

A patient presents to the ER with fever, chills, and Roth spots and reports a history of rheumatic valvular disease. The nurse practitioner is suspicious of infective endocarditis. It is appropriate to order which of the following tests initially? A. Chest x-ray and lipid panel B. Blood culture and transthoracic echocardiogram C. Cardiac MRI and guaiac stool test D. Initial blood culture and cardiac stress test

B. Blood culture and transthoracic echocardiogram

15-6. The nurse practitioner is assessing a patient who reports new onset of diplopia and inability to move certain muscles. Which diagnosis does this suggest? A. Anthrax B. Botulism C. Ebola virus D. Lassa fever

B. Botulism

11-4. The nurse practitioner asks the patient to "open up and say AHH." This assesses which cranial nerves? A. CN IX and XI B. CN IX and X C. CN X and XI D. CN XI and XII

B. CN IX and X

11-1. Audrey, a 53-year-old woman, reports a rapid onset of right-sided facial weakness. She is unable to turn a smile on the right side and unable to close her right eye fully. This presentation is likely paralysis of which cranial nerve? A. CN V B. CN VII C. CN IX D. CN X

B. CN VII

13-27. Risk factors for developing metabolic syndrome include all of the following except: A. Obesity B. Cardiovascular disease C. Smoking D. Sedentary lifestyle

B. Cardiovascular disease

17-3. At the two-week visit, the nurse practitioner notes clear drainage from the right eye with mild lower lid redness. Which treatment is appropriate? A. Oral antibiotics B. Topical antibiotics C. Nasolacrimal duct massage D. Referral to ophthalmology

C. Nasolacrimal duct massage

An 18-year-old male patient presents for routine follow-up on asthma that was diagnosed at age 6. The current medication regimen includes fluticasone (Flovent) 44mcg/actuation 2 puffs BID inhaled with albuterol (ProAir FHA) as a rescue inhaler. The patient tells you he uses fluticasone daily as prescribed, and he has used the albuterol inhaler twice daily for the last month. What would be your plan of care for this patient using a stepwise approach? A. Add Montelukast (Singulair) to his current regimen B. Change fluticasone (Flovent) to 110mcg/actuation, 2 puffs BID C. Have patient keep track of peak flow measurements and return to clinic for follow-up in 1 week D. Refer to a pulmonologist

B. Change fluticasone (Flovent) to 110mcg/actuation, 2 puffs BID

3-9. The CDC reports that the costliest healthcare expenditures in the U.S. are attributed to: A. Ebola and dengue fever B. Chronic, preventable diseases C. Tick-borne diseases, such as Lyme disease D. Motor vehicle accidents

B. Chronic, preventable diseases

16-35. Which description of clinical manifestations best characterizes acne vulgaris? A. Blackened skin on certain body parts B. Comedones, papules, pustules C. Widespread facial redness D. Pruritus, erythema, flakiness

B. Comedones, papules, pustules

4-13. Jennifer P. is a 58-year-old female being treated with cisplatin for breast cancer. The nurse practitioner knows that part of the management plan should include frequent testing of which lab test? A. Serum glucose B. Complete blood count (CBC) C. Urinalysis D. Uric acid

B. Complete blood count (CBC)

Which is a true statement about viral hepatitis? A. 1 in 2 acute cases of hepatitis E will end with liver failure B. Concurrent hepatitis B and hepatitis D infection tends to result in a poorer prognosis than hepatitis B infection alone C. Hepatitis D can only be contracted by a patient who already has hepatitis A D. There is no immunization for hepatitis A

B. Concurrent hepatitis B and hepatitis D infection tends to result in a poorer prognosis than hepatitis B infection alone

18-8. Certain medical conditions can place an elderly patient at risk for a fall. These conditions include all of the following except: A. Dementia B. Constipation C. Dehydration D. Aortic stenosis

B. Constipation

11-18. During the neurological exam, the nurse practitioner asks the patient to walk heel to toe. This test assesses which function of the cerebellar system? A. Mental status B. Coordination C. Motor strength D. Reflex system

B. Coordination

16-16. Which of the following is not a common irritant in contact dermatitis? A. Perfumes B. Cotton C. Plants D. Nickel

B. Cotton

According to the guidelines by the American College of Chest Physicians (ACCP), annual screening with low-dose computed tomography (LDCT) for lung cancer should occur in which of the following patient populations? A. Current smokers aged 50 and older with at least a 20 pack-year smoking history B. Current smokers aged 55-74 years with at least a 30 pack-year smoking history C. Former smokers aged 55-74 years with at least a 30 pack-year smoking history who have quit within the past 20 years D. Current smokers aged 50 and older with at least a 20 pack-year smoking history and at least one additional risk factor

B. Current smokers aged 55-74 years with at least a 30 pack-year smoking history

19-35. Which of the following is false regarding reimbursement? A. Nurse practitioners are typically reimbursed at 85% of physician rates B. Diagnostic testing will always be reimbursed as long as the reason for testing is documented in the plan of care C. Rates of reimbursement for Medicare are based on RBRVS D. Private insurance typically reimburses an E&M visit at 105% of what Medicare pays

B. Diagnostic testing will always be reimbursed as long as the reason for testing is documented in the plan of care

12-31. Treatment of depression may include all of the following except: A. Selective serotonin reuptake inhibitors B. Dialectical behavior therapy C. Cognitive behavioral therapy D. Interpersonal therapy

B. Dialectical behavior therapy

You examine a 56-year-old patient with type 2 DM who is at the office for follow-up 4 weeks after initiating statin therapy with atorvastatin. The patient states, "I have had mild aches and pains since starting this medication." The NP should perform which of the following actions? A. Discontinue statin use and initiate niacin treatment B. Discontinue statin and initiate a lower dose of different statin until adverse effects improve C. Encourage daily muscle strengthening exercises D. Explain to the patient that the addition of fenofi- brate will help improve symptoms

B. Discontinue statin and initiate a lower dose of different statin until adverse effects improve

17-1. The nurse practitioner notes a single palmar crease on a young infant. This finding is likely to be associated with which disorders? A. Down syndrome and Turner syndrome B. Down syndrome and fetal alcohol spectrum disorder C. Klinefelter syndrome and Turner syndrome D. Fetal alcohol spectrum disorder and Klinefelter syndrome

B. Down syndrome and fetal alcohol spectrum disorder

Ninety percent of patients are asymptomatic at the time of primary tuberculosis (TB) infection. As the disease progresses, clinical findings include all of the following except: A. Night sweats B. Dyspnea C. Diurnal fever D. Dry cough

B. Dyspnea

Which of the following statements regarding colorectal cancer is false? A. Colon cancer is more common than cancers involving the rectum B. Early manifestations include colicky abdominal pain and constipation W C. Most colorectal cancers are not found during physical examination D. Colorectal cancer is the third leading cause of cancer deaths for both sexes in the U.S.

B. Early manifestations include colicky abdominal pain and constipation W

11-43. Which of the following is accurate regarding the workup of GBS? A. An erythrocyte sedimentation rate within nor- mal levels rules out GBS B. Electromyography and nerve conduction studies are routinely measured in the workup of GBS C.A cerebrospinal fluid (CSF) level with normal protein levels and elevated WBC is suggestive of GBS D. MRI is both sensitive and specific for diagnosis of GBS

B. Electromyography and nerve conduction studies are routinely measured in the workup of GBS

16-10. A 5-year-old patient has mild eczema on her cheeks. Her mother prefers to use a leftover prescription for topical clobetasol cream 0.05% ( Temovate ) instead of going to the pharmacy for a new prescription. The nurse practitioner uses the following elements to determine if this medication can be used except: A. Patient's age B. Expiration date of the medication C. Location of the condition D. Severity of the condition

B. Expiration date of the medication

16-34. The FNP student recalls that onychomycosis is an infection of the toenail or fingernail and recognizes that the infection is caused by a: A. Bacteria B. Fungus C. Parasite D. Virus

B. Fungus

A 42-year-old male presents to clinic complaining of fatigue and mild edema. Upon auscultation of his chest wall, the NP hears a loud systolic murmur, but it has no palpable thrill. Upon documentation, how would the nurse practitioner grade this murmur? A. Grade 1 B. Grade 3 C. Grade 4 D. Grade 6

B. Grade 3

11-26. Of the following bacterial causes of meningitis, which is not vaccine-preventable? A. Neisseria meningitidis B. Group B Streptococcus C. Haemophilus influenzae type B D. Streptococcus pneumoniae

B. Group B Streptococcus

19-33. CPT codes for an evaluation and management (E&M) visit are based on documentation of: A. HPI, exam findings, diagnosis, and time spent with the patient B. HPI, ROS, exam findings, and diagnosis C. Patient's demographic information, HPI, PMFSH, and diagnosis D. HPI, exam findings, insurance information, diagnosis, and treatment plan

B. HPI, ROS, exam findings, and diagnosis

17-7. A 4-year-old has a five-day history of high fever and a rash. Which set of additional signs would be necessary for a diagnosis of Kawasaki disease? A. Strawberry tongue, scarlatina-form rash on trunk, pharyngitis B. Hand desquamation, conjunctivitis, strawberry tongue C. Football-shaped lesions on hands, feet, and buccal mucosa D. Vesicular lesions in various stages, headache

B. Hand desquamation, conjunctivitis, strawberry tongue

4-11. Lucas M. is a 65-year-old male who presents with elevated liver enzymes, and further testing indicates he has hepatitis C (HCV). All of the following are true of pharmacologic treatment of HCV except. A. Test all patients for evidence of current or prior hepatitis B virus (HBV) infection before initiating treatment with HCV direct acting antivirals B. Hepatitis C treatment is generally swift, simple, and straightforward C. The most common initial treatments are combination medications based on the genotype of the virus D. Patients should be evaluated for co-infection with HIV and the presence of cirrhosis

B. Hepatitis C treatment is generally swift, simple, and straightforward

Which common findings should the NP monitor for in a patient diagnosed with infectious mononucleosis? A. Hepatomegaly, splenomegaly, heart failure B. Hepatomegaly, lymphocytosis, splenomegaly C. Heart failure, lymphocytosis, hepatomegaly D. Renal failure, hepatomegaly, splenomegaly

B. Hepatomegaly, lymphocytosis, splenomegaly

14-7. The five-year survival rate for lymphoma is: A. Higher for non-Hodgkin lymphoma (NHL) than for Hodgkin lymphoma (HL) B. Higher for Hodgkin lymphoma (HL) than for non-Hodgkin lymphoma (NHL) C. The same for Hodgkin lymphoma (HL) and non-Hodgkin lymphoma (NHL) D. Poor, as with all cancers of the blood

B. Higher for Hodgkin lymphoma (HL) than for non-Hodgkin lymphoma (NHL)

Risk factors for developing a pulmonary embolism including Virchow's triad, genetics, and a previous history of a pulmonary embolism. Which three conditions comprise Virchow's triad? A. Prolonged immobility, hypercoagulability, and venous stasis B. Hypercoagulability, vessel wall injury, and venous stasis C. Trauma, venous stasis, and peripheral vascular disease D. Vessel wall injury, dehydration, and Factor V Leiden

B. Hypercoagulability, vessel wall injury, and venous stasis

13-2. Which of the following is a sign of Addison's disease? A. Glucose intolerance B. Hypotension C. Acne D. Hypertension

B. Hypotension

Thomas is a 55-year-old with a history of diabetes and COPD. His medications include metformin (Glucophage) 500 mg BID, simvastatin (Zocor) 10 mg nightly, lisinopril/HCTZ (Zestoretic) 20/12.5 mg every AM and salmeterol (Serevent) 50 mg/dose 1 puff BID albuterol (ProAir HFA) 90 mcg/actuation prn, which he uses 2-3 times daily. He presents for follow-up on HTN. His home readings have been 145-155/92-105 for the last 3 months. What would be the next step in the treatment plan? A. Advise to start walking 45 mins/day, 5 days/week B. Increase lisinopril to 30 mg daily C. Increase the HCTZ to 25 mg daily and change to PM dosing D. Add carvedilol (Coreg) 6.25 mg daily and have patient follow up in 1 month

B. Increase lisinopril to 30 mg daily

16-37. Moderate acne is best classified by which of the following? A. Closed comedones B. Inflammation and occasional papules/pustules C. Open comedones D. Cysts and nodules

B. Inflammation and occasional papules/pustules

11-45. Atonic seizures are known to typically: A. Involve uncoordinated movements/thrashing. thus putting the patient at risk for accidentally hitting nearby objects B. Involve muscle weakness/limpness, thus putting the patient at high risk for falls C. Last more than five minutes, thus putting the patient at risk for status epilepticus D. Involve a loss of awareness resembling day- dreaming, thus putting the patient at risk for poor academic performance

B. Involve muscle weakness/limpness, thus putting the patient at high risk for falls

Crohn's disease typically: A. Is confined to the colon (beginning at the rectum) B. Involves noncontiguous inflammation of all tissue layers C. Affects only the mucosal layer of the colon lining D. Is noted for its hallmark feature of bloody diarrhea

B. Involves noncontiguous inflammation of all tissue layers

13-4. Which statement is true regarding Addison's disease? A. It is easily diagnosed, as its classic signs of hyperkalemia, hyponatremia, hyperpigmentation, and fatigue are readily recognizable B. It is difficult to pinpoint due to its vague symptoms, such as weakness or fatigue-symptoms that can be found in conditions as wide-ranging as depression or anorexia nervosa C. It is similar to Cushing syndrome, especially because it features hyperglycemia D. It only involves the hypothalamus

B. It is difficult to pinpoint due to its vague symp- toms, such as weakness or fatigue-symptoms that can be found in conditions as wide-ranging as depression or anorexia nervosa

20-4. Which of the following statements is false regarding culturally congruent care? A. To provide competent care, it is important to understand one's own cultural biases and competencies B. It is permissible to delay medical care until a translator can be located for a patient that does not speak English C. In caring for special populations and making follow-up plans, it is important to recognize special populations may have difficulty with transportation for follow-up appointments D. It is important to listen to a patient's perspective and understanding of their health concerns

B. It is permissible to delay medical care until a translator can be located for a patient that does not speak English

3-2. The Lesbian, Bay, Bisexual, and Transgender (LGBT) population is included as a special population in the Healthy People 2020 initiative due to which of the following health concerns? A. LGBT population experiences few social barriers to healthcare caused by discrimination B. LGBT youth can be at special risk, with increased suicide rates 2-3 times higher than the general population C. LGBT individuals have less access to healthcare due to living in poorly served rural areas D. LGBT individuals experience a lower risk of violence and victimization, which has long-lasting effects on the individual and the community

B. LGBT youth can be at special risk, with increased suicide rates 2-3 times higher than the general population

After performing the Leopold maneuvers, you determine that the fetus is in the LOA position. Based on this information, you listen for fetal heart tones in which abdominal quadrant? A. RLQ B. LLQ C. LUQ D. RUQ

B. LLQ

You are interpreting ECG findings in a 44-year-old male who complains of palpitations, cough, and peripheral edema. You notice that the QRS in lead I is upright, and in AVF it faces down. How would you describe the axis of this ECG? A. Extreme right axis deviation B. Left axis deviation C. Normal axis D. Right axis deviation

B. Left axis deviation

12-10. A diagnosis of major depressive disorder must include either a depressed mood or which of the following symptoms? A. Feelings of worthlessness B. Loss of interest or pleasure C. Sleep issues D. Recurrent thoughts of death

B. Loss of interest or pleasure

15-8. The nurse practitioner is assessing a patient who presented with fever and rash. The nurse practitioner notes a red circular area with central clearing and concentric circles. What is the likely diagnosis? A. Marburg virus B. Lyme disease C. Rocky Mountain spotted fever D. Smallpox

B. Lyme disease

11-13. The NP is aware that which of the following are appropriate techniques for communicating with patients with dementia? A. Ask open-ended questions B. Maintain good eye contact and use a relaxed and unhurried approach C. Rephrase a statement if the patient does not seem to understand D. Finish the patient's statements

B. Maintain good eye contact and use a relaxed and unhurried approach

Koplik's spots are associated with: A. Poxvirus infections B. Measles C. Kawasaki's disease D. Reye's syndrome

B. Measles: S/S of the measles includes 3 C's! Cough, coryza, conjunctivitis, rash, fever over 101F and Koplik's spots in buccal mucosa

3-17. In which of the following scenarios might it be acceptable to postpone immunization? A. Low-grade fever or mild illness B. Moderate to severe illness C. Prior localized reaction D. All of the above

B. Moderate to severe illness

12-20. Which of the following are first-line medication treatments for patients with a bipolar disorder diagnosis? A. Antidepressants B. Mood stabilizers C. Anxiolytics D. Mood stabilizers and antidepressants together

B. Mood stabilizers

28 y.o. student presents with "hacking cough." Productive with small amounts sputum, runny nose. Doesn't take any meds, no allergies, no significant PMH. Temp: 99.9F, Resp: 16/min, HR: 90 BPM, diffuse fine crackles. CXR shows diffuse infiltrates in lower lobe of right lung. CBC shows WBC count 10,500. What is most likely diagnosis? A. Strep pneumo B. Mycoplasma pneumo C. Acute Bronchitis D. Legionnaires disease

B. Mycoplasma: most commonly seen in children and young adults. spread through droplets, sneezing and coughing at close distance. DX made through sx and CXR showing infiltrate to lower lobes

11-30. Primary progressive multiple sclerosis (PPMS) is notable for: A. Being the MS type featuring the most inflammatory activity B. Not featuring periods of remission; patients experience progressive deterioration C. Being the most common type of MS D. Always leading to cognitive deficits

B. Not featuring periods of remission; patients experience progressive deterioration

The NP is evaluating a patient with a 2-day history of fever, poor appetite, and mouth pain. The NP notes red vesicles inside the mouth. Which additional assessment is most important for the determining the patient's diagnosis? A. Assess for a sandpaper-like rash on the abdomen B. Observe the hands and feet for erythematous lesions C. Palpate lymph nodes for swelling, determining if tender D. Inspect genital area for presence of ulcerations

B. Observe the hands and feet for erythematous lesions

19-25. Which statement is true regarding the NPI? A. An NPI must be renewed every five years B. Once issued, an NPI never changes C. A provider can only apply for an NPI by paper application D. NPI stands for National Provider Indicator

B. Once issued, an NPI never changes

While discussing the need for immunizations with a 54-year-old diabetic man with chronic obstructive pulmonary disease (COPD), you would recommend which of the following? A. Inactivated influenza vaccine B. PPSV 23 pneumococcal vaccine C. PCV 13 pneumococcal vaccine D. Annual Td vaccination

B. PPSV 23 pneumococcal vaccine

Which is the correct description of Rovsing sign? A. Palpate RLQ, holding pressure for 30 seconds with palm of hand and then release B. Palpate LLQ and patient complains of pain in RLQ C. Flex right hip and right knee and internally rotate right hip D. Palpate RUQ and monitor for inspiratory pause during palpation

B. Palpate LLQ and patient complains of pain in RLQ

A 60-year-old male presents to the office with complaints of sexual dysfunction. He reports that over the past 6 months he has had difficulty maintaining an erection. He denies any UTI symptoms and denies any recent trauma or psychological component, but does feel anxious about his situation. After examining the patient, you find no physical abnormalities and decide to start him on which of the following medications? A. sertraline (Zoloft) B. sildenafil (Viagra) C. ciprofloxacin (Cipro) D. testosterone (AndroGel)

B. sildenafil (Viagra)

A 36-year-old G3P2 female presents to the office with pelvic and back pain, as well as painful intercourse and burning on urination. Your exam reveals tenderness in the LLQ and RLQ of the abdomen, cervical motion tenderness, and exquisite pain on palpation of the uterus and right adnexa. Her urine dipstick is positive for leukocytes and ketones and negative for blood and nitrites. Based on these findings, you diagnose and treat her with which of the following? A. Urinary tract infection: nitrofurantoin (Macro- bid) 100 mg PO BID x 5 days B. Pelvic inflammatory disease: ceftriaxone (Rocephin) 250 mg IM x 1 and doxycycline (Vibramycin) 100 mg PO BID x 14 days C. Trichomoniasis: tinidazole (Tindamax) 2 g PO x 1 D. Endometriosis: continuous combined oral contraceptives

B. Pelvic inflammatory disease: ceftriaxone (Rocephin) 250 mg IM x 1 and doxycycline (Vibramycin) 100 mg PO BID x 14 days

A 35-year-old female presents to the clinic for evaluation of recent onset of right-nipple discharge. This discharge is elicited with manipulation and is green to yellow in color. Her paternal grandmother was diagnosed with breast cancer at the age of 65. Which of the following is best for the nurse practitioner to include in the treatment plan? A. Schedule the patient for a diagnostic mammogram B. Perform a hemoccult of the nipple discharge C. Reassure the patient that her nipple discharge is benign D. Refer the patient to a breast specialist

B. Perform a hemoccult of the nipple discharge

19-3. The goal of the Privacy Rule is to: A. Allow for a patient's privacy when performing sensitive procedures B. Protect an individual's personal health information C. Protect a patient's right to decline treatment D. Allow an individual to make a decision about their healthcare at the end of life

B. Protect an individual's personal health information

The nurse practitioner is reviewing the results ofa urinalysis. Which set of results is most indicative of urinary tract infection? A. Protein 2+, blood 2+, nitrites negative B. Protein negative, nitrites 2+, leukocyte esterase 2+ C. Bacteria 1+, nitrites negative, leukocyte esterase negative D. Protein negative, blood negative, WBC 4

B. Protein negative, nitrites 2+, leukocyte esterase 2+

20-8. Which of the following is a strategy to decrease barriers to healthcare access? A. Limited office hours B. Provider cultural competence C. Only accepting commercial insurance D. Collecting co-pays at the visit

B. Provider cultural competence

19-30. The Affordable Care Act helps make healthcare affordable to lower-income patients by: A. Allowing health insurance companies to charge more for patients with pre-existing conditions, thus evening out the costs for all B. Providing subsidies that help cover the cost of health insurance for lower-income patients C. Relegating all lower-income patients to low-cost "bronze" plans D. Encouraging lower-income patients to seek emergency room care for routine healthcare needs

B. Providing subsidies that help cover the cost of health insurance for lower-income patients

A 72-year-old male presents to the clinic with a 2-month history of severe abdominal pain; greasy, malodorous stools; fatigue; and a 20-pound unin- tentional weight loss. He also complains of dry, itchy skin and was diagnosed with diabetes mellitus 3 months ago. You suspect pancreatic cancer as your leading differential based on which of the following diagnostic clues? A. Abdominal pain B. Recent diagnosis of diabetes mellitus C. Dry, itchy skin D. Greasy, malodorous stools

B. Recent diagnosis of diabetes mellitus

A 30-year-old female was recently seen for her annual well-woman exam. As is recommended by the ASCCP, she had a Pap smear with HPV co-testing completed. Her results are ASCUS and HPV negative. You take which of the following actions? A. Refer the patient for a colposcopy B. Repeat Pap with co-testing in three years C. Return for cytology only in six months D. Repeat Pap with co-testing in one year

B. Repeat Pap with co-testing in three years

You would advise an 18 y.o. at the student health clinic who just got an MMR booster which of the following: A. She may have low grade fever over next 24-48 hrs B. Not to get pregnant within the next 4 weeks C. Her arm will be very sore for the next 48 hrs at injection site D. Her arm will have some induration at IM site for 48 hrs

B. Women should not get MMR while pregnant, and need at least 28 days between vaccination and conception, preferrably at least 4 weeks

4-7. A 56-year-old male presents to the office as a new patient. When reviewing his medical history you notice "medication-induced tendonitis with subsequent tendon rupture" listed under past problems. Even though he connote remember the medication, you know that it is a serious adverse effect of which of the following drugs? A. ceftriaxone (Rocephin) B. ciprofloxacin (Cipro) C. darifenacin (Enablex) D. phenazopyridine (Pyridium)

B. ciprofloxacin (Cipro)

What is the appropriate treatment of community-acquired pneumonia in a 52-year-old male who has a dry cough, fatigue, no recent antimicrobial use, and no comorbidity? The patient currently takes no routine medications. A. amoxicillin (Amoxil) B. clarithromycin (Biaxin) C. levofloxacin (Levaquin) D. ceftriaxone (Rocephin)

B. clarithromycin (Biaxin)

A 56-year-old female patient who is allergic to penicillin with a prosthetic heart valve is visiting her dentist to undergo a tooth implant. Which antibiotic is recommended at this time? A. doxycycline (Vibramycin) B. clindamycin (Cleocin) C. ampicillin (Amoxil) D. cefaclor (Ceclor)

B. clindamycin (Cleocin)

The nurse practitioner is caring for a patient with Addison disease with low aldosterone levels. Which medication will the nurse practitioner prescribe for aldosterone replacement? A. cortisone (Cortone) B. fludrocortisone (Florinef) C. hydrocortisone (Cortef) D. prednisone (Deltasone)

B. fludrocortisone (Florinef)

Which of the following medications is the only one FDA-approved for the management of amyotrophic lateral sclerosis (ALS)? A. pramipexole (Mirapex) B. riluzole (Rilutek) C. tetrabenazine (Xenazine) D. topiramate (Topamax)

B. riluzole (Rilutek)

16-6. The patient shows the nurse practitioner a lesion on his third finger, right hand. It is a 1 cm, dome-shaped verrucous papule with small black punctate spots. The patient is to be referred to dermatology. Treat- ment for his lesion may include daily application of: A. mupirocin (Bactroban) B. salicylic acid (Durasal) C. ammonium lactate (Lac-Hydrin) D. calamine lotion

B. salicylic acid (Durasal)

An infant who does not have a history of reactive airway disease and allergy has both insiratory and expiratory wheezing accompanied by fever and profuse clear nasal discharge, which of the following are most likely? A. Tracheobronchitis B. Bronchiolitis C. Croup D. A small foreign body that is lodged in the left main bronchus

B: Bronchiolitis: Tracheobronchitis include prominent dry, non productive cough; later coughing up sputum is common. Bronchiolitis is a viral infection caused by RSV that is commonly seen during the winter/ spring months in infants & young children. Typical S/S include fever/ inspiratory/ expiratory wheezing, with clear drainage. Croup is viral infection with classic "barking" cough; the pt may have runny nose but no fever with croup. Foreign body ruled out due to fever and runny nose

67 y.o female presents to the clinic with 50 pk year hx of smoking for physical. C/o being easily short of breath/ frequent fatigue. Physical reveals diminished breath sounds, hyperresonance and hypertrophied resp muscles. Her CBC shows that her hematocrit level is slightly elevated. Her pulmonary function test shows increased total lung capacity. What is most likely DX for this pt? A. Bronchogenic carcinoma B. COPD C. Chronic bronchitis D. CHF

B: COPD: results in polycythemia (Increased Hematocrit) due to chronic hypoxia. Also you may note digital clubbing, dyspnea on exertion, accessory muscle use

What test would you use on a 7 y.o. to screen for color blindness? A. Snellen chart B. Ishihara chart C. Cover/uncover test D. Red Reflex

B: Ishihara chart

RMSF is caused by bite of: A. Mosquito B. Tick C. Insect D. Flea

B: Tick with parasite Rickettsia rickettsii

All of the following are implicated in causing chronic cough except: A. Chronic bronchitis B. Allergic rhinitis C. Acute viral URI D. GERD

C Acute viral URI: The key in this question is where it says what causes "CHRONIC COUGH." An acute viral illness will resolve, while chronic bronchitis, GERD, and allergic rhinitis can all cause chronic coughs.

A 39-year-old migrant worker presents to the clinic 2.5 days after a purified protein derivative (PPD) test. What minimum size of induration would be considered posi- tive for this patient? A) 3 mm B) 5 mm C) 10 mm D) 15 mm

C) 10 mm The PPD is administered on the volar aspect of the lower arm and read 48 hours after the test is given. The PPD result must have induration and measure 10 mm or greater to be positive in a low-risk patient. Induration (firmness with palpation) must be present. If the site has erythema but no induration, result would be negative. Color is not important.

When does an infant triple its birth weight? A) 3 months B) 6 months C) 12 months D) 15 months

C) 12 months By the age of 12 months, an infant is expected to have tripled its birth weight.

What is the caloric content of infant formula and breast milk? A) 10 kcal/30 mL B) 15 kcal/30 mL C) 20 kcal/30 mL D) 25 kcal/30 mL

C) 20 kcal/30 mL The caloric content of infant formula and breast milk is 20k cal/30 mL.

Mrs. J. L. is a 55-year-old female with a body mass index (BMI) of 24 and a history of asthma. She has hypertension that has been under control with hydrochlorothiazide 12.5 mg PO daily. Her total cholesterol is 230 g/dl. How many risk factors for coronary heart disease (CAD) does she have? A) 1 risk factor B) 2 risk factors C) 3 risk factors D) 4 risk factors

C) 3 risk factors Risk factors for coronary heart disease include hypertension, hypercholesterolemia, and women 55 years of age and older

At what age can a child copy a circle and ride a tricycle? A) 1 year B) 2 years C) 3 years D) 4 years

C) 3 years Developmental stages in children include the following: 1 year: walk; 2 years: walks up steps with the same foot; 3 years: pedals a tricycle and copies a circle; 4 years: rides a bicycle and copies a cross and draws a person with 2 parts.

The following are patients who are at high risk for complications due to urinary tract infections. Who does not belong in this category? A) A 38-year-old diabetic patient with a HbA1C of 7.5% B) A woman with a history of rheumatoid arthritis who is currently being treated with a regimen of methotrexate and low-dose steroids C) A 21-year-old woman who is under treatment for 2 sexually transmitted infections D) Pregnant women

C) A 21-year-old woman who is under treatment for 2 sexually transmitted infections Risk factors for complications due to urinary tract infection include pregnancy, diabetes, and steroid therapy.

What is the best description of Cullen's sign? A) The onset of hyperactive bowel sound before the onset of ileus B) A reddish-purple discoloration that is located on the flank area C) A bluish discoloration or bruising that is located on the umbilical area D) The acute-onset subcutaneous bleeding seen during acute pancreatitis

C) A bluish discoloration or bruising that is located on the umbilical area Cullen's sign is the acute onset of bluish discoloration that is located on the umbilical/periumbilical area; it is caused by bruising underneath the skin. The bluish discoloration that is located on the flank area is called the Grey-Turner sign. It is a sign of a severe case of pancreatitis.

Which of the following is a CDC-recommended treatment for a case of uncomplicated gonorrheal and chlamydial infection? A) Metronidazole (Flagyl) 250 mg PO TID x 7 days B) Valacyclovir (Valtrex) 500 mg PO BID x 10 days C) Azithromycin 1 g orally OR doxycycline 100 mg orally twice a day for 7 days D) 1 dose of oral fluconazole (Diflucan) 150 mg

C) Azithromycin 1 g orally OR doxycycline 100 mg orally twice a day for 7 days Drug therapy is based on the 2010 CDC STD guidelines for treatment.

All of the following pharmacologic agents are used to treat inflammation in the lungs of asthmatics except: A) Nedocromil sodium (Tilade) two sprays QID B) Cromolyn sodium inhaler (Intal) two puffs QID C) A long-acting oral theophylline (Theo-Dur) 200 mg every 12 hours D) Fluticasone inhaler (Flovent) two puffs BID

C) A long-acting oral theophylline (Theo-Dur) 200 mg every 12 hours Theophylline is used to control inflammation of the lungs. Tilade, Intal, and Flovent help to treat inflammation.

The following skin findings are considered macules except: A) A freckle B) Petechiae C) Acne D) A flat, 0.5-cm brown birthmark

C) Acne A macule is a flat, non-raised lesion on the skin. Acne lesions are pap- ules because they consist of raised, erythemic lesions on the skin. A freckle, petechiae, and a flat birthmark are all considered macules.

A 12-year-old girl is complaining of a 2-week history of facial pressure that worsens when she bends over. She complains of tooth pain in her upper molars on the right side of her face. On physical exam, her lung and heart sounds are normal. Which of the following is the most likely diagnosis? A) An acute dental abscess B) Chronic sinusitis C) Acute sinusitis D) Severe allergic rhinitis

C) Acute sinusitis Signs/symptoms of acute sinusitis includes headache, facial pain that worsens with bending over, eye/ear pressure and pain, aching in upper jaw/teeth, reduced smell and taste, cough (especially at night due to the nasal drainage), sore throat, bad breath, and fatigue.

All of the following are implicated in causing chronic cough except: A) Chronic bronchitis B) Allergic rhinitis C) Acute viral upper respiratory infection D) Gastroesophageal reflux disease

C) Acute viral upper respiratory infection Chronic cough can be caused by chronic bronchitis, allergic rhinitis, and GERD.

While performing a sports physical on a 16-year-old girl, the nurse practitioner notes a split S2 during inspiration that disappears during expiration. The girl is active and her growth and development have been uneventful. What is the best recommendation for the child's mother? A) Her daughter needs to be referred to a pediatric cardiologist B) Her daughter needs to be referred for a stress EKG C) Advise the mother that this is a normal finding D) The child should avoid strenuous physical exertion until further evaluation

C) Advise the mother that this is a normal finding Explain to the mother that in healthy adolescents, a split S2 during inspiration that disappears during expira- tion is a normal variation.

An 13-year-old girl is brought in by the mother because her daughter is complaining of vaginal discharge and pain. The mother tells the nurse practitioner that her daughter is not sexually active yet. The mother is divorced and lives with her boy- friend and works full time. During the exam, the nurse practitioner notes that the vaginal introitus is red, with tears and a torn hymen. The cervix is covered with green discharge. The nurse practitioner suspects that the child has been sexually abused by the mother's boyfriend. What is the best action for the nurse practitioner to take? A) Ask the mother questions about her boyfriend's behaviors B) Advise the mother to watch how her boyfriend interacts with her daughter and to call within 1 week to discuss his behavior with her C) Advise the mother that you suspect that her daughter has been sexually abused and that she is legally required to report the case to the child protection program D) Report the child abuse to the local police department

C) Advise the mother that you suspect that her daughter has been sexually abused and that she is legally required to report the case to the child protection program The NP is legally required to report the case to the child protection program. If the child is in danger, child protective services may ask for a court order to take the child away for protection until the investigation is completed. Talking about the boyfriend's behavior will not be effective and may put the child and/or mother in danger if the boyfriend suspects that he is being watched.

Which of the following is appropriate follow-up for this 70-year-old patient? A) Referral to an optometrist B) Referral to an ophthalmologist C) Advise the patient that it is a benign condition and will resolve spontaneously D) Prescribe an ophthalmic antibiotic solution

C) Advise the patient that it is a benign condition and will resolve spontaneously A subconjunctival hemorrhage is a benign disorder that resolves without any treatment

A 62-year-old female complains of chronic severe low back pain. She also reports mild episodes of fecal incontinence and numbness to her lower legs over the past week. You would suspect which of the following? A) Fracture of the lower spine B) A herniated disc C) Cauda equina syndrome D) Ankylosing spondylitis

C) Cauda equina syndrome Cauda equina syndrome is a surgical emergency. Signs and symptoms include a change in bowel and bladder control (incontinence). Acute pressure on the sacral nerve root causes inflammatory and ischemic changes to the nerve and can lead to permanent nerve damage.

A 44-year-old patient with Down syndrome starts to develop impaired memory and difficulty with his usual daily life routines. He is having problems functioning at the job that he has done for the past 10 years. The physical exam and routine labs are all negative. The vital signs are normal. His appetite is normal. The most likely diagnosis is: A) Tic douloureux B) A stroke C) Alzheimer's disease D) Delirium

C) Alzheimer's disease Delirium is an acute decline in mental status and is temporary. Common causes are fever, shock, drugs, alcohol, and dehydration. Alzheimer's disease is a permanent change to the brain that causes short-term memory loss, agnosia, apraxia, and aphasia. In this case, the patient's physical exam is normal; however, he is having memory loss and difficulty working and carrying out his normal tasks

A new mother reports to you that her 6-month-old infant has a cold and has a fever of 99.8 degrees Fahrenheit. The infant is not irritable and is feeding well without problems. The mother wants to know whether it is okay for him to be immunized at this time. Which of the following statements is true? A) The infant should not be immunized until he is afebrile B) An infant with a cold can be immunized at any time C) An infant with a cold can be immunized as long as the infant's temperature is no higher than 100.4 degrees Fahrenheit D) Because immunization is so important, it should be given to the infant as scheduled

C) An infant with a cold can be immunized as long as the infant's temperature is no higher than 100.4 degrees Fahrenheit Immunization is acceptable as long as the temperature is not higher than 100.4°F.

The NP suspects that a middle-aged woman may have systemic lupus erythematosus (SLE). Which of the following laboratory tests is strongly associated with this disease? A) Sedimentation rate B) C-reactive protein C) Antinuclear antibody (ANA) D) Systemic lupus erythematosus (SLE)

C) Antinuclear antibody (ANA) The ANA is usually positive in lupus patients. Other types of autoantibody testing recommended for these patients, in addition to antinuclear antibody (ANA) tests, are antiphospholipid antibodies, antibodies to double-stranded DNA, and anti-Smith (Sm) antibodies. Patient with suspected lupus should be referred to a rheumatologist. The sedimentation rate and the C-reactive protein are nonspecific findings of inflammation and are present in autoimmune diseases, infections, and others.

When palpating a woman who is at 20 weeks of gestation, the nurse practitioner should be able to feel the uterine fundus at what level? A) Just rising above the level of the pubic symphysis B) Between the pubic symphysis and the umbilicus C) At the level of the umbilicus D) Above the level of the umbilicus

C) At the level of the umbilicus Uterine growth at 12 weeks gestation, palpable just above the symphysis pubis; 20 weeks gestation, at the level of the umbilicus; after this, measurements in cm should approximately equal weeks of gestation.

An older male is diagnosed with conductive hearing loss in the left ear by the nurse practitioner. Which of the following is the expected result on this patient when performing a Rinne test? A) AC (air conduction) > BC (bone conduction) B) Lateralization to the bad ear C) BC > AC D) Lateralization to the good ear

C) BC > AC A normal result in the Rinne test is AC greater than BC. When there is a conduction hearing loss (i.e., cerumenosis, otitis media), the result will be BC greater than AC. The reason is that the sound waves are blocked (i.e., cerumen, fluid in middle ear). Therefore, the patient cannot hear them as well as through bone conduction.

A middle-aged hypertensive male presents to a public health clinic with complaints of an acute onset of fever, chills, and cough that is productive of rusty-colored sputum. The patient reports episodes of sharp pains on the left side of his back and chest whenever he is coughing. His temperature is 102.2 degrees Fahrenheit, the pulse is 100/min, and the BP is 130/80. The urinalysis does not show leukocytes, nitrites, or blood. This finding is most consistent with: A) Atypical pneumonia B) An upper urinary tract infection C) Bacterial pneumonia D) Acute pyelonephritis

C) Bacterial pneumonia Symptoms of bacterial pneumonia include high fever, chills, productive cough, and pain in back/chest area with deep inspiration and cough.

Which of the following classes of drugs is implicated with blunting the signs and symptoms of hypoglycemia in diabetics? A) Calcium channel blockers B) Diuretics C) Beta-blockers D) ARBs (angiotensin receptor blockers)

C) Beta-blockers Beta-blockers are known to blunt the signs/symptoms of hypoglycemia in patient with diabetes.

A new mother is breast feeding her full-term 4-week-old infant. She wants to know whether she should give the infant vitamin supplements. The best advice is: A) Since she is breast feeding, the infant does not need any vitamin supplements until he is at least 6 months of age B) Breast milk gives the infant all the vitamins he needs until 12 months of age C) Breastfed infants require iron and Vitamin D supplementation right away D) BreastfedinfantsrequireironandVitaminEsupplementationat3monthsof age

C) Breastfed infants require iron and Vitamin D supplementation right away While infants do receive Vitamin D through human milk, the AAP recom- mends a daily supplement of 200 IU Vitamin D for all breastfed infants.

A first-grader presents to a school nurse practitioner with a few blisters on one arm and on the face. The child keeps scratching the affected areas. Some of the lesions have ruptured with yellow serous fluid that crusts easily. This best describes: A) Acute cellulitis B) Herpes zoster C) Bullous impetigo D) Erysipelas

C) Bullous impetigo Bullous impetigo is a skin infection, commonly seen on the face and hands, with yellow, honey-colored fluid blisters with drainage that turn into scabbed lesions that continue to spread until treated.

Balanitis is caused by: A) Staphyloccocus aureus B) Streptococcus pyogenes C) Candida albicans D) Trichomonads

C) Candida albicans Balanitis is an yeast infection of the glans of the penis. Candida albicans is the causative source. Staphyloccocus aureus and Streptococcus pyogenes are bacterial infections. Trichomonads are protozoans that cause infection.

When the nurse practitioner is evaluating a patient for intermittent claudication, he/she would first: A) Order a venogram B) Order T.E.D. anti-embolism stockings C) Check the ankle and brachial blood pressure before and after exercise D) Check the pedal and posterior tibial pulses

C) Check the ankle and brachial blood pressure before and after excercise Initial evaluation for intermittent claudication would include checking the ankle and brachial blood pressure before and after exercise.

All of the following may help relieve the symptom(s) of GERD except: A) Losing weight B) Stopping caffeine intake C) Chewing breath mints D) Stopping alcohol intake

C) Chewing breath mints GERD is a condition in which food comes up from the stomach/esophagus through a weak sphincter. It usually worsens with lying down, from the reflux, and can cause a cough and esophageal irritation if not treated. Effective treatment may include weight loss, decreased caffeine intake, and avoidance of alcohol.

A cauliflower-like growth with foul-smelling discharge is seen during an otoscopic exam of the left ear of an 8-year-old boy with a history of chronic otitis media. No tympanic membrane or ossicles are visible and the patient seems to have difficulty hearing the nurse practitioner's instructions. Which of the following conditions is best described? A) Chronic perforation of the tympanic membrane with secondary bacterial infection B) Chronic mastoiditis C) Cholesteatoma D) Cancer of the middle ea

C) Cholesteatoma Cholesteatoma is an abnormal growth that occurs in the middle ear that can cause hearing loss. Difficulty hearing is common.

Asthmatics may have all of the following symptoms during an exacerbation except: A) Rapid pulse B) Wheezing C) Chronic coughing D) Tachypnea

C) Chronic coughing Signs and symptoms of an acute exacerbation of asthma include breathlessness, coughing, wheezing, and chest tightness. Agitation, increased respiratory and pulse rate, and decreased lung function are also noted. A "chronic" cough is not a symptom.

Patients who are considered mentally competent have a right to consent or refuse medical treatment. What is the legal term for this right? A) Informed consent B) Durable power of attorney C) Competence D) Advance directives

C) Competence Competence is having the ability to make one's own decisions regarding one's own course of health care, as long as one is documented as being mentally competent. Informed consent is giving permission to perform a particular procedure, understanding the purpose of the procedure/treatment, success/failure rate, other alternatives to the procedure, risks and benefits of the procedure, prognosis, and success rate. To give informed consent, the patient must be 18 years old or older. Durable power of attorney is the legal document giving one person the authority to make decisions regarding another person's health care and personal affairs on that person's behalf. Advance directives are written instructions to be per- formed in the event the patient is not mentally competent and not able to make decisions for himself/herself. The Living Will is an example of an advance directive.

Which of the following structures of the eyes is responsible for color vision? A) Rods B) Macula C) Cones D) Pupils

C) Cones Rods and cones are photoreceptor cells of the retina. The cones of the eyes are responsible for color vision. Cones are very sensitive to colors (red, blue, or green) and work better in brighter light. Rods are good for night vision and for vision in low light conditions because they are sensitive to light and dark. To remember them, note that both cone and color start with the letter "C."

A 62-year-old male with chronic obstructive pulmonary disease (COPD) complains to the nurse practitioner that his prescription of ipratropium bromide (Atrovent) is not working. He reports that he still feels short of breath even after using it 4 times a day for 3 months. Which of the following actions is the next step for the nurse practitioner? A) Increase the patient's dose of ipratropium bromide (Atrovent) to 3 inhalations QID B) Continue the ipratropium bromide and start the patient on oxygen by nasal cannula C) Continue ipratropium bromide (Atrovent) and add albuterol (Ventolin) inhaler 2 inhalations QID D) Start the patient on oxygen by nasal cannula at bedtime and PRN during the daytime

C) Continue ipratropium bromide (Atrovent) and add albuterol (Ventolin) inhaler 2 inhalations QID Treatment of COPD starts with an anticholinergic (ipratropium bromide). The next step is to add a short-acting beta-2 agonist (albuterol)

The Rinne and the Weber tests are used to assess which of the following cranial nerve(s)? A) Cranial nerves 3, 4, and 6 B) Cranial nerve 7 C) Cranial nerve 8 D) Cranial nerves 9 and 10

C) Cranial nerve 8 The Rinne and Weber tests are used to assess cranial nerve 8 or the acoustic nerve. The patient's hearing is tested by air conduction (Rinne and Weber) and bone conduction (Rinne only).

Mr. Brown is a 65-year-old carpenter complaining of morning stiffness and pain in both his hands and right knee upon awakening. He feels some relief after warming up. On exam, the nurse notices the presence of Heberden's nodes. Which of the fol- lowing is most likely? A) Osteoporosis B) Rheumatoid arthritis C) Degenerative joint disease D) Reiter's syndrome

C) Degenerative joint disease Heberden's nodes are commonly seen in degenerative joint disease. Heberden's nodes are bony nodules located on the distal interphalangeal joints (DIPs).

All of the following are considered instrumental activities of daily living (IADLs) except: A) Grocery shopping B) Managing one's finances C) Grooming and hygiene D) Using a telephone and a computer

C) Grooming and hygiene Grooming and hygiene are classified as basic activities of daily living (ADLs). Grocery shopping, paying bills, using telephones, and driving a car are all IADLs.

At what time of the day would you recommend a scotch tape test be done to evaluate for a suspected case of enterobiasis? A) In the evening after dinner B) At night before bed C) Early in the morning D) It does not matter what time of the day the test is done

C) Early in the morning The scotch tape test is best done in the morning for several days in a row. The females do not lay eggs every day and therefore testing for several days will be more accurate.

A possible side effect from the use of nifedipine (Procardia XL) is: A) Hyperuricemia and hypoglycemia B) Hyperkalemia and angioedema C) Edema of the ankles and headache D) Dry hacking cough

C) Edema of the ankles and headache Common side effects of calcium channel blockers, such as Procardia, include edema of the ankles, dizziness, headaches, flushing, and weakness. ACE inhibitors tend to have the side effects of angioedema and a dry hacking cough. Diuretics can cause hyperkalemia and hyperuricemia.

A new mother who is on her fourth day of breastfeeding complains to the nurse practitioner of very sore breasts. The nurse practitioner would: A) Recommend a decrease in the number of times she breastfeeds her infant per day B) Recommend that she stop breastfeeding and use infant formula for the next 48 hours C) Educate the mother that this is normal during the first week or 2 of breastfeeding and the soreness will eventually go away D) Recommend that she purchase plastic nipple pads for her nursing bra and use them daily

C) Educate the mother that this is normal during the first week or 2 of breastfeeding and the soreness will eventually go away Nursing during the first 2 weeks after delivery may cause tenderness and soreness of the nipples and usu- ally resolves after this. The mother should continue to breastfeed as she has been advised, and she should make sure the infant is latching on appropriately.

When confirming a case of temporal arteritis, the erythrocyte sedimentation rate (ESR) is expected to be: A) Normal B) Lower than normal C) Elevated D) Indeterminate result

C) Elevated Temporal arteritis can be diagnosed by having an elevated erythrocyte sedimentation rate.

The nurse practitioner who suspects that one of her hypertensive patients has Cushing's syndrome would expect to find which of the following laboratory results? A) Hyponatremia B) Hypoglycemia C) Elevated serum cortisol levels D) Decreased urine 17-ketosteroids

C) Elevated serum cortisol levels Elevated serum cortisol levels are seen with Cushing's syndrome.

All of the following factors are associated with a higher risk of osteopenia and osteoporosis except: A) Excessive alcohol intake and cigarette smoking B) Asian or Caucasian ancestry C) Estrogen and progesterone deficiency D) Older age

C) Estrogen and progesterone deficiency Bone loss is not associated with progesterone deficiency (only estrogen deficiency). Risk factors include excessive alcohol intake, cigarette smoking, ancestry (Asian or Caucasian), older age, anorexia nervosa, small bone frame, and others.

Which of the following drugs is classified as a 5-alpha reductase inhibitor? A) Terazosin (Hytrin) B) Tamsulosin (Flomax) C) Finasteride (Proscar) D) Sildenafil (Viagra)

C) Finasteride (Proscar) Finasteride (Proscar) belongs to the drug class called 5-alpha reductase inhibitors. It helps to lower serum testosterone, which helps to decrease the size of the prostate. It is also used for male-pattern baldness. A smaller prostate results in less obstructive voiding symptoms such as weak stream, frequency, and nocturia. Both terazosin (Hytrin) and tamsulosin (Flomax) are alpha-blockers and may start to control symptoms in as little as 3 days. They work by relaxing the smooth muscle tissue of the prostate gland, which enlarges the diameter of the urethra.

All of the following are true statements about sexuality in the older adult except: A) Erectile dysfunction is very common B) It may take longer to become aroused C) Frail elderly are not interested in sexuality anymore D) Dyspareunia is a common symptom of atrophic vaginitis

C) Frail elderly are not interested in sexuality anymore Some studies have shown that older adults in their 80s can remain sexually active. By the age of 70 years, about 80% of males have ED.

All of the following groups are classified as "vulnerable populations" and have additional protections as human subjects except: A) Prisoners B) Pregnant women, fetuses, and children C) Frail elders D) Mentally incompetent individuals

C) Frail elders Frail elders who are mentally competent are not considered a vulnerable population group.

A multigravida who is at 34 weeks of gestation wants to know at what level her uterine fundus should be. The best answer is to advise the mother that her fun- dus is: A) Midway between the umbilicus and the lower ribs B) At the level of the umbilicus C) From 33 to 35 cm D) From 32 to 34 cm

C) From 33 to 35 cm After 20 weeks gestation, fundal height in centimeters should measure approximately the same as the number of weeks of gestation

Which of the following drugs is most likely to relieve the patient's symptoms? A) Atenolol (Tenormin) B) Trimethoprim/sulfamethoxazole (Bactrim DS) C) Furosemide (Lasix) D) Hydrocodone/guaifenesin syrup (Hycotuss)

C) Furosemide (Lasix) Lasix would be used to help remove the extra fluid load.

While assessing for a cardiac murmur, the first time that a thrill can be palpated is at: A) Grade II B) Grade III C) Grade IV D) Grade V

C) Grade IV A thrill is palpated at grade IV.

A middle-aged male who is homeless reports to the local Public Health Clinic com- plaining of a painless and shallow ulcer on the penile shaft for the past 2 weeks. He is sexually active and had unprotected intercourse with two male partners over the past few months. The patient is tested for HIV, syphilis, gonorrhea, hepatitis B, and herpes types 1 and 2. The syphilis and HIV tests are both positive. The gonorrhea, hepatitis B, and herpes tests are negative. The nurse practitioner is aware of Nationally Notifiable Infectious Conditions. Which of the following is true regarding reporting of any of these sexually transmitted infections? A) Obtain the patient's permission before reporting the positive HIV and syphilis test results to the local public health department B) Obtain the patient's and sexual partner's permission before reporting the posi- tive test results to the local health department C) Health care providers are mandated by law to report certain types of diseases to the local health department even if the patient does not give permission D) The nurse practitioner should consult with the supervising physician about this issue

C) Health care providers are mandated by law to report certain types of diseases to the local health department even if the patient does not give permission Physicians and laboratories are legally mandated to report certain types of diseases. STDs, HIV infection/AIDS, gonorrhea, and syphilis must be reported to the local health department even if the patient does not give permission. Partner tracing and notification are done by the local health department. The CDC web- site contains a list of nationally reportable diseases. Other diseases that are on the CDC 2012 list of reportable diseases (i.e., diseases that must be reported) are TB, diphtheria, hepatitis A, B, and C, measles, mumps, pertussis, Lyme disease, Rocky Mountain spotted fever, and many others.

All of the following are true about strawberry hemangiomas found in infants except: A) Most will involute spontaneously by the age of 18 to 24 months B) Watchful waiting is the most useful strategy C) Hemangiomas should be treated with laser therapy if they have not resolved by the age of 12 months D) Strawberry hemangiomas are benign

C) Hemangiomas should be treated with laser therapy if they have not resolved by the age of 12 months True strawberry hemangiomas will eventually resolve by the time the child goes to kindergarten. Most will reduce or disappear in the first 2 years. Laser treatment is rarely needed.

Which of the following is indicated for the prophylactic treatment of migraine headache? A) Ibuprofen (Motrin) B) Naproxen sodium (Anaprox) C) Propranolol (Inderal) D) Sumatriptan (Imitrex)

C)Propranolol (Inderal) Medications used to prophylactically prevent migraine headaches include beta-blockers (propranolol) and tricyclic antidepressants (ami- triptyline). Motrin, Anaprox, and Imitrex are all medications used to treat migraine headache.

A 19-year-old female has recently been diagnosed with acute hepatitis B. She is sexually active and is monogamous. She reports using condoms inconsistently. What would you recommend for her male sexual partner who was also tested for hepatitis with the following results: HBsAg (−), anti-HBs (−), anti-HCV (−), anti-HAV (+)? A) A hepatitis B vaccination B) Hepatitis B immunoglobulin C) Hepatitis B vaccination and hepatitis B immunoglobulin D) No vaccination is needed at this time

C) Hepatitis B vaccination and hepatitis B immunoglobulin Hepatitis B vaccination is given for long-term prophylaxis treatment to prevent hepatitis B infection. Hepatitis B immunoglobulin is given for prevention of hepatitis B infection when the person has been directly exposed to the hepatitis B infection. The immnoglobulin is not a vaccine and does not protect against long-term prophylaxis.

A patient diagnosed with bacterial vaginosis should be advised that her sexual partner be treated with: A) Ceftriaxone(Rocephin)250mgIMwithdoxycycline100mgBIDfor14days B) Metronidazole (Flagyl) 500 mg PO BID for 7 days and 1 dose of azithromycin (Zithromax) C) Her partner does not need treatment D) Clotrimazole cream (Lotrimin) on his penis BID for 1 to 2 weeks

C) Her partner does not need treatment Bacterial vaginosis is a bacterial infection, but is not considered an STD for which the partner needs treatment. Studies show that men rarely carry this infection.

All of the following signs and symptoms are present with an anticholinergic drug overdose except: A) Dilated pupils B) Flushing and tachycardia C) Hypertension D) Confusion

C) Hypertension Drugs with strong anticholinergic properties include diphenhydramine, scopolamine, TCAs, antipsychotics, and others. The mnemonic is "dry as a bone, red as a beet, mad as a hatter, blind as a bat." Look for a low-grade temperature.

According to Erik Erikson, adolescents are at what psychosocial developmental stage? A) Autonomy vs. shame B) Industry vs. inferiority C) Identity vs. role confusion D) Intimacy vs. isolation

C) Identity vs. role confusion Adolescents (age 12 to 18 years) are in the stage known as "identity vs. role confusion." At this time, the teen is transitioning into adulthood and reexamining his/her identity and beliefs. The teen wonders about himself/herself (e.g., "who am I"). Peers are highly valued.

The first teeth to erupt during infancy are which of the following? A) First molars B) Second molars C) Incisors D) Canines

C) Incisors The incisors are the first teeth to erupt during infancy.

All of the following physiologic changes are present in the lungs of the elderly except: A) Decrease in the forced expiratory volume (FEV1) B) Slight increase in the residual volume (RV) C) Increase in lung compliance D) Airways tend to collapse earlier (than young patients) with shallow breathing

C) Increase in lung compliance There is a decrease in lung compliance as we get older; therefore, the FEV1 also decreases. There is minimal change in the total lung volume. Airways tend to collapse earlier (than young patients) with shallow breathing, which increases the risk of pneumonia.

Which of the following drugs is effective therapy for treating pain in patients who are having an acute exacerbation of gout? A) Acetaminophen (Tylenol) B) Systemic steroids C) Indomethacin (Indocin) D) Allopurinol (Zyloprim)

C) Indomethacin (Indocin) Common medications used for acute flare-ups of gout include two NSAIDs, such as indomethacin and naproxen sodium, BID prn. Colchicine may be added to the NSAIDs if relief is not obtained. Maintenance therapy consists of allopurinol and/or probenecid.

A coworker calls the nurse practitioner and wants to know about a patient's progress. She tells the NP that they are neighbors and she is worried about the patient's health status. The coworker works in the same facility, but is not directly involved with this patient's care. Which of the following actions is the most appropriate? A) Share with the coworker the patient's health status B) Advise the coworker to call the patient right away and ask her for verbal permission C) Inform the coworker that you cannot release any information about this patient because the coworker is not directly involved in the patient's care D) Reassure the coworker that the patient is doing fine and is getting better quickly

C) Inform the coworker that you cannot release any information about this patient because the coworker is not directly involved in the patient's care Releasing any information about this patient to a coworker who is not directly involved in the patient's care would violate the HIPAA privacy rules

AllofthefollowingarecorrectstatementsregardingtheS3componentoftheheart sound except: A) It occurs very early in diastole and is sometimes called an opening snap B) It is a normal finding in some children, healthy young adults, and athletes C) It can be a normal variant if heard in a person age 40 or older D) It signifies CHF (congestive heart failure)

C) It can be a normal variant if heard in a person age 40 or older S3 heart sounds occur early in diastole and are sometimes referred to as an "opening snap." It is a normal variant in children, healthy young adults, and athletes. Bibasilar crackles in lung bases and the presence of S3 heart sounds are classic findings of CHF.

Mary, who is 65 years of age, comes into the clinic during the first week of November for her annual wellness visit. Her last Td booster was 9 years ago. Which immunization(s) would you recommend for this visit? A) Influenza vaccine B) Tetanus and influenza vaccine C) Pneumococcal (Pneumovax) and influenza vaccines D) She does not need any vaccinations to be administered in this visit

C) Pneumococcal (Pneumovax) and influenza vaccines At 65 years of age, Pneumovax is recommended. Influenza is recommended annually. Td is recommended every 10 years.

A 17-year-old high school student is considering her birth control options. She wants to know more about Seasonale. Which of the following statements is false? A) Taking Seasonale results in only 4 periods per year B) Her period will occur within the 7 days when she is on the inert pills C) It is a progesterone-only method of birth control and does not contain estrogen D) Take 1 tablet daily for 84 consecutive days followed by 7 days of inert pills

C) It is a progesterone-only method of birth control and does not contain estrogen Seasonale is an extended-cycle form of birth control. It contains both levonorgestrel and ethinyl estradiol. There are 84 pink tablets (active) and 7 white pills (inert). In general, more spotting (breakthrough bleeding) is experienced with extended-cycle pills during the first few months of use (compared with the monthly birth control pills).

Which of the following is correct regarding the best site to listen for mitral regurgitation? A) The apical area during S2 B) It is best heard at the base at S1 C) It is best heard at the apex at S1 D) It is best heard at the base at S2

C) It is best heard at the apex at S1 Mitral regurgitation is best heard at the apical area; it is a high-pitched, blowing pansystolic murmur.

A new patient who is a 40-year-old female postal worker is being evaluated for complaints of a new onset of an erythematous rash on both her cheeks and on the bridge of the nose that is accompanied by fatigue. She reports a history of Hashimoto's thyroiditis and is currently being treated with Synthroid 1.25 mg daily. Which of the following conditions is most likely? A) Atopic dermatitis B) Thyroid disease C) Lupus erythematosus D) Rosacea

C) Lupus erythematosus Classic symptoms of lupus erythematosus are butterfly rash across both cheeks and bridge of nose and fatigue. Risk factors also include being female and 40 years old.

What is the first-line class of antibiotics recommended by the American Thoracic Society (ATS) for patients younger than 60 years of age who are diagnosed with community-acquired pneumonia with no comorbidity? A) First-generation cephalosporins B) Second-generation cephalosporins C) Macrolides D) Beta-lactam antibiotics

C) Macrolides The American Thoracic Society recommends macrolides as the first line of therapy for community-acquired pneumonia without comorbidity.

A 65-year-old woman's bone density result shows severe demineralization of cortical bone. All of the following pharmacologic agents are useful in treating this condition except: A) Raloxifene (Evista) B) Calcitonin (Miacalcin) C) Medroxyprogesterone (Depo-Provera) D) Calcium with Vitamin D

C) Medroxyprogesterone (Depo-Provera) Common treatments for osteoporosis include calcium and Vitamin D, Evista, Miacalcin, and bisphosphonates. Depo- Provera is a type of contraception.

Beta thalassemia minor is considered a: A) Macrocytic anemia B) Normocytic anemia C) Microcytic anemia D) Hemolytic anemia

C) Microcytic anemia Beta thalassemia minor is a genetic disorder in which the bone marrow produces small, pale RBCs in which mild hypochromic, microcytic anemia occurs.

Lead poisoning can cause which type of anemia? A) A mild macrocytic anemia B) Normocytic anemia C) Microcytic anemia D) A mild hemolytic anemia

C) Microcytic anemia Lead poisoning can cause microcytic anemia. Signs and symptoms of lead poisoning are abdominal pain, constipation, vomiting, and blue-black line on the gums. Lead causes anemia by mimicking healthful minerals such as calcium, iron, and zinc. It is absorbed by the bones, where it interferes with the production of RBCs. This absorption can also interfere with calcium absorption that is needed to keep the bones healthy.

A 12-year-old male's peak expiratory flow results indicate 60 to 80% of the predicted range. How would you classify his asthma? A) Mild intermittent asthma B) Mild persistent asthma C) Moderate persistent asthma D) Severe asthma

C) Moderate persistent asthma Asthma classifications: Intermittent: normal FEV between exacerbations, FEV > 80%; mild persistent, FEV > 80%; moderate persistent, FEV 60-80%; severe persistent, FEV < 60%.

An elderly Hispanic male has been taking finasteride (Proscar) for several months. The nurse practitioner decides to check the prostate specific antigen (PSA). The PSA result is 10 ng/mL. The patient's baseline PSA was 30 ng/mL. Which of the following is the next step? A) Add the baseline PSA value to the treatment PSA value B) The treatment PSA value is the correct value C) Multiply the treatment PSA value by 2 D) Divide the baseline PSA value by the treatment PSA value

C) Multiply the treatment PSA value by 2 Before starting a prescription of Proscar, obtain the baseline PSA. Recheck the PSA again within 2 to 3 months during treatment to assess the patient's response (treatment PSA). For this example, the corrected treatment PSA is 20 (multiply 10 x 2 = 20). When com- paring the corrected treatment PSA (20) with the baseline PSA (30), the value is lower (means the prostate has shrunk in size). The patient's symptoms will also improve, including less nocturia, less dribbling, and stronger urinary stream.

The Health Insurance Portability and Accountability Act (HIPAA) was passed by Congress in 2003. All of the following statements about HIPAA are correct except: A) It provides federal protections for personal health information B) It is applicable to all health care providers and payers who bill electronically and transmit health information over the Internet C) Patients have the right to view their mental health and psychotherapy-related health information D) It gives patients the right to view and correct errors in their medical records

C) Patients have the right to view their mental health and psychotherapy-related health information Mental health and psychotherapy/psychiatric records do not have to be released to patients even if they request those records. Otherwise, any type of medical records can be released if it is requested by the health insurance or health plan for billing purposes and reimbursement. HIPAA applies to all health care providers, health plans, health insurance companies, medical clearinghouses, and others who bill electronically and transmit health information over the Internet.

A 35-year-old male has a history of an upper respiratory viral infection 4 weeks ago. He reports that he started feeling short of breath and now complains of sharp pain in the middle of his chest that seems to worsen when he lies down. The patient's physical exam is within normal limits with the exception of a precordial rub on auscultation. The most likely diagnosis would be: A) Pulmonary embolism B) Dissecting aneurysm C) Pericarditis D) Esophageal reflux

C) Pericarditis Pericarditis is inflammation of the sac around the heart. Common signs/symptoms include chest pain over the center/left side of the chest; shortness of breath, especially with lying down; low-grade fever; weakness; fatigue; dry cough; and abdominal or leg swelling. Pericardial rub may be auscultated.

The nurse practitioner orders an ankle-brachial index (ABI) test for a patient. Which of the following disorders is the ABI test used for? A) Venous insufficiency B) Osteoarthritis of the arm or the ankle C) Peripheral arterial disease (PAD) D) Rheumatoid arthritis

C) Peripheral arterial disease (PAD) The ankle-brachial index (ABI) is a test that is used to stratify the severity of arterial blockage in the lower extremities for patients with peripheral arterial disease (PAD). An ABI score of 1.0 to 1.4 is normal. Any value less than 1.0 is abnormal. A score of 0.5 or less is indicative of severe PAD.

All of the following conditions are contraindications for bupropion (Wellbutrin, Zyban) except: A) Anorexia nervosa and bulimia B) Seizure disorders C) Peripheral neuropathy D) Within 14 days after discontinuation of a monoamine oxidase inhibitor (MAOI)

C) Peripheral neuropathy Bupropion increases the risk of seizures. Contraindications are seizures, anorexia nervosa, and bulimia. Avoid with any condition that increases seizures, such as after abrupt withdrawal of alcohol or sedatives and certain head injuries. For peripheral neuropathy, atypical antide- pressants such as duloxetine (Cymbalta) and TCAs such as amitriptyline (Elavil) are used.

A 25-year-old male who was involved in a car accident is brought in to the local emergency department. He reports wearing a seat belt and was the driver of the vehicle. The patient is complaining of pain on his right leg. The skin is intact, but the right foot is abducted slightly. The right knee is tender and swollen. Which of the following tests is/are the best choice for evaluating for possible fractures and joint damage? A) Plain radiographs of the right hip and knee B) Ultrasound with a CT scan of the right leg C) Plain radiograph of the right hip and leg with an MRI of the knee joint D) Radiograph of the right hip and knee with special view of the hip

C) Plain radiograph of the right hip and leg with an MRI of the knee joint The stem is asking for two things " . . . evaluating for possible fractures joint damage." Plain x-rays are the first exam ordered for suspected or obvious bony fractures. An MRI scan is the best test for any type of joint pathology (swollen right knee).

Multiple myeloma is a malignancy of the: A) White blood cells (WBC) B) Red blood cells (RBC) C) Plasma cells D) Platelets

C) Plasma cells Myeloma is a cancer of the plasma cells (or mature B-cells/ lymphocytes), which affects the bone marrow. Plasma cells produce antibodies and reside mainly in the bone marrow. Signs/symptoms are bone pain, fractures, hypercalcemia, depressed immunity, and anemia. The bone marrow produces WBCs (neutrophils, lymphocytes, eosinophils, basophils), RBCs, and platelets. The typical patient is an older adult who is age 60 years or older.

Which of the following diseases is associated with a high risk of giant cell arteritis? A) History of transient ischemia attacks (TIA) B) Frequent migraine headaches with focal neurologic findings C) Polymyalgia rheumatica D) Systemic lupus erythematosus (SLE)

C) Polymyalgia rheumatica Giant cell arteritis (also known as temporal arteritis) is more common among patients with PMR. It can cause permanent blindness if it is not treated. PMR patients are taught how to recognize it. The quick onset of vision loss in one eye, accompanied by a tender indurated artery and scalp tenderness on the same side, are classic symptoms. The screening test is the sedimentation rate and C-reactive protein test. Both will be markedly elevated.

A 76-year-old woman reports that for the previous 4 months, she has noticed severe stiffness and aching in her neck and both shoulders and hips that is worsened by movement. She reports having a difficult time getting out of bed because of the severe stiffness and pain. It is difficult for her to put on a jacket or blouse or to fasten her bra. Along with these symptoms, she also has a low-grade fever, fatigue, loss of appetite, and weight loss. Starting yesterday, the vision in her right eye has progressively worsened. She has annual eye exams and denies that she has glaucoma. Which of the following conditions is most likely? A) Rheumatoid arthritis (RA) B) Degenerative joint disease C) Polymyalgia rheumatica (PMR) D) Fibromyalgia

C) Polymyalgia rheumatica(PMR) Polymyalgia rheumatica (PMR )is associated with a high risk of giant cell arteritis (GCA) or temporal arteritis (15%-30%). The new onset of vision loss and the location of the pain (neck, both shoulders/hips) are the most important clues. PMR is a rheumatic condition that involves joints and the arteries. Temporal arteritis can cause permanent blindness. The sedimentation rate is very high (40 mm/hr to 100 mm/hr). Almost all will have elevated C-reactive protein levels (up to 99%). These patients are managed by rheumatologists via long-term steroids.

Which of the following conditions are possible causes of secondary hypertension? A) Leukemia and thalassemia major B) Hashimoto's thyroiditis and polycystic ovaries C) Renal stenosis and adrenal tumors D) Myocardial infarction and coronary artery disease

C) Renal stenosis and adrenal tumors Secondary hypertension is most likely seen following renal stenosis and adrenal tumors. Renal stenosis causes secondary hypertension by plaque formation in the arteries, causing damage to coronary arteries (atherosclerosis). Adrenal tumors will initiate secondary hypertension by releasing a large amount of aldosterone, which will cause water and salt retention and loss of too much potassium.

A 13-year-old boy wants to be treated for his acne. He has a large number of closed and open comedones on his face. The patient has been treating himself with OTC benzoyl peroxide and salicylic acid topical products. Which of the following would be recommended next? A) Retinoic acid (Accutane) B) Tetracycline C) Retin A 0.25% gel D) Wash face carefully with medicated soap at bedtime

C) Retin A 0.25% gel Topical agents are the first-line treatment for acne vulgaris. Retin A 0.25% gel would be the next step. Oral preparations would then be offered for the next step (tetracycline), and Accutane would be the final step of therapy.

While checking for the red reflex on a 3-year-old boy during a well child visit, a white reflection is seen on the child's left pupil. Which of the following conditions should be ruled out? A) Unilateral strabismus B) Unilateral cataracts C) Retinoblastoma of the left eye D) Color blindness of the left eye

C) Retinoblastoma of the left eye Retinoblastoma is a rare type of cancer in which a cancerous tumor of the retina is present. This is diagnosed by noting a pupil that appears white or has white spots on it. One or both eyes may be affected. It is often seen in photographs, where there will be a white glow in the eye instead of the usual "red eye" noted in pictures from the flash.

A new patient who recently visited a relative in North Carolina complains of an onset of fever and red rashes that started 2 days ago. The rash first appeared on the wrist and the ankles and included the palms of the hands. The patient reports that it is spreading toward his trunk. The patient's eyes are not injected and no enlarged nodes are palpated on his neck. There is no desquamation of the skin. Which of the following is most likely? A) Kawasaki's disease B) Meningococcemia C) Rocky Mountain spotted fever D) Measles

C) Rocky Mountain spotted fever Given the location in the East, Rocky Mountain spotted fever is most likely the infection causing symptoms of fever, rash on ankles/wrists moving to the palms of the hands and the trunk. Rocky Mountain spotted fever is a tickborne disease caused by the bacterium Rickettsia rickettsii, which is transmitted by a bit from an infected tick.

A 28-year-old G3P2 at 16w5d presents to the office for a prenatal visit. You are sure to offer which test appropriate for her gestational age before she leaves? A. Group B strep swab B. Cell-free DNA C. Alpha-fetoprotein D. Ultrasound anatomy scan

C. Alpha-fetoprotein

Which of the following tests would you order for an older diabetic male with the following CBC results? Hb 11 g/dL, Hct 38%, and an MCV 105 fl. His reticulocyte count is normal. A) Serum ferritin and a peripheral smear B) Hemoglobin electrophoresis C) Serum folate acid and B12 level D) Schilling test

C) Serum folate acid and B12 level Serum folate acid and B12 levels would be ordered to evaluate him for folic acid deficiency anemia. Many patients who are deficient in folic acid are also deficient in B12.

Mary Walker is a 16-year-old who presents in the clinic for a physical exam. She tells the triage nurse that she would like contraception as well. How should the nurse practitioner proceed? A) Refuse to see the patient until consent can be obtained from her parent or legal guardian B) Perform a physical exam and discuss contraceptive options C) Speak with the patient about contraception and have her obtain parental consent for the physical exam D) Have the nurse discuss contraception with the patient

C) Speak with the patient about contraception and have her obtain parental consent for the physical exam In the United States, state laws and policies may vary regarding contraceptive counseling and treatment for anyone under the age of 18 years. It is required that you be familiar with the state laws. In all states, anyone under the age of 18 years must have parental consent for the physical exam.

Carol, a 73-year-old patient, complains of episodic vertigo, slight confusion, and weakness that last nearly an hour each time. Movement does not worsen the vertigo. She "rests" and her symptoms subside, but she is puzzled because the weakness "jumps from side to side," sometimes on the right and sometimes on the left of her body. Her symptoms suggest: A) Benign paroxysmal positional vertigo B) Ménière's disease C) TIA (transient ischemic attack) D) CVA (cerebrovascular accident)

C) TIA (transient ischemic attack) Transient ischemic attack (TIA) is caused by vascular occlusion. Symptoms of a TIA usually last less than 1 hour; however, the symptoms may also be permanent. Common signs/symptoms include neurologic deficits, vertigo, confusion, weakness, hemiparesis, temporary monocular blindness, ataxia, and diplopia.

What Tanner stage is a girl at when her breasts form a secondary mound? A) Tanner Stage II B) Tanner Stage III C) Tanner Stage IV D) Tanner Stage V

C) Tanner Stage IV Tanner stages for breast development: I, prepuberty; II, breast bud; III, breast and areola one mound; IV, breast and areola secondary mound; V, adult pattern.

A 10-year-old boy complains of a sudden onset of scrotal pain when he woke up that morning. He is also complaining of severe nausea and vomiting. During the physical examination, the nurse practitioner finds a tender, warm, and swollen left scrotum. The cremasteric reflex is negative and the urine dipstick is negative for leukocytes, nitrites, and blood. The most likely diagnosis is: A) Acute epididymitis B) A severe salmonella infection C) Testicular torsion D) Acute orchitis

C) Testicular torsion Testicular torsion signs and symptoms include sudden onset of unilateral scrotal pain, nausea, vomiting, and abdominal pain. Acute epididymitis causes fever, chills, nausea, and unilateral pain and is most com- monly seen in sexually active men. Unilateral scrotal pain does not occur with Salmonella infection. Acute orchitis is often based on having a recent mumps infec- tion or parotitis with testicular edema.

Candidal intertrigo is the name for an infection that is caused by the yeast Candida albicans. What is the location of this type of candida infection? A) The scalp B) The flexor areas of the elbows and the knees C) The areas of the body where skin is rubbing together, such as under the breast or in the groin area D) The hands

C) The areas of the body where skin is rubbing together, such as under the breast or in the groin area Candidal intertrigo infections are more common in the obese and in women with pendulous breasts. It is found in areas where skin rubs against skin (under breasts, in the groin area, and on stomach folds in the obese). It is more common in warm and humid weather (summer).

All of the following are correct statements regarding the Healthy People 2020 objectives except: A) The document's objectives are not only applicable nationally, but also internationally B) One of the objectives of the document is to help people achieve physical health C) The document's objectives are applicable only to people of the United States of America D) The document is formulated by a special committee formed by an alliance of all the state health departments

C) The document's objectives are applicable only to people of the United States of America Healthy People 2020 was written for all people

Some nurse practitioners bill directly for their services. Regarding reimbursement, who is considered a third-party payer? A) The patient B) The health care provider C) The health insurance companies, health plans, Medicare and Medicaid D) The federal government

C) The health insurance companies, health plans, Medicare and Medicaid Third- party payers are health insurance companies, health plans (HMO or PPO), Medicare and Medicaid. The "first party" is the patient. The "second party" is the health care provider

A 19-year-old student who is on a prescription of Triphasil is being seen for an annual gynecologic exam in the college health center. The nurse practitioner has obtained the Pap smear and is about to perform the bimanual exam. She gently removes the plastic speculum from the vagina. While the NP is performing the bimanual vaginal exam, the patient complains of slight discomfort during deep palpation of the ovaries. Which of the following is a true statement? A) The uterus and the ovaries are both very sensitive to any type of palpation B) The fallopian tubes and ovaries are not sensitive to light or deep palpation C) The ovaries are sensitive to deep palpation but they should not be painful D) The uterus and the ovaries are not important organs of reproduction

C) The ovaries are sensitive to deep palpation but they should not be painful The ovaries are usually slightly sensitive to deep palpation, but they should not be painful. Unilateral adnexal pain accompanied by cervical motion tenderness and purulent endocervical discharge is suggestive of PID.

A sexually active 22-year-old man is asking to be screened for hepatitis B because his new girlfriend has recently been diagnosed with hepatitis B infection. His lab results are the following: anti-HBV is negative, HBsAg is positive, and HBeAg is negative. Which of the following is indicated? A) The patient is immune to the hepatitis B virus B) The patient is not infected with hepatitis B virus C) The patient needs hepatitis B vaccine and hepatitis B immunoglobulin D) The patient needs only hepatitis B immunoglobulin

C) The patient needs hepatitis B vaccine and hepatitis B immunoglobu- lin Because he is HBsAg positive, and anti-HBV negative and HBeAg negative, he needs hepatitis B immunoglobulin and hepatitis B vaccine.

You suspect an enterobiasis infection in a 6-year-old girl. Which of the following tests would you recommend? A) Stool culture and sensitivity B) Stool for ova and parasites C) The scotch tape test D) A Hemoccult test

C) The scotch tape test Enterobiasis infection (pinworms) is caused by small worms that infect the intestines. Symptoms include itching around the anus, which is usually worse at night. The scotch tape test is done by applying the scotch tape on the anal area in the morning; the worms commonly come out at night and will stick to the tape, which is used for diagnosis.

All of the following are false statements regarding acute gastritis except: A) Chronic intake of nonsteroidal anti-inflammatory drugs (NSAIDs) can cause the disorder B) Chronic lack of dietary fiber is the main cause of the disorder C) The screening test for the disorder is the barium swallow test D) The gold standard to evaluate the disorder is a colonoscopy

C) The screening test for the disorder is the barium swallow test The gold standard for diagnosing gastric disease is biopsy of gastric and/or duodenal tissue by upper endoscopy. Chronic use of NSAIDs disrupts the production of prostaglandins, which decreases blood supply and then breaks down the protective layer of the mucosa, which leads to gastric disease.

A 13-year-old male is brought in by his mother for a physical exam. During the genital exam, the nurse practitioner notices that the patient is at Tanner Stage II. Which of the following is the best description of this Tanner stage? A) The penis is growing more in length than in width and the testicles become larger with darker scrotal skin and the pubic hair is starting to curl B) The penis is growing more in width than in length with darker scrotal skin and more numerous pubic hairs that are darker, curly, and coarser C) The testicles become larger and the skin of the scrotum starts to become darker with straight, fine, countable hairs on the genitals and the axilla D) The testicles, penile width and length are all developing quickly

C) The testicles become larger and the skin of the scrotum starts to become darker with straight, fine, countable hairs on the genitals and the axilla Tanner Stage II is when the testicles start to grow. The scrotal skin becomes thicker and starts to get darker (hyperpigmentation). The pubic hair is of a fine texture and straight and there are few countable hairs on the genitals and the axilla.

All of the following are physiologic changes that occur in the body as we age except: A) The half-life of some drugs is prolonged B) There is an increase in cholesterol production by the liver C) There is a mild increase in renal function D) There is a slight decrease in the activity of the immune system

C) There is a mild increase in renal function Physiologic changes that occur in the elderly include decrease in renal function, increased half-life of some medica- tions, increase in cholesterol production by the liver, and slight decrease in the immune system.

All of the following are correct statements regarding physiologic changes found in the elderly with the exception of: A) There is an increase in the fat-to-lean body ratio B) There is a decrease in the ability of the liver to metabolize drugs C) There is an increase in renal function D) Loss of hearing for sounds in the high-frequency range (presbycusis)

C) There is an increase in renal function Physiological changes in the elderly include increase in the fat-to-lean body ratio, decrease in the ability of the liver to metabolize drugs, loss of hearing for sounds in the high-frequency range, and a decrease in renal function.

All of the following drug classes are approved for treating hypertension. Which of the following antihypertensive drug classes is associated with the largest number of research studies? A) ACE inhibitors B) Angiotensive receptor blockers (ARBs) C) Thiazide diuretics D) Calcium channel blockers (CCBs)

C) Thiazide diuretics Thiazide diuretics have been used to treat hypertension for many decades and numerous placebo-controlled studies have documented their effectiveness as an antihypertensive drug class.

During a sports physical exam of a 14-year-old high school athlete, the nurse practitioner notices a split of the S2 component of the heart sound during deep inspiration. She notes that it disappears upon expiration. The heart rate is regular and no murmurs are auscultated. Which of the following is correct? A) This is an abnormal finding and should be evaluated further by a cardiologist B) A stress test should be ordered C) This is a normal finding in some young athletes D) An echocardiogram should be ordered

C) This is a normal finding in some young athletes It is common to hear a split of the S2 over the pulmonic area of the heart with inspiration as long as it disappears with expiration, with no other abnormal symptoms. This is caused by split- ting of the aortic and pulmonic components.

An elderly patient with a productive cough and fever is diagnosed with pneumonia. All of the following organisms are capable of causing community-acquired pneumonia except: A) Haemophilus influenzae B) Mycoplasma pneumoniae C) Treponema pallidum D) Streptococcus pneumoniae

C) Treponema pallidum Treponema pallidum is a gram-negative spirochete bacterium that causes syphilis

Which of the following benzodiazepines has the shortest half-life? A) Lorazepam (Ativan) B) Alprazolam (Xanax) C) Triazolam (Halcion) D) Clonazepam (Klonopin)

C) Triazolam (Halcion) Triazolam (Halcion) has an average half-life of about 2 hours. Xanax has a half-life of 12 hours. Ativan has a half-life of 15 hours. Klonopin has a half-life of 34 hours

Julia, a 16-year-old patient, is being treated for her first urinary tract infection. Julia had an allergic reaction with hives after taking sulfa as a child. Which of the following antibiotics would be contraindicated? A) Cephalexin (Keflex) B) Ampicillin (Amoxil) C) Trimethoprim-sulfamethoxazole (Bactrim) D) Nitrofurantoin crystals (Macrobid)

C) Trimethoprim-sulfamethoxazole (Bactrim) With the allergic history to sulfa drugs, it would be safest to avoid Bactrim.

A mother brings in her 4-year-old daughter who just started attending preschool. She tells the nurse practitioner that her child is complaining of burning and itching in both eyes along with a runny nose. During the physical exam, the child's eyes appear injected bilaterally. The throat is red and her inferior nasal turbinates are swollen. Which of the following is most likely? A) Herpes keratitis B) Corneal ulcer C) Viral conjunctivitis D) Bacterial conjunctivitis

C) Viral conjunctivitis Viral conjunctivitis is inflammation of the conjunctiva with common complaints of red, itchy eyes, without signs of infection.

Symptoms suggestive of ulcerative colitis include all of the following except: A) Bloody diarrhea mixed with mucus B) Nausea and vomiting C) Weight gain D) Abdominal pain

C) Weight gain Symptoms of ulcerative colitis includes bloody diarrhea mixed with mucus, nausea/vomiting, abdominal pain, and possible weight loss with long-term diarrhea

During an annual wellness check of an asymptomatic 22-year-old male patient, the nurse practitioner notes an S3 sound that is short in duration. Which statement should the nurse include in her teaching instructions to patient? A. "Please refrain from exercise at this time. B. "A prompt evaluation with a cardiologist is needed" C. "This is a normal variant and no further testing is required at this time." D. "A transthoracic echocardiogram will be ordered today"

C. "This is a normal variant and no further testing is required at this time."

11-11. A 72-year-old male is brought in by his daughter who reports that her father seems to have had some changes in his memory. He recently lost his car downtown and had to take a cab home. She asks if her father has Alzheimer's dementia. What response by the NP is best? A. "Don't worry. This is a normal part of aging" B. "As long as he can still perform activities of daily living, this is not dementia." C. "We will need further testing" D. Probably not, since women are more likely to develop dementia."

C. "We will need further testing"

Opal calls the clinic to ask for advice about OTC medications. She is complaining of runny nose and sneezing with no other symptoms. Past medical history is remarkable for hypertension. You would intervene if you over heard the office nurse tell her which of the following? A. "You may take an over-the-counter antihistamine like diphenhydramine or loratadine." B. "If your symptoms don't improve in 7 to 10 days, or if they get worse, come into the clinic." C. "You can take any of the over-the-counter cold remedies." D. "Be sure to wash your hands after sneezing or blowing your nose."

C. "You can take any of the over-the-counter cold remedies."

17-9. For which of the following children should universal dyslipidemia screening be performed? A. 20-month-old B. 6-year-old C. 10-year-old D. 16-year-old

C. 10-year-old

17-11. Which of these girls should be referred to the ortho pedic surgeon for evaluation of scoliosis? A. Preteen with BMI of 82nd percentile for age and scoliometer angle of trunk rotation of 6 degrees B. Preteen with BMI of 87th percentile for age and scoliometer angle of trunk rotation of 4 degrees C. 12-year-old with Cobb angle 22 degrees who has not reached menarche D. 17-year-old with Cobb angle 24 degrees who reached menarche at age 13

C. 12-year-old with Cobb angle 22 degrees who has not reached menarche

13-28. In a patient with metabolic syndrome, a goal blood pressure is equal to or less than: A. 120/80 B. 135/85 C. 140/90 D. 150/80

C. 140/90

15-2. Which of the following patients is at highest risk for significant infection? A. 27-month-old with a temperature of 102.7°F (39.3°C) B. 4-year-old with a temperature of 102.0°F (38.9°C) C. 15-day-old with a temperature of 100.8°F (38.2°C) D. 13-year-old with a temperature of 101.9°F (38.8°C)

C. 15-day-old with a temperature of 100.8°F (38.2°C)

11-17. When assessing the patient's deep tendon reflexes, the nurse practitioner notes the reflexes are normal. The nurse practitioner documents them as A. 0 B. 1 C. 2 D. 3

C. 2

A 68-year-old male presents to the clinic with complaints of abdominal pain and increased flatulence. He has only had 2 bowel movements in the past week. You diagnose him with constipation and recommend lifestyle changes for initial treatment. You recommend that he increase his water intake to at least 8 cups a day and ingest how many grams of fiber per day? A. 10-20 g/day B. 15-25 g/day C. 20-35 g/day D. 35-40 g/day

C. 20-35 g/day

A 45-year-old patient with known ASCVD prefers to forgo statin use and make lifestyle modifications. The nurse practitioner knows that the average LD reduction with diet changes only is approximately A. 1-3% B. 3-5% C. 5-10% D. 10-20%

C. 5-10%

A peak age group most likely to suffer from PUD is: A. 14-18, typically due to poor diet and junk food B. 20-30, typically due to ETOH use and stress C. 55-65, typically due to H. pylori infection and NSAID use D. 40-50, typically due to disruption of gut bacteria and food allergies

C. 55-65, typically due to H. pylori infection and NSAID use

Which of the following patients is at risk of developing pancreatitis? A. 14-year-old female with history of iron defi- ciency, anemia, and strong family history of alcoholism B. 52-year-old female who smokes 1/2 pack per day, diagnosed with breast cancer 1 month ago C. 65-year-old male with serum triglyceride level of 1200 mg/dL D. 5-year-old with history of formula intolerance and esophageal reflux

C. 65-year-old male with serum triglyceride level of 1200 mg/dL

18-7. The greatest risk factor for a fall in the elderly population is: A. Diminished lighting B. Poor eyesight C. A personal history of a previous fall D. Osteoporosis

C. A personal history of a previous fall

A 22-year-old male complains of RLQ abdominal pain that started last night. The pain has progressively worsened throughout the night and he is now nauseated and vomiting. Vital signs: HR 110, BP 118/76, RR 16, SpO2 98 % , and temp 101.8°F (38.8°C). Based on this information, which diagnosis is most likely? A. Viral gastroenteritis B. Acute diverticulitis C. Acute appendicitis D. Renal calculi

C. Acute appendicitis

What is sumatriptan used for? A. Treatment of a severe headache B. Prophylaxis of migraine headaches C. Acute treatment of a migraine headache D. Acute treatment of a tension headache

C. Acute treatment of a migraine headache

12-24. The Abnormal Involuntary Movement Scale (AIMS) should be administered to: A. Patients you suspect are having symptoms of tardive dyskinesia B. Patients receiving psychotropic medications C. All patients receiving an antipsychotic medication D. All mental health patients

C. All patients receiving an antipsychotic medication

According to the rule of prescribing scheduled medications, refills for schedule IV drugs such as lorazepam: A. Are allowed, and a new Rx is needed after one year of treatment B. Are allowed, and a new Rx is needed after three months of treatment C. Are allowed, and a new Rx is needed after six months of treatment D. Are not allowed

C. Are allowed, and a new Rx is needed after six months of treatment

18-10. Fall risk assessment tools can be used to assess an elderly patient's risk for falls. The FNP can perform the Timed Up and Go (TUG) test. Which of the fol- lowing accurately describes the TUG? A. Ask the patient to walk 10 feet in a straight line with arms stretched out from their body at shoulder height B. From a standing position, ask the patient to walk 15 feet, turn, walk back, and then sit down in a chair C. Ask the patient to stand from a sitting position, walk 10 feet, turn, walk back to the chair, and sit down D. From a sitting position, ask the patient to stand, walk 15 feet, turn, walk back to the chair, and sit down

C. Ask the patient to stand from a sitting position, walk 10 feet, turn, walk back to the chair, and sit down

16-1. A 19-year-old patient was seen by the nurse practitioner to get refills for her asthma and hay fever medications. At that visit, the patient shows the NP a "rash" behind her knees and in her antecubital areas. When asked what else has changed in her routine, the patient tells the NP she started playing soccer and is taking a hot shower before heading home after practice each day. Based on the patient's symptoms, the care plan may include: A. Washing as usual and using soothing oatmeal lotion on affected areas B. Avoiding foods with peanuts C. Avoiding overly hot baths D. Advising to stop scratching the areas so the rash will go away

C. Avoiding overly hot baths

12-14. A 20-year-old male is evaluated and reports recurrent episodes of binge eating, vomiting, and the use of laxatives to control weight several times a week for the past three months. The NP determines the criteria for a diagnosis of: A. Binge eating disorder is met B. Anorexia nervosa with binge-eating/purging type is met C. Bulimia nervosa is met D. Binge-purge disorder is met

C. Bulimia nervosa is met

Mammograms do not always identify cancer in dense breast tissue. All of the following are more effective at detecting breast cancers in women with dense breasts except: A. Ultrasound В. MRI C. CT scan with contrast D. 3-D mammogram

C. CT scan with contrast

11-15. The nurse practitioner asks the patient to perform a rapid alternating movement (RAM) test to evaluate: A. Cerebral functioning B. Deep tendon reflexes C. Cerebellar functioning D. Frontal lobe functioning

C. Cerebellar functioning

18-2. A 75-year-old patient believes she has suffered neurosensory hearing loss. After examining the patient, the nurse practitioner makes the patient aware that her condition is reversible and does not represent loss of sensory function. The NP is referring to which condition? A. Hyposmia B. Ageusia C. Cerumen impaction D. Presbycusis

C. Cerumen impaction

19-21. The Family Nurse Practitioner might be exceeding scope of practice if he or she: A. Chooses to work in a non-acute family care clinic based in a hospital B. Opens an independent practice in a full-practice state C. Chooses to see only patients whose primary diagnoses are psychiatric in nature and is primarily prescribing psychiatric medications D. Writes prescriptions without the co-signature of an MD

C. Chooses to see only patients whose primary diagnoses are psychiatric in nature and is pri- marily prescribing psychiatric medications

A 56-year-old male presents to the office with complaints of painful, frequent, and urgent urination. He also states that sometimes when he tries to void he has difficulty emptying. This is the third time he has been seen with these same symptoms in the last 8 months, and each time his urine culture showed E. coli. Based on his symptoms, you know that your patient is most likely suffering from which of the following conditions? A. Asymptomatic bacteriuria B. Interstitial cystitis C. Chronic prostatitis D. Varicocele

C. Chronic prostatitis

16-7. A new patient complains of new itchy "bumps" in her genital area. After consulting the Internet for advice, the patient applied a small amount of vinegar to each lesion to help identify their cause. She found this produced acetowhite changes on the surfaces of the lesions. The nurse practitioner visualizes flesh-colored papules with a cauliflower appearance in the patient's genital area and determines that the lesions are most likely: A. Herpetic whitlow and can be treated with a course of acyclovir B. Molluscum contagiosum and can be treated with applications of Canthacur C. Condyloma acuminatum and can be treated with a course of imiquimod (Aldara) D. Verruca vulgaris and can be treated with cryotherapy

C. Condyloma acuminatum and can be treated with a course of imiquimod (Aldara)

21-3. The third step involved in the process of evidence-based practice is: A. Evaluating patient outcomes before and after implemented practice change B. Performing a literature search for the best evidence C. Critically appraising and synthesizing the evidential findings D. Composing a clinical inquiry

C. Critically appraising and synthesizing the evidential findings

13-10. Diabetic ketoacidosis is a potentially life-threatening sequela of diabetes. It features: A. A plasma glucose level of >600 mg/dL B. Serum bicarbonate >15 mEq/L C. Dehydration D. Serum pH>7.3

C. Dehydration

21-6. In a study where a researcher has developed a medication he knows will work to extend the life of lung cancer patients with only minor side effects, it would not be ethical to withhold the medication from the control group. In this instance, the researcher might conduct the study using which type of control group? A. Randomized, meaning placement of subjects in either the control or intervention group should be left to chance B. Placebo, using a pill that has no active ingredient C. Dose-response, where the intervention group receives a larger dose of the cancer medication than the control group D. Wait list, where the intervention group receives the new medication while the control group receives an established older medication

C. Dose-response, where the intervention group receives a larger dose of the cancer medication than the control group

13-25. In addition to waist circumference, blood pressure, triglycerides, and fasting blood sugar, what other lab value is diagnostic criteria for metabolic syndrome? A. Albumin B. Total cholesterol C. HDL-C D. LDL-C

C. HDL-C

19-1. All of the following are rules included in HIPAA except: A. Privacy Rule B. Breach Notification Rule C. Health Insurance Rule D. Security Rule

C. Health Insurance Rule

Rob is a 65-year-old with a 5-year history of bilateral lower leg arterial disease. BMI is 24, HR 75, BP 126/75, RR 18, and SpO2 99%. His medications include metoprolol tartrate (Lopressor) 50 mg BID and ASA 325 mg daily. He has smoked 1 pack-per-day for 20 years and drinks a six-pack of beers nightly. Fasting labs from one week ago: total cholesterol 250, triglycerides 176, LDL 180, HDL 30, and blood sugar 95. He presents for follow-up and lab review. Which of the following would not be included in the treatment plan? A. Atorvastatin (Lipitor) 20 mg every night B. Counseling regarding smoking cessation C. Hemoglobin AlC level 3 D. Counseling regarding weight loss

C. Hemoglobin AlC level

16-30. A 72-year-old presents with a painful rash. The FNP notes a 3-inch strip of vesicles wrapping slightly around the left torso, which is indicative of: A. Cellulitis B. Erysipelas C. Herpes zoster D. Impetigo

C. Herpes zoster

13-33. After a complete thyroidectomy, a patient complains that she is experiencing muscle twitches, abdominal pain, polydipsia, and polyuria. The nurse practitioner thinks that the patient is experiencing a complication from her surgical procedure. The nurse practitioner believes the patient is suffering from: A. Thyroid storm from excessive hormones released during the thyroidectomy B. Diabetes insipidus from damage to the hypothalamus during the thyroidectomy C. Hypercalcemia from damage to the parathyroid glands during the thyroidectomy D.Hypercortisolemia from damage to the adrenal glands during the thyroidectomy

C. Hypercalcemia from damage to the parathyroid glands during the thyroidectomy

An adult male has been diagnosed with nephrolithiasis. Which of the following is the correct patient education? A. Take cranberry supplements on a daily basis B. Once the stone passes, recurrence is unlikely C. Increase daily amounts of fluid intake D. Avoid eating bananas, potatoes, and oranges

C. Increase daily amounts of fluid intake

Which of the following objective findings would be consistent with the diagnosis of otitis externa? A. Bullae on the tympanic membrane B. Red, bulging tympanic membrane C. Increased pain with tragus palpation D. Tympanic membrane immobility

C. Increased pain with tragus palpation

3-3. Many objectives of Healthy People 2020 focus on broader population issues including which of the following? A. Increasing health disparities B. Reducing equal access to quality healthcare C. Increasing services to identified minority populations D. Decreasing the availability and dissemination of health-related information

C. Increasing services to identified minority populations

19-14. Which of the following is covered by Medicaid? A. Eyeglasses B. Respiratory services C. Inpatient hospital care D. Hospice

C. Inpatient hospital care

14-15. Multiple myeloma typically affects patients in: A. Childhood B. Adolescence C. Late adulthood D. Early adulthood

C. Late adulthood

12-7. When starting therapy with an SSRI for GAD, the nurse practitioner should consider starting with a(n): A. SSRI, as the patient will see a decrease in symptoms within two weeks B. SSRI, which will not affect any other medications taken C. Low dose and slowly increasing the dose if needed D. SSRI that is known to be more energizing

C. Low dose and slowly increasing the dose if needed

11-25. Of the following cerebrospinal fluid (CSF) results, which is most consistent with bacterial meningitis? A. Low opening pressure B. Lymphocytosis C. Low glucose, high protein D. Normal protein, normal glucose

C. Low glucose, high protein

13-30. Which of the following is true about initiation of levothyroxine therapy in the elderly? A. Response time is much quicker B. Levels should be checked in 1 week C. Lower doses are required D. Higher doses are required

C. Lower doses are required

12-18. You are concerned your patient is exhibiting possible symptoms of bipolar disorder and need to evaluate further. The best choice of screening tool is: A. PHQ-9 B. GAD-7 C. MDQ D. DAST-10

C. MDQ

What is the first line class of ABX recommended by the American Thoracic Society for patients younger than 60 with CAP with no co morbidities? A. First gen cephalosporins B. Second Gen cephalosporins C. Macrolides D. Beta Lactam antibiotics

C. Macrolides

19-10. Which of the following is true about Medicaid and Medicare? A. Medicare and Medicaid only cover Americans over 65 years old B. Medicare Part B covers most prescription costs C. Medicaid is a program in which the federal government gives states matching funds to cover healthcare services D. Medicare and Medicaid can expect a decline in enrollment numbers since the number of elderly is expected to decrease by 2050

C. Medicaid is a program in which the federal government gives states matching funds to cover healthcare services

14-1. Folate deficiency anemia causes which of the follow- ing changes in the RBC indices? A. Normocytic, normochromic B. Macrocytic, hypochromic C. Microcytic, normochromic D. Microcytic, hypochromic

C. Microcytic, normochromic

12-12. You are assessing a 24-year-old female who received a score of 12 on the PHQ-9. According to her questionnaire, the severity of her depression is: A. Minimal B. Mild C. Moderate D. Severe

C. Moderate

13-3. The gold standard test for Addison's disease is: A. Elevated BUN B. Elevated plasma renin C. Morning serum cortisol D. Serum electrolytes (showing hypokalemia and hyponatremia)

C. Morning serum cortisol

Which statement is true concerning pharyngitis? A. All sore throats require antibiotic treatment B. Most cases of pharyngitis are bacterial in nature C. Most cases of pharyngitis are viral in nature D. Empiric treatment of pharyngitis is important

C. Most cases of pharyngitis are viral in nature

16-2. The nurse practitioner's 65-year-old patient is homeless and has been staying in a local shelter. He complains of pruritus on his ventral wrists and in the webbed areas of his fingers. The nurse practitioner examines his wrists. The most likely description of his lesions is: A. Multiple well-demarcated plaques with overlying silvery-white scale B. Erythematous papules with coalescing/edema- tous plaques (wheals) C. Multiple erythematous vesicles and papules with accompanying silvery linear burrows and secondary excoriations D. Expanding circular red rash with central clearing

C. Multiple erythematous vesicles and papules with accompanying silvery linear burrows and secondary excoriations

18-14. Which form of elder abuse is incorrectly matched with its description? A. Self-neglect: when the elderly fail to take precautions to preserve their own health and safety B. Sexual abuse: nonconsensual sexual contact C. Neglect: the unlawful appropriation or use of another's property for one's own benefit D. Physical abuse: intentionally inflicted physical harm, such as bruising, lacerations, fractures, or broken bones

C. Neglect: the unlawful appropriation or use of another's property for one's own benefit

The nurse practitioner is assessing the results of a urine dipstick for a patient with a suspected renal disorder. The urine is 3+ for blood. This finding rules out which disorder? A. Glomerulonephritis B. Nephrolithiasis C. Nephrotic syndrome D. Urinary tract infection

C. Nephrotic syndrome

Risk factors for infection with DRSP include all of the following except: A. Comorbidities B. Systemic antibiotic use in the last 90 days C. Non-smoking D. Age >65

C. Non-smoking

The nurse practitioner has just diagnosed a patient with mild rheumatoid arthritis. Which medication will the nurse practitioner likely prescribe? A. Corticosteroid B. Disease-modifying anti-rheumatic drug (DMARD) C. Nonsteroidal anti-inflammatory drug (NSAID) D. Opioid

C. Nonsteroidal anti-inflammatory drug (NSAID)

12-25. The nurse practitioner is evaluating a 10-year- old male for ADHD per his parents' and teacher's request. Symptoms include decreased level of focus, impulsivity, and hyperactivity occurring in school. His grades have also declined. With this information, the nurse practitioner will: A. Diagnose ADHD and start medications B. Diagnose ADHD and refer the family for behavioral modification therapy C. Not make any diagnosis, but evaluate further for ADHD D. Not make any diagnosis, and refer back to the school guidance department

C. Not make any diagnosis, but evaluate further for ADHD

16-18. Which of the following is true regarding contact dermatitis? A. Continued exposure to the irritant is allowable B. Medication can control the response to contin- ued irritant exposure C. Oatmeal baths and calamine lotion can be helpful D. Baths may worsen the response to the irritant and should be avoided

C. Oatmeal baths and calamine lotion can be helpful

14-18. When calling a patient back regarding a platelet count of 120,000 platelets/microL and a recheck two weeks later of 99,000 platelets/microL, it is most important to do which of the following? A. Advise the patient to avoid alcohol B. Advise the patient to have platelets rechecked in one month C. Obtain a thorough and up-to-date medication list D. Inquire if they are bruising more easily than usual

C. Obtain a thorough and up-to-date medication list

A 62-year-old patient with a history of mild aortic stenosis reports a recent episode of syncope and more frequent angina. Which action by the nurse practitioner is best? A. Obtain a chest x-ray to look for heart enlargement B. Explain to patient that this is a normal finding with aortic stenosis C. Obtain an echocardiogram and refer to a cardiologist D. Discuss the addition of nitroprusside for angina pain

C. Obtain an echocardiogram and refer to a cardiologist

11-34. The nurse practitioner is considering a clinical diag- nosis of Parkinson disease for her 62-year-old male patient. Which of the following statements would lead to a reconsideration of the diagnosis and a look at alternative diagnoses? A. On a smell test, the patient shows signs of hypos- mia (reduced ability to smell) B. On gait assessment, the patient walks in short shuffling steps C. On a levodopa medication trial, the patient shows no improvement in motor function D. On a motor assessment, the patient shows rest- ing tremor in one hand

C. On a levodopa medication trial, the patient shows no improvement in motor function

A subtle sign of ongoing hemorrhage in silent peptic ulcer disease (PUD) is: A. Gnawing abdominal pain B. Repeated episodes of hematochezia C. Ongoing bouts of orthostasis D. Excessive salivation

C. Ongoing bouts of orthostasis

A 68-year-old male presents to the clinic with complaints of a recent change in the shape and size of his bowel movements, along with lower abdominal pain. He is given a home stool collection kit and is instructed to collect at least two samples from three consecutive specimens. The results are positive for occult blood. What should be the nurse practitioner's next step? A. Order a CT scan B. Order a CEA, CBC, and abdominal x-ray series C. Order a colonoscopy D. Perform a DRE to confirm the gFOBT

C. Order a colonoscopy

All of the following are risk factors for plantar fasciitis except: A. Flat feet B. Obesity C. Osteoporosis D. Prolonged standing

C. Osteoporosis

Which subjective and objective findings suggest an arterial cause of leg pain? A. Complaint of aching to legs that improves with elevation, varicose veins, and dependent rubor B. Dorsalis pedis and posterior tibial pulses 1+ bilaterally and right calf pain, warmth, and edema that started 2 days after driving from Michigan to Florida C. Pain in buttocks and thighs with walking, capil- lary refill greater than 5 secs, ABI 0.80 D. Bilateral leg edema that improves with elevation, legs with increased pigmentation and multiple telangiectasias

C. Pain in buttocks and thighs with walking, capil- lary refill greater than 5 secs, ABI 0.80

A diagnosis of benign prostatic hyperplasia includes all of the following except: A. Urinary urgency and frequency B. Slow urination, occasional spraying of urine C. Painful urination D. Enlarged prostate on digital rectal exam

C. Painful urination

Which meal choice will the nurse practitioner recommend for the patient to have when taking a dose of ivermectin? A. Baked lean chicken and asparagus B. Grilled whitefish and broccoli C. Pork chop with baked potato and butter D. Vegetable salad with lemon juice

C. Pork chop with baked potato and butter

3-15. Which of the following is not a permanent contraindication to vaccination? A. Systemic allergic reaction to a vaccine component B. Encephalopathy following pertussis immunization C. Pregnancy in a younger female D. Anaphylaxis to a vaccine component

C. Pregnancy in a younger female

12-4. A patient describes his alcohol use as 2-3 drinks per night, 4-5 nights per week. He has been missing work and family functions because of his drinking. He reports wanting to reduce his drinking, but not being able to. He admits to having a problem and has vague plans to make changes. Which stage of change does this represent? A. Precontemplation B. Contemplation C. Preparation D. Action

C. Preparation

Tyler is a 68-year-old male who complains of fatigue and shortness of breath. He states his heart "feels funny" His exam shows HR 140 and irregular, BP 105/62, RR 18, SpO, 98% , BMI 31. Medications include lisinopril 20 mg daily and tamsulosin 0.4 mg daily. A 12-lead ECG is obtained and the rhythm is atrial fibrillation. What would be your next course of action? A. Refer patient for a same-day ECHO B. Place transcutaneous pacing pads on patient C. Prepare the patient for immediate transport D. Refer to cardiology

C. Prepare the patient for immediate transport

A 22-year-old nulliparous female presents to the office with complaints of burning on urination and vaginal itching. She broke up with her boyfriend three months ago and recently became sexually active with someone new. Her symptoms began three days ago and she has already tried OTC miconazole, which made the burning worse. She denies any urinary frequency, urgency, pressure, vaginal odor or postcoital bleeding. On exam, you note multiple blistering lesions with red, swollen bases, in varying stages of healing. You culture the lesions, but tell her you are most confident she has which diagnosis? A. Molluscum B. Secondary syphilis C. Primary HSV infection D. Condyloma acuminatum caused by HPV

C. Primary HSV infection

12-22. It is important to obtain labs on patients taking atypical antipsychotics: A. Prior to initiating medications, every six months and more frequently if needed B. After being on the medication for six months and yearly thereafter C. Prior to initiating medications, at three months, six months, yearly, and more frequently if needed D. After being on the medication for three months, at six months, and yearly

C. Prior to initiating medications, at three months, six months, yearly, and more frequently if needed

11-22. In a patient using zolmitriptan 2.5 mg tablets more than twice per week, the nurse practitioner knows to start which of the following medications? A. Amlodipine (Norvasc) 10 mg daily B. Trazodone (Desyrl) 150 mg daily C. Propranolol (Inderal) 80 mg per day D. Tramadol (Ultram) 50 mg twice daily

C. Propranolol (Inderal) 80 mg per day

19-17. Which of the following is not an example of patient-centered ethical nursing care? A. Providing for patient safety and maintaining an appropriate nurse-patient relationship B. Providing for patient privacy and respecting patient wishes C. Providing evidence-based nursing care and advocating for personal vacation time D. Respecting patient dignity and advocating for patients in the policy and research arenas

C. Providing evidence-based nursing care and advocating for personal vacation time

What is the least common pathogen found in community-acquired atypical pneumonia? A. Moxarella catarrhalis B. Strep Pneumo C. Pseudomonas aeruginosa D. Mycoplasma Pneumonia

C. Pseudomonas: is an uncommon cause of CAP but is very difficult bacteria to treat

An IV drug user is being treated for infective endocarditis (IE) and complains of dyspnea, chest pain, and lightheadedness. The nurse practitioner understands that these are common signs of which finding seen in IE? A. Splenic emboli event B. Myocardial infarction C. Pulmonary emboli event D. Renal emboli event

C. Pulmonary emboli event

20-5. All of the following are considered to be social determinants of health except: A. Built environment B. Education level C. Race and ethnicity D. Social context

C. Race and ethnicity

The FNP has diagnosed a patient with allergic rhinitis. What diagnostic testing would be appropriate? A. Blood chemistries B. Computed tomography of the sinuses C. Radioallergosorbent testing D. Plain x-ray films of the sinuses

C. Radioallergosorbent testing

The NP is caring for a 10-year-old with a score throat and rash. Which test should be ordered to verify the most common cause of bacterial pharyngitis? A. Anti-streptolysin O B. Heterophile antibody test C. Rapid antigen detection test D. Rapid plasma reagin

C. Rapid antigen detection test

18-5. Gastrointestinal changes associated with aging can lead to less efficient emptying of the bowel. This can likely cause: A. Diarrhea B. Aspiration pneumonia C. Reduced drug clearance D. Odynophagia

C. Reduced drug clearance

A 30-year-old breastfeeding patient was diagnosed with mastitis and treated with dicloxacillin (Dynapen) for 7 days. She presents for reassess- ment. The patient reports, "I think things are getting better." The affected breast remains erythematous throughout. Which of the following is best for the nurse practitioner to do? A. Discontinue the dicloxacillin and prescribe trimethoprim/sulfamethoxazole (Bactrim) B. Advise the patient to use cabbage leaves for her engorgement C. Refer the patient to a breast surgeon D. Schedule the patient for a mammogram

C. Refer the patient to a breast surgeon

11-42. In speaking with a patient with suspected Guillain- Barré syndrome, the nurse practitioner asks if the patient has a history of: A. Head injury without follow-up treatment B. Damage to the spinal cord C. Respiratory illness in the past four weeks D. Seizure activity diagnosed at birth

C. Respiratory illness in the past four weeks

In a child with ototis media with effusion (OME), which characteristic will the tympanic membrane have upon otoscopic examination? A. Erythematous B. Bulging C. Retracted D. Vascular in appearance

C. Retracted

New patient who recently visited relative in north carolina c/o a new onset fever, and red rashes that started 2 days ago. First rash appeared on wrists and ankles, and included palms of hands. Pt reports the rash is spreading to his trunk. His eyes are not injected, and no enlarged nodes are palpated on his neck. There is no desquamation of the skin. Which of the following is most likely? A. Kawasaki's disease B. Meningococcemia C. Rocky Mountain spotted fever D. Measles

C. Rocky Mountain Spotted fever: given recent travel, rash location. Tick born illness

3-7. A patient with type 2 diabetes who sees a podiatrist for monthly foot examinations is practicing what type of prevention? A. Tertiary prevention B. Primary prevention C. Secondary prevention D. Diabetic surveillance

C. Secondary prevention

Which of the following is not a symptom of herpes keratitis? A. Eye pain B. Photophobia C. Seeing halos D. Blurred vision

C. Seeing halos

11-54. Which statement is true regarding brain tumors? A. Secondary brain tumors are most common in adolescents B. High-dose glucocorticoids are used in treatment if lymphoma is suspected C. Seizures are the most common symptom associ- ated with a brain tumor D. CT scan with contrast is the imaging modality of choice to diagnose a brain tumor

C. Seizures are the most common symptom associ- ated with a brain tumor

13-19. A patient presents with a blood sugar of 51 mg/dL. You would anticipate seeing the following symptoms except: A. Slurred speech, unsteadiness, and vomiting B. Confusion, syncope, and blurred vision C. Seizures, diarrhea, and chest pain D. Headache, palpitations, and nausea

C. Seizures, diarrhea, and chest pain

A 43-year-old female with HIV has just been diagnosed with active pulmonary tuberculosis (TB). She has a 19-year-old daughter that lives with her and has been caring for her. Which of the following actions should be taken with regard to the patient's daughter? A. Because of her age, TB prophylaxis is contraindicated, even if she has a positive TST B. If her TST is positive, but her chest x-ray is negative and she displays no clinical symptoms, no further evaluation or treatment is indicated C. She should receive TB prophylaxis if her TST is >=5mm induration D. She should be treated for active pulmonary TB with a multidrug regimen

C. She should receive TB prophylaxis if her TST is >=5mm induration

19-18. Which of the following is an example of nonmaleficence? A. A nurse practitioner who has been seeing a long-time dialysis patient makes sure to remember the patient's birthday and helps staff arrange to have a sugar-free birthday cake waiting for the patient in the dayroom for after treatment B. A patient calls the nurse practitioner and informs her that she is not going to go through with scheduled gallbladder surgery. She tells the nurse practitioner she would prefer to wait and see if taking ursodiol (Actigall) might help. After making sure that the patient clearly understands the pros and cons of this decision, the nurse practitioner respects the patient's choice C. Staff in a nursing home contact the nurse practitioner, stating that a patient who recently began taking an antibiotic has developed signs of angioedema. The nurse practitioner orders the staff to withhold the medication D. A nurse practitioner works in a gynecology clinic that predominantly serves teens. The nurse practitioner has noticed that at-risk teens are not receiving equal access to services because they mostly go to a school across town and can't make it to the clinic before closing time. The nurse practitioner shifts the start time at the clinic so that the at-risk teens have equal access to clinic appointments

C. Staff in a nursing home contact the nurse practitioner, stating that a patient who recently began taking an antibiotic has developed signs of angioedema. The nurse practitioner orders the staff to withhold the medication

16-27. The nurse practitioner caring for an older adult has noted a pressure ulcer exhibiting full-thickness skin loss with a crater-like appearance. The nurse practitioner documents this ulcer as which stage? A. Stage 1 B. Stage 2 C. Stage 3 D. Stage 4

C. Stage 3

13-22. A 24-year-old patient with diabetes mellitus type 1 presents to the clinic with complaints of a recent severe hypoglycemic episode. Which of the following symptoms would you expect the patient to report? A. Anxiety, dizziness, indigestion B. Nausea, vomiting, diarrhea C. Sweating, shakiness, headache D. Confusion, weakness, dry mouth

C. Sweating, shakiness, headache

Every prenatal visit includes the following information except: A. Fundal height B. Fetal heart rate C. Temperature D. Blood pressure

C. Temperature

14-3. Martha has a low mean cell volume (MCV), low mean cell hemoglobin concentration (MCHC), and normal red cell distribution (RDW). What should you consider in terms of diagnosis? A. Renal disease B. Iron deficiency anemia C. Thalassemia D. Pernicious anemia

C. Thalassemia

12-28. It is important to screen for autism spectrum disorders (ASD) at: A. The 18- and 24-month well-child check B. Every well-child check C. The 18- and 24-month well-child check and any time there is a concern for delays D. The 12-, 18-, 24-, and 36-month well-child check

C. The 18- and 24-month well-child check and any time there is a concern for delays

19-23. The two national organizations that are permitted to grant FNP board certification status are: A. The American Nurses Credentialing Center (ANCC) and the American Nurses Association (ANA) В. The American Academy of Nurse Practitioners (AANP) and the nurse practitioner's local state board of nursing C. The American Nurses Credentialing Center (ANCC) and the American Academy of Nurse Practitioners (AANP) D. The American Academy of Nurse Practitioners (AANP) and the National Council of State Boards of Nursing (NCSBN)

C. The American Nurses Credentialing Center (ANCC) and the American Academy of Nurse Practitioners (AANP)

19-6. Which of the following statements is true? A. The NP previously worked for a company from 1/2014-1/2018. She had a claim-based malpractice policy through this employer. She is covered for a malpractice claim that occurred 12/2017 even though she no longer has this policy and does not have tail coverage B. The NP is preparing to retire and decides he will not need tail coverage for his claims-based malpractice policy because he will no longer be in practice C. The NP had an occurrence-based policy from 3/2012-4/2018. She is covered for a malpractice claim that occurred 2/2014, even though she is no longer employed and no longer has coverage D. The NP had a claims-based policy with a previous employer for 11/2008-11/2018. He starts a new job on 2/2019 and obtains an occurrence policy that starts 2/2019. He is covered for a malpractice claim that occurs 1/2019

C. The NP had an occurrence-based policy from 3/2012-4/2018. She is covered for a malpractice claim that occurred 2/2014, even though she is no longer employed and no longer has coverage

The FDA is in the process of eliminating the pregnancy categories (A, B, C, D, and X) and replacing them with a new labeling system. All of the following are benefits of the new system, except: A. It will include information for females and males of reproductive potential, pregnancy, and lactation B. It will summarize the specific risks associated with each medication C. The Organization will be much simpler than using the five categories D. If a medication is absorbed systemically, it will state whether the information is based on animal or human studies

C. The Organization will be much simpler than using the five categories

20-1. Standard 8 of the ANA Scope of Practice and Standards of Care addresses culturally congruent care. Culturally congruent care considers all of the following except: A. Linguistics and language translation B. Ethnopharmacology and pharmacogenetics C. The notion that Western medicine has superior science and should be adhered to when caring for patients from all ethnically diverse backgrounds D. Culture-bound syndromes and alternative approaches to disease etiology and treatment options

C. The notion that Western medicine has superior science and should be adhered to when caring for patients from all ethnically diverse backgrounds

12-17. You are evaluating a 20-year-old college student who presents with symptoms of moodiness, not sleeping well, feeling more down than usual. The student asks if he has bipolar disorder. What is true about this patient? A. The patient meets the criteria for bipolar disor- der, and you start medication B. The patient meets the criteria for bipolar disorder, and you refer to a mental health provider C. The patient does not meet the criteria for bipolar disorder but does warrant further evaluation D. The patient does not meet the criteria for bipolar disorder but does meet criteria for depressive disorder

C. The patient does not meet the criteria for bipolar disorder but does warrant further evaluation

12-30. The FNP is asked to evaluate a new patient who presents as clinically depressed. The FNP develops a concern about the patient's risk of suicide based on all of the following factors except: A. The patient stating, "Everything is fine" B. The patient describing his impulsive nature C. The patient's wife sending him to see the NP D. The patient having taken fluoxetine for several months

C. The patient's wife sending him to see the NP

16-28. The FNP is assessing a teen boy's scalp and notes black dots in a quarter-sized circular pattern. This finding is indicative of A. Melanoma B. Early onset male pattern baldness C. Tinea capitis D. Alopecia areata

C. Tinea capitis

Which finding is most consistent with the diagnosis of acute bacterial rhinosinusitis? A. Upper respiratory symptoms lasting less than 7 days B. Mild tenderness and fullness with sinus palpation C. Upper respiratory symptoms lasting longer than 10 days D. Thick nasal drainage that has become discolored

C. Upper respiratory symptoms lasting longer than 10 days

A 23-year-old female presents to the office with a desire for birth control. She is generally healthy and has no drug allergies but is lactose intolerant. She denies any PMH/PSH and denies smoking cigarettes. She takes a multivitamin "most of the time," though she often forgets. She reports her period is regular and she likes having it because it reassures her she's not pregnant. Based on this information, the nurse practitioner recommends which of the following birth control methods? A. Combined oral contraceptive pills B. DMPA (Depo-Provera) injections C. Vaginal ring (NuvaRing) D. LNG-IUD (Skyla)

C. Vaginal ring (NuvaRing)

11-37. Myasthenia gravis occurs when antibodies destroy the __________ receptors at the neuromuscular junction causing __________. A. adrenergic; muscle contraction B. dopaminergic adrenergic; muscle contraction C. acetylcholine; muscle weakness D. metabotropic; muscle weakness

C. acetylcholine; muscle weakness

A patient has recently been diagnosed with hyperthyroidism. Which medication will the nurse practitioner prescribe? A. hydrocortisone (Cortef) B. levothyroxine (Levoxyl) C. methimazole (Tapazole) D. metformin (Glucophage)

C. methimazole (Tapazole)

A 54-year-old woman with known rheumatoid arthritis is in the office for a routine visit. She has swelling in her 2nd and 3rd MCP joints bilaterally and reports morning stiffness lasting for more than 1 hour. She has been taking naproxen twice daily for the past several years, but she no longer feels this medication is helping with her arthritis. Which DMARD medication would you consider using for treatment of this patient? A. leflunomide (Arava) B. etanercept (Enbrel) C. methotrexate (Rheumatrex) D. adalimumab (Humira)

C. methotrexate (Rheumatrex)

A 75-year-old man presents to the office with complaints of blurry vision, urinary retention, dry mouth, and constipation. Based on these symptoms, you know he is most likely experiencing an adverse reaction to which of the following? A. nifedipine (Procardia) B. baby aspirin (Bayer low dose) C. solifenacin (VESIcare) D. omega-3 fish oil

C. solifenacin (VESIcare)

Following a pulmonary embolism diagnosis, anticoagulant therapy should be continued for __________ month(s) and then reevaluated. A. one B. two C. three D. six

C. three

4-10. Phillip R. is a 42-year-old male who presents to the clinic with recurrent genital herpes. Which of the following is recommended for the treatment of genital herpes in this individual? A. ganciclovir (Zirgan) B. atazanavir (Reyataz) C. valacyclovir (Valtrex) D. ribavirin (Virazole)

C. valacyclovir (Valtrex)

4 y.o. female comes in after just starting pre school complaining of burning/ itching in both eyes along with runny nose. Child's eyes appear injected bilaterally, the throat is red and her inferior nasal turbinates are swollen. Which is most likely? A. Herpes Keratitis B. Corneal Ulcer C. Viral Conjunctivitis D. Bacterial conjunctivitis

C: Viral- no signs of infection but pts often complain of red/ itchy/ swollen eyes

Beta thalassemia minor is considered a: A. Macrocytic anemia B. Normocytic anemia C. Microcytic anemia D. Hemolytic anemia

C; Microcytic anemia: Beta thalassemia is a genetic disorder in which the bone marrow makes small pale RBCs (microcytic hypochromic), and microcytic anemia occurs.

Asthmatics may have all of the following symptoms during an asthma exacerbation except: A. Rapid Pulse B. Wheezing C. Chronic coughing D. Tachypnea

Chronic coughing

When a domestic dog is suspected to be infected with the rabies virus, it can either be killed for a brain biopsy or it can be quarantined. What is the minimum number of days that a dog suspected of rabies must be quarantined? A) 4 weeks B) 21 days C) 14 days D) 10 days

D) 10 days The minimum number of days to quarantine an animal suspected of rabies is 10 days. If the animal is healthy and has no symptoms of rabies at 10 days, it is not infected with the rabies virus and can be returned to the owner.

The earliest age that an MMR can be administered is at: A) 4 months B) 6 months C) 8 months D) 12 months

D) 12 months The earliest age that MMR is recommended is 12 months. This age is recommended because giving it any earlier may be less effective because the infant still has antibodies from the mother. Antibodies still present from the mother may interfere with the production of the antibodies stimulated by the MMR vaccine.

All of the following are considered emancipated minors except: A) 15-year-old male who is married B) 14-year-old female who is a single parent C) 17-year-old male who is enlisted in the U.S. Army D) 13-year-old being treated for a sexually transmitted disease

D) 13-year-old being treated for a sexually transmitted disease There are three primary ways for a minor to become emancipated: marriage, court order, and military service.

At what age can a child copy a cross and ride a bicycle? A) 1 year B) 2 years C) 3 years D) 4 years

D) 4 years Developmental stages in children include the following: 1 year: walk; 2 years: walks up steps with the same foot; 3 years: pedals a tricycle and copies a circle; 4 years: rides a bicycle and copies a cross and draws a person with 2 parts.

At what age can a child ride a bicycle? A) 2 to 3 years of age B) 3 to 4 years of age C) 4 to 5 years of age D) 5 to 6 years of age

D) 5 to 6 years of age At about the age of 5 to 6 years, most children can ride a bicycle with training wheels. Helmets should always be worn (primary prevention).

All of the following patients are at higher risk for suicide except: A) A 66-year-old White male whose wife of 40 years recently died B) A high school student with a history of bipolar disorder C) A depressed 45-year-old female with family history of suicide D) A 17-year-old teen who has only 1 close friend in school

D) A 17-year-old teen who has only one close friend in school Risk factors for suicide include: 1) elderly white males (especially after the death of a spouse); 2) past history of suicide; 3) family history of suicide; 4) plans for use of a lethal weapon such as a gun or knife; 5) female gender has a higher attempt rate, but males have a higher success rate; 6) personal history of bipolar disorder or depression.

All of the following patients should be screened for diabetes mellitus except: A) An obese man of Hispanic descent B) An overweight middle-aged Black woman whose mother has type 2 diabetes C) A woman who delivered an infant weighing 9.5 lb D) A 30-year-old White man with hypertension

D) A 30-year-old White man with hypertension The 30-year-old White man with hypertension would be the last patient to be screened for diabetes. Not having any information about him also puts him lower on the list. Obesity, ethnicity (Hispanic/Latino Americans, African Americans, Native Americans, Asian Americans, Pacific Islanders, and Alaska natives), family history of diabetes, and gestational diabetes (mother of the infant weighing 9.5 lbs) are all risk factors. These were present in all of the other selections. Other risk factors for diabetes include impaired glucose tolerance test, sedentary lifestyle, PCOS, and hypertension.

Which of the following situations is considered emergent? A) A laceration on the lower leg of a patient on aspirin (Bayer) 81mg every other day B) Rapid breathing and tachycardia in a patient with a fever C) An elderly man with abdominal pain whose vital signs appear stable D) A 37-year-old male biker with a concussion due to a fall who appears slightly agitated and does not appear to understand instructions given by the medical assistant checking his vital signs

D) A 37-year-old male biker with a concussion due to a fall who appears slightly agitated and does not appear to understand instructions given by the medical assistant checking his vital signs The biker who had the concussion is the emergent situation due to his agitation and his inability to follow directions, which could mean he has some type of brain trauma. The laceration can be treated with pressure to stop the bleeding until help arrives. The man with abdominal pain has normal vital signs. Tachycardia is common with fever.

What is the pedigree symbol for a diseased or affected female? A) An empty square B) An empty circle C) A filled-in square D) A filled-in circle

D) A filled-in circle A filled-in circle is a diseased/affected female and an empty circle is a healthy female. A tip to remember is that females make eggs (or follicles), which resemble a circle. By default, the square symbol is the male.

A 35-year-old sexually active male presents with a 1-week history of fever and pain over the left scrotum. It is accompanied by frequency and dysuria. The scrotum is edematous and tender to touch. He denies flank pain, nausea, and vomiting. He reports that the pain is lessened when he uses scrotal support briefs. His urinalysis shows 2+ blood and a large number of leukocytes. What is the most likely diagnosis? A) Acute urinary tract infection B) Acute pyelonephritis C) Acute orchitis D) Acute epididymitis

D) Acute epididymitis Acute epididymitis is the infection presented here. Scrotal edema and pain with palpation do not occur in UTI or pyelonephritis. Acute orchitis symptoms include testicular pain and edema, are usually associated with the mumps, but do not have frequency and dysuria.

Prophylaxis for Pneumocystis carinii pneumonia includes all of the following drugs except: A) Trimethoprim-sulfamethoxazole B) Dapsone C) Aerosolized pentamidine D) Aerosolized albuterol sulfate (Ventolin)

D) Aerosolized albuterol sulfate (Ventolin) Prophylaxis for Pneumocystis carinii pneumonia includes the use of Bactrim, dapsone, and aerosolized pentamidine. Aerosolized pentamidine is a antimicrobial treatment for prevention, along with Bactrim and dapsone, which are antibacterial medications.

All of the following are clinical signs and symptoms seen early in testicular torsion except: A) Nausea and vomiting B) Absence of the cremasteric reflex C) Affected testicle is elevated compared with the normal testicle D) Affected testicle is swollen and feels cold to touch

D) Affected testicle is swollen and feels cold to the touch The affected testicle will be swollen, but it will feel very warm to the touch. A cold testicle is abnormal and is indicative of gangrene (after 24 hours).

When a patient is suspected of having acute pancreatitis, initial testing should include all of the following except: A) Electrolyte panel B) Serum amylase level C) Serum lipase level D) A barium swallow

D) Barium swallow For an acute abdominal pain, initial labs performed are serum electrolytes, amylase, and lipase. Barium swallow would not be performed initially.

Koilonychia is associated with which of the following conditions? A) Lead poisoning B) Beta thalassemia trait C) B12 deficiency anemia D) Iron-deficiency anemia

D) Iron-deficiency anemia Koilonychia is an abnormal shape of the nail. The nailbed is thin, with irregular edges, and curves inward. Koilonychia is commonly associated with iron-deficiency anemia.

Which of the following is a true statement regarding the first-pass metabolism process? A) Drugs that are administered by intramuscular injection all go through the process of first-pass metabolism B) After being swallowed, oral drugs are absorbed by the GI tract and metabolized by the bacteria in the small intestines before being released into the general circulation C) Drugs administered through the skin (patches) are metabolized by the dermis of the skin D) After a drug is taken by the oral route, it is absorbed in the small intestines and enters the liver through the portal circulation, where it is metabolized before being released into the general circulation

D) After a drug is taken by the oral route, it is absorbed in the small intestines and enters the liver through the portal circulation, where it is metabolized before being released into the general circulation After drugs are taken by the oral route, they are absorbed by the small intestines and enter the liver through the portal circulation, where they are metabolized before being released into the general circulation. First-pass metabolism (first-pass effect) determines how much of the active drug is available to the body (bioavailability). Depending on the drug and other factors, a drug may be poorly metabolized or extensively metabolized by the liver.

Jim Wheeler is obese (BMI of 33), fatigued, and complaining of excessive thirst and hunger. You suspect type 2 diabetes mellitus. Initial testing to confirm diagnosis can include: A) Fasting plasma glucose level B) Glycosylated hemoglobin level (HbA1 c) C) Glucose tolerance testing D) All of the above

D) All of the above Type 2 diabetes mellitus screening tests include: fasting plasma glucose level (> 126 mg/dL), random plasma glucose level (> 200 mg/dL), and glucose tolerance testing (2 hr blood glucose level > 200 mg/dL) with 75 g glucose load. Normal HbA1c levels are < 6%

Carol M. is a 40-year-old bank teller who has recently been diagnosed with obsessive-compulsive disorder by her therapist. Her symptoms would include: A) Ritualistic behaviors that the patient feels compelled to repeat B) Attempts to ignore or suppress the repetitive behaviors, which increase anxiety C) Frequent intrusive and repetitive thoughts and impulses D) All of the above

D) All of the above Signs and symptoms of obsessive-compulsive disorder include ritualistic behaviors that are repeated, increased anxiety with attempting to ignore repetitive behaviors, and frequent intrusive and repetitive thoughts and impulses.

Ted, who is 15 years old, has just moved into the community and is staying in a foster home temporarily. There is no record of his immunizations. His foster mother wants him to be checked before he enters the local high school. Which of the fol- lowing does this patient need? A) Meningococcal B) MMR C) Tdap D) All of the above

D) All of the above The measles, mumps, and rubella (MMR) is recommended as one of the "catch-up" immunizations for this patient's age group. The tetanus immunization that is recommended as a "catch-up" for that age group is the Tdap instead of just the Td.

All of the following are covered under Medicare Part B except: A) Persons age 65 years or older B) Durable medical equipment C) Mammograms annually starting at age 50 D) Anesthesiologist's services

D) Anesthesiologist's services Medicare Part B covers: 1) outpatient physician visits, labs, x-rays; 2) durable medical equipment; 3) mammograms/colonoscopy after age 50 years annually; 4) rehabilitation. Anesthesiologist's services are covered by Medicare Part A.

Which of the following is used to confirm a diagnosis of Hashimoto's thyroiditis? A) Serum TSH B) Free T4 test C) Antimicrosomal antibody test D) Any of the above

D) Any of the above In addition to conducting a physical examination and taking a thorough history and symptoms into account, 1 or more laboratory tests are used to diagnose Hashimoto's thyroiditis. The 3 most common diagnostic tests that detect this common thyroid disorder are: serum thyroid-stimulating hormone test (TSH), anti-thyroid antibodies tests, and the free T4 hormone test.

Signs and symptoms of depression include all of the following except: A) Anhedonia and changes in appetite B) Decreased energy and irritability C) Apathy and low self-esteem D) Apraxia and fatigue

D) Apraxia and fatigue Apraxia is a disorder of the nervous system in which the brain is affected and the patient is unable to move the arms/legs when asked to do so. Common signs of depression include anhedonia (loss of interest in activities that the patient finds pleasurable), unintentional weight loss or gain, fatigue, change in appetite, insomnia or hypersomnia, feelings of guilt and worthlessness, and recurrent thoughts of suicide.

Which of the following drugs that are used to treat attention deficit hyperactivity disorder (ADHD) is not classified as an amphetamine/stimulant? A) Dexmethylphenidate (Focalin XR) B) Mixed salts of amphetamine (Adderall) C) Methylphenidate (Ritalin) D) Atomoxetine (Strattera)

D) Atomoxetine (Strattera) Strattera is classified as a norepinephrine reuptake inhibitor. It is not a stimulant or an amphetamine. Strattera is contraindicated during/within 14 days of taking an MAOI, narrow-angle glaucoma, heart disorder where increases in BP or heart rate will worsen it, or pheochromocytoma. Children and teenagers should be monitored for suicidal thoughts/plans.

All of the following tests require the patient's voice to perform correctly except: A) Egophony B) Tactile fremitus C) Whispered pectoriloquy D) Auscultation

D) Auscultation Egophony, tactile fremitus, and whispered pectoriloquy require the patient to speak. Auscultation is listening to an organ with the use of a stethoscope.

All of the following statements are false regarding the rehabilitation of alcoholics except: A) Al-Anon is not designed for family members of alcoholics B) Disulfiram (Antabuse) is always effective C) Alcoholics Anonymous is not an effective method for treating this condition D) Avoid foods or drinks that contain alcohol, such as cough syrups

D) Avoid foods or drinks that contain alcohol, such as cough syrups Alcoholics Anonymous is designed for families and is an effective treatment for alcoholism. Antabuse is effective for some patients. The intake of foods/medications that contain alcohol must be avoided.

All of the following are not recommended for the outpatient treatment of mild preeclampsia except: A) Severe sodium restriction B) Restrict fluid intake to less than 1 liter per 24 hours C) Aldomet (methyldopa) 250 mg PO (orally) BID D) Bed rest on the left side with bathroom privileges

D) Bed rest on the left side with bathroom privileges Outpatient treatment for mild pre-eclampsia includes bed rest on the left lateral side with bathroom privileges.

Which type of hepatitis virus infection is more likely to result in chronic hepatitis and increased risk of developing hepatocellular carcinoma? A) Hepatitis Avirus B) Hepatitis B virus C) Hepatitis C virus D) Both hepatitis B and hepatitis C

D) Both hepatitis B and hepatitis C Of the primary hepatitis viruses, only B and C are associated with hepatocellular cancer.

A 55-year-old female with a history of migraine headaches has recently been diagnosed with Stage II hypertension. Her EKG strips reveal second-degree heart block. The chest x-ray is normal. Which of the following drugs should this patient avoid? A) ACE inhibitors B) Angiotensin receptor blockers C) Diuretics D) Calcium channel blockers

D) Calcium channel blockers Common side effects of calcium channel blockers include headaches, edema of the lower extremities, and heart block or bradycardia. Contraindications for calcium channel blockers include second- or third-degree AV block, bradycardia, and congestive heart failure.

A medical assistant is calling out the names of patients who are in the waiting room. The medical assistant is following the HIPAA Privacy Rule if she performs which of the following actions? A) Call patients by their full name to show respect B) Call patients by their last name or surname C) If the patient prefers to be called by a nickname, use that name to call him/her inside D) Call patients by using the first name only

D) Call patients by using the first name only Patients who are in waiting rooms or rooms with other people should be called by their first names only, to protect their privacy.

A 14-year-old is brought in by his mother who reports that her child has been complaining for several months of recurrent bloating, stomach upset, and occasional loose stools. She reports that her son has difficulty gaining weight and is short for his age. She has noticed that his symptoms are worse after eating large amounts of crackers, cookies, and breads. She denies seeing blood in the child's stool. Which of the following conditions is most likely? A) Amebiasis B) Malabsorption C) Crohn's colitis D) Celiac disease

D) Celiac disease Celiac disease is also known as celiac sprue. Patients should avoid foods containing gluten, which causes malabsorption (diarrhea, gas, bloat- ing, abdominal pain, etc.). Foods to avoid are wheat, rye, and barley. Oats do not damage the mucosa in celiac disease. Antigliadin IgA and IgG are elevated in almost all patients (90%).

All of the following are considered benign physiologic variants except: A) Internal tibial torsion B) Supernumerary nipples C) Split uvula D) Cheilosis

D) Cheilosis Cheilosis is a skin disorder in which fissures and maceration occur in the corner of the mouth

A 16-year-old male with a recent history of a cat bite is brought in by his father. The bite occurred about 2 hours before the visit. The nurse practitioner evaluates the wound and notes 2 small puncture wounds. There is no redness or purulent discharge. The father reports that the teenager received a tetanus booster when he was 12 years old. Which of the following is the correct action to take? A) Clean the wound with soap and water and apply topical antibiotic and a bandage B) Because the wound is clean and does not appear infected, there is no need for antibiotics C) Give the patient a tetanus booster using the Tdap form of the vaccine D) Clean the wound with soap and water and prescribe Augmentin 500mg PO BID x 10 days for the patient

D) Clean the wound with soap and water and prescribe Augmentin 500 mg PO BID x 10 days for the patient Cat wounds are more likely to become infected compared with dog bites; in addition, this patient's bite is located on an extremity. These facts justify the prescription of Augmentin 500 mg PO BID x 10 days for the patient.

All of the following are considered risk factors for UTIs in women except: A) Diabetes mellitus B) Diaphragms and spermicide use C) Pregnancy D) Intrauterine device

D) Intrauterine device Risk factors for UTIs include diabetes mellitus, pregnancy, and use of diaphragms and spermicide.

What type of testing is recommended before starting a patient on a prescription of hydroxychloroquine (Plaquenil)? A) CBC B) Serum creatinine and urine for microalbumin C) Liver function tests D) Comprehensive eye exam

D) Comprehensive eye exam A comprehensive eye exam by an ophthalmologist is recommended because hydroxychloroquine can adversely affect the retina (scotomas or visual field defects, loss of central vision, loss of color vision). Higher doses and long-term use increase the risk of retinal toxicity.

A 4-year-old boy is brought in by his mother for a wellness visit. His mother tells the family nurse practitioner that he had a bad case of chickenpox 1 year ago. Which of the following immunizations are indicated? A) IPV, Hib (hemophilus influenzae type B), hepatitis B B) IPV, Hib, MMR C) DTaP, IPV, MMR, hepatitis B D) DTaP, IPV

D) DTaP, IPV DTap and IPV would be due for this 4-year-old child.

A 15-month-old infant is in your office for a well baby check. The mother denies history of chickenpox infection. Which of the following immunizations is indicated at this visit? A) DTaP, Hib, IPV, hepatitis B B) DTaP, Hib, PCV, IPV, MMR C) MMR, hepatitis B, varicella D) DTaP, IPV, MMR, varicella

D) DTap, IPV, MMR, varicella Immunizations for a 15-month-old child include DTap, IPV, MMR, and varicella vaccine.

All of the following are true statements regarding elder abuse except: A) Those aged 80 years or older are at the highest risk for abuse B) A delay in medical care is a common finding C) A new onset of an STD in an elderly patient may signal sexual abuse D) Decreased anxiety and depression are common symptoms of abuse in the elderly

D) Decreased anxiety and depression are common symptoms of abuse in the elderly Elder abuse is commonly seen in elderly patients over the age of 80 years. Common signs/symptoms include anxiety and depression. A new onset of an STD may indicate signs of sexual abuse. These patients will commonly delay treatment for acute/chronic conditions.

According to DSM-5, all of the following are some of the criteria that must be present to diagnose a child with autistic disorder. Which of the following criterion is incorrect? A) Onset of symptoms before age 3 years B) Lack of social interaction or social reciprocity C) Stereotyped and repetitive movements such as hand flapping D) Depressed affect

D) Depressed affect Depression is not included in the criteria for diagnosing autistic spectrum disorders. Signs and symptoms include avoidance of eye contact and social interaction, marked delay or absence of verbal communication, repetitive movements, fixed rituals, and so on. Autism can range from mild to severe. Early diagnosis is important. For evaluation, refer the patient for psychological testing by a psychologist who specializes in autistic spectrum disorders.

Which of the following is considered a relative contraindication for combined oral contraceptive pills? A) Undiagnosed vaginal bleeding B) A hepatoma of the liver C) Suspected history of TIAs D) Depression

D) Depression Depression is a relative contraindication for combined OCPs due to the hormonal effects that can affect mood. Absolute contraindications include hepatoma of the liver, history of embolic episode, history of TIAs, and undiag- nosed vaginal bleeding. OCP should not be considered in these instances due to the high risk factors and health risks.

A 21-year-old woman complains of left-sided pelvic pain accompanied by dyspareunia. During the gynecological exam, the nurse practitioner notices green cervical discharge. The patient mentions a new onset of a painful and swollen left knee and denies a history of trauma. This best describes: A) Septic arthritis B) Reiter's syndrome C) Chondromalacia of the patella D) Disseminated gonorrheal infection

D) Disseminated gonorrheal infection Symptoms of PID with painful, swollen joints of extremities indicate disseminated gonorrheal infection. Untreated disseminated gonorrhea can lead to septic arthritis. Symptoms may be mild from slight joint pain and no fever to severe joint pain with high fever. PID symptoms do not occur with septic arthritis, Reiter's syndrome, or chondromalacia of the patella.

A postmenopausal female complains of random episodes of vaginal bleeding for the past 6 months. Which of the following is recommended management for this condition? A) Cervical biopsy B) Pap smear C) Colposcopy D) Endometrial biopsy

D) Endometrial biopsy Random episodes of vaginal bleeding in a postmenopausal woman are not normal. Endometrial biopsy is needed to biopsy the endometrial lining for abnormal cells, which may indicate cancer.

Medicare Part B will pay for all of the following services except: A) Outpatient physician visits that are medically necessary B) Durable medical equipment C) Outpatient laboratory and radiology tests D) Eyeglasses and routine dental care

D) Eyeglasses and routine dental care Medicare Part B covers: 1) outpatient physician visits, labs, x-rays; 2) durable medical equipment; 3) mammograms/ colonoscopy after age 50 years annually; and 4) rehabilitation.

What structure of the eye is responsible for 20/20 vision (sharpest vision)? A) Rods B) Cones C) Optic disc D) Fovea of the macula

D) Fovea of the macula The fovea is located on the center of the macula and is responsible for the sharpest vision ("20/20 vision") in the eyes. On the fovea, the only receptors are the cones, which allow us to see things in color and in detail. The macula is responsible for central vision.

Erythromycin inhibits the cytochrome P-450 system. The following drugs should be avoided because of a potential for a drug interaction except: A) Theophylline (Theo-Dur) B) Warfarin (Coumadin) C) Diazepam (Valium) D) Furosemide (Lasix)

D) Furosemide (Lasix) There are many medications that are contraindicated with the cytochrome P-450 system. Lasix is one medication that can be used.

You would recommend the pneumococcal vaccine (Pneumovax) to patients with all of the following conditions except: A) Sickle cell anemia B) Splenectomy C) Patients infected with HIV D) G6PD deficiency anemia

D) G6PD deficiency anemia Pneumococcal vaccine should not be given to patients with G6PD deficiency anemia.

A common side effect of metformin (Glucophage) therapy is: A) Weight gain B) Lactic acidosis C) Hypoglycemic episodes D) Gastrointestinal problems

D) Gastrointestinal problems Common side effects of metformin include diarrhea/gastrointestinal problems.

All of the following factors increase the risk of mortality for patients diagnosed with bacterial pneumonia except: A) Alcoholism B) Very young age or the elderly C) Multiple lobar involvement D) Hypertension

D) Hypertension Factors that increase the risk of mortality for patients diagnosed with bacterial pneumonia include alcoholism, very young or elderly patients, and multiple lobar involvement.

A lipid profile done on a newly diagnosed hypertensive patient shows a triglyceride level of 650 mg/dL, total cholesterol 240 mg/dL, LDL 145 mg/dL, and an HDL of 35 mg/dL. What is the best intervention for this patient? A) Educate the patient about lifestyle changes that will help lower cholesterol levels B) Initiate a prescription of pravastatin (Pravachol) C) Recommend that the patient exercise at least every other day and avoid eating fatty or fried foods D) Initiate a prescription of nicotinic acid (Niacin, Niaspan)

D) Initiate a prescription of nicotinic acid (Niacin, Niaspan) Niacin is recommended for treatment of high triglyceride and cholesterol levels. Exercise and healthy eating lifestyle is also recommended, but with the high level of triglycerides at 650 mg/dL and total cholesterol 240 mg/dL, niacin is recommended.

What is the best description of a variable? A) It is an important part of every research study B) It is the probability that a factor is important for the research data C) It is the value or number that occurs the most frequently D) It is a condition, characteristic, or factor that is being measured

D) It is a condition, characteristic, or factor that is being measured A variable is a condition, characteristic, or factor that is being measured. An independent variable is the one being manipulated that is not affected by the others. A dependent variable changes depending on the manipulation of the independent variable.

An 18-year-old male is found to have a 47, XXY karyotype and is diagnosed with Klinefelter's syndrome. The patient is most likely to have all of the following physical characteristics except: A) Gynecomastia B) Long limbs C) Lack of secondary sexual characteristics D) Large testes

D) Large testes Signs/symptoms of Klinefelter's syndrome include gynecomastia, long limbs, and lack of secondary sexual characteristics. Testes are usually small. Infertility is a major concern for these boys. If treated early, they may have a normal sexual/reproductive system in the future.

Which of the following findings is associated with the chronic use of chewing tobacco? A) Cheilosis and xerostomia B) Glossitis C) A geographic tongue D) Leukoplakia and oral cancer

D) Leukoplakia and oral cancer The chronic use of tobacco increases the risk of oral cancer and leukoplakia. Cheilosis is skin fissures/maceration in the corner of the mouth, most commonly caused by anemia, bacterial infection, vitamin deficiencies, or oversalivation.

The first nurse practitioner program was started by: A) Alfred Bandura B) President John F. Kennedy C) Federal government D) Loretta Ford, PhD

D) Loretta Ford, PhD The first NP program was started by Loretta Ford, PhD, at the University of Colorado; it was a certificate program for NPs.

The following are acceptable methods of birth control for breastfeeding mothers except: A) Diaphragm with spermicidal gel B) Progesterone-only pills (Micronor) C) Condoms D) Low-dose oral contraceptives with at least 20 mcg of estradiol (Alesse, Lo-estrin)

D) Low-dose oral contraceptives with at least 20 mcg of estradiol (Alesse, Lo-estrin) Low-dose oral contraceptives that contain estradiol are contraindi- cated for breastfeeding mothers.

Which of the following findings are seen in a patient with folate-deficiency anemia? A) Microcytic and hypochromic RBCs B) Microcytic and normochromic RBCs C) Normal size and color of the RBCs D) Macrocytic and normocytic RBCs

D) Macrocytic and normocytic RBCs Folate-deficiency anemia is diagnosed by macrocytic, normocytic red blood cell.

Carpal tunnel syndrome is due to inflammation of the: A) Ulnar nerve B) Radial nerve C) Brachial nerve D) Median nerve

D) Median nerve Carpal tunnel syndrome is due to inflammation of the median nerve.

Patients with Down syndrome are at higher risk for all of the following except: A) Atlantoaxial instability B) Congenital heart disease C) Early onset of Alzheimer's disease D) Melanoma

D) Melanoma Patients with the diagnosis of Down syndrome are at higher risk for atlantoaxial instability, congenital heart defects, and early onset of Alzheimer 's disease. Children with Down syndrome who participate in sport activities must be carefully examined for these conditions prior to participation to prevent injury.

All of the following are considered Category X drugs except: A) Misoprostol (Cytotec) B) Isotretinoin (Accutane) C) Finasteride (Proscar) D) Meperidine (Demerol)

D) Meperidine (Demerol) Demerol is in drug Category C and and treated by the FDA as a controlled drug Schedule II.

All of the following are clinical eye findings found in some patients with chronic uncontrolled hypertension. Which of the following findings is not associated with this disorder? A) AV nicking B) Copper wire arterioles C) Flame-shaped hemorrhages D) Microaneurysms

D) Microaneurysms "Keith Wagener Barker (KWB) Grades" for uncontrolled hypertension: Grade 1: generalized arteriolar constriction seen as "silver wiring" and vascular tortuosities; Grade 2: grade 1 plus irregularly located, tight constric- tions known as "AV nicking" or "AV nipping"; Grade 3: grade 2 plus cotton wool spots and flame-shaped hemorrhages; Grade 4, grade 3 but with swelling of the optic disk (papilledema). Microaneurysms occur with diabetic retinopathy.

Three of the following are eye findings associated with chronic uncontrolled hypertension. Which one of the following is associated with diabetic retinopathy? A) AV nicking B) Copper wire arterioles C) Flame hemorrhages D) Microaneurysms

D) Microaneurysms Microaneurysms are seen with diabetic retinopathy. AV nicking, copper wire arterioles, and flame hemorrhages are seen with uncontrolled hypertension.

A 70-year-old male with open-angle glaucoma is prescribed Betimol (timolol) ophthalmic drops. All of the following are contraindications to Betimol ophthalmic drops except: A) Overt heart failure or sinus bradycardia B) Asthmatic patients C) Second- or third-degree AV block D) Migraine headaches

D) Migraine headaches Contraindications with Betimol (timolol) include heart failure or sinus bradycardia, asthmatic patients, and second- or third-degree AV block

Potential complications of mitral valve prolapse (MVP) include all of the following except: A) Severe mitral regurgitation B) Endocarditis C) Increased risk of stroke and TIA D) Mitral stenosis

D) Mitral stenosis Complications of MVP include mitral regurgitation, endocarditis, and increased risk of stroke and TIAs.

A 40-year-old woman is in the office complaining of palpitations and some light headedness for the past 6 months. These are random episodes. The nurse practitioner notices a mid-systolic click with a late systolic murmur that is best heard in the apical area during auscultation of the chest. The NP would suspect: A) Atrial fibrillation B) Sinus arrhythmia C) Mitral stenosis D) Mitral valve prolapse

D) Mitral valve prolapse Mitral valve prolapse occurs when the mitral valve does not close all the way, causing a late systolic murmur heard best in the apical area during auscultation of the chest. Symptoms patients may experience at times include palpitations and dizziness.

The following are treatment plans related to migraine headaches. Which one of the following would not be considered effective therapy? A) Propranolol (Inderal) B) Cold packs to the forehead C) Trimethobenzamide (Tigan) D) Moderate sodium restriction

D) Moderate sodium restriction Effective treatments for migraine headaches include propranolol, trimethobenzamide, cold packs to the forehead/neck, and hydration.

The differential diagnosis for genital ulceration includes all of the following except: A) Syphilis B) Genital herpes C) Chancroid D) Molluscum contagiosum

D) Molluscum contagiosum Genital ulcers may occur with syphilis, genital herpes, and chancroid. Molluscum contagiosum is a viral infection that causes smooth, round tiny papules, approximately 5 mm or less, that have a central umbilication with a white plug present.

Which of the following findings is associated with thyroid hypofunction? A) Graves disease B) Eye disorder C) Thyroid storm D) Myxedema

D) Myxedema Myxedema is a rare, and sometimes fatal, disease in which the thyroid is severely underactive and causes life-threatening symptoms. These symptoms include low blood pressure, decreased breathing, decreased body temperature, unresponsiveness, and even coma. Graves' disease, thyroid storm, and eye disorder are seen with an overactive thyroid disease known as hyperthyroidism.

An adult patient is being evaluated for tuberculosis infection with a Mantoux test. The PPD result is 10.5 mm. The patient denies weight loss, cough, and night sweats and has a negative chest x-ray. The patient reports that he is in the United States illegally and is fearful about discovery. What is the most appropriate action for the nurse practitioner? A) The patient is an illegal alien/migrant and the nurse practitioner has the legal duty to report the patient to the local State Department responsible for illegal migrants B) Health care workers are legally mandated to report illegal migrants to state authorities. C) The nurse practitioner should call the state health department and report that the patient has a TB infection D) Nurse practitioners have the ethical duty to provide quality health care to patients

D) Nurse practitioners have the ethical duty to provide quality health care to patients Currently, health caregivers are not legally required to report illegal aliens to the state or local authorities. This patient does not have the signs and symptoms of active TB disease (cough, weight loss, night sweats) and has a negative chest x-ray. Therefore, he has latent TB infection and is not contagious. Only patients with active TB disease (has signs/symptoms) must be reported to the state public health department.

A college student is seen as a walk-in appointment in a college health clinic. She complains of the abrupt onset of sore throat, nasal congestion, runny nose, and malaise. Vital signs are a temperature of 99.8 degrees Fahrenheit, pulse 84, and respiratory rate 14 breaths/minute. The physical exam reveals an erythematous throat, swollen nasal turbinates, and rhinitis. The NP suspects viral URI. All of the following treatments are appropriate except: A) Saline nasal spray (Ocean nasal spray) B) Pseudoephedrine (Sudafed) C) Ibuprofen (Advil) D) Oral prednisone (Medrol Dose Pack)

D) Oral prednisone (Medrol Dose Pack) Symptomatic treatment for viral URI are saline nasal sprays (Ocean spray), decongestants (pseudoephedrine), NSAIDs (Advil), increased fluid intake, and alternative herbal remedies (echinachea, astralagus, elderberry syrup, high doses of Vitamin C).

All of the following clinical findings are classified as major criteria that are necessary to diagnose pelvic inflammatory disease (PID). Which of the following is classified as a minor criterion? A) Cervical motion tenderness B) Adnexal tenderness C) Uterine tenderness D) Oral temperature of more than 101°F (more than 38°C)

D) Oral temperature of more than 101°F (more than 38°C) PID is a clinical diagnosis. The presence of at least 1 of the major criteria (cervical motion tenderness, adnexal tenderness, uterine tenderness) when combined with the history is highly suggestive of PID. Minor criteria are not necessary, but they help to support the diagnosis of PID (oral temperature of more than 101°F or more than 38°C), mucopurulent cervical or vaginal discharge, elevated sedimentation rate, elevated C-reactive protein, large amount of WBCs on saline microscopy of the vaginal fluid, or laboratory documentation of cervical infection with N. gonorrhoeae or C. trachomatis).

Women with a history of pelvic inflammatory disease (PID) have an increased risk for all of the following complications except: A) Ectopic pregnancy B) Scarring of the fallopian tube(s) C) Infertility D) Ovarian cysts

D) Ovarian cysts Women with a history of pelvic inflammatory disease (PID) have a higher risk of ectopic pregnancy, scarring of the fallopian tubes, and infertility due to the scarring and trauma caused by the pelvic inflammation.

A woman at 32 weeks gestation has a positive throat culture for strep pyogenes. She denies allergies but gets very nauseated with erythromycin. Which of the fol- lowing is the best choice for this pregnant patient? A) Clarithromycin (Biaxin) B) Trimethoprim/sulfamethoxazole (Bactrim DS) C) Ofloxacin (Floxin) D) Penicillin (Pen VK)

D) Penicillin (Pen VK) Pen VK is safe to use for strep throat during pregnancy. Pen VK is a category B medication for pregnancy and lactation.

A nurse practitioner is teaching a 54-year-old woman with stress urinary incontinence about Kegel exercises. The patient is instructed to tighten her pelvic floor muscles for a count of 10 and then to relax them for a count of 10. The nurse practitioner instructs the patient that Kegel exercises should be done consistently every day at what frequency? A) Perform 30 exercises each time in the morning and the evening B) Perform 20 exercises each time 2 times a day C) Perform 15 exercises each time 3 to 4 times a day D) Perform 10 exercises each time 3 times a day

D) Perform 10 exercises each time 3 times a day Weak pelvic floor muscles increase the risk of urinary and fecal incontinence. Educate a female patient that the pelvic floor muscles are the ones that she uses when she consciously holds/ stops the flow of urine when she urinates. Warn her that the anal sphincter will also tighten with the vaginal muscles. Advise the patient to relax the abdomen and the thighs when doing the exercises. Kegel exercises are also recommended as conservative treatment for patients (male and female) with fecal incontinence.

All of the following conditions are associated with an increased risk for normocytic anemia except: A) Rheumatoid arthritis B) Lupus C) Chronic autoimmune disorders D) Pregnancy

D) Pregnancy During pregnancy, women may experience microcytic, hypochromic anemia due to the dilutional effect of the increased blood volume during the pregnancy.

A patient with a history of mitral valve prolapse (MVP) is requesting prophylaxis before her dental surgery. Which of the following would you prescribe this patient? A) Amoxicillin a half hour before and 2 hours after the procedure B) Amoxicillin 1 hour before the procedure C) Amoxicillin 1 hour before and 3 hours after the procedure D) Prophylaxis is not recommended for this patient

D) Prophylaxis is not recommended for this patient The American Heart Association does not recommend routine antibiotic prophylaxis prior to dental procedures except for patients who are at high risk for bacterial endocarditis (a prosthetic cardiac valve, previous history of bacterial endocarditis, congenital heart defects, etc.).

A skilled nursing facility (SNF) can provide all of the following except: A) Physical therapy and other types of rehabilitation B) Skilled nursing care and medical care C) Reimbursement by Medicare D) Provision of custodial care

D) Provision of custodial care Custodial care is done by nursing homes. SNFs are reimbursed by Medicare and can provide skilled nursing and medical care. If a patient is discharged from a hospital, but needs therapy and skilled care, he/she is usually transferred to an SNF. SNFs have transfer agreements with local hospitals. Nursing home patients are usually medically stable and need skilled or medical care. These patients are unable to perform from 2 to 6 ADLs. Patients with Alzheimer's and other types of dementia are usually cared for in nursing homes.

Which of the following conditions is associated with a positive Auspitz sign? A) Contact dermatitis B) Seborrheic dermatitis C) Systemic lupus erythematosus D) Psoriasis

D) Psoriasis Auspitz sign is the presence of pinpoint bleeding spots from psoriasis where the skin is scraped off.

Mary, who has recently been diagnosed with lupus, complains that her hands and feet always feel cold even in the summertime. Sometimes her fingertips become numb and turn a blue color. The fingertips eventually turn to a dark red color. Which of the following is most likely? A) Chronic arterial insufficiency B) This is a normal reaction when one feels very cold C) Peripheral vascular disease D) Raynaud's syndrome

D) Raynaud's syndrome Raynaud's syndrome occurs from vasospasms of the blood vessels, leading to decreased blood supply to the hands/feet, which causes bluish discoloration, with fingertips turning a dark red color if severe. Stress and cold weather are classic triggers for this.

The mother of an 11-year-old boy with sickle cell anemia calls on the phone complaining that her son woke up because of a painful penile erection that will not go away. The nurse practitioner's most appropriate intervention is: A) Insert a Foley catheter and measure the child's intake and output for the next 24 hours B) Insert a Foley catheter to obtain a specimen for a urinalysis and urine for C&S (culture and sensitivity) C) Recommend an increase in the child's fluid intake D) Recommend immediate referral to the ER

D) Recommend immediate referral to the ED Priapism (painful penile erection not related to sexual activity) is a true urologic emergency that may lead to permanent erectile dysfunction and penile necrosis if not treated appropriately. It can be associated with a number of medical conditions (sickle cell anemia, leukemia, or spinal cord injury) and/or some pharmacologic agents

The mother of an 11-year-old boy with sickle cell anemia calls on the phone because her son woke up with a painful penile erection that will not go away. The nurse practitioner's most appropriate intervention is: A) Insert a Foley catheter and measure the child's intake and output for the next 24 hours B) Insert a Foley catheter to obtain a specimen for a urinalysis and urine for C&S (culture and sensitivity) C) Recommend an increase in the child's fluid intake D) Recommend immediate referral to the emergency department

D) Recommend immediate referral to the emergency department Priapism (painful penile erection not related to sexual activity) is a true urological emergency that may lead to permanent erectile dysfunction and penile necrosis if not treated appropriately. It can be associated with a number of medical conditions (sickle cell anemia, leukemia, or spinal cord injury) and/or some pharmacological agents.

A patient with chronic obstructive pulmonary disease (COPD) is referred for pulmonary function testing. Which of the following pulmonary function tests are abnormal in patients with COPD? A) Reduction of the TLC (total lung capacity) and the RV (residual volume) B) Complaints of mild to severe dyspnea with hypoxemia C) Normal forced vital capacity (FVC) with no changes in the FEV1 (forced expiratory volume in 1 second) D) Reduction of the FEV1 (forced expiratory volume in 1 second) with increase in the TLC (total lung capacity) and RV (residual volume)

D) Reduction of the FEV1 (forced expiratory volume in 1 second) with increase in the TLC (total lung capacity) and RV (residual volume) COPD findings during pulmonary function testing are the reduction of the FEV1 (forced expiratory volume in 1 second) and reduction of the FVC (forced vital capacity). There is an increase in the TLC (total lung capacity) and RV (residual volume). The lungs of patients with emphysema have lost their recoil (decreases FEV1). The lungs are always full of air that is hard to "squeeze out" of the lungs (increases residual volume and total lung capacity). To summarize, COPD=reduction in FEV1 and FVC with increased RV and TLC.

Your 35-year-old patient is being worked up for microscopic hematuria. All of the following are differential diagnoses of microscopic hematuria except: A) Kidney stones B) Bladder cancer C) Acute pyelonephritis D) Renal stenosis

D) Renal stenosis Renal stenosis is a narrowing of the renal artery. No blood would be noted on exam. Evidence of blood in the urine can be seen with kidney stones, bladder cancer, and acute pyelonephritis.

An elderly woman with a history of rheumatoid arthritis reports to the nurse practitioner that she had been taking ibuprofen BID for many years. Which of the following organ systems has the highest risk of damage from chronic non- steroidal anti-inflammatory drug (NSAID) use? A) Cardiovascular system B) Neurological system C) Gastrointestinal system D) Renal system

D) Renal system The 2 main adverse drug reactions associated with NSAIDs relate to gastrointestinal (GI) effects and renal effects of the agents. The main adverse drug reactions associated with use of NSAIDs relate to direct and indirect irritation of the GI tract. Ulceration risk increases with therapy duration and with higher doses. NSAIDs can induce 2 different forms of acute kidney injury: hemodynamically mediated, and acute interstitial nephritis, which is often accompanied by nephrotic syndrome.

In addition to surgical repair of a compound fracture that has broken through the skin, which of the following treatment plans is important to consider in this patient? A) Application of a topical antibiotic BID until the wound is healed B) Wound irrigation C) Tdap vaccine D) Tetanus vaccine and systemic antibiotics

D) Tetanus vaccine and systemic antibiotics A break in the skin with a compound fracture is an indication for use of a tetanus vaccine (if last dose is more than 5 years ago) and systemic antibiotics.

An 18-year-old waitress is diagnosed with pelvic inflammatory disease (PID). The cervical Gen-Probe result is positive for Neisseria gonorrhoeae and negative for Chlamydia trachomatis. All of the following statements are true regarding the man- agement of this patient except: A) This patient should be treated for chlamydia even though the Gen-Probe for chlamydia is negative B) Ceftriaxone 250 mg IM and doxycycline 100 mg PO BID x 14 days are appropriate treatment for this patient C) Advise the patient to return to the clinic for a repeat pelvic exam in 48 hours D) Repeat the Gen-Probe test for Chlamydia trachomatis to ensure that the previous test was not a false-negative result

D) Repeat the Gen-Probe test for Chlamydia trachomatis to ensure that the previous test was not a false-negative result Treatment for both gonorrhea and chlamydia are recommended for the diagnosis of PID, regardless if the chlamydia test was negative.

A new patient is complaining of severe pruritus that is worse at night. Several family members also have the same symptoms. Upon examination, areas of excoriated papules are noted on some of the interdigital webs of both hands and on the axillae. This finding is most consistent with: A) Contact dermatitis B) Impetigo C) Larva migrans D) Scabies

D) Scabies The classic symptom of scabies is severe pruritis that worsens at night. The pruritic rash is commonly seen on the trunk and between the fingers, and appears as excoriated, erythemic papules. Scabies is contagious and can affect other family members.

A 22-year old is going on a 5-day cruise for her honeymoon. She reports a history of severe motion sickness. Which of the following medicines can be prescribed for motion sickness? A) Dimenhydrinate (Dramamine) B) Metochlopramide (Reglan) C) Ondansetron (Zofran) _ D) Scopolamine patch (Transderm Scop)

D) Scopolamine patch (Transderm Scop) Scopolamine patch (Transderm Scop) is a prescription medicine that is used for motion/sea sickness. It is a small, circular patch that is placed behind the ear and is effective for 3 days. Advise the patient to apply it 4 hours before the trip to be effective. Because the question is asking about a "prescribed" med, answers with OTCs such as Dramamine are incorrect. Zofran is indicated for cancer-related nausea and vomiting (chemo, radiation, surgery).

At level of prevention would you classify screening for lung cancer? A) Primary prevention B) Secondary prevention C) Tertiary prevention D) Screening for lung cancer is not currently recommended

D) Screening for lung cancer is not currently recommended

The nurse practitioner would refer all of the following to a physician except: A) Severe facial burns B) Electrical burns C) Burns that involve the cartilage of the ear D) Second-degree burns on the lower arm

D) Second-degree burns on the lower arm First- and second-degree burns are appropriately treated by a nurse practitioner. Third-degree burns should be referred to a physician. Examples of third-degree burns include electrical burns, severe burns on the face, and burns involving cartilage, such as the ear and nose.

Acute prostatitis can present with all of the following signs and symptoms except: A) Fever and chills B) Tenderness of the scrotum on the affected side C) Perineal pain D) Slow onset of symptoms

D) Slow onset of symptoms Acute prostatitis symptoms include fever, chills, tenderness of the scrotum on the affected side, and abrupt onset of symptoms.

A faun tail nevus is a sign of which of the following? A) Down syndrome B) Infantile scoliosis C) Congenital heart disease D) Spina bifida

D) Spina bifida A faun tail nevus is an abnormal tuft of hair in the lumbosacral area, which can be a sign of spina bifida.

A nurse practitioner is giving dietary counseling to a male alcoholic who has recently been diagnosed with folic acid deficiency anemia. Which of the following foods should the nurse practitioner recommend to this patient? A) Tomatoes, oranges, and bananas B) Cheese, yogurt, and milk C) Lettuce, beef, and dairy products D) Spinach, liver, and whole wheat bread

D) Spinach, liver, and whole wheat bread Foods that are higher in folic acid include spinach, liver, and whole wheat bread.

What would you advise him regarding his fluvastatin (Lescol) prescription? A) Continue taking the medicine until the lab results are available B) Take half the usual daily dose until the lab results are available C) Take the medicine every other day instead of daily until the lab results are available D) Stop taking the medicine until the lab results are available

D) Stop taking the medicine until the lab results are available When liver enzymes are affected by medications, such as statin medications, the patient is advised to stop taking the medicine until the lab results are available.

A 68-year-old woman complains of leaking a small amount of urine whenever she sneezes, laughs, and/or strains. The problem has been present for many months. The patient denies dysuria, frequency, and nocturia. The urine dipstick test is neg- ative for white blood cells, red blood cells, ketones, and urobilinogen. What is the name of this condition? A) Urge incontinence B) Overflow incontinence C) Urinary incontinence D) Stress incontinence

D) Stress incontinence The signs and symptoms of stress incontinence occur when the stress caused by sneezing, laughing, and/or straining results in the leaking of a small amount of urine through a weakened sphincter.

While performing a routine physical exam on a 60-year-old male, the nurse practitioner notices a soft bruit over the carotid area on the left side of the neck. The patient has a history of hypertension. The patient is at higher risk for: A) Temporal arteritis and brain aneurysms B) Dizziness and headaches C) Abdominal aneurysm and congestive heart failure D) Stroke and coronary heart disease

D) Stroke and coronary heart disease Carotid stenosis puts the patient at risk for stroke and coronary heart disease

The cremasteric reflex is elicited by: A) Asking the patient to open his or her mouth and touching the back of the pharynx with a tongue blade B) Hitting the biceps tendon briskly with a reflex hammer and watching the lower arm for movement C) Hitting the patellar tendon briskly with a reflex hammer and watching the lower leg for movement D) Stroking the inner thigh of a male client and watching the testicle on the ipsilateral side rise up toward the body

D) Stroking the inner thigh of a male client and watching the testicle on the ipsilateral side rise up toward the body The cremasteric reflex test is done by stroking the inner thigh of a male client and watching for the testicle on the ipsi- lateral side to rise up toward the body.

A 70-year-old male patient complains of a bright red spot in his left eye for 2 days. He denies eye pain, visual changes, or headaches. He has a new onset of cough from a recent viral upper respiratory infection. The only medicine he is on is Bayer aspirin 1 tablet a day. Which of the following is most likely? A) Corneal abrasion B) Acute bacterial conjunctivitis C) Acute uveitis D) Subconjunctival hemorrhage

D) Subconjunctival hemorrhage Subconjunctival hemorrhage is a benign disorder that occurs from an increase in intraocular pressure that may be caused by coughing, vomiting, forceful exerrtion in labor during childbirth, straining while having a bowel movement, weight lifting, or lifting a heavy object.

All of the following statements about common health beliefs of many traditional Asian cultures are true except: A) An imbalance of the hot and cold (yin/yang) vital forces can cause illness, and treating a hot disease with a "cold" treatment (i.e., certain foods/herbs) can help to restore balance and cure the illness B) If the patient is very ill or dying, immediate family and extended family members will visit the patient daily in "shifts" to provide emotional support C) Babies and small children may wear an amulet such as a red string on the wrist or a piece of cloth on the neck or the wrist D) Surgical procedures are regarded as important treatment for many illnesses

D) Surgical procedures are regarded as important treatment for many illnesses The Vietnamese regard surgery as a last resort and consider loss of blood as depleting the vital forces of the body and causing illness. Western medicine is considered "hot," and patients may discontinue or reduce the doses of their medicine without asking. An imbalance of the hot and cold (yin/yang) is believed to cause illness. Treating a "yin" disease (common cold) means avoiding eating yin foods (melons, cucumbers) because they will worsen it. Instead, yin diseases are treated with yang foods (meat, spicy foods) so that the body becomes more balanced. If the patient is very ill or dying, immediate family and extended family members will visit the patient daily in "shifts" to provide emotional support. Babies and small children may wear an amulet such as a red string on the wrist or a piece of cloth that ties on the neck or the wrist.

When an adolescent female's areola and nipples elevate above the level of the breasts to form a secondary mound, which of the following is the correct Tanner stage for this phase of breast development? A) Tanner Stage I B) Tanner Stage II C) Tanner Stage III D) Tanner Stage IV

D) Tanner Stage IV At Tanner Stage IV, the breast tissue is still growing. This is the stage in which the areola and the nipple separate to form a distinct mound. The most important clue is "secondary mound." During Tanner Stage III, the breast and areola and nipples grow together in one mound. There is no separation yet.

A 45-year-old gardener is seen as a walk-in patient in a private clinic. He reports stepping on a nail that morning. His last tetanus vaccine was 7 years ago. Which of the following vaccines is recommended? A) DTaP B) DT C) Td D) Tdap

D) Tdap The CDC recommends that 1 of the tetanus boosters be replaced with the Tdap (once in a lifetime). Thereafter, the Td form of the vaccine is indicated every 10 years. The DTaP (diphtheria-tetanus-acellullar pertussis) and DT (diphtheria-tetanus) forms of the tetanus vaccine are not given after the age of 7 years. Puncture wounds are at higher risk for tetanus because Clostridium tetani bacteria are anaerobes (deep puncture wounds are not exposed to air compared with superficial wounds).

19-13. Which Medicare type is not correctly matched with its description? A. Medicare Part A: hospital insurance for the elderly B. Medicare Part B: supplementary medical insurance C. Medicare Part C: also known as Medicare Advantage; involves privately run medical groups providing Medicare services D. Medicare Part D: provision of dental services

D. Medicare Part D: provision of dental services

A nurse practitioner is evaluating an 80-year-old woman from a nursing home. She is instructing the patient to remember the words "orange," "house," and "world." A few minutes later, the patient is told to recall these 3 words. Which of the following is being described? A) The Lachman test B) A neurologic exam C) The Romberg test D) The Mini Mental Status Exam

D) The Mini Mental Status Exam The Lachman test is performed on the knee to assess stability; the Romberg test is performed to assess for balance; a neurologic exam is performed by assessing the 12 cranial nerves; the MMSE is performed to assess for cognitive impairment.

The Romberg test is done to check for problems with balance. Which area of the brain is responsible for balance? A) Frontal lobe B) Temporal lobe C) The midbrain D) The cerebellum

D) The cerebellum The Romberg test evaluates the cerebellum, which is responsible for balance. This test is performed by having the client stand up straight with feet together and then having him/her close the eyes and stand still for at least 20 seconds. A positive Romberg exam would demonstrate loss of balance.

Which of the following is a true statement regarding the effect of aspirin on platelet function? A) The effect on platelets is reversible B) The effect on platelets is reversible and lasts only 1 week C) It has a minimal effect on platelet function D) The effect on platelet function is irreversible and lasts 15 to 20 days

D) The effect on platelet function is irreversible and lasts 15 to 20 days The use of aspirin affects the platelet function, is irreversible, and can last up to 15-20 days

The apex of the heart is located at: A) Second ICS (intercostal space) to the right of the sternal border B) Second ICS to the left of the sternal border C) The left lower sternal border D) The left side of the sternum at the fifth ICS by the midclavicular line

D) The left side of the sternum at the fifth ICS by the midclavicular line The apex of the heart is located at the left side of the sternum at the fifth ICS by the midclavicular line.

An adult female presents with complaints of "bad burns" that are very painful. A large pot of boiling water tipped over and spilled on her arms and her anterior chest and abdomen. During the physical exam, the nurse practitioner notices bright red skin with numerous bullae on the left arm and hand and large patches of bright red skin on the anterior chest and abdominal area. On a pain scale of 1-10, she reports the pain as 9. Her vital signs are stable with tachycardia (pulse is 100/minute). She does not appear to be in shock. Using the Rule of Nines, what is the total body surface area (TBSA) and the depth of the burns in this patient? A) The patient has a TBSA of 15% with full-thickness burns of the left arm and left hand, and partial-thickness burns of the anterior chest and abdominal area B) The patient has a TBSA of 20% with partial-thickness burns on the left arm, left hand, and mild burns on the anterior chest and abdominal area C) The patient has a TBSA of 27% with partial-thickness burns on the left arm and left hand with superficial burns on the anterior chest and abdominal area D) The patient has a TBSA of 18% with full-thickness burns of the left arm and left hand, and superficial burns on the anterior chest

D) The patient has a TBSA of 18% with full-thickness burns of the left arm and left hand, and superficial burns on the anterior chest

All of the following are correct statements regarding the role of the person named in a durable power of attorney except: A) The agent's decisions are legally binding B) The agent can make decisions in other areas of the patient's life such as financial issues C) The agent can decide for the patient who is on life support when that life support can be terminated D) The patient's spouse has a right to override the agent's decisions

D) The patient's spouse has a right to override the agent's decisions The person named in a durable power of attorney (the agent) is designated by the patient to make all medical decisions, as well as any decisions regarding the patient's private affairs in the event that the patient becomes incompetent and unable to make his/her own decisions. No one has the ability to override the agent's decision.

The following statements are true about Wilms tumor except: A) The most frequent clinical sign is a palpable abdominal mass B) It is a congenital tumor of the kidney C) Microscopic or gross hematuria is sometimes present D) The tumor commonly crosses the midline of the abdomen when it is discovered

D) The tumor commonly crosses the midline of the abdomen when it is discovered A Wilm's tumor is a congential tumor of the kidney that should never be palpated, once diagnosed, to avoid spread of the tumor cells. Microscopic or gross hematuria may be present.

An Rh-negative pregnant woman with negative rubella titers should be vaccinated at what time period in pregnancy? A) She can be vaccinated at any time in her pregnancy B) During the second trimester C) During the third trimester D) During the postpartum period

D)During the post partum period Rubella should be administered to the woman during the postpartum period. Rubella is contraindicated during pregnancy.

3-13. The ability of a test to correctly identify those with-out the disease (true negative rate) is known as: A. Mortality rate B. Prevalence rate C. Sensitivity D. Specificity

D. Specificity

A fracture on the navicular area of the wrist is usually caused by falling forward and landing on the hands. The affected wrist is hyperextended to break the fall. The nurse practitioner is aware that all of the following statements are true except: A) It has higher rate of nonunion compared with the other bones in the wrist when it is fractured B) The fracture frequently does not show up on an x-ray film when it is taken immediately after the injury C) The x-ray film will show the fracture if the film is repeated in 2 weeks D) These fractures always require surgical intervention to stabilize the joint

D) These fractures always require surgical intervention to stabilize the joint A fracture of the navicular area of the wrist occurs when one tries to break a fall with the hands, thus hyperextending the wrist. Immediately following the injury, the fracture may not show on x-ray; however, fracture is usually visible on x-ray if the x-ray is repeated in 2 weeks. The need for surgery is dependent on the extent of the fracture.

Acanthosis nigricans is associated with all of the following disorders except: A) Obesity B) Diabetes C) Colon cancer D) Tinea versicolor

D) Tinea versicolor Acanthosis nigricans is a benign skin condition. It appears as hyperpigmented velvety areas of skin that are usually located on the neck and the axillae. It is a sign of insulin resistance. It is rarely associated with some types of adenocarcinoma of the gastrointestinal tract. Tinea versicolor is a superficial infection of the skin (stratum corneum layer) that is caused by dermatophytes (fungi) of the tinea family. Another name for it is "sunspots."

Which of the following symptoms is associated with B12 deficiency anemia? A) Spoon-shaped nails and pica B) Abnormal neurological exam C) Vegan diet D) Tingling and numbness of both feet

D) Tingling and numbness of both feet Vitamin B12 deficiency anemia can cause nerve cell damage if not treated. Symptoms of B12 deficiency anemia may include tingling or numbness in fingers and toes, difficulty walking, mood changes or depression, memory loss, disorientation, and dementia.

Which of the following symptoms is associated with B12 deficiency anemia? A) Spoon-shaped nails and pica B) An abnormal neurological exam C) A vegan diet D) Tingling and numbness of both feet

D) Tingling and numbness of both feet Vitamin B12 deficiency anemia can cause nerve cell damage if not treated. Symptoms of B12 deficiency anemia may include tingling or numbness in fingers and toes, difficulty walking, mood changes or depression, memory loss, disorientation, and dementia.

A 74-year-old man presents with recurrent abdominal cramping associated with diarrhea that occurs from 4 to 5 times per day. He reports that currently he is having an exacerbation. The stools are bloody with mucus and pus. The patient reports that he has lost a little weight and is always fatigued. The patient denies recent travel or outdoor camping. Which of the following conditions is most likely? A) Giardiasis B) Irritable bowel syndrome (IBS) C) Diverticulitis D) Ulcerative colitis

D) Ulcerative colitis The most important clue for ulcerative colitis is bloody stools that are covered with mucus and pus along with the systemic symptoms (fatigue, low-grade fever).

A 17-year-old boy reports feeling something on his left scrotum. On palpation, soft and movable blood vessels that feel like a "bag of worms" are noted underneath the scrotal skin. It is not swollen or reddened. The most likely diagnosis is: A) Chronic orchitis B) Chronic epididymitis C) Testicular torsion D) Varicocele

D) Varicocele Palpation of varicose veins, known as "bag of worms" in the scrotum, is a classic symptom of a varicocele. Chronic epididymitis and chronic orchi- tis are caused by a bacterial infection and commonly have burning, frequency, and pain. Testicular torsion is an emergent condition in which the testicle becomes twisted, interrupting the blood supply to the testis; to avoid damage, the condition must be corrected within 6 hours.

Auscultation of normal breath sounds of the chest will reveal: A) Bronchial breath sounds heard at the lower bases B) High-pitched vesicular breath sounds heard over the upper lobes C) Vesicular breath sounds heard over the trachea D) Vesicular breath sounds in the lower lobe

D) Vesicular breath sounds in the lower lobe Normal sounds of the chest wall include vesicular breath sounds in the lower lobes. Bronchial breath sounds are heard best at the second and third intercostal spaces. Tracheal breath sounds are heard over the trachea.

Peak expiratory flow (PEF) meters are used to monitor asthma by using personal best measurements. All of the following factors are used to determine the PEF except: A) Age B) Gender C) Height D) Weight

D) Weight Peak expiratory flow volume is determined by using height, gender, and age. Weight is not used in the formula.

11-46. The nurse practitioner provides instructions for the family of a patient ("Joe") with a seizure disorder. Which of the following statements by a family member suggests further teaching is needed? A. "I will not restrain Joe when he is having a seizure." B. "I will assist Joe to the floor when he is having a seizure." C. "I will turn Joe's head to the side when he is hav- ing a seizure." D. "I will hold Joe's head to protect it when he is having a seizure."

D. "I will hold Joe's head to protect it when he is having a seizure."

A 55-year-old female with a history of mitral valve prolapse is being discharged from the hospital and sent home with antibiotic therapy for infective endocarditis (IE). Which statement indicates an under- standing of the patient's future prophylactic needs? A. "I will not require antibiotic prophylaxis after I have completed my current antibiotic therapy B. "I will require antibiotic prophylaxis but only for the next 6 months after discharge." C. "I will only require antibiotic prophylaxis for dental procedures that require general anesthesia." D. "I will require antibiotic prophylaxis before all dental procedures with the exception of x-rays and minor orthodontic adjustments."

D. "I will require antibiotic prophylaxis before all dental procedures with the exception of x-rays and minor orthodontic adjustments."

11-31. Which of the following statements by a patient with suspected multiple sclerosis indicates an understanding of blood work and the diagnosis of multiple sclerosis? A. "A CBC will show an underlying hematologic disorder." B. "A TSH will show evidence of thyroid issues." C. "A CMP will show a metabolic disturbance" D. "No blood test will confirm the diagnosis of multiple sclerosis."

D. "No blood test will confirm the diagnosis of multiple sclerosis."

Which patient would not warrant a workup for sec ondary hypertension? A. 5-year-old with blood pressure greater than the 99th percentile for height with a positive family history of HTN B. 35-year-old with a 2-month history of BP 150/90s on amlodipine 10 mg daily and bena epril 40 mg daily; BP on follow-up in the ofice s 165/100 with repeat measurements C. 65-year-old with blood pressure 172/98 with right upper quadrant abdominal bruit D. 25-year-old female with BMI of 37 and a blood pressure of 145/98 one month after starting phentermine

D. 25-year-old female with BMI of 37 and a blood pressure of 145/98 one month after starting phentermine

19-29. Before the Affordable Care Act was enacted, the number of Americans estimated to be without healthcare coverage was: A. 10 million B. 20 million C. 30 million D. 50 million

D. 50 million

To perform the Phalen test, the nurse practitioner maintains the patient's shoulder in neutral and the elbow in flexion. The NP then passively brings the patient's wrist to maximum flexion for what length of time? A. 3 minutes B. 5 minutes C. 30 seconds D. 60 seconds

D. 60 seconds

12-16. Evidence-based practice treatment for eating disorders include(s): A. Medications to decrease symptoms B. Therapy to address unhealthy thoughts regarding food and weight in order to decrease symptoms C. Strict monitoring of food intake and limiting exercise D. A combination of medications, therapy, and nutritional and medical care

D. A combination of medications, therapy, and nutritional and medical care

21-2. According to the hierarchy of evidence, which of the following is considered level I (strongest-level) evidence? A. A cohort study B. An expert opinion C. A randomized control trial D. A meta-analysis of a randomized control trial

D. A meta-analysis of a randomized control trial

In Alzheimer's disease, donepezil is used to increase which chemical in the brain? A. Serotonin B. Norepinephrine C. Dopamine D. Acetylcholine

D. Acetylcholine

11-51. Which of the following is not a differential diagnosis for a brain tumor? A. Meningitis B. Toxoplasmosis C. Pseudotumor cerebri D. Acute angle glaucoma

D. Acute angle glaucoma

All of the following are risks for IBD except for: A. Being of Caucasian or Ashkenazi Jewish descent B. The patient's age C. Having a first-degree relative with IBD D. Alcohol use

D. Alcohol use

Which testing is needed prior to initiation of biologic response modifiers for rheumatoid arthritis? A. Complete blood count B. Chest x-ray C. Hepatitis B and C D. All of the above

D. All of the above

Screening guidelines for colorectal cancer begin at age 50 and include a(n): A. Annual DRE and colonoscopy every 5 years B. Flexible sigmoidoscopy every 5-10 years C. Double-contrast barium enema every 10 years D. Annual GFOBT and a colonoscopy every 10 years

D. Annual GFOBT and a colonoscopy every 10 years

The nurse practitioner is evaluating a patient who has presented with knee pain. The Lachman test is positive. Which of the following structures is likely involved? A. Medial collateral ligament B. Lateral collateral ligament C. Posterior cruciate ligament D. Anterior cruciate ligament

D. Anterior cruciate ligament

The nurse practitioner notes a grade 3 systolic murmur with an ejection click at the right sternal border. Which finding does the nurse practitioner expect to see in this patient? A. Pericarditis B. Mitral valve prolapse C. Aortic regurgitation D. Aortic stenosis

D. Aortic stenosis

A 47-year-old obese man presents to the clinic with nocturnal acute onset pain and swelling to the right foot. He denies previous incidence of symptoms and denies known injury. The patient is on high blood pressure medication, is a borderline diabetic, and consumes alcohol regularly. What diagnostic test is essential for diagnosis of this patient's condition? A. CBC B. Uric acid C. X-ray of the foot D. Aspiration of the joint

D. Aspiration of the joint

Based on the Rome III criteria, all of the following are considered diagnostic for constipation except: A. Hard, lumpy stool B. Straining C. Tenesmus D. Bright red rectal bleeding

D. Bright red rectal bleeding

Rectal bleeding associated with hemorrhoids is usually described as: A. Bloody clot with mucus B. Dark brown/black stool C. Tarry black stool D. Bright red streaks with normal appearing stool

D. Bright red streaks with normal appearing stool

11-3. Which cranial nerves are associated with extraocular movements? A. CN II, III, IV B. CN II, IV, V C. CN III, IV, V D. CNIII, IV, VI

D. CNIII, IV, VI

A 45-year-old male presents with low back pain that radiates down the posterior right buttock and the thigh to his knee. He noticed the pain when lifting heavy boxes while helping a friend move late yesterday afternoon. He reports urinary incontinence since the onset of pain and now notices decreased sensation in the perianal region. On exam, straight-leg raises cause an increase in pain in the leg and posterior thigh. These symptoms suggest: A. Spondylolisthesis B. Lordosis C. Sciatica D. Cauda equina syndrome

D. Cauda equina syndrome

19-26. Which organization is responsible for issuing an NPI? A. Department of Health and Human Services B. State Medical Examiner C. National Insurance Regulator D. Center for Medicare and Medicaid Services

D. Center for Medicare and Medicaid Services

The first-line treatment for alcoholic hepatitis is: A. Paracentesis for the ascites B. Topical steroid ointment for the pruritic skin C. Opiates for the abdominal pain D. Cessation of alcohol intake

D. Cessation of alcohol intake

Which type of bacteria is a common cause of reactive arthritis? A. Pseudomonas aeruginosa B. Mycobacterium tuberculosis C. Beta-hemolytic Streptococcus D. Chlamydia trachomatis

D. Chlamydia trachomatis

11-49. The nurse practitioner is providing patient education to a newly diagnosed epileptic patient. The nurse practitioner lets the patient know that seizure triggers can include: A. Hypersomnolence B. Herbal tea C. Inadequate insulin D. Cigarette use

D. Cigarette use

13-11. Differential diagnoses for diabetes mellitus (DM) may include the following except: A. Alcoholism B. Diabetes insipidus C.. Polycystic ovary syndrome (PCOS) D. Cirrhosis

D. Cirrhosis

16-2. While examining a child, the nurse practitioner notes decreased blood pressure and weakness in the lower extremities as compared to the upper extremi- ties. What is the likely explanation? A. Ventricular septal defect B. Tetralogy of Fallot C. Normal expectation D. Coarctation of the aorta

D. Coarctation of the aorta

Hemorrhoids and perianal/anal fissures have many characteristics in common, yet each condition has unique qualities. Which of the following statements is true regarding hemorrhoids? A. Hemorrhoids commonly affect infants and older adults B. Risks for hemorrhoids include constipation, trauma, and childbirth C. Blood from hemorrhoids tends to be tarry black 39 D. Complications may include iron deficiency anemia

D. Complications may include iron deficiency anemia abon

16-29. Which of the following is the best definition of impetigo? A. Noncontagious bacterial infection affecting the skin's subcutaneous layers B. Noncontagious bacterial infection affecting the epidermal superficial layers C. Contagious bacterial infection affecting the skin's subcutaneous layers D. Contagious bacterial infection affecting the epi- dermal superficial layers

D. Contagious bacterial infection affecting the epi- dermal superficial layers

14-2. Mary, age 30, had a CBC drawn at her last visit, It indicated that she has microcytic hypochromic anemia. Further testing showed iron deficiency anemia. She was treated with ferrous sulfate 325 mg TID. The following month, a CBC was done, with the following results: -Hgb 12.5 g/dL (normal (12-16 g/dL) -Hct 36% (normal 35-46% ) -MCV 84 fL (normal 81-96 fL) -Ferritin 25 ng/dL (normal 12-156 ng/dL) The most appropriate action is: A. Cut back on the iron supplement to BID B. Perform a fecal occult blood test C. Start her on vitamin B12 supplementation D. Continue the present course of treatment

D. Continue the present course of treatment

Which of the following diagnostic tests provides a definitive diagnosis for lung cancer? A. MRI B. CT scan C. Chest x-ray D. Cytology

D. Cytology

21-7. Which research term is matched correctly with its description? A. Hypothesis: a well-established model, based on accumulated evidence, to explain a phenomenon B. Theory: a provisional (and testable) proposal by a researcher to explain a phenomenon C. Independent variable: an extraneous variable not controlled for by the researcher that can influence the outcome of an experiment D. Dependent variable: an extraneous variable not controlled for by the researcher that can influence the outcome of an experiment

D. Dependent variable: an extraneous variable not controlled for by the researcher that can influ- ence the outcome of an experiment

Initial diagnostic workup for a DVT includes: A. Obtaining a D-dimer B. Ordering a doppler ultrasound of the affect extremity C. Ordering a spiral CT of the chest D. Determining pretest probability

D. Determining pretest probability

13-29. Which of the following signs and symptoms are con- sistent with hyperthyroidism? A. Brittle nails and constipation B. Dry skin and cold intolerance C. Edema and weight gain D. Diarrhea and weight loss

D. Diarrhea and weight loss

4-18. Mrs. K. comes to the clinic because she has experienced nausea, vomiting, and diarrhea for three days. She has lost two pounds. Her medications include rosuvastatin, lisinopril, vitamin D, digoxin, and zolpidem. Which lab is most critical for the nurse practitioner to draw? A. Lipid profile B. Urinalysis C. Vitamin D level D. Digoxin level

D. Digoxin level

14-20. Kira is a 55-year-old female with a history of type 2 diabetes and hypertension. Her meds include metformin (Glucophage) 1000 mg BID, glipizide (Glucotrol) 10 mg BID, glargine insulin (Lantus) 20 units subcutaneously, lisinopril (Zestril) 20 mg daily, rosuvastatin (Crestor) 20 mg daily, and sulfamethoxazole-trimethoprim (Bactrim) 400/80 mg once daily. She was recently started on rosuvastatin for cardiovascular protection and Bactrim for recurrent cystitis. The results of recent laboratory tests are as follows: -WBC 8.2 cells/mm3 -RBC 4.3 million cells/mm3 -hemoglobin 13 g/dL -hematocrit 42 % -platelets 110,000/microL -A1C 6.8 -TC 195 -TG 110 mg/dL -LDL 75 mg/dL -HDL 42 mg/dL Based on the lab results, what would be the next step in the plan of care? A. Increase rosuvastatin dose B. Discontinue lisinopril C. Decrease glipizide dose D. Discontinue Bactrim

D. Discontinue Bactrim

13-34. The nurse practitioner knows that risk factors for thyroid cancer include all of the following except: A. A family history of multiple endocrine neoplasia B. Being female C. A personal history of being exposed to head or neck radiation D. Inherited genetic predisposition to neurological conditions

D. Inherited genetic predisposition to neurological conditions

A 41-year-old female calls the answering service to report a possible anxiety attack. She is feeling short of breath and has palpitations and mild chest pain. The symptoms began this evening when she got home from the airport after traveling on a business trip. A thorough review of her chart reveals she is a healthy nonsmoker and that her only medication is combined oral contraceptives. Based on this information, you give her which advice? A. Make an appointment in the office as soon as it opens tomorrow B. Practice slow deep breaths, take a warm shower, and try to rest C. Call in a prescription for alprazolam to the pharmacy D. Do not take any more COCS and go immediately to the nearest emergency department

D. Do not take any more COCS and go immediately to the nearest emergency department

19-8. Which of the following is not needed to establish a malpractice case? A. An injury B. A duty to provide care C. A standard of care that was not followed D. Documentation that is not complete and accurate

D. Documentation that is not complete and accurate

Manny is a 45-year-old male who comes to your clinic complaining of shortness of breath since starting his new job at a saw mill. He smokes one pack per day and states he had asthma when he was younger but "grew out of it." Which objective finding is most suggestive of a diagnosis of asthma? A. Productive cough noted during exam, respiratory rate of 16, and wheezing on auscultation of posterior left upper lobe B. Pale, boggy nasal turbinates, clear air fluid level behind bilateral tympanic membranes, barrel chest, FEV1 <80% predicted and FEV1 increased 5% after administration of albuterol C. SpO2=94%, heart rate of 88, and bilateral wheezes on auscultation D. Dry cough noted during exam, SpO2=95%, FEV1 <80% predicted, and FEV1 increased 15% after administration of albuterol

D. Dry cough noted during exam, SpO2=95%, FEV1 <80% predicted, and FEV1 increased 15% after administration of albuterol

16-19. Which of the following best describes atopic dermatitis? A. Furuncles and carbuncles in clusters with redness B. Redness with significant scaling, yet well-tolerated C. Dry skin with pruritic rashes, related to a fungus D. Dry, pruritic skin with rashes, related to allergy

D. Dry, pruritic skin with rashes, related to allergy

12-27. A 9-year-old female is following up with you after starting a trial of a stimulant medication for a diagnosis with ADHD. She has lost 1 pound over the past month, has a decreased appetite, and is doing better in school, but her guardian is concerned about the decrease in appetite. What is your plan of treatment? A. Stop the medication since this is not a common side effect to stimulants and reevaluate in one month B. Wean off the current stimulant, start another stimulant to minimize weight loss, and reevaluate in one month C. Stop the stimulant, prescribe another classification of medication, and reevaluate in one month D. Explain that loss of appetite is a common side effect of stimulant medications, help the family identify healthy and calorie-rich snacks, and reevaluate in one month

D. Explain that loss of appetite is a common side effect of stimulant medications, help the family identify healthy and calorie-rich snacks, and reevaluate in one month

A 33-year-old presents to the office with the desire to start combined oral contraceptives. This method is contraindicated if which of the following is part of her health history? A. First-degree relative with breast cancer B. Gestational hypertension during her second pregnancy C. Smoking five cigarettes per day D. Factor V Leiden

D. Factor V Leiden

The nurse practitioner is prescribing antipyrine and benzocaine for a child with otitis externa. Which is the correct prescription? A. Fill affected ear canal and repeat daily B. 2 drops to affected ear canal twice daily C. 2 drops to affected ear canal 4 times daily D. Fill affected ear canal, repeat every 2 hours as needed

D. Fill affected ear canal, repeat every 2 hours as needed

14-8. Lymphoma survivors need to be vigilant for the appearance of adverse sequelae. Which of the following is not an adverse sequela? A. Breast cancer B. Cardiac failure C. Reduced fertility D. Frequent infections

D. Frequent infections

A 19-year-old female presents to the office for a well-woman exam. She has been sexually active with her first partner for 6 months. She denies any complaints and states, "I'm just here for my birth control refill." Her exam should include which of the following? A. Pap smear, gonorrhea test, and chlamydia test B. Full physical exam including clinical breast exam and pelvic exam C. Complete health history and vital signs only; no physical exam is necessary D. Gonorrhea test and chlamydia test

D. Gonorrhea test and chlamydia test

14-4. Vitamin B12 deficiency anemia causes which of the following changes in the RBC indices? A. Normal MCV, MCHC, RDW B. Low MCV, MCHC, high RDW C. Low MCV, MCHC, normal RDW D. High MCV, normal MCHC, high RDW

D. High MCV, normal MCHC, high RDW

12-1. The nurse practitioner is speaking with a parent who abused her child. Which of the following risk factors would the nurse practitioner expect to find? A. Single-parent home situation B. History of parental mental illness C. Consistent communication patterns D. History of a parent having been abused

D. History of a parent having been abused

The FNP is assessing a young child and notes a 2mm pustule on the left lower eyelid margin accompanied by erythema, swelling, and a small amount of purulent discharge. This finding is most consistent with which condition? A. Blepharitis B. Chalazion C. Conjunctivitis D. Hordeolum

D. Hordeolum

13-35. The nurse practitioner is educating a patient on typical expectations status post thyroidectomy. The nurse practitioner makes the patient aware of the need for lifelong: A. Antineoplastic medication-such as lenvatinib (Lenvima)-due to a high thyroid cancer recurrence B. Hoarseness due to the unavoidable vocal cord damage C. Dysphagia due to the unavoidable esophageal damage D. Hormone replacement with levothyroxine (Synthroid), as thyroidectomy eliminates the body's ability to produce any thyroid hormone

D. Hormone replacement with levothyroxine (Synthroid), as thyroidectomy eliminates the body's ability to produce any thyroid hormone

19-19. Ethical nursing care considerations should take into account: A. Hospital rules B. Concern of what one's nursing colleagues will think C. Nursing actions that might result in punishment D. Human rights overall

D. Human rights overall

3-18. Which type of vaccine is always contraindicated in pregnancy? A. Polysaccharide B. Toxoid C. Killed D. Live

D. Live

12-29. Which of the following statements is true about the risk of suicide? A. Suicidal patients will present with overt statements about self-harm B. A screening tool such as SPS is the primary means of assigning a patient for suicide potential C. Married individuals are more likely to attempt suicide than single individuals D. Mental illness, including substance use, contributes to suicidality

D. Mental illness, including substance use, contributes to suicidality

13-13. The nurse practitioner knows conditions mimicking Cushing syndrome include: A. Anxiety B. Hyperthyroidism C. Tuberculosis D. Metabolic syndrome

D. Metabolic syndrome

17-6. A 3-year-old with a history of impetigo has a low-grade fever. His mother states he is not running around like his usual self. Upon physical examination, the 3-year-old is guarding his left upper leg. The WBC and CRP are elevated. The child should be referred to which of the following specialists? A. Physical therapist B. Occupational therapist C. Rheumatologist D. Orthopedist

D. Orthopedist

4-6. An 88-year-old male is being treated for complicated pyelonephritis with gentamicin IM daily. His wife called the office to report that he didn't hear his alarm clock go off this morning. In addition, when their granddaughter called earlier that day, he had to hand the phone to his wife because he couldn't understand what she was saying. Based on this information, you are most concerned about which possible serious adverse effect of gentamicin? A. Dementia B. Hepatotoxicity C. Xerophthalmia D. Ototoxicity

D. Ototoxicity

Which of the following describes a peritonsillar abscess? A. Inflammation of the tonsils B. Decreased blood supply to the tonsils C. Autoimmune destruction of the tonsils D. Palpable collection of pus around the tonsils

D. Palpable collection of pus around the tonsils

14-16. Diagnostic tests for multiple myeloma may include all of the following except: A. Bone marrow biopsy B. Serum free light chain testing C. Serum protein electrophoresis D. Peripheral blood smear

D. Peripheral blood smear

14-26. The family nurse practitioner (FNP) is reviewing a complete blood count (CBC) report, which shows a white blood cell (WBC) count of 19,000 cells/mm3 for a patient complaining only of fatigue. Upon repeat CBC, the WBC count is 19,700 cells/mm3. Which lab test should the FNP order next? A. Bone marrow aspiration B. Blood culture and sensitivity C. Epstein-Barr virus titer D. Peripheral blood smear

D. Peripheral blood smear

11-44. Which of the following is accurate regarding the treatment of GBS? A. Patients frequently require treatment for tachycardia; they rarely require treatment for bradycardia B. Long-term antihypertensive medications will be part of the treatment plan C. Corticosteroids (oral and intravenous) are com- monly used as monotherapy D. Plasma exchange and IVIG (immunoglobulin therapy) are commonly used to hasten recovery

D. Plasma exchange and IVIG (immunoglobulin therapy) are commonly used to hasten recovery

A 65-year-old female with congestive heart failure and diabetes mellitus presents with productive cough, fever, and dyspnea. Which diagnosis is most likely? A. Asthma exacerbation B. Acute bronchitis C. Influenza D. Pneumonia

D. Pneumonia

The NP has diagnosed an older adult patient with benign paroxysmal positional vertigo (BPPV). Which of the following tests aids in diagnosis? A. Positive Epley maneuver B. Pneumatic otoscopy C. Computed tomography D. Positive Dix-Hallpike test

D. Positive Dix-Hallpike test

19-4. Which of the following does not represent protected health information? A. Address, surgical history, phone number B. Diagnosis, medication list, billing and payment information C. Patient's smoking status, laboratory results, psy chiatric record D. Review of systems for a particular visit, student's immunization record held by the school, family history

D. Review of systems for a particular visit, student's immunization record held by the school, family history

The nurse practitioner ordered a diagnostic mammogram and breast ultrasound to further evaluate a palpable area of concern for a 55-year-old female patient. The patient reported feeling a mass during her recent self-breast exam. The nurse practitioner appreciated thickening on exam but no obvious mass. The imaging report notes that there is no visible correlate for the palpable finding with a BIRADS category 2 and recommendation for a screening mammogram in one year. What will the nurse prac- titioner do next? A. Reassure the patient, as her imaging was normal and she has no family history of breast cancer B. Schedule the patient for return appointment with a screening mammogram in one year C. Schedule a short-term follow up appointment in 6 months D. Schedule a consult with a breast surgeon

D. Schedule a consult with a breast surgeon

A 36-year-old man presents to the office with complaint of a lump in his chest. On exam, you note a 1 cm x 1 cm hard, non-tender mass beneath the right areola at 3 oclock. Based on these findings, you make which of the following plans? A. Schedule the patient for a right-breast ultrasound B. Refer to a breast surgeon for removal C. Continue to monitor for one month, and return for care if still present D. Schedule the patient for a diagnostic mammogram and breast ultrasound

D. Schedule the patient for a diagnostic mammogram and breast ultrasound

4-8. A 33-year-old female presents to the office with complaints of urinary urgency, frequency, pressure, and dysuria. You would like to prescribe nitrofurantoin (Macrobid) 100mg PO BID X 5 days for her uncomplicated UTI. Which medical information would cause you to choose a different antibiotic? A. She reports bright orange urine after taking Pyridium earlier today B. She sometimes takes diphenhydramine to sleep at night C. She has a history of an allergic reaction to amoxicillin D. She is currently 39 weeks pregnant

D. She is currently 39 weeks pregnant

15-5. Doxycycline is NOT recommended as a treatment for which of the following? A. Anthrax B. Lyme disease C. Rocky Mountain spotted fever D. Smallpox

D. Smallpox

20-6. Why must the nurse practitioner consider the social determinants of health? A. In the United States, all people are treated equally B. Considering these factors is the right thing to do C. Negative social determinants decrease the cost of healthcare D. Social determinants affect a wide range of health and quality-of-life outcomes

D. Social determinants affect a wide range of health and quality-of-life outcomes

3-5. Which organization, created by Congress in 1984, is an independent, volunteer panel of national experts that develops screening recommendations based on evidence-based medicine? A. The World Health Organization B. American Association of Family Physicians C. National Screening Program D. The U.S. Preventive Services Task Force (USPSTF)

D. The U.S. Preventive Services Task Force (USPSTF)

11-38. A patient experiencing unexplained muscle weak- ness, diplopia, ptosis, and difficulty breathing is to receive a Tensilon test (edrophonium). Which find- ing is suggestive of myasthenia gravis? A. The patient experiences an exacerbation of paralysis B. The patient experiences a sense of warmth with facial flushing C. The patient reports a ringing in the ears D. The patient experiences improved muscle strength

D. The patient experiences improved muscle strength

21-5. The independent variable in an experiment refers to: A. The variable that is measured to see if the intervention had an effect B. A provisional (and testable) proposal by a researcher to explain a phenomenon C. An extraneous variable that was not controlled for by the researcher D. The variable that is manipulated by the researcher to produce an effect

D. The variable that is manipulated by the researcher to produce an effect

11-9. The NP is aware that in order to make a diagnosis of dementia as per DSM-5 criteria, a decline in memory and a decline in at least one of the following must be present except the ability to: A. Generate coherent speech and understand spoken or written language B. Recognize or identify objects C. Execute motor activities D. Think objectively without prejudice

D. Think objectively without prejudice

Which of the following is associated with Vitamin B-12 anemia? A. Spoon shaped nails and Pica B. An abnormal neuro exam C. Vegan diet D. Tingling/ numbness in both feet

D. Tingling and numbness in both feet: B12 can cause nerve damage if left untreated, also dementia, memory loss, difficulty walking, mood changes, depression also

16-26. Which medication is appropriate for preventing burn wound infection? A. Oral erythromycin B. Oral moxifloxacin C. Topical corticosteroid D. Topical silver sulfadiazine

D. Topical silver sulfadiazine

A stool culture reveals Shigella. What medication should the nurse practitioner prescribe? A. acyclovir (Zovirax) B. amoxicillin clavulanate (Augmentin) C. rimantadine (Zantac) D. trimethoprim-sulfamethoxazole (Bactrim DS)

D. trimethoprim-sulfamethoxazole (Bactrim DS)

11-47. The patient with epilepsy is morbidly obese and lets the nurse practitioner know she is trying to lose weight. The patient states that she is willing to take an anti-seizure medication, but she does not want to take any medication associated with weight gain. The nurse practitioner knows to avoid prescribing: A. lamotrigine (Lamictal) B. topiramate (Topamax) C. phenytoin (Dilantin) D. valproic acid (Depakote)

D. valproic acid (Depakote)

A patient that has been taking ranitidine and omeprazole for 2 years is at risk of ________________ deficiency? A. potassium B. vitamin C C. calcium D. vitamin B12

D. vitamin B12

All of the following factors increase risk of mortality for patients diagnosed with bacterial pneumonia except: A. Alcoholism B. Very young age or the elderly C. Multiple lobar involvement D. Hypertension

D: Hypertension

Which of the following findings are seen in a patient with folate deficiency anemia? A. Microcytic and Hypochromic RBCs B. Microcytic and normochromic RBCs C. Normal size and color RBCs D. Macrocytic and Normocytic RBCs

D: Macrocytic and Normocytic RBCs are found in Folate deficiency anemia

All of the following are associated with an increased risk for normocytic anemia except: A. Rheumatoid arthritis B. Lupus C. Chronic immune disorders. D. Pregnancy

D: Pregnancy, during pregnancy women at risk to develop microcytic, hypochromic anemia due to dilutional effect of increased blood volume in pregnancy.

A 70 y.o male c/o bright red spot in his left eye for 2 days. He denies pain, visual changes, or headaches. New onset cough from recent viral illness. The only med he was on was Bayer Aspirin 1 tablet daily. Which of the following is most likely? A. Corneal Abraison B. Acute bacterial conjunctivitis C. Acute uveitis D. Subconjunctival hemorrhage

D: Subconjunctival hemorrhage: caused by increased intraocular pressure due to straining, coughing, heavy lifting etc. Self limiting- disappears in 1-3 weeks


Kaugnay na mga set ng pag-aaral

NUR1025 Most Missed Questions Exam 2

View Set

IXL compound complex simple and compound complex sentences

View Set